Case Digest Criminal Law Draft Final

You might also like

Download as docx, pdf, or txt
Download as docx, pdf, or txt
You are on page 1of 292

CASE DIGEST IN

CRIMINAL LAW
1

Submitted by:

VIVAR, NELSON M.

Submitted to:

PROSECUTOR JEDREK NG

A.Y. 2022 – 2023


Second Semester
CRIMINAL LAW DIGESTS

Subj Art. G.R. No. Date Title Ponente Badge Keyword

Nullum Crimen,
September LEONARDO-DE Sec. guard
CRIM1 1 176364 Rimando v. Comelec Nulla Poena Sine
18, 2009 CASTRO, J (election)
Lege
June 26, Utilitarian Theory- Check-
CRIM1 1 96132 1992 Magno v. CA PARAS, J.
Protective Theory collateral
Construction of Business-
CRIM1 1 180016 April 29, 2014 Corpuz v. People PERALTA, J.
Penal Laws jewelry
December 10, Del Socorro v. Van Married-
CRIM1 1 193707 2014 Wilsem PERALTA, J. Territoriality Holland
January 11, BBB- chef
CRIM1 1 212448 2018 AAA vs. BBB TIJAM, J Transitory Crime Singapore
March 23, Raid in Cebu-
CRIM1 1 184355 2015 Jacaban v. People PERALTA, J Intent illegal firearms
March 11, Complex Crime/ Ex Robbery-
CRIM1 1 123696 2004 People v. Hijada AZCUNA, J
Post Facto stabbed
November 19, Estrada vs.
CRIM1 1 148560 2011 Sandiganbayan BELLOSILLO J: Mala In Se plunder

Diplomatic Agent Scalzo-


CRIM1 1 142396 Feb. 11, 2003 Minucher vs. Scalzo VITUG, J
Immunity drugs
March 26, YNARES- Diplomatic
CRIM1 1 125865 2011 Liang vs. People ADB employee
SANTIAGO, J Immunity
Kuratong
CRIM1 1 149453 April 1, 2003 People vs. Lacson CALLEJO, SR., J Prospectivity Baleleng
June 21,
CRIM1 1 151876 Go vs. Dimagiba PANGANIBAN J Prospectivity BP22
2005
June 21, Animals-
CRIM1 2 5270 2005 US vs. Bull ELLIOTT, J Terrotoriality foreign ship
CRIM1 3 193169 April 06, 2015 Roque v. People DEL CASTILLO, J: Criminal Intent Gun shooting
November 26, Specific Intent- Stabbed while
CRIM1 3 178512 2014 De Guzman v. People BERSAMIN, J:
Intent to Kill fetching water
January 25,
CRIM1 3 166326 2006 Rivera v. People CALLEJO, SR. J: Criminal Intent Hollow blocks

Culpable Felony: Aquilian


February 1,
CRIM1 3 151258 Villareal v. People SERENO, C J: Negligence and initiation-
2012 Ateneo
Imprudence
January 27, Criminal Intent-
CRIM1 3 163927 2006 Gaviola v. People CALLEJO, SR. J: coconut
Intent to gain
March 19, Intruder- knife-
CRIM1 3 L-5272 1910 US v. Ah Chong CARSON J: Mistake of Fact killed
Police killed
CRIM1 3 47722 July 27, 1943 People v. Oanis MORAN J: Mistake of Fact diff. person
120744- June 25, Yapyuco v. Hearsay NPA-
CRIM1 3 46 2012 Sandiganbayan PERALTA J: Mistake of Fact Tamaraw jeep
Double Jeopardy-
February 10,
CRIM1 3 152644 Loney v. People CARPIO J: mala in se and mala Mine waste
2006
prohibita
March 20, PERLAS-BERNABE Non remittance
CRIM1 3 224943 2017 Jorge Navarra v. People Mala Prohibita of SSS
J:
CRIM1 3 209464 July 1, 2015 Dungo v. People MENDOZA J: Mala Prohibita APO Fraternity
February 9, CHICO-NAZARIO
CRIM1 3 152133 2006 Calimutan vs. People Criminal Intent Hitting stone
J:
November 19, Estrada vs.
CRIM1 3 148560 2011 Sandiganbayan BELLOSILLO J: Mala In Se plunder

Intentional Felony-
August 28, Wrongful Act
CRIM1 4 171951 Garcia v. People QUISUMBING J: videoke
2009 Different From That
Intended
Intentional Felony-
Wrongful Act Bukidnon town
CRIM1 4 201858 June 4, 2014 People v. Jenny Likiran REYES J:
Different From That fiest
Intended
June 20, Elbowed wife
CRIM1 4 L-36858 1988 People v. Ulep GANCAYCO J: Proximate Cause internal malady
Stabbed- not
November 14,
CRIM1 4 181052 Belbis Jr. v. People PERALTA J: Proximate Cause hospitalized-
2012 money concern
Body dropped
CRIM1 4 116736 July 24, 1997 People v. Ortega PANGANIBAN J: Praeter Intentionem to well
January 7, GUTIERREZ, JR. Efficient intervening Palay flooded-
CRIM1 4 72964 1988 Urbano v. IAC
J: cause bolo
September 7, LEONARDO-DE Efficient intervening Stabbed-
CRIM1 4 186412 2011 People v. Villacorta CASTRO J: cause bamboo stick

VIVAR, 2
CRIMINAL LAW DIGESTS

Subj Art. G.R. No. Date Title Ponente Badge Keyword


Police
CRIM1 4 117954 April 27, 2000 People v. Acuram QUISUMBING J: Proximate Cause passenger-
gunshots
ABERRATIO ICTUS Eduardo took
November 23,
CRIM1 4 180219 People v. Talampas BERSAMIN, J: (mistake in the refuge to
2011 Ernesto- shoot
blow)
ABERRATIO ICTUS
June 23, Bodoy and
CRIM1 4 125909 People v. Flora QUISUMBING, J: (mistake in the Boboy
2000
blow)
ABERRATIO ICTUS
Stray bullet-
CRIM1 4 205228 July 15, 2015 People v. Adriano PEREZ, J: (mistake in the bystander
blow)
ABERRATIO ICTUS
114003- January 14, Pait,
CRIM1 4 People v. Violin BELLOSILLO, J: (mistake in the
06 1997 Catbalogan
blow)
October 3, Beaten two
CRIM1 4 177218 2011 People v. Sales DEL CASTILLO, J: Proximate Cause sons
October 21, Impossible Crime Gunshots,
CRIM1 4 103119 1992 Intod v. CA CAMPOS, JR J:
Legal Impossibility victim not home
Impossible Crime Check- no
CRIM1 4 162540 July 13, 2009 Jacinto v. People PERALTA J: Factual value, NBI
Impossibility entrapment
March 1, Kidnapped –
CRIM1 4 95322 1993 People v. Tan PERALTA J: Impossible Crime child of doctor
Unlicensed
CRIM1 6 210710 July 27, 2016 People v. Gaborne PEREZ, J Frustrated Murder firearm
Attempted
August 3, tresspass to
CRIM1 6 L-43530 People v. Lamahang RECTO, J Press Bar
1935 dwelling instead of
attempted robbery
Light Coercion as
February 22,
CRIM1 6 138033 Baleros v. People GARCIA, J opposed to Squeezed
2006
attempted rape
143468- January 24, Sisters- video
CRIM1 6 71 2003 People v. Lizada CALLEJO, SR., J Attempted Rape shop
October 7, Frustrated
CRIM1 6 198400 2013 Abella v. People REYES, J scythe
Homicide
Attempted Murder
Father and 2
CRIM1 6 202867 July 15, 2013 People v. Labiaga CARPIO, J not Frustrated daughters
Murder
No frustrated theft,
June 21, Tide,
CRIM1 6 160188 Valenzuela v. People TINGA, J only attempted or
2007 Supermarket
consummated
Ponds Cream
CRIM1 6 206442 July 1, 2015 Canceran v. People TINGA, J Attempted theftinside a box of
Magic Flakes
CRIM1 6 126148 May 5, 1999 People v. Quinanola VITUG, J Consummated Rape NPA
141724- November 12,
CRIM1 6 27 2003 People v. Orande CORONA, J No Frustrated Rape Step-daughter

Acts of failed to
January 15, LEONARDO-DE Lasciviousness penetrate his
CRIM1 6 202122 People v. Pareja
2014 CASTRO, J instead of thing- 13y/o
attempted rape sister
Acts of
October 08,
CRIM1 6 166441 Cruz v. People BERSAMIN, J Lasciviousness vs. Tent
2014
attempted rape
December 2, Defensor Santiago v.
CRIM1 6 109266 1993 QUIASON, J Continued Crime 32 aliens
Justice Garchitorena
Consummated Philippine
CRIM1 6 88724 April 3, 1990 People v. Orita MEDIALDEA, J Rape- No Constabulary
Frustrated Rape Officer soldier
March 30, Statutory Rape
CRIM1 6 129433 2000 People v. Campuhan BELLOSILLO, J Attempted Rape (4 yr old)

VIVAR, 3
CRIMINAL LAW DIGESTS

Subj Art. G.R. No. Date Title Ponente Badge Keyword

Light Coercion as
February 22,
CRIM1 6 138033 Baleros v. People GARCIA, J opposed to Squeezed
2006
attempted rape

Acts of
October 8,
CRIM1 6 166441 Cruz v. People BERSAMIN, J Lasciviousness vs. Tent
2014
attempted rape
YNARES- Three Stages of collecting
CRIM1 6 122099 July 5, 2000 People v. Listerio money
SANTIAGO, J Felony
Kidnap or 3
131926 June 18,
CRIM1 8 People v. Pagalasan CALLEJO, SR., J Conspiracy Handwritten
138991 2003 Letters
138874- February 3,
CRIM1 8 75 2004 People v. Larrañ aga SERENO, C.J: Conspiracy Gang rape

LEONARDO-DE 2 police officers


CRIM1 8 175926 July 6, 2011 People v. Carandang Conspiracy
CASTRO, J shot and killed
CRIM1 8 195196 July 13, 2015 People v. Octa SERENO, C.J Conspiracy Lancer car
CRIM1 8 196735 May 5, 2014 People v. Feliciano LEONEN, J Conspiracy Frat War
August 10, Pawnshop/
CRIM1 8 187536 2011 People vs. Bokingco PEREZ, J Conspiracy Hammer
March 10,
CRIM1 8 132895 2004 People v. Castillo PER CURIAM Conspiracy birthday party

August 24, Fernan Jr, et al vs. Cebu Highway


CRIM1 8 145927 2007 People VELASCO, JR., J Conspiracy Scam
February 17, AUSTRIA-
CRIM1 10 141066 2005 Ladonga vs. People Conspiracy in BP 22 UCPB
MARTINEZ, J
Dangerous Drugs tea bags of
CRIM1 10 93028 July 29, 1994 People vs. Simon REGALADO, J marijuana
Act and RPC
September AUSTRIA- VAWC and
CRIM1 10 168852 30, 2008 Go-Tan vs. Tan MARTINEZ, J Conspiracy
VAWC

Absence of Unlawful
January 15, Aggression;
CRIM1 11 135981 People v. Genosa PANGANIBAN, J Lead pipe
2004 Battered Wife
Syndrome
March 31, LEONARDO-DE Absence of Unlawful
CRIM1 11 174483 People v. Regalario nightstick
2009 CASTRO, J Aggression
June 29, Absence of Unlawful Atulayan Norte,
CRIM1 11 171284 2015 People v. Dulin BERSAMIN, J Tuguegarao
Aggression
Absence of
January 25, reasonable
CRIM1 11 177743 People v. Fontanilla BERSAMIN, J bellang wood
2012 necessity of means
employed.
September Absence of Unlawful Noe stoning the
CRIM1 11 158057 Toledo v. People CALLEJO, SR., J
24, 2004 Aggression house
June 30,
CRIM1 11 L-7020 US vs. Ramos TRENT, J Defense of Honor Couple- bolo
1954
Incomplete Defense;
L-33466- Defense of Property;
CRIM1 11 April 20, 1983 People vs. Narvaez MAKASIAR, J fence
67 Lack of Reasonable
Means Employed
Presence of all
March 21, Drunk police
CRIM1 11 191913 Nacnac vs. People MAKASIAR, J elements of self
2012 colleague
defense
Privileged
Mitigating
September LEONARDO-DE Nogi- Rape,
CRIM1 12 169641 People v. Sarcia Circumstance-
10, 2009 CASTRO, J Backyard
Child in Conflict
with the Law
Exempting
February People v. Victoriano Circumstances Victoriano
CRIM1 12 187683 NACHURA, J Wife Beating
11, 2010 Dela Cruz Elements of
Accident
Exempting Bouncing
September
CRIM1 12 149275 Ty v. People TINGA, J. Circumstances Check-
27, 2004 Uncontrollable Fear Hospital bills

VIVAR, 4
CRIMINAL LAW DIGESTS

Subj Art. G.R. No. Date Title Ponente Badge Keyword

Exempting
June 29, YNARES- Circumstances
CRIM1 12 172695 People v. Castillo Sling & Arrow
2007 SANTIAGO, J Elements of
Accident
Exempting
Stabbing
February 7, SANDOVAL- Circumstances
CRIM1 12 124392 People v. Abrazaldo public
2003 GUTIERREZ, J Elements of authorities
Accident
Exempting
June 23, VILLARAMA, JR., Circumstances Wife shot
CRIM1 12 186128 People v. Latosa
2010 J Accident- Intent to husband
kill
Mitigating
Photocopy,
December 7, Circumstances
CRIM1 13 210810 Del Poso v. People PERALTA, J Iron, child
2016 Grave wrong; not
abuse
passion/ obfuscation
Mitigating
Circumstances No Barfight, Hot-
January 20, intention to headed
CRIM1 13 182750 Urbano v. People VELASCO, JR., J
2009 commit so grave a Victim, Lucky
wrong; sufficient Punch
provocation
Mitigating
Circumstances
Ignas killed
140514- September Immediate
CRIM1 13 People v. Ignas QUISUMBING, J husband-
15 30, 2003 vindication; passion
Bakery, Affair
or obfuscation,
voluntary surrender
T-shirt
Mitigating printing; Brgy.
December 7, Nizurtado v.
CRIM1 13 107838 VITUG, J Circumstances Captain,
1994 Sandiganbayan Voluntary surrender Malversation
of Funds
Mitigating
January 15, Circumstances Battered
CRIM1 13 135981 People v. Genosa PANGANIBAN, J Passion or
2004 woman
obfuscation; Illness
Mayor &
Mitigating
Treasurer
February 8, Manuel v. Circumstances No
CRIM1 13 158413 MENDOZA, J Malversation,
2012 Sandiganbayan Payment of the
Personal
amount malversed
Loans
Mitigating
CRIM1 13 135701 May 9, 2002 People v. Callet PUNO, J Circumstances cara y cruz
Voluntary Surrender
Mitigating
January 30, YNARES- fenced disco
CRIM1 13 131839 People vs. Adlawan Circumstances
area in Oril
2002 SANTIAGO, J passion or obfuscation
Mitigating
July 16, YNARES- Urinated on
CRIM1 13 132325 People v. Espina Circumstances
2002 SANTIAGO, J Immediate vindication the victim
Aggravating
October 3, Circumstances Sumpak,
CRIM1 14 216671 Dorado v. People MENDOZA, J No evident
2016 Bully, Minor
premeditation
Aggravating Drinking,
February Circumstances Stabbing,
CRIM1 14 198954 People v. Macaspac BERSAMIN, J No evident Went Into
22, 2017
premeditation Hiding
Aggravating Bilibid
January 24, MELENCIO-
CRIM1 14 L-40757 People v. Macariola Circumstances Gambling,
1983 HERRERA, J Quasi-recidivism Stabbing
Aggravating
October 24, Circumstances
CRIM1 14 196434 People v. Nazareno ABAD, J Wake, Brawl
2012 Abuse of superior
strength
Aggravating
Fighting
June 29, Circumstances
CRIM1 14 171284 People v. Dulin BERSAMIN, J Incomplete Self-
Cock, Self-
2015 Defense
Defense- Absent
Aggravating Tomahwak
January 12, Circumstances Axe, Abuse Of
CRIM1 14 190912 Fantastico v. Malicse PERALTA, J
2015 Abuse of superior Superior
strength Strengt
Aggravating
August 10, Construction
CRIM1 14 187536 People v. Bokingco PEREZ, J Circumstances
2011 Failure of prosecution Worker
Aggravating Live-in
February LEONARDO-DE
CRIM1 14 168169 People v. Tabarnero Circumstances Partner,
24, 2010 CASTRO, J Treachery Stoppage of

VIVAR, 5
CRIMINAL LAW DIGESTS

Subj Art. G.R. No. Date Title Ponente Badge Keyword


Planned
Marriage
Aggravating
February 1, LEONARDO-DE Urinated on
CRIM1 14 186541 People v. Vilbar Circumstances
Table
2012 CASTRO, J Treachery- Absent
Aggravating Unlicensed
March 25, PERLAS- Circumstances Fire-arm,
CRIM1 14 206381 People v. Matibag Treachery; Sub-division
2015 BERNABE, J
unlicensed firearms Altercation
Aggravating
Fraternity
CRIM1 14 196735 May 5, 2014 People v. Feliciano LEONEN, J Circumstances
Attack
Disguise; Treachery
Aggravating
September CHICO-NAZARIO,
CRIM1 14 171018 People v. Naelga Circumstances Drug Pusher
11, 2009 J Entrapment
Aggravating
Robbery and
October 26, Circumstances
CRIM1 14 134802 People v. Dizon PER CURIAM Rape- Penis
2001 Cruelty &
bolitas
uninhabited place
Aggravating
138306- December 30 cavans of
CRIM1 14 People v. Ancheta BELLOSILLO, J Circumstances
07 21, 2001 Treachery; Band
clean palay
Mayor
16- 204481- October 14,
CRIM1 Ambagan, Jr. v. People VELASCO, JR., J induced
20 82 2015
Shootout
Rape,
16- September Prostitution
CRIM1 193854 People v. Dulay PERALTA, J
20 24, 2012 in Kubuhan,
Speed
Fishdealer,
16- October 1,
CRIM1 172707 People v. Gambao PEREZ, J Chowking,
20 2013
Ransom
Three people
16- April 03, ganging up
CRIM1 125688 People v. Cupino PANGANIBAN, J
20 2000 on victim
with bolo
16- June 27,
CRIM1 20 121828 2003 People v. Agulos CALLEJO, SR., J
16- 121651- August 16, YNARES-
CRIM1 20 52 2000 People v. Watimar
SANTIAGO, J
Murder or
21- January 31,
CRIM1 130492 People v. Arrojado MENDOZA, J Suicide of
88 2001 Cousin
Ambush of
21- September Mayor of
CRIM1 184500 People v. Nelmida PEREZ, J
88 11, 2012 Tawan-
Tawan
Drunk
21- December LEONARDO-DE Philippine
CRIM1 199892 People v. Punzalan
88 10, 2012 CASTRO, J Navy, Run
Over
Miriam
21- December 2, Santiago v. Santiago,
CRIM1 109266 QUIASON, J
88 1993 Garchitorena Unqualified
Aliens
21- September People v. Patriarca, et.
CRIM1 88 135457 BUENA, J
29, 2000 al.
21- September YNARES-
CRIM1 88 139857 15, 2006 Batulanon v. People SANTIAGO, J
21- April 20,
CRIM1 88 197562 2015 Fransdilla v. People BERSAMIN, J
21- July 23,
CRIM1 88 176317 2014 Zafra v. People BERSAMIN, J
21- November
CRIM1 88 181441 People v. Lopez CARPIO, J
14, 2008
21-
CRIM1 88 181036 July 6, 2010 People v. Leonardo PEREZ, J
21- June 13, PERLAS-
CRIM1 88 186469 2012 People v. Matias
BERNABE, J
21- June 13, Legrama v.
CRIM1 88 178626 2012 Sandiganbayan PERALTA, J
21- March 3,
CRIM1 88 186441 2010 Flordeliz v. People NACHURA, J

VIVAR, 6
CRIMINAL LAW DIGESTS

Subj Art. G.R. No. Date Title Ponente Badge Keyword

21- April 20,


CRIM1 88 197712 2015 Imbo v. People PEREZ, J
PD February 1,
CRIM1 151258 2012 Villareal v. People SERENO, J
968
PD December
CRIM1 182748 Colinares v. People ABAD, J
968 13, 2011
PD January 29,
CRIM1 968 L-67301 1990 Bala v. Hon. Martinez SARMIENTO, J
PD August 10,
CRIM1 168550 2006 Urbano v. Comelec TINGA, J
968
PD October 09,
CRIM1 193237 2012 Jalosjos v. Comelec CARPIO, J
968
April 18,
CRIM1 89 200030 2012 People v. Bayot PEREZ, J
April 27, AUSTRIA-
CRIM1 89 168641 2007 People v. Bautista
MARTINEZ, J
June 13,
CRIM1 89 152662 2012 People v. Pangilinan PEREZ, J
October 6, SEC v. Interport CHICO-NAZARIO,
CRIM1 89 135808 2008 Resource Corporation J
165510- July 28, YNARES-
CRIM1 89 33 2006 Romualdez v. CA SANTIAGO, J
September People v. Patriarca, et.
CRIM1 89 135457 BUENA, J
29, 2000 al.

VIVAR, 7
CRIMINAL LAW DIGESTS

Nullum Crimen, Nulla Poena Sine Lege <<<

Rimando v. Comelec
G.R. No. 176364, September 18, 2009
LEONARDO-DE CASTRO, J:

Doctrine:
Nullum crimen sine lege is the principle in criminal law and international criminal law
that a person cannot or should not face criminal punishment except for an act that was
criminalized by law before he/she performed the act.

Facts:

 During the 2001 Election Period, RIMANO, as the President and General Manager
of the Illustrious Security and Investigation Agency Inc., despite COMELEC
denial of application for a Firearms & Deadly Weapons Ban Exemption, permit
his security guards Carag and Enaya to carry firearms which lead to MAGNO, a 19-
year old student being shot to death. With that being said, MAGNO (respondent)
filed a complaint with COMELEC accusing Carag, Enaya and Rimando (petitioner)
for violating COMELEC Resolution No. 3328. The Provincial Election Supervisor
DISMISSED the complaint on the grounds that the licensed firearms were carried
and used within the guard’s place of work without the need for exception nor
permit. Consequently, COMELEC AFFIRMED the PROVINCIAL ELECTION
SUPERVISOR’s DISMISSAL of the complaint; but directed its law department to file
the proper information against RIMANDO for his failure to acquire a permit from
the Commission as his capacity as the President and General Manager of the
agency. RIMANDO filed a motion for reconsideration which was GRANTED and
REVERSED and SET ASIDE the previous resolution, it was held that:

 “There is no dispute that the security agency concerned, as represented by


respondent Rimando, is required by law to secure the necessary permit from
the Commission. In fact, the records show that the said agency represented by
respondent Rimando did in fact apply for exemption from the gun ban, but the
same was denied for failure to comply with all the requirements.”

 RIMANDO moved for reconsideration. COMELEC DENIED and directed its law
department to file the proper information against RIMANDO. RIMANO filed a
motion accusing COMELEC of acting in grave abuse of discretion and/or without
or excess of jurisdiction in holding RIMANDO criminally liable for his failure to
secure a written authority from the COMELEC to bear firearms during the
election period.

Issue:

Whether or not COMELEC of acted in grave abuse of discretion and/or without or


excess of jurisdiction in holding RIMANDO criminally liable for his failure to secure a
written authority from the COMELEC to bear firearms during the election period.

Ruling:

No
RIMANDO should be absolved of any criminal liability, consistent with the doctrine of
nullum crimen, nulla poena sine lege— there is no crime when there is no law
punishing it. What is punished or prohibited under Section 261 (s) is merely the
bearing of arms by a member of a security agency outside the immediate vicinity of his
place of work without the approval of the COMELEC. The provision does not prohibit
certain acts or provide penalties for its violation; neither does it describe the nature of
a crime and its punishment. Consequently, the above quoted phrase cannot be
considered a penal provision. Authorization is not required under Section 261 (s) of
the Omnibus Election

VIVAR, 8
CRIMINAL LAW DIGESTS

Code, but rather an added regulatory measure, the same is likewise not a penal
provision. At most, it is an administrative requirement.

Fallo:
WHEREFORE, The Resolutions of the COMELEC En Banc issued on October 11, 2005
and January 5, 2007 in Election Case No. 01-130 are hereby REVERSED and SET
ASIDE.

VIVAR, 9
CRIMINAL LAW DIGESTS

Utilitarian Theory- Protective Theory <<<

Magno v. CA
G.R. No. 96132, June 26, 1992
PARAS, J:

Doctrine:
Under the utilitarian theory, the "protective theory" in criminal law, y" in criminal law,
"affirms that "affirms that the primary function of punishment is the protective (sic) of
society against actual and potential wrongdoers." Corollary to the above view, is the
application of the theory that of the theory that "criminal law is founded upon th
"criminal law is founded upon that moral disapproba at moral disapprobation . . . of
tion . . . of actions which are immoral, i. e., which are detrimental (or dangerous) to
those conditions upon which depend the conditions upon which depend the existence
and prog existence and progress of human society. This ress of human society. This
disapprobation is inevitable to the extent that morality is generally moral opinions of
all . . . That which we call punishment is only an external means of emphasizing moral
disapprobation: the method of punishment is in reality the amount of punishment.

Facts:

This is an appeal by certiorari from the decision of the court of appeals which affirmed
the decision of the regional trial court of Quezon city, branch 104 finding Magno
(accused petitioner) guilty of violations of batas Pambansa bilang 22.

Sometime in April 1983, Magno was in the process of putting up a car repair shop
business but was having incomplete equipment and lacked funds to purchase the
necessary equipment to make his business operational. Thus, Magno representing
Ultra Sources International Corporation approached one Corazon Teng Vice president
of Mancor Industries to get his needed car repair service equipment of which Mancor
was a distributor. He also discloses his insufficiency of funds to buy the equipment he
needs. Teng after learning Magno’s predicament immediately referred him to LS
Finance and Management Corporation advising its vice president Joey Gomez that
Mancor was willing and able to supply the pieces of equipment needed if LS Finance
could accommodate Magno and provide him credit facilities.

Arrangement went through on condition that Magno will put up a warranty deposit
equivalent to thirty per centum (30%) of the total value of the pieces of equipment to be
purchased, amounting to P29,790.00.

Magno could not come up with such an amount, so he asked Joey Gomez to look for a
third party who could lend him the warranty deposit amount. Unknown to him
(Magno) it was Teng who advanced the deposit in question. But will have to be paid as
short- term loan at 3% interest.

Part of the arrangement, there is this leasing agreement which Magno and LS Finance
entered, whereby LS Finance would lease the garage equipment and Magno will pay
the corresponding rent with the option to buy the same. Documentation is done and
the equipment were delivered to Magno who in turn issued 2 sets of six post dated
checks on two different occasions.

The first set of which was given to Joey Gomez who delivered to Teng without Magno’s
knowledge. When this first set of 6 checks matured he requested Joey Gomez to not to
deposit them as Magno was no longer banking with Pacific Bank. To replace them he
issued the second set, two of them were deposited and cleared while four others were
being held by Teng per Magnos request as they were not covered with sufficient funds.
On a different note, LS Finance pulled out the garage equipment as Magno could not
pay anymore. Then later on became aware that Teng was the one who paid the warranty
deposit. Magno with his wife came to see Teng and promised her to pay the latter but
payment never came so Teng deposited the check but returned for the reason of closed
account.

VIVAR, 1
CRIMINAL LAW DIGESTS

Because of this Magno gets convicted before the regional trial court of Quezon city
branch 104 for four counts of offense violating BP 22 which was affirmed by the court
of appeals.

Hence, this appeal

Issue:

Whether or not Magno should be punished for violation of Bouncing Checks Law.

Ruling:

No
No. Thus, it behooves behooves upon a court of law that in applying applying the
punishment imposed upon the accused, the objective of retribution of a wronged society,
should be directed against the "actual and potential wrongdoers." In the instant case,
there is no doubt that petitioner's four (4) checks were used to collateralize an
accommodation, and not to cover the receipt of an actual "account or credit for value"
as this was absent, and therefore petitioner should not be punished for mere issuance
of the checks in question. Following the aforecited theory, in petitioner's stead the
petitioner's stead the "potential wrongdoer," whos "potential wrongdoer," whose
operation could be e operation could be a menace to society, should not be glorified to
society, should not be glorified by convicting t by convicting the petitioner

Fallo:

WHEREFORE, the appealed decision is REVERSED and the accused-petitioner is


hereby ACQUITTED of the crime charged.

VIVAR, 1
CRIMINAL LAW DIGESTS

Construction of Penal Laws <<<

Corpuz v. People
G.R. No. 180016, April 29, 2014
PERALTA, J:

Doctrine:
The framers of the RPC anticipated this matter by including Art. 5, which is the duty of
the court in connection with acts, which should be repressed, but which are not
covered by the law, and in cases of excessive penalties.

Facts:

Danilo Tangcoy, private complainant, and Lito Corpuz met at the Admiral Royale
Casino in Olongapo City sometime in 1990. Tangcoy was then engaged in the business
of lending money to casino players and, upon hearing that Tangcoy had some pieces of
jewelry for sale, Corpuz approached him on May 2, 1991 at the same casino and
offered to sell the said pieces of jewelry on commission basis. Tangcoy agreed, and as a
consequence, he turned over to petitioner the following items: an 18k diamond ring
for men; a woman's bracelet; one (1) men's necklace and another men's bracelet, with
an aggregate value of P98,000.00. They both agreed that petitioner shall remit the
proceeds of the sale, and/or, if unsold, to return the same items, within a period of 60
days. The period expired without petitioner remitting the proceeds of the sale or
returning the pieces of jewelry. When Tongcoy was able to meet petitioner, the latter
promised the former that he will pay the value of the said items entrusted to him, but
to no avail.

A criminal complaint for estafa was filed against Corpuz. On the prosecution, it was
established that Tongcoy and Corpuz were collecting agents of Antonio Balajadia.
Petitioner denied having transacted any business with Tongcoy but admitted obtaining
a loan from Balajadia for which he was made to sign a blank receipt. He claimed that
the same receipt was used as evidence against him for the supposed agreement to sell
the subject pieces of jewelry, which he did not even see. RTC found petitioner guilty
beyond reasonable doubt. CA affirmed with modification the indeterminate penalty of 4
years 2 months of prison correccional, as minimum, to 8 years of prison mayor, as
maximum, plus 1 year for each additional 10,000 pesos or a total of 7 years.

Issue:
Whether or not there is a perceived injustice brought about by the range of penalties
(excessive fines) that the courts continue to that the courts continue to impose against
property committed today epecially in estafa.

Ruling:

Legislature pegged these penalties to the value of money and property in 1932 when
the RPC was enacted. There seems to be a perceived injustice brought about by the
range of penalties that the court continues to impose on crime against property
committed today, based on the amount of damage measured by the value of money 80
years ago in 1932. The court however cannot modify the range of penalties because
that would constitute juridical legislation. As the Constitution vests the power to enact
laws on the legislature, the courts cannot arrogate the power to enlarge the scope of
the crime, introduce matters that the legislature clearly did not intend, redefine a
crime in a manner that does not hew to the statutory language, or modify the penalty
to conform to the courts' notion (out of the innumerable number of notions) of justice
and fairness.

However, this does not render the whole situation without remedy. The framers of the
RPC anticipated this matter by including Art. 5, which is the duty of the court in
connection with acts, which should be repressed, but which are not covered by the
law, and in cases of excessive penalties.

VIVAR, 1
CRIMINAL LAW DIGESTS

Fallo:

WHEREFORE, the Petition for Review on Certiorari dated November 5, 2007 of petitioner
Lito Corpuz is hereby DENIED. Consequently, the Decision dated March 22, 2007 and
Resolution dated September 5, 2007 of the Court of Appeals, which affirmed with
modification the Decision dated July 30, 2004 of the Regional Trial Court, Branch 46,
San Fernando City, finding petitioner guilty beyond reasonable doubt of the crime of
Estafa under Article 315, paragraph (1), sub-paragraph (b) of the Revised Penal Code,
are hereby AFFIRMED with MODIFICATION that the penalty imposed is the
indeterminate penalty of imprisonment ranging from THREE (3) YEARS, TWO (2)
MONTHS and ELEVEN DAYS of prison correccional, as minimum, to FIFTEEN (15)
YEARS ofreclusion temporal as maximum.

VIVAR, 1
CRIMINAL LAW DIGESTS

Territoriality <<<

Del Socorro v. Van Wilsem


G.R. No. 193707, December 10, 2014
PERALTA, J:

Doctrine:
Doctrine of Processual Presumption; Foreign Laws; If the foreign law involved is not
properly pleaded and proved, our courts will presume that the foreign law is the same
as our local or domestic or internal law.

Facts:
Petitioner Norma A. Del Socorro and respondent Ernst Johan Brinkman Val Wilsem
contracted marriage in Holland on September 25, 1990, and 4 years after their
marriage, they were blessed with a son named Roderigo Norjo Van Wilsem.
Unfortunately, their marriage bond ended on July 19,1995 by virtue of a Divorce
Decree issued by the appropriate Court of Holland. Thereafter, the petitioner and her
son came home to the Philippines.

According to Norma, Ernst made a promise to provide monthly support to their son.
However, since their arrival in the Philippines, the respondent never gave support to
their son. Not long thereafter, respondent came to the Philippines and remarried in
Pinamungahan, Cebu City, and resided there since then, which coincidentally it is
where the petitioner also resides.

On August 28, 2009, petitioner sent a letter demanding support from respondent, but
the latter refused to receive the letter. Because of the foregoing circumstances, petitioner
filed a complaint against the respondent for violation of Section 5, paragraph E(2) of
R.A. No. 9262 for the latter’s unjust refusal to support his minor child with the
petitioner.

Upon motion and after notice and hearing, the RTC-Cebu issued a Hold Departure
Order against the respondent, and was consequently arrested, but posted bail.
Petitioner also filed a Motion/Application of Permanent Protection Order but was
dismissed because the facts charged do not constitute an offense with respect to the
accused, who is an alien.

Thereafter, petitioner filed her Motion for Reconsideration thereto reiterating the
obligation to support their child under Article 195 of the Family Code, thus failure to
do so makes him liable under R.A. No. 9262 which “equally applies to all persons in the
Philippines who are obliged to support their minor children regardless of the obligor’s
nationality.” The motion for reconsideration was subsequently denied for lack of
merit.

Hence, the petitioner filed for a petition for review on certiorari.

Issue:
1. Whether or not a foreign national has an obligation to support his minor child
under Philippine law; and
2. Whether or not a foreign national can be held criminally liable under R.A.
No.9262 for his unjustified failure to support his minor child.SC HELD:*Note:
petition was allowed to be directly appealed to the SC without violating
hierarchy of courts since the petition involves purely questions of law.

Ruling:
Yes
In the present case, respondent hastily concluded that being a national of the
Netherlands, he is governed by such laws on the matter of provision and of capacity to
support. Since the petitioner invoked Article 195 of the Family Code in demanding
support from the respondent, who is a foreign citizen, the Court, however, agreed with
the respondent’s contention since Article 15 of the New Civil Code stresses the
principle of nationality. In international law, the party who wants to have a foreign law

VIVAR, 1
CRIMINAL LAW DIGESTS

applied to a dispute or case has the burden of proving the foreign law. While
respondent pleaded

VIVAR, 1
CRIMINAL LAW DIGESTS

the laws of the Netherlands in advancing his position that he is not obliged to support
his son, he never proved the same.

Thus, since the law of the Netherlands as regards the obligation to support has not
been properly pleaded and proved in the instant case, it is presumed to be the same
with Philippine law, which enforces the obligation of parents to support their children
and penalizes non-compliance therewith. Accordingly, the respondent, who is a
foreign national, has an obligation to support his minor child under Philippine law
because it would be of great injustice to the child to be denied financial support when
the latter is entitled thereto.

Furthermore, to determine whether or not a person is criminally liable under R.A.


9262, it is imperative that the legal obligation to support exists. In taking cognizance of
the case, the Supreme Court recognizes that it presents a novel question of law
concerning liability of a foreign national who allegedly commits acts and omissions
punishable under special criminal laws, specifically in relation to family rights and
duties. Despite Van Wilsem’s plea that the law of Netherlands does not oblige him to
support his son, the Court ruled that such obligation is still duly enforceable in the
Philippines, as justified in the provision of the third paragraph of the Article 17 of the
New Civil Code.

Fallo:

WHEREFORE, the petition is GRANTED. The Orders dated February 19, 2010 and
September 1, 2010, respectively, of the Regional Trial Court of the City of Cebu are
hereby REVERSED and SET ASIDE. The case is REMANDED to the same court to
conduct further proceedings based on the merits of the case.

VIVAR, 1
CRIMINAL LAW DIGESTS

Transitory Crime <<<

AAA vs. BBB


G.R. No. 212448, January 11,
2018 TIJAM, J:

Doctrine:
A person charged with a continuing or transitory crime may be validly tried in any
municipality or territory where the offense was in part committed. In the present
scenario, the offended wife and children of respondent husband are residents of Pasig
City since March 2010.

Facts:
AAA and BBB were married on August 1, 2006 in Quezon City. Their union produced 2
children. In May 2007, BBB started working in Singapore as a chef, where he acquired
permanent resident status in September 2008. This petition nonetheless indicates his
address to be in Quezon City where his parents reside and where AAA also resided
from the time they were married until March 2010, when AAA and their children
moved back to her parents’ house in Pasig City.

AAA claimed, albeit not reflected in the Information, that BBB sent little to no financial
support, and only sporadically. This allegedly compelled her to fly extra hours and take
on additional jobs to augment her income as a flight attendant. There were also
allegations of virtual abandonment, mistreatment of her and their CCC, and physical
and sexual violence. To make matters worse, BBB supposedly started having an affair
with a Singaporean woman named Lisel Mok with whom he allegedly has been living
in Singapore. Things came to a head on April 19, 2011 when AAA and BBB had a
violent altercation at a hotel room in Singapore during her visit with their kids. As can
be gathered from earlier cited Information, despite the claims of varied forms of
abuses, the investigating prosecutor found sufficient basis to charge BBB with causing
AAA mental and emotional anguish through his alleged marital infidelity.

A warrant of arrest and hold departure order were issued but BBB continued to evade
arrest. Consequently, the case was archived. However, on November 6, 2013, an Entry
of Appearance as Counsel for the Accused With Omnibus Motion to Revive Case, Quash
Information, Lift Hold Departure Order and Warrant of Arrest was filed on behalf of BBB.
The motion to quash was granted on ground of lack of jurisdiction (acts complained of
had occurred in Singapore).

AAA’s motion for reconsideration was denied so she sought direct recourse to the
Supreme Court via petition for review under Rule 45 on pure question of law. In the
main, AAA argues that mental and emotional anguish is an essential element of the
offense charged against BBB, which is experienced by her wherever she goes, and not
only in Singapore where the extra-marital affair takes place; thus, the RTC of Pasig City
where she resides can take cognizance of the case. In support of her theory, AAA
specifically cites Section 7 on Venue of R.A. 9262 and Section 4 on liberal construction
of the law to promote the protection and safety of victims of violence against women and
their children.

In his Comment, BBB contends that the grant of the motion to quash is in effect an
acquittal; that only the civil aspect of a criminal case may be appealed by the private
offended party, and that the petition should be dismissed for having been brought before
the Court by AAA instead of the Office of the Solicitor General (OSG) as counsel for the
People in appellate proceedings. BBB also asserts that the petition is belatedly filed.

Issue:
Whether or not Philippine courts have jurisdiction over complaint for psychological
abuse under Republic Act No. 9262 otherwise known as the Anti-Violence Against
Women and their Children Act of 2004 when committed through marital infidelity
outside the country.

VIVAR, 1
CRIMINAL LAW DIGESTS

Ruling:
Yes
“Physical violence is only the most visible form of abuse. Psychological abuse,
particularly forced social and economic isolation of women, is also common.” Section 3
of R.A.No. 9262 encompass in a non-limiting manner the various forms of violence that
may be committed against women and their children.

What R.A. 9262 criminalizes is not the marital infidelity per se but the psychological
violence causing mental or emotional suffering on the wife. Otherwise stated, it is the
violence inflicted under the said circumstances that the law seeks to outlaw.

In Section 7 of R.A. No. 9262, venue undoubtedly pertains to jurisdiction. It provides


that the case may be filed where the crime or any of its elements was committed at the
option of the complainant. While the psychological violence as the means employed by
the perpetrator is certainly an indispensable element of the offense, equally essential
also is the element of mental or emotional anguish which is personal to the
complainant.

What may be gleaned from Section 7 of R.A. No. 9262 is that the law contemplates that
acts of violence against women and their children may manifest as transitory or
continuing crimes, meaning that some acts material and essential thereto and
requisite in their consummation occur in one municipality or territory, while some
occur in another.

Thus, a person charged with a continuing or transitory crime may be validly tried in
any municipality or territory where the offense was in part committed. In the present
scenario, the offended wife and children of respondent husband are residents of Pasig
City since March 2010.

Hence, the RTC of Pasig City may exercise jurisdiction over the case. We say that even
if the alleged extra marital affair causing the offended wife mental and emotional
anguish is committed abroad, the same does not place a prosecution under R.A.
No.9262 absolutely beyond the reach of Philippine courts.

Fallo:

IN VIEW OF THE FOREGOING, the petition is GRANTED. The Resolutions dated


February 24, 2014 and May 2, 2014 of the Regional Trial Court of Pasig City, Branch
158, in Criminal Case No. 146468 are SET ASIDE. Accordingly, the Information filed in
Criminal Case No. 146468 is ordered REINSTATED.

VIVAR, 1
CRIMINAL LAW DIGESTS

Intent <<<

Jacaban v. People
G.R. No. 184355, March 23, 2015
PERALTA, J:

Doctrine:
Animus posidendi or the intent to possess. This is a state of mind, dependent on
attendant events in the case. It may be inferred from prior or contemporaneous acts of
the accused, or the surrounding circumstances.

Facts:
According to the prosecution:
 On July 15, 1999, P/SInsp. Ipil Dueñ as of the Presidential Anti-Organized Crime
Task Force filed an application for Search Warrant before Branch 22 of the Cebu
RTC, to search the residence at J. Labra St., Guadalupe. The warrant was issued.
 Around July 16, 1999, 12:45am in Cebu City, a raid was done on the premises,
where illegal firearms and ammunition (slightly different from the items listed
in the warrant).
o According to the raiding officers, the search warrant was served to the
petitioner, who was inside the residence with his wife and other ladies.
Upon receipt of the warrant, the officers allege the petitioner became angry
and denied having committed anything illegal, but was in turn assured he
need not worry if the raiding officers do not find anything.
o During the search, one of the officers found a 45 caliber pistol in the
ceiling. The petitioner allegedly rushed from the living room to the
bedroom and grappled for the possession of the gun.
o When the search concluded, the seized items were inventoried and taken
back to the team’s office along with the arrested petitioner.
o Later (2002) a certification held that the petitioner is not licensed to
possess any of the firearms or ammunition seized.

According to the defense:


 The defense presented the petitioner’s wife (Felipenerie Jacaban) as a witness.
o She alleged that the policemen conducted a raid in the house of Gabriel
Arda, the petitioner’s unlce.
o She alleged that she fetched the appellant from his house, since the
officers were looking for him.
o She claims that the recovered pistol was pledged to her father by a
policeman
o She also claims that during the whole of the raid, her husband the
petitioner merely observed and did not protest.

Both the RTC and CA found the petitioner guilty of the crime charged.

Issue:
Whether or not the petitioner is guilty of the crime charged?

Ruling:
Yes

 There are only two essential elements for illegal possession of firearms and
ammunition:
o The existence of subject firearm
o The fact that the accused who possessed or owned the same does not have
corresponding license for it.
 Ownership is not an essential element, so the petitioner’s defense that he owns
neither the gun nor the residence holds no water.
o There are two types of possession:
 Actual physical possession

VIVAR, 1
CRIMINAL LAW DIGESTS

 Constructive possession, or the subjection of the thing to one’s


control and management. The prosecution proved the petitioner
had constructive possession of the contraband.
 There is also the concept of animus posidendi or the intent to possess.
o This is a state of mind, dependent on attendant events in the case. It may
be inferred from prior or contemporaneous acts of the accused, or the
surrounding circumstances.
 In this case, the act of rushing to the room and trying to wrest control
of the gun points to the petitioner’s animus posidendi.
 The argument that the petitioner is not a resident of the home where the
contraband was found is not persuasive, since according to the testimony of the
defense, he did not protest his arrest. Neither did he contact the alleged owner
of the house. Also, he and his wife were present in the residence at an unholy
hour.
o The alleged owner of the house, Gabrie Arda, did not testify due to health
issues. However, the defense testified that he was not present in the
house, undermining his ownership of the said residence.
 The Court also dismissed the error in the testimony of the prosecution witness
placing the time of the raid at 12:45PM instead of AM. The Court held the
discrepancy did not affect the material aspects of the case, and it was not
actuated by improper motives.
 Discrepancy is alright as long as it does not affect the material aspects of the case.

Fallo:

WHEREFORE, in view of all the foregoing, the Decision of the Court of Appeals, dated
July 30, 2008, is AFFIRMED WITH MODIFICATION. Petitioner is sentenced to suffer the
indeterminate penalty of imprisonment ranging from SIX (6) YEARS of prision
correccional in its maximum period, as minimum, to SIX (6) YEARS, EIGHT (8)
MONTHS and ONE (1) DAY of prision mayor minimum in its medium period, as
maximum, and to pay a fine of P30,000.00.

VIVAR, 2
CRIMINAL LAW DIGESTS

Complex Crime/ Ex Post Facto <<<

People v. Hijada
G.R. No. 123696, March 11, 2004
AZCUNA, J:

Doctrine:
The crime of Robbery with Homicide is a special complex crime punishable under
Article 294 of the Revised Penal Code with reclusion perpetua to death. However, at
the time the crime was committed, on September 14, 1992, the death penalty could
not be imposed in view of Article III, Section 19(1) of the Constitution.

Facts:
 SEPTEMBER 14, 1992 the named accused went to the residence of Filonila
Tupaz and on the occasion of said robbery the accused did and there willfully,
unlawfully and feloniously stab Filonila Tupaz, Filomena Garcia. And Rosemarie
Diaz. Inflicting upon them serious and grave wounds which were the direct and
immediate cause of their death.
 On September 19, 1992. SPO1 Rolando Aguilar testified as the police investigator
who took the statement of Dante confessing to having robbed the house of Filonila
Tupaz, and implicated Ricky and Rodel as having taken part in the robbery and
pointed to Ricky as the person who killed the three victims
 On September 17, 1992 SPO1 Rolando Aguilar handed a cartographic sketch of
one of the suspects to SPO4 Juan Aguilar who testified as the head of the
investigation team. The police were able to obtain the address of Ricky in
Marikina. They proceeded to that address where they met Jeffrey Ambrosio who
told them that he knew something about the crime. He also gave the addresses
of Dante and Rodel whom he identified as the participants in the robbery.
 The police recovered from Dante and Rodel a necklace and a travelling bag. Dante
and Rodel voluntarily disclosed that they were on their way to 8th street in New
Manila to meet Ricky. The police immediately proceeded to the aforesaid location,
spotted Ricky walking along 8th street and placed him under arrest. Ricky was
there after brought to his residence in Tondo where he surrendered a bladed
weapon stained with blood.
 Mrs. Trinidad Albaracan testified that Ricky was once hired by Filonila Tupaz to
fix the rood of the latter house.
 Alvin Monares the witness who gave a physical description of Ricky, declared that
in the morning of September 14, 1992 at around 8:30 he saw all three appellants
outside the residence of FIlonila Tupaz, with Ricky standing in front of the house
while Dante and Rodel sat beside a gutter.
 Dr. Emmanuel Aranas reported that one knife was found to be stained with
human blood while the other resulted in negative findings.
 Jeffrey Ambrosio testified that on September 12, 1992 he was in the company of
the appellants mixing cement from 8:00am to 5:00 pm. It was in the morning of
September 15, 1992 that he saw the appellants again and stated that he saw
Ricky wearing a white shirt stained with blood.

Facts of the evidence for the defense:


 Victor Hijada testified that on September 18, 1992 at about 5:00 a.m, he was
awakened by his wife who told him that there were armed men inside their
house looking for Ricky.
 Apellant Dante testified that he was in Laguna on September 18 1992. He
denied knowing Ricky prior to his arrest on September 18 1992 and further
claimed that he was never arrested by counsel all throughout his investigation.
 Apellant Rodel testimony substantially corroborated the testimony of Dante.
 Apellant Ricky testifies that he was in Bicol from September 1 1992. He stated
that he left for Manila on September 19 1992 because his child was sick and
needed to be brought to the hospital.
 Teresita Pena that ricky was in Bagamanoc, Catanduanes from September 1
1992.

VIVAR, 2
CRIMINAL LAW DIGESTS

Issue:
1. WON the appellants be held criminally liable for committing robbery with
homicide?
2. WON the supreme court fail to consider the evidences of the appellants after
imposing death penalty

Ruling:
1. Yes. The Regional Trial Court , speaking through Justice Azcona, declared that
the crime of Robbery with Homicide is a special complex crime punishable
under Article 294 of the Revised Penal Code with Reclusion Perpetua to death.
2. No. The trial court, speaking through Justice Azcona, erred in imposing upon
them the supreme death penalty. The crime of Robbery with Homicide is a
special complex crime punishable under Article 294 of the Revised Penal Code
with Reclusion Perpetua to death. However, at the time the crime was
committed, on September 14, 1992, the death penalty could not be imposed in
view of Article III, Section 19(1) of the Constitution. Subsequently, the penalty
of Reclusion Perpetua to death for Robbery with Homicide was again imposed
in 1993 with the enactment of Republic Act No. 7695.[35] The provisions of
Republic Act No. 7695, however, cannot be applied retroactively, for that would
violate Article III, Sec. 22 of the Constitution stating that no ex post facto law
shall be enacted, as well as Article 21 of the Revised Penal Code.[36]
Consequently, the single indivisible penalty of reclusion perpetua should be
imposed on each of appellants. Based on these reasons, the Regional Trial
Court found the appellants guilty beyond reasonable doubt of the special
complex crime of Robbery with Homicide and are sentenced to suffer the
penalty of Reclusion Perpetua

Fallo:

WHEREFORE, in view of the foregoing, the decision of the Regional Trial Court of
Quezon City, Branch 84, in Criminal Case No. Q-92-35539, is hereby MODIFIED.
Appellants Ricky Hijada y Villanueva, Danilo Alcera y Alfon and Rodelio Villamor y
Rabanes are found guilty beyond reasonable doubt of the special complex crime of
Robbery with Homicide and are sentenced to suffer the penalty of reclusion perpetua.
Appellants are each further ordered to pay jointly and severally the heirs of Filonila M.
Tupaz, Filomena P. Garcia and Rosemarie C. Diaz the amounts of P50,000 as civil
indemnity and P50,000 as moral damages. No costs.

VIVAR, 2
CRIMINAL LAW DIGESTS

Mala In Se <<<

Estrada vs. Sandiganbayan


GR No. 148560, November 19, 2011
BELLOSILLO, J:

Doctrine:
The legislative declaration in R.A. No. 7659 that plunder is a heinous offense implies
that it is a malum in se. For when the acts punished are inherently immoral or
inherently wrong, they are mala in se and it does not matter that such acts are
punished in a special law, especially since in the case of plunder the predicate crimes
are mainly mala in se

Facts:
On April 25, 2001, the Sandiganbayan issued a resolution in Criminal Case No. 26558,
finding probable cause that petitioner Joseph Ejercito Estrada, then the President of
the Philippines has committed the offense of plunder, and that he be prosecuted under
RA 7080 (An Act Defining and Penalizing the Crime of Plunder).

The petitioner wishes to impress upon the Court that the assailed law is so defectively
fashioned that it crosses that thin but distinct line which divides the valid from the
constitutionally infirm. His contentions are mainly based on the effects of the said law
that it suffers from the (1) vice of vagueness; (2) it dispenses with the "reasonable doubt"
standard in criminal prosecutions; and (3) it abolishes the element of mens rea in
crimes already punishable under The Revised Penal Code saying that it violates the
fundamental rights of the accused.

The focal point of the case is the alleged “vagueness” of the law in the terms it uses.
Particularly, this terms are: combination, series and unwarranted. Because of this, the
petitioner uses the facial challenge on the validity of the mentioned law.

Issue:
Whether or not Plunder as defined in RA 7080 is a malum prohibitum, and if so,
whether it is within the power of Congress to so within the power of Congress to so
classify it.

Ruling:
No.
Any doubt as to whether the crime of plunder is malum in se must be deemed to have
been resolved in the affirmative by the decision of Congress in 1993 to include it
among the heinous crimes punishable by reclusion perpetua to death. The legislative
declaration in R.A. No. 7659 that plunder is a heinous offense implies that it is a
malum in se. For when the acts punished are inherently immoral or inherently wrong,
they are mala in se and it does not matter that such acts are punished in a special law,
especially since in the case of plunder the predicate crimes are mainly mala in se

Fallo:

PREMISES CONSIDERED, this Court holds that RA 7080 otherwise known as the
Plunder Law, as amended by RA 7659, is CONSTITUTIONAL. Consequently, the
petition to declare the law unconstitutional is DISMISSED for lack of merit.

VIVAR, 2
CRIMINAL LAW DIGESTS

Diplomatic Immunity <<<

Minucher vs. Scalzo


G.R. no. 142396 Feb. 11,
2003 VITUG, J:

Doctrine:
Under the Vienna Convention on Diplomatic Relations, a diplomatic agent, assuming
petitioner is such, enjoys immunity from criminal jurisdiction of the receiving state
except in the case of an action relating to any professional or commercial activity
exercised by the diplomatic agent in the receiving state outside his official functions.
As already mentioned above, the commission of a crime is not part of official duty.

Facts:
Petitioner Khosrow Minucher, an Iranian national, was charged for violation of Section
4 of Republic Act No. 6425, otherwise known as the "Dangerous Drugs Act of 1972."
The narcotic agents who raided the house of Minucher were accompanied by private
respondent Arthur Scalzo. Minucher was acquitted by the trial court of the charges.
Minucher filed a civil case before the Regional Trial Court of Manila for damages on
account of what he claimed to have been trumped-up charges of drug trafficking made
by Arthur Scalzo. Scalzo filed a motion to dismiss the complaint on the ground that,
being a special agent of the United States Drug Enforcement Administration, he was
entitled to diplomatic immunity. The trial court denied the motion to dismiss. Scalzo
filed a petition for certiorari with injunction with the Court, asking that the complaint
be ordered dismissed. The case was referred to the Court of Appeals. The appellate
court promulgated its decision sustaining the diplomatic immunity of Scalzo and
ordering the dismissal of the complaint against him. Minucher filed a petition for
review with the Court, appealing the judgment of the Court of Appeals. The Supreme
Court reversed the decision of the appellate court and remanded the case to the lower
court. The Manila RTC continued with its hearings on the case. After trial, the court
rendered a decision in favor of petitioner Khosrow Minucher and adjudged private
respondent Arthur Scalzo liable in actual and compensatory damages of P520,000.00;
moral damages in the sum of P10 million; exemplary damages in the sum of
P100,000.00; attorney's fees in the sum of P200,000.00 plus costs. On appeal, the
Court of Appeals reversed the decision of the trial court and sustained the defense of
Scalzo that he was sufficiently clothed with diplomatic immunity during his term of
duty and thereby immune from the criminal and civil jurisdiction of the "Receiving
State" pursuant to the terms of the Vienna Convention. Hence, the present petition for
review.

The Supreme Court denied the petition. According to the Court, a foreign agent,
operating within a territory, can be cloaked with immunity from suit but only as long
as it can be established that he is acting within the directives of the sending state. The
consent of the host state is an indispensable requirement of basic courtesy between
the two sovereigns. The official exchanges of communication between agencies of the
government of the two countries, certifications from officials of both the Philippine
Department of Foreign Affairs and the United States Embassy, as well as the
participation of members of the Philippine Narcotics Command in the "buy-bust
operation" conducted at the residence of Minucher at the behest of Scalzo, may be
inadequate to support the "diplomatic status" of the latter, but they give enough
indication that the Philippine government has given its imprimatur, if not consent, to
the activities within Philippine territory of agent Scalzo of the United States Drug
Enforcement Agency. The job description of Scalzo has tasked him to conduct
surveillance on suspected drug suppliers and, after having ascertained the target, to
inform local law enforcers who would then be expected to make the arrest. In conducting
surveillance activities on Minucher, later acting as the poseur-buyer during the buy-
bust operation, and then becoming a principal witness in the criminal case against
Minucher, Scalzo hardly can be said to have acted beyond the scope of his official
function or duties.

Issue:

VIVAR, 2
CRIMINAL LAW DIGESTS

Whether or not Arthur Scalzo Whether or not Arthur Scalzo is indeed entitled to is
indeed entitled to diplomatic immunity diplomatic immunity.

Ruling:
Yes.
A foreign agent, operating within a territory A foreign agent, operating within a territory,
can be cloaked with immunity from suit as long be cloaked with immunity from suit as
long as it can as it can be established that he is acting within the dire be established
that he is acting within the directives of the sending state. ives of the sending state. The
consent or imprimatur of the Philippine government to the activities of the United
States Drug Enforcement Agency Enforcement Agency, however, can be gleaned ,
however, can be gleaned from the undisputed facts from the undisputed facts in the
ca in the case.

 The official exchanges of communication between agencies of the government of


the two countries.
 Certifications from officials of both the Philippine Department of Foreign Affairs
and the United States Embassy.
 Participation of members of the Philippine Narcotics Command in the “buy-bust
operation” conducted at the residence of Minucher at the behest of Scalzo.

These may be inadequate to support the “diplomatic status” of the latter but they give
enough indication that the Philippine government has given its imprimatur, if not
consent, to the activities within Philippine territory of within Philippine territory of agent
Scalzo of agent Scalzo of the United States Drug Enforcement the United States Drug
Enforcement Agency. Agency The job description of Scalzo has tasked him to conduct
surveillance on suspected drug suppliers and, after having ascertained the target, to
inform local law enforcers who would then be expected to make the arrest.

In conducting surveillance activities on Minucher, later acting as the poseur-buyer


during the buy- bust operation, and then becoming a principal witness in the criminal
case against Minucher ess in the criminal case against Minucher,

Scalzo hardly can be said to have acted beyond the scope of his official function or duties.

Fallo:

WHEREFORE, on the foregoing premises, the petition is DENIED. No costs.

VIVAR, 2
CRIMINAL LAW DIGESTS

Diplomatic Immunity <<<

Liang vs. People


G.R. No. 125865 March 26, 2011
YNARES-SANTIAGO, J:

Doctrine:
Under the Vienna Convention on Diplomatic Relations, a diplomatic agent, assuming
petitioner is such, enjoys immunity from criminal jurisdiction of the receiving state
except in the case of an action relating to any professional or commercial activity
exercised by the diplomatic agent in the receiving state outside his official functions.
As already mentioned above, the commission of a crime is not part of official duty.

Facts:
For allegedly uttering defamatory words against a fellow Asian Development Bank (ADB)
worker, petitioner, an economist at ADB, was charged before the Metropolitan Trial
Court (MeTC) with two counts of grave oral defamation. The MeTC judge received an
"office protocol" from the Department of Foreign Affairs (DFA) stating that petitioner is
covered by immunity from legal process under Section 45 of the Agreement between the
ADB and the Philippine Government regarding the Headquarters of the ADB in the
country. Based on the said communication, the MeTC judge, without notice to the
prosecution, dismissed the two criminal cases. The motion for reconsideration filed by
the respondent was denied by the MeTC, hence, a petition for certiorari was filed before
the Regional Trial Court (RTC). The RTC set aside the MeTC ruling and ordered the
enforcement of the warrant of arrest earlier issued. Thus, petitioner elevated the case to
the Supreme Court after his motion for reconsideration was denied. He argued that he
is covered by immunity and that no preliminary investigation was held.

The Supreme Court denied the petition. According to the Supreme Court, slandering a
person could not possibly be covered by the immunity agreement because our laws do
not allow the commission of a crime, such as defamation; the mere invocation of the
immunity clause does not ipso facto result in the dropping of the charges; and that
preliminary investigation is not a matter of right in cases cognizable by the MeTC such
as the one at bar.

Issue:
Whether or not the petitioner is covered by immunity under the Agreement between
the ADB and the Philippine Government.

Ruling:
No.
Slandering a person could not possibly be covered by the immunity agreement because
our laws do not allow the commission of a crime, such as defamation, in the name of
official duty. The imputation of theft is ultra vires and cannot be part of official
functions. It is well-settled principle of law that a public official may be liable in his
personal private capacity for whatever damage he may have caused by his act done with
malice or in bad faith or beyond the scope of his authority or jurisdiction. It
appears that even the government's chief legal counsel, the Solicitor General, does not
support the stand taken by petitioner and that of the DFA. Under the Vienna
Convention on Diplomatic Relations, a diplomatic agent, assuming petitioner is such,
enjoys immunity from criminal jurisdiction of the receiving state except in the case of
an action relating to any professional or commercial activity exercised by the
diplomatic agent in the receiving state outside his official functions. As already
mentioned above, the commission of a crime is not part of official duty.

Fallo:

WHEREFORE, the petition is DENIED.

VIVAR, 2
CRIMINAL LAW DIGESTS

Prospectivity <<<

People vs. Lacson


GR 149453, April 1, 2003
CALLEJO, SR., J:

Doctrine:
The Court is not mandated to apply Section 8 retroactively simply because it is
favorable to the accused. It must be noted that the new rule was approved by the Court
not only to reinforce the constitutional right of the accused to a speedy disposition of
the case. The time-bar under the new rule was fixed by the Court to excise the malaise
that plagued the administration of the criminal justice system for the benefit of the
State and the accused; not for the accused only.

Facts:
1. THE CHARGE
a. The Ombudsman filed before the Sandiganbayan eleven informations for
murder against respondent Panfilo M. Lacson and 25 other accused. They
were all charged as principals in the alleged "rub-out" of eleven members
of the Kuratong Baleleng Gang. Upon motion of the respondent, the
criminal cases were remanded to the Ombudsman for reinvestigation.
2. REINVESTIGATION - DOWNGRADE FOR LACSON
a. After reinvestigation, the case against Lacson was downgraded from
principal to accessory. With the downgrading of charges against him,
Lacson questioned the jurisdiction of the Sandiganbayan to hear the
cases as none of the accused was a government official with Salary Grade
27 or higher. Accordingly, the Sandiganbayan ordered the cases
transferred to the Regional Trial Court.
3. WITNESSES WITHDRAW (ERAP PROTECTION)
a. However, before the accused could be arraigned, prosecution witnesses
recanted their affidavits implicating respondent Lacson in the murder
cases.
b. Meanwhile, the other private complainants executed their respective
affidavits of desistance. Because of these developments, the cases against
the 26 accused were dismissed on March 29, 1999.
4. NEW WITNESSES APPEAR (ERAP IS OUSTED)
a. In 2001, the same criminal cases were again ordered to be investigated on
the strength of affidavits of new witnesses. Thereafter, eleven informations
for murder were filed before the Regional Trial Court of Quezon City. The
new informations charged as principals 34 people, including the 26
original accused in the previous criminal cases.
5. CA – CERTIORARI
a. Respondent Lacson filed a petition for certiorari before the Court of
Appeals. The Court Appeals issued a temporary restraining order enjoining
the issuance of a warrant of arrest and conducting hearing on the criminal
charges filed. Then the Court of Appeals issued a decision which made
permanent the temporary restraining order earlier issued and also
dismissed all the 11 criminal cases filed, stating that the filing of the 11
informations falls within the purview of the prescriptive period barring a
revival of the earlier cases filed against the respondent in accordance with
the amendments on the Rules of Criminal Procedure. Hence, this petition
for review on certiorari to reverse and set aside the above decision of the
Court of Appeals.
b. In sum, this Court is of the considered view that the subject dismissal of
[the] criminal cases was provisional in nature and that the cases
presently sought to be prosecuted by the respondents are mere revival or
re-opening of the dismissed cases. The present controversy, being one
involving; "provisional dismissal" and revival of criminal cases, falls
within the purview of the prescriptive period provided under Section 8,
Rule 117 of the 2000 Revised Rules of Criminal Procedure.

VIVAR, 2
CRIMINAL LAW DIGESTS

Issue:
Whether or not the time-bar in said rule be applied retroactively.

Ruling:
No.
Time bar should not be applied retroactively. Though procedural rules may be applied
retroactively, it should not be if to do so would work injustice or would involve
intricate problems of due process. Statues should be construed in light of the purposes
to be achieved and the evils to be remedied. This is because to do so would be
prejudicial to the State since given that the Judge dismissed the case on March 29,
1999 and the new rule tool effect on Dec 1, 2000, it would only in effect give them 1
year and three months to work instead of 2 years.

At that time, they had no knowledge of the said rule and therefore they should not be
penalized for that. The 2 year period fixed in the new rule is for the benefit of both the
State and the accused. It should not be emasculated and reduced by an inordinate
retroactive application of the time bar therein provided merely to benefit the accused.
To do so would cause an injustice of hardship to the state and adversely affect the
administration of justice. To require the State to give a valid justification as a condition
sine qua non to the revival of a case provisionally dismissed with the express consent
of the accused before the effective date of the new rule is to assume that the State is
obliged to comply with the time bar under the new rule before it took effect. This
would be a rank denial of justice. The State must be given a period of one year or two
years as the case may be from December 1, 2000 to revive the criminal case without
requiring the State to make a valid justification for not reviving the case before the
effective date of the new rule. Although in criminal cases, the accused is entitled to
justice and fairness, so is the State.

Fallo:
IN VIEW OF THE FOREGOING, the case at bar is remanded to the RTC-Quezon City,
Branch 81 so that the State prosecutors and the respondent Lacson can adduce
evidence and be heard on whether the requirements of Section 8, Rule 117 have been
complied with on the basis of the evidence of which the trial court should make a
ruling on whether the Informations in Criminal Cases Nos. 01-101102 to 01-101112
should be dismissed or not. Pending the ruling, the trial court is restrained from
issuing any warrant of arrest against the respondent Lacson. Melo and Carpio, JJ.,
take no part.

VIVAR, 2
CRIMINAL LAW DIGESTS

Prospectivity <<<

Go vs. Dimagiba
GR 151876 June 21,
2005 PANGANIBAN J:

Doctrine:
The rule on retroactivity states that criminal laws may be applied retroactively if
favorable to the accused. This principle, embodied in the Revised Penal Code, has been
expanded in certain instances to cover special laws.

Facts:
The case is a petition for review filed by Susan Go (Petitioner) assailing the orders of
Baguio RTC Branch 5.

In July 1999 Baguio City MTCC Branch 4 convicted Fernando Dimagiba (respondent) of
13 counts of violation of BP 22. Dimagiba appealed to the RTC, but the RTC later
denied his appeal and upheld the conviction. No further appeals were filed before the
CA, hence the RTC issued a Certificate of Finality of the Decision on February 1, 2001.

Following the issuance of the Certificate of Finality, the MTCC issued an arrest warrant
against Dimagiba for the service of his sentence. Dimagiba, filed a Motion for
Reconsideration of the MTCC order but was denied and later on arrested and imprisoned.

Respondent, in October 2001, filed a Petition for Writ of Habeas Corpus before the
Baguio City RTC. The RTC, after hearing the case, issued an order to release the
respondent from confinement and ordered to pay a fine of PhP 100,000 in lieu of
imprisonment.

The RTC invoked Vaca v. CA and SC-AC No. 12-2000 in justifying its decision, which
allegedly “required the imposition of a fine only instead of imprisonment for BP 22
Violations, if the accused was not a recidivist or a habitual delinquent.” The RTC held
that this should be applied in favor of Dimagiba.

In response, Go filed a Motion for Reconsideration of the RTC Orders, which was

denied. Hence, this Petition filed directly before the SC (on pure question of law).

Issue:
Whether or not SA-SC No. 12-2000 should be applied retroactively in this case. (NO)

Ruling:
No.
The Court notes that the Petition for a writ of habeas corpus relied mainly on the alleged
retroactivity of SC-AC No. 12-2000, which supposedly favored BP 22 offenders. On this
point, Dimagiba contended that his imprisonment was violative of his right to equal
protection of the laws, since only a fine would be imposed on others similarly situated.

The rule on retroactivity states that criminal laws may be applied retroactively if
favorable to the accused. This principle, embodied in the Revised Penal Code, has been
expanded in certain instances to cover special laws.

The issue of retroactivity of SC-AC No. 12-2000 was settled in De Joya v. Jail Warden
of Batangas City - “First. SC Admin. Circular No. 12-2000 is not a penal law; hence,
Article 22 of the Revised Penal Code is not applicable. The circular applies only to
those cases pending as of the date of its effectivity and not to cases already terminated
by final judgment.”

Because the Circular merely lays down a rule of preference, it serves only as a
guideline for the trial courts. Thus, it is addressed to the judges, who are directed to
consider the factual circumstances of each case prior to imposing the appropriate
penalty. In other

VIVAR, 2
CRIMINAL LAW DIGESTS

words, the Administrative Circular does not confer any new right in favor of the accused,
much less those convicted by final judgment.

The doctrine of equal protection of laws does not apply for the same reasons as those
on retroactivity. Foremost of these reasons is that the Circular is not a law that deletes
the penalty of imprisonment. As explained earlier, it is merely a rule of preference as
to which penalty should be imposed under the peculiar circumstances of a case.

Fallo:
Dispositive Portion: WHEREFORE, the Petition is GRANTED and the assailed Orders
NULLIFIED. Respondent’s Petition for habeas corpus is hereby DENIED. Let this case
be REMANDED to MTCC of Baguio City for the re-arrest of respondent and the
completion of his sentence. No pronouncement as to costs. SO ORDERED.

VIVAR, 3
CRIMINAL LAW DIGESTS

Territoriality <<<

US vs. Bull
G.R. No. L-5270, 15 January
1910 ELLIOTT, J:

Doctrine:
When a foreign merchant ship enters territorial waters, the ship's officers and crew
are subject to the jurisdiction of the territorial courts, subject to such limitations only
as have been conceded by the territorial sovereign through the proper political
agencies.

Facts:
Accused H. N. Bull, master of vessel, willfully, unlawfully, and wrongly carry, transport,
and bring into the port and city of Manila, aboard said vessel, from the port of
Ampieng, Formosa, 677 head of cattle and carabaos, without providing suitable means
for securing the animals while in transit, so as to avoid cruelty and unnecessary
suffering. Animals to be tied by means of rings passed through their noses, and allow
and permit others to be transported loose in the hold and on the deck of said vessel
without being tied or secured in stalls, and all without bedding.

Neglect and failure of the accused to provide suitable means for securing said animals
while so in transit, the noses of some of said animals were cruelly torn, and many of
said animals were tossed about upon the decks and hold of said vessel, and cruelly
wounded, bruised, and killed.

All contrary to the provisions of Acts No. 55 and No. 275 of the Philippine Commission.
Section 1 of Act No. 55, which went into effect January 1, 1901, provides that — The
owners or masters of steam, sailing, or other vessels, carrying or transporting cattle,
sheep, swine, or other animals, from one port in the Philippine Islands to another, or
from any foreign port to any port within the Philippine Islands, shall carry with them,
upon the vessels carrying such animals, sufficient forage and fresh water to provide for
the suitable sustenance of such animals during the ordinary period occupied by the
vessel in passage from the port of shipment to the port of debarkation, and shall cause
such animals to be provided with adequate forage and fresh water at least once in
every twenty-four hours from the time that the animals are embarked to the time of
their final debarkation.

H.N. Bull, a Norwegian vessel, conceded that it was not registered or licensed in the
Philippine Islands under the laws thereof so it is not within the jurisdiction of the
Philippines.

Issue:
Whether or not the court had jurisdiction over an offense committed on board a
foreign ship while inside the territorial waters of the Philippines.

Ruling:
Yes.
No court of the Philippines has jurisdiction over any crimes committed in a foreign
ship on the high seas, but the moment it entered into territorial waters, it
automatically would be subject to the jurisdiction of the country. The offense,
assuming that it originated in Formosa, which the Philippines would have no
jurisdiction, continued until it reached Philippine territory which is already under
jurisdiction of the Philippines. Every state has complete control and jurisdiction over
its territorial waters. The Supreme Court of the United States has recently said that
merchant vessels of one country visiting the ports of another for the purpose of trade
would subject themselves to the laws which govern the ports they visit, so long as
they remain.

Defendant is thereby found guilty, and sentenced to pay a fine with subsidiary
imprisonment in case of insolvency, and to pay the costs.

VIVAR, 3
CRIMINAL LAW DIGESTS

Fallo:

VIVAR, 3
CRIMINAL LAW DIGESTS

Criminal Intent <<<

Roque v. People
G.R. No. 193169, April 06, 2015
DEL CASTILLO, J:

Doctrine:
Usually, the intent to kill is shown by the kind of weapon used by the offender and the
parts of the victim's body at which the weapon was aimed, as shown by the wounds
inflicted. Hence, when a deadly weapon, like a bolo, is used to stab the victim in the
latter's abdomen, the intent to kill can be presumed (Reyes, The Revised Penal Code,
13TH ED., P. 431)

Facts:

 Petitioner Rogelio Roque was charged with the crime of frustrated homicide.
 On/about November 22, 2001: P attacked, assaulted and shot complainant,
Reynaldo Marquez, hitting him on his right ear and nape and kicking him on the
face and back causing serious physical injuries.
 Prosecution version of incident: C (Reynaldo) and his brother (Rodolfo) were in
the house of Bella, and spotted De la Cruz with his wife, and shouted for him to
join them. At that instant, petitioner and his wife were passing-by on board a
tricycle. Believing that Rodolfo's shout was directed at him, petitioner stopped
the vehicle and cursed the former. C apologized for misunderstanding, but
petitioner warned them that something would happen to them if they continue
to make him anxious. C went to the barangay captain (Tayao) and asked for
assistance. C was fetched by De la Cruz and went to the house of Tayao. Tayao
was not around, and C proceeded to P’s house. C apologized but P did not
respond. He later came out of the house with a gun and started attacking P.
 Defense version of incident : P went to house of Bella and brothers who were
visibly intoxicated cursed him. P ignored the two and went home. Brothers
appeared in his front yard. P tried to pacify them but they threatened to kill him
would they return. Brothers came back challenging P to a gun duel. C fired his
gun and in an act of self-defense, P fired back twice.
 March 12, 2007: RTC found P guilty as charged (6 to 10 years of imprisonment)
 August 16, 2007: P filed a motion for reconsideration = denied
 P appealed to the CA, and CA affirmed in full RTC’S decision

Issue:

Whether or not there is an intent to kill?

Ruling:

Yes. Usually, the intent to kill is shown by the kind of weapon used by the offender and
the parts of the victim's body at which the weapon was aimed, as shown by the
wounds inflicted. Hence, when a deadly weapon, like a bolo, is used to stab the victim
in the latter's abdomen, the intent to kill can be presumed (Reyes, The Revised Penal
Code, 13TH ED., P. 431)

It is worth highlighting that the victim received two gunshot wounds in the head.
Indeed the location of the wounds plus the nature of the weapon used are ready
indications that the accused-appellant's objective is not merely to warn or incapacitate
a supposed aggressor. Verily, had the accused-appellant been slightly better with his
aim, any of the two bullets surely would have killed him outright. Also, the intent to kill
is further exhibited by the fact that the accused-appellant even prevented barangay
officials from intervening and helping x x x the bleeding victim. Indeed, the fact that
Reynaldo Marquez was miraculously able to live through the ordeal and sustain only
modicum injuries does not mean that the crime ought to be downgraded from
frustrated homicide to less serious physical injuries. After all, as was mentioned
above, what should be

VIVAR, 3
CRIMINAL LAW DIGESTS

determinative of the crime is not the gravity of the resulting injury but the criminal
intent that animated the hand that pulled the trigger.

Fallo:

WHEREFORE, the Petition is DENIED. The Decision dated February 27, 2009 of the
Court of Appeals in CA-G.R. CR No. 31084 affirming in its entirety the March 12, 2007
Decision of the Regional Trial Court of Malolos, Bulacan, Branch 84 in Criminal Case
No. 3486-M-2002 convicting petitioner Rogelio Roque of the crime of frustrated homicide,
is AFFIRMED with the MODIFICATION that the petitioner is ordered to pay the victim
Reynaldo Marquez moral damages and temperate damages in the amount of
P25,000.00 each, with interest at the legal rate of 6% per annum from the date of
finality of this Resolution until fully paid.

VIVAR, 3
CRIMINAL LAW DIGESTS

Criminal Intent <<<

De Guzman v. People
G.R. No. 178512, November 26, 2014
BERSAMIN, J:

Doctrine:
Usually, the intent to kill is shown by the kind of weapon used by the offender and the
parts of the victim's body at which the weapon was aimed, as shown by the wounds
inflicted. Hence, when a deadly weapon, like a bolo, is used to stab the victim in the
latter's abdomen, the intent to kill can be presumed (Reyes, The Revised Penal Code,
13TH ED., P. 431)

Facts:

 On the evening of Dec. 24, 1997, Alexander Fojo was fetching water below his
rented house in Mandaluyong City when he was suddenly hit on the nape by
Alfredo De Guzman, brother of Alexander’s land lady, Lucila Bautista.
 Alexander informed Lucila about what happened and she apologized on behalf
of Alejandro and told the latter to just go up.
 Later on, at around 12:00 to 12:15 pm, he went down and continued fetch
water. While pouring water, Alfredo suddenly appeared and stabbed him on left
face and chest.
 Cirilino Bantaya, son-in-law of Alexander, saw the victim bleeding and begging
for help. Alexander told Cirilino that Alfredo stabbed him.
 Cirilino brought and rushed him to the Mandaluyong City Medical Center.
Alexander two (2) stabbed wounds, one in zygoma (FACE), left part and one in
the upper left chest which was fatal. The attending physician, Francisco
Obmerga, stated that the second wound could have caused Alexander’s death if
he did not get rushed to the hospital.
 Alfredo appealed to the CA, contending that his intent to kill (CRITICAL
ELEMENT OF THE CRIME CHARGED) was not established, and that any person
could have inflicted the wounds.
 Petitioner also insisted that he should be guilty of slight physical injuries, not
frustrated homicide.

Issue:

Whether or not the INTENT TO KILL, which is a criminal element of the crime charged,
is established in the case.

Ruling:

Yes. The wounds sustained by Alexander were not mere scuff-marks inflicted in the
heat of anger or as the result of a fistfight between them. The petitioner wielded and
used a knife in his assault on Alexander. There is also to be no doubt about the wound
on Alexander’s chest being sufficient to result into his death were it not for the timely
medical intervention.

INTENT TO KILL is determined by the ff. factors:


1. Nature and number of the weapon used by the offender in the commission of
the crime – STABBED THE VICTIM USING A KNIFE.
2. Nature, number and location of wounds inflicted/sustained by the victim – 2
wounds: One in the ZYGOMA (left side), one in the upper left chest.
3. The manner of committing the crime – Alfredo stabbed him 2 times
4. Acts, deeds or words stated by the offender before, during or immediately after
the commission of the crime – Alejandro was hit on the nape by Alfredo.
5. Evidence of the motive (before or after but NOT during the commission of the
crime – direct evidence is not available)

VIVAR, 3
CRIMINAL LAW DIGESTS

Specific Criminal Intent – it must be proven by the prosecution beyond reasonable


doubt since the VICTIM did not die in this case. If the VICTIM died, the intent to kill
becomes a GENERAL CRIMINAL INTENT which is presumed by law.

It is worth highlighting that the victim received two gunshot wounds in the head.
Indeed the location of the wounds plus the nature of the weapon used are ready
indications that the accused-appellant's objective is not merely to warn or incapacitate
a supposed aggressor. Verily, had the accused-appellant been slightly better with his
aim, any of the two bullets surely would have killed him outright. Also, the intent to kill
is further exhibited by the fact that the accused-appellant even prevented barangay
officials from intervening and helping x x x the bleeding victim. Indeed, the fact that
Reynaldo Marquez was miraculously able to live through the ordeal and sustain only
modicum injuries does not mean that the crime ought to be downgraded from
frustrated homicide to less serious physical injuries. After all, as was mentioned above,
what should be determinative of the crime is not the gravity of the resulting injury but
the criminal intent that animated the hand that pulled the trigger.

 RTC found Alfredo guilty beyond unreasonable reason in the crime of FRUSTATED
HOMICIDE and was sentenced 6 MONTHS AND 1 DAY of PRISION CORRECTION as
MINIMUM and6 YEARS AND ONE DAY of PRISION MAYOR as MAXIMUM and pay
the private complainant compensatory damages in amount of P14,170.35
 CA dismissed the instant appeal and hereby affirmed the RTC’s decision.
 The SC affirmed the decision promulgated finding Alexander GUILTY beyond
reasonable doubt of FRUSTRATED HOMICIDE and was sentenced 4 YEARS of
PRISION CORRECTIONAL as MINIMUM and 8 YEARS AND ONE DAY of PRISION
MAYOR as MAXIMUM and orders the petitioner to pay Alexander P30,000 (CIVIL
INDEMITY), P30,000 (MORAL DAMAGES) and P14,170.35 plus interest 6% annum
(COMPENSATORY DAMAGES)

Fallo:

WHEREFORE, the Petition is DENIED. The Decision dated February 27, 2009 of the
Court of Appeals in CA-G.R. CR No. 31084 affirming in its entirety the March 12, 2007
Decision of the Regional Trial Court of Malolos, Bulacan, Branch 84 in Criminal Case
No. 3486-M-2002 convicting petitioner Rogelio Roque of the crime of frustrated homicide,
is AFFIRMED with the MODIFICATION that the petitioner is ordered to pay the victim
Reynaldo Marquez moral damages and temperate damages in the amount of
P25,000.00 each, with interest at the legal rate of 6% per annum from the date of
finality of this Resolution until fully paid.

VIVAR, 3
CRIMINAL LAW DIGESTS

Criminal Intent <<<

Rivera v. People
G.R. No. 166326 January 25,
2006 CALLEJO, SR. J:

Doctrine:
Usually, the intent to kill is shown by the kind of weapon used by the offender and the
parts of the victim's body at which the weapon was aimed, as shown by the wounds
inflicted. Hence, when a deadly weapon, like a bolo, is used to stab the victim in the
latter's abdomen, the intent to kill can be presumed (Reyes, The Revised Penal Code,
13TH ED., P. 431)

Facts:

Ruben Rodil and the Rivera brothers are neighbors. One fine day, a heated verbal
altercation ensued between Rodil and Edgardo Rivera. The following day, Rodil went
to a store near his house in Imus Cavite, tagging along his three year old daughter.
Suddenly, in an unexpected manner, the Rivera brothers (Esmeraldo, Ismael and
Edgardo) emerged from their house and ganged up on Rodil.

Ismael and Esmeraldo mauled Rodil with fist blows which caused the latter to fall to
the ground. While in that helpless position, Ruben was hit three times with a hollow
block on the parietal area by Edgardo. When policemen on board a mobile car arrived,
the Rivera brothers fled to their house. Rodil was then brought to the hospital for
treatment. According to the medical certificate, Rodil sustained only a superficial
wound in the parietal area.

An Information was filed in RTC Cavite, charging the Rivera brothers of attempted
murder.

RTC rendered judgment finding all the accused guilty of FRUSTRATED murder.
However, the CA modified the RTC decision, finding all the accused guilty of
ATTEMPTED murder. Hence, the accused filed to the SC the instant petition for review
under R45, insisting that the prosecution failed to prove intent to kill on their part.
More importantly, accused aver that based on the medical certificate, Rodil only
suffered a superficial wound in the parietal area; hence, accused should be criminally
liable only for physical injuries.

Issue:

Whether or not prosecution was able to prove intent to kill on the part of the accused?

Ruling:

Yes. Prosecution was able show that the accused had the intent to kill the victim.
Evidence to prove intent to kill may consist in the means used, the nature, location and
number of wounds sustained by the victim. In the case at bar, Esmeraldo and Ismael
pummeled Rodil with fist blows. Even as Rodil fell to the ground, unable to defend
himself, Edgardo still hit him on the parietal area of the head three times with a hollow
block.

An overt or external act is defined as some physical activity or deed, indicating the
intention to commit a particular crime, more than a mere planning or preparation,
which if carried out to its complete termination following its natural course, without
being frustrated by external obstacles nor by the spontaneous desistance of the
perpetrator, will logically and necessarily ripen into a concrete offense...It is necessary
that the overt act should have been the ultimate step towards the consummation of the
design. The act done need not constitute the last proximate one for completion. It is
necessary, however, that the attempt must have a causal relation to the intended
crime. The overt acts must have an immediate and necessary relation to the offense.

VIVAR, 3
CRIMINAL LAW DIGESTS

In the case at bar, the accused, who acted in concert, commenced the felony of murder
by mauling the victim and hitting him three times with a hollow block, just narrowly
missing the middle portion of Rodil’s head.

Fallo:

IN LIGHT OF ALL THE FOREGOING, the petition is DENIED for lack of merit. The
Decision of the Court of Appeals is AFFIRMED WITH THE MODIFICATION that
petitioners are sentenced to suffer an indeterminate penalty of from two (2) years of
prision correccional in its minimum period, as minimum, to nine (9) years and four (4)
months of prision mayor in its medium period, as maximum. No costs

VIVAR, 3
CRIMINAL LAW DIGESTS

Culpable Felony: Negligence and Imprudence <<<

Villareal v. People
G.R. NO. 151258, February 1, 2012
SERENO, C J:

Doctrine:
Reckless imprudence consists in voluntary, but without malice, doing or falling to do
an act from which material damage results by reason of inexcusable lack of precaution
on the part of the person performing or failing to perform such act, taking into
consideration his employment or occupation, degree of intelligence, physical condition
and other circumstances regarding persons, time and place.

If death resulted from an act executed without malice or criminal intent — but with
lack of foresight, carelessness, or negligence — the act must be qualified as reckless or
simple negligence or imprudence resulting in homicide.

Facts:

In February 1991, seven freshmen law students of the Ateneo de Manila University
School of Law were subjected to traditional forms of Aquilan “initiation rites” which
lasted for three days. They were tormented physically and psychologically.

Accused non-resident or alumni fraternity members Fidelito Dizon and Artemio Villareal
demanded that the rites be reopened. The head of initiation rites, Nelson Victorino,
initially refused but he reopened the initiation rites. The fraternity members, including
Dizon and Villareal, then subjected the neophytes to “paddling” and to additional
rounds of physical pain. Lenny received several paddle blows, one of which was so
strong it sent him sprawling to the ground. The neophytes heard him complaining of
intense pain and difficulty in breathing.

After their last session of physical beatings, Lenny could no longer walk. After an hour
of sleep, the neophytes were suddenly roused by Lenny’s shivering and incoherent
mumblings. When his condition worsened, the Aquilans rushed him to the hospital.
Lenny was pronounced dead on arrival.

The trial court rendered judgment holding the 26 accused guilty beyond reasonable
doubt of the crime of homicide, penalized with reclusion temporal under Article 249 of
the Revised Penal Code.

The CA set aside the finding of conspiracy by the trial court and modified the criminal
liability of each of the accused according to individual participation. Nineteen of the
accused-appellants were acquitted, as their individual guilt was not established by
proof beyond reasonable doubt. Four of the accused-appellants– Vincent Tecson,
Junel AnthonyAma, Antonio Mariano Almeda, and Renato Bantug, Jr. (Tecson et al.)–
were found guilty of the crime of slight physical injuries and sentenced to 20 days of
arresto menor. Two of the accused-appellants– Fidelito Dizon and Artemio Villareal
were found guilty beyond reasonable doubt of the crime of homicide under Article 249 of
the Revised Penal Code.

Issue:

Whether or not prosecution was able to prove intent and negligence to kill on the part
of the accused?

Ruling:

Yes. The intentional infliction of physical injuries on Villa was nonetheless a felonious
act under Articles 263 to 266 of the Revised Penal Code according to the trial court,
although hazing was not at the time punishable as a crime. Thus, in ruling against the
accused, the court a quo found that pursuant to Article 4(1) of the Revised Penal Code,

VIVAR, 3
CRIMINAL LAW DIGESTS

the accused fraternity members were guilty of homicide, as it was the direct, natural
and logical consequence of the physical injuries they had intentionally inflicted.

The CA modified the trial court’s finding of criminal liability. It ruled that there could
have been no conspiracy since the neophytes, including Lenny Villa, had knowingly
consented to the conduct of hazing during their initiation rites. The accused fraternity
members, therefore, were liable only for the consequences of their individual acts.

Consequently, the collective acts of the fraternity members were tantamount to


recklessness, which made the resulting death of Lenny a culpable felony. It must be
remembered that organizations owe to their initiates a duty of care not to cause them
injury in the process. Reckless imprudence or negligence consists of a voluntary act
done without malice, from which an immediate personal harm, injury or material
damage results by reason of an inexcusable lack of precaution or advertence on the part
of the person committing it. In this case, the danger is visible and consciously
appreciated by the actor. Since the NBI medico-legal officer found that the victim’s death
was the cumulative effect of the injuries suffered, criminal responsibility redounds to all
those who directly participated in and contributed to the infliction of physical injuries.
Wherefore, the court ruled that the accused are guilty of reckless imprudence resulting
in homicide.

Fallo:

WHEREFORE, premises considered, the Motion for Partial Reconsideration of petitioner


Gerarda H. Villa in connection with G.R. Nos. 178057 & 178080 is hereby DENIED. The
Motion for Reconsideration filed by the Office of the Solicitor General concerning G.R.
Nos. 155101 and 154954 is also DENIED.

The respective Motions for Clarification or Reconsideration of Antonio Mariano


Almeda, Junel Anthony D. Ama, Renato Bantug, Jr., and Vincent Tecson are likewise
DENIED. In light of the finding that Caloocan City Regional Trial Court Branch 130
acted without or in excess of its jurisdiction in taking cognizance of the
aforementioned Applications for Probation, we hereby ANNUL the entire probation
proceedings and SET ASIDE all orders, resolutions, or judgments issued in connection
thereto. We, however, CLARIFY that Antonio Mariano Almeda, Junel Anthony D. Ama,
Renato Bantug, Jr., Vincent Tecson, and Fidelito Dizon are eligible to apply or reapply
for probation in view of our recent ruling in Colinares v. People of the Philippines, 88
without prejudice to their remaining civil liability, if any.

Furthermore, we issue a CORRECTION of the dispositive portion of our Decision dated


1 February 2012 and hereby delete the phrase "and one (1) day" located in the fourth
sentence of the first paragraph thereof. The sentence shall now read as follows: "They
are hereby sentenced to suffer an indeterminate prison term of four (4) months of arresto
mayor, as minimum, to four (4) years and two (2) months of prisió n correccional, as
maximum."

VIVAR, 4
CRIMINAL LAW DIGESTS

Criminal Intent- Intent to Gain <<<

Gaviola v. People
G.R. No. 163927, January 27,
2006 CALLEJO, SR. J:

Doctrine:
For one to be guilty of theft, the accused must have an intent to steal (animus furandi)
personal property, meaning the intent to deprive another of his ownership/lawful
possession of personal property which intent is apart from, but concurrent with the
general criminal intent which is an essential element of a felony of dolo (dolos malus).
The animo being a state of the mind may be proved by direct or circumstantial evidence,
inclusive of the manner and conduct of the accused before, during and after the taking
of the personal property. General criminal intent is presumed or inferred from the very
fact that the wrongful act is done since one is presumed to have willed the natural
consequences of his own acts. Likewise, animus furandi is presumed from the taking of
personal property without the consent of the owner or lawful possessor thereof. The
same may be rebutted by the accused by evidence that he took the personal property
under a bona fide belief that he owns the property.

Facts:
This case is a petition for review of the Decision of the CA affirming the ruling of the
RTC of Biliran convicting petitioner Gaviola of qualified theft. Elias Gaviola filed a
complaint against Eusebio Mejarito involving the recovery of parcel of coconut land.
Eusebio, for his part, claimed ownership over the property. In the mean time, Eusebio
died intestate and was survived by his son, Cleto. Elias Gaviola also died intestate and
was survive by his son, Alfonso. Cleto filed a suit involving the recovery of possession
of land however, it was dismissed because the property is different from that which is
adjudicated to Eusebio Mejarito. The court ruled that he had no cause of action in the
case. At 7AM on Sept 6, 1997, Jovencio, a nephew of Cleto Mejarito, saw the Gavino
Gaviola climbing the coconut trees and gathered P3,000 worth of coconuts. Alfonso
admitted that the coconuts were taken upon his instructions, but insisted that the
trees from which they were taken were planted on Lot 1311, the property he had
inherited from his father. He asserts the he had been taking coconuts from the
property in broad day light three times a year since Aug 5, 1993 on his honest belief
that he was the owner of the land. RTC convicted him of the crime of qualified theft
which was affirmed by the CA.

Issue:
Whether the prosecution proved beyond reasonable doubt that he had intent to gain
when the coconuts were taken upon his instruction.

Ruling:
Yes. In all cases where one in good faith takes another’s property under claim of title in
himself, he is exempt from the charge of larceny, however puerile or mistaken the claim
may in fact be. And the same is true where the taking is on behalf of another, believed to be
the true owner. Still, if the claim is dishonest, a mere pretense, it will not protect the taker.
Gaviola cannot feign ignorance or even unfamiliarity with the location, identity and the
metes and bounds of the properties involved as it is categorically stated clearly that the
three parcels of land are distinct and separate from each other. Hence, Gaviola claim of
good faith in taking the coconuts is a mere pretense to escape criminal liability and was
guilty not only of simple theft but of qualified theft.

Thus, the elements of theft are: (1) that there be taking of personal property; (2) that said
property belongs to another; (3) that the taking be done with intent to gain; (4) that the
taking be done without the consent of the owner; and (5) that the taking be accomplished
without the use of violence against or intimidation of persons or force upon things.

Fallo:
IN LIGHT OF ALL THE FOREGOING, the petition is DENIED for lack of merit. Costs against
the petitioner

VIVAR, 4
CRIMINAL LAW DIGESTS

Mistake of Fact <<<

US v. Ah Chong
GR No. L-5272, March 19, 1910
CARSON J:

Doctrine:
The Court held that there is no criminal liability when one commits an offense or act
due to ignorance of facts (mistake of facts) provided that it was not due to negligence or
bad faith.

Facts:

The defendant Ah Chong worked as a cooked while the deceased Pascual Gilberto who
was a house boy. The two of them shared a room having a door with no permanent
lock. As a means of securing it, a chair was placed against the door. At around 10 in the
evening, Ah Chong who was sleeping was awakened by someone trying to forcefully open
the door. He called twice but there was no response. Fearing that the intruder might be
a thief, Ah Chong took his knife and struck the intruder when it entered the room. It
turned out that the said intruder was his roommate Pascual. Despite his plea of self-
defense, said defendant was found guilty with homicide by the Court of First Instance.

Issue:

Whether or not the defendant by reason of mistake of facts criminally liable.

Ruling:

No. The Court held that there is no criminal liability when one commits an offense or
act due to ignorance of facts provided that it was not due to negligence or bad faith.
Such ignorance of the fact is sufficient to negative the particular intent which under
the law, is an essential element to the crime of murder charged cancels the
presumption of intent and works for an acquittal. In the case, the defendant struck the
fatal blow on the belief that the intruder was a robber, on which his life and property
was in danger. It is clear that he acted in good faith without negligence and without
any criminal intent in exercising his right to self-defense. There can be no crime, large
or small, without an evil mind. The author of the Penal Code deemed criminal intent or
malice to be an essential element of the various crimes and misdemeanors. It is a
principle that the essence of an offense is the wrongful intent, without which it cannot
exist. In other words, punishment is the sequence of wickedness, without which it
cannot be. And neither in philosophical speculation nor in religious or moral sentiment
would any people in any age allow that a man should be deemed guilty unless his mind
was so. This doctrine confirmed by the maxim actus non facit reum nisi mens sit rea in
which the act itself does not make a man guilty unless his intention were so. Thus, the
Court held that the defendant should be acquitted.

Fallo:

The judgment of conviction and the sentence imposed by the trial court should be
reversed, and the defendant acquitted of the crime with which he is charged and he is
bail bond exonerated, with the costs of both instances de oficio.

VIVAR, 4
CRIMINAL LAW DIGESTS

Mistake of Fact <<<

People v. Oanis
G.R. No. 47722, July 27, 1943
MORAN J:

Doctrine:
The theory of non-liability by reason of honest mistake of fact or the maxim ignorantia
facti excusat applies only when the mistake is committed without fault or carelessness.

Facts:

Captain Godofredo Monsod (Provincial Inspector of Cabanatuan) was instructed to


arrest Balagtas (at large), a notorious criminal, and, if overpowered, to get him dead or
alive. The same instruction was given to the Chief of Police Oanis who knew the
whereabouts of Irene, the paramour of Balagtas. Upon arriving at Irene’s house, Oanis
approached Mallare and asked her where Irene’s room was. Mallare indicated the
place and upon further inquiry also said that Irene was sleeping with her paramour.

Defendants Oanis and Galanta (Corporal of the Philippine Constabulary) then went to
the room of Irene, and upon seeing a man sleeping with his back towards the door
where they were, simultaneously or successively fired at him with their .32 and .45
caliber revolvers. Awakened by the gunshots, Irene saw her paramour already
wounded, and looking at the door where the shots came, she saw the defendants still
firing at him. It turned out later that the person shot and killed was not Balagtas but a
peaceful and innocent citizen named Tecson, Irene’s paramour.

The trial court found appellants guilty of homicide through reckless imprudence.
Hence, the present appeal. It is contended that, as appellants acted in innocent mistake
of fact in the honest performance of their official duties, both of them believing that
Tecson was Balagtas, they incur no criminal liability. Appellants rely on the case of U.S.
v. Ah Chong.

Issue:

Whether or not the defendant by reason of mistake of facts criminally liable.

Ruling:

Yes. The crime committed by appellants is not merely criminal negligence, the killing
being intentional and not accidental.

1. The theory of non-liability by reason of honest mistake of fact or the maxim


ignorantia facti excusat applies only when the mistake is committed without fault or
carelessness.
2. In this case, there are no circumstances whatsoever which would press the police
to immediate action. The person in the room being then asleep, they had ample
time and opportunity to ascertain his identity without hazard to themselves, and
could even effect a bloodless arrest if any reasonable effort to that end had been
made, as the victim was unarmed. This, indeed, is the only legitimate course of
action for them to follow even if the victim was really Balagtas, as they were
instructed not to kill Balagtas at sight but to arrest him, and to get him dead or alive
only if resistance or aggression happens.

Fallo:
For all the foregoing, the judgment is modified and appellants are hereby declared guilty
of murder with the mitigating circumstance above mentioned, and accordingly
sentenced to an indeterminate penalty of from five (5) years of prision correccional to
fifteen (15) years of reclusion temporal, with the accessories of the law, and to pay the
heirs of the deceased Serapio Tecson jointly and severally an indemnity of P2,000, with
costs.

VIVAR, 4
CRIMINAL LAW DIGESTS

Mistake of Fact <<<

Yapyuco v. Sandiganbayan
G.R. Nos. 120744-46, June 25,
2012 PERALTA J:

Doctrine:
Mistake of fact rests on the inquiry into the mistaken belief of the defendant, and it
does not look at all to the belief or state of mind of any other person. It also requires
that (a) that the mistake be honest and reasonable; (b) that it be a matter of fact; and
(c) that it negate the culpability required to commit the crime or the existence of the
mental state which the statute prescribes with respect to an element of the offense.

Facts:
The accused-petitioners were members of the Integrated National Police (INP) stationed
at the Sindalan Substation in San Fernando, barangay captains of Quebiawan and De,
Carmen members of the Civil Home Defense Force (CHDF) or civilian volunteer
officers in Barangays Quebiawan, Del Carmen and Telebastagan. They allegedly
received information concerning a reported presence of armed NPA members in
Quebiawan. It was so unfortunate that the Tamaraw jeepney conveying the victims
would make an inevitable turn to which the accused all await. Believing that the
victims were the armed NPA members, the accused opened fire to the passengers of
the said Tamaraw. Such shooting incident on April 5, 1988 in Barangay Quebiawan,
San Fernando, Pampanga caused the death of Leodevince Licup (Licup) and injured
Noel Villanueva (Villanueva). The accused were all charged with murder, multiple
attempted murder and frustrated murder. Upon conviction, Yapyuco disputed that he
cannot be exonerated since he responded to the scene in fulfillment of his duty as a
member of the police force and he invoked mistake of fact as caused by his co-accused
in the belief that the victims are members of the NPA.

Issue:
Whether or not the principle of Mistake of Fact is applicable in the instant case.

Ruling:
No, the principle of Mistake of fact is not applicable. In the context of criminal law, a
mistake of fact is a misapprehension of a fact which, if true, would have justified the
act or omission which is the subject of the prosecution. Generally, a reasonable
mistake of fact is a defense to a charge of crime where it negates the intent component
of the crime. It may be a defense even if the offense charged requires proof of only
general intent. The inquiry is into the mistaken belief of the defendant, and it does not
look at all to the belief or state of mind of any other person. A proper invocation of this
defense requires
(a) that the mistake be honest and reasonable; (b) that it be a matter of fact; and (c)
that it negate the culpability required to commit the crime or the existence of the
mental state which the statute prescribes with respect to an element of the offense. In
the present case, he relied merely on the statement of his co-accused therefore it
cannot be invoked.

Fallo:
WHEREFORE, the instant petitions are DENIED. The joint decision of the Sandiganbayan in
Criminal Case Nos. 16612, 16613 and 16614, dated June 30, 1995, are hereby AFFIRMED with
the following MODIFICATIONS:

a. In Criminal Case No. 16612, petitioners are sentenced to suffer the indeterminate penalty
of six (6) years and one (1) day of prision mayor, as the minimum, to twelve (12) years
and one (1) day of reclusion temporal, as the maximum; in Criminal Case No. 16614, the
indeterminate sentence is hereby modified to Two (2) years and four (4) months of prision
correccional, as the maximum, and Six (6) months of arresto mayor, as the minimum.
b. Petitioners are DIRECTED to indemnify, jointly and severally, the heirs of Leodevince
Licup in the amount of P77,000.00 as actual damages, P50,000.00 in moral damages, as
well as Noel Villanueva, in the amount of P51,700.00 as actual and compensatory
damages, and P20,000.00 as moral damages.

VIVAR, 4
CRIMINAL LAW DIGESTS

Double Jeopardy- mala in se and mala prohibita <<<

Loney v. People
G.R. No. 152644. February 10, 2006
CARPIO J:

Doctrine:
As early as the start of the last century, this Court had ruled that a single act or
incident might offend against two or more entirely distinct and unrelated provisions of
law thus justifying the prosecution of the accused for more than one offense. The only
limit to this rule is the Constitutional prohibition that no person shall be twice put in
jeopardy of punishment for "the same offense”.

Facts:
Petitioners John Eric Loney, Steven Paul Reid and Pedro B. Hernandez are the Pres. and
CEO, Senior Manager, and Resident Manager for Mining Operations, respectively, of
Marcopper Mining Corp., a corporation engaged in mining in the province of Marinduque.

Marcopper had been storing tailings (mine waste) from its operations in a pit in Mt.
Tapian, Marinduque. At the base of the pit ran a drainage tunnel leading to the Boac
and Makulapnit rivers. It appears that Marcopper had placed a concrete plug at the
tunnel’s end. On March 24, 1994, tailings gushed out of or near the tunnel’s end. In a
few days, Mt. Tapian pit had discharged millions of tons of tailings in to the Boac and
Makalupnit rivers.

In August 1996, the DOJ separately charged petitioners in the MTC of Boac,
Marinduque with violation of Art. 91 (B), subparagraphs 5 and 6 of P.D. No. 1067 or
the Water code of the Phil., Sec. 8 of P.D. No. 984 or the National Pollution Decree of
1976, Sec. 108 of R.A. No. 7942 or the Phil. Mining Act of 1995, and Art. 365 of the
RPC for Reckless Imprudence Resulting to Damage to Property.

In the Consolidated Order of MTC, granting partial reconsideration to its Joint Order
quashing the information for violation of PD 1067 and PD 984. The MTC maintained the
Information for violation of RA 7942 and Art. 365 of the RPC. Petitioners subsequently
filed a petition for certiorari with the RTC assailing that the portion of the Consolidated
Order maintaining the Information for violation of RA 7942 and the petition was raffled
to Br. 94 while public respondent’s appeal assailing that portion of the Consolidated
Order quashing the Info. for violation of P.D. 1067 and P.D. 984 and this appeal was
consolidated with petitioners petition.

RTC Br. 94 granted the public respondent’s appeal but denied petitioner’s petition.
Petitioners then filed for certiorari with the Court of Appeals alleging that Br. 94 acted
with grave abuse of discretion because 1. the Information for violation of PD 1067, PD
984, RA 7942 and the Art. 365 of the RPC “proceeded from are based on a single act or
incident of polluting the rivers thru dumping of mine tailings, and the charge for
violation of Art 365 of the RPC absorbs the other charges since the element of “lack of
necessary or adequate protection, negligence, recklessness and imprudence” is
common among them, 2. theduplicitous nature of the Information contravenes the
ruling in People v. Relova. The Court of Appeals affirmed the Br. 94 ruling.

Issue:
Whether or not all the charges filed against petitioners except one should be quashed
for duplicity of charges and only the charge for Reckless Imprudence Resulting in
Damage to Property should stand.

Ruling:
No, Duplicity of charges simply means a single complaint or information charges more
than one offense, as Sec. 13 of Rule 110 of the 1985 Rules of Criminal Procedure. As
early as the start of the last century, the court ruled that a single act or incident might
offend against two or more entirely distinct and unrelated provisions of law thus
justifying the prosecution of the accused for more than one offense and the only limit
is

VIVAR, 4
CRIMINAL LAW DIGESTS

the Constitutional prohibition that no person shall be twice put in jeopardy of


punishing for the same offense. In People vs. Doriquez, the court held that two or more
offenses arising form the same act are not the same. And so, double jeopardy is not an
issue because not all its elements are present.

On petitioners claim that the charges for violation of Art. 365 of the RPC “absorbs” the
charges for violation of PD 1067, PD 984 and RA 7942, suffice it to say that a mala in
se felony (such as Reckless Imprudence Resulting to Damage in Property) cannot
absorb mala prohibita crimes (such as those violating PD 1067, PD 984 and RA 7942).
What makes the former felony is criminal intent (dolo) or negligence (culpa) and what
makes the latter crimes are the special laws enacting them.

Petitioners reiterate their contention in that their prosecution contravenes ruling in


People vs. Relova. In particular, petitioners cite the court’s statement in Relova that
the law seeks to prevent harassment of the accused by “multiple prosecutions for
offenses which though different from one another are nonetheless each constituted by
a common set or overlapping sets of technical elements. Thus, Relova is no authority for
petitioners’ claim against multiple prosecutions based on a single act not only because
the question of double jeopardy is not an issue here, but also because, as the Court of
Appeals held, petitioners are being prosecuted for an act or incident punished by four
national statutes and not by an ordinance and a national statute. In short,
petitioners, if ever fall under the first sentence of Sec. 21, Art. III which prohibits
multiple prosecution for the same offense, and not, as in Relova, for offenses arising
from the same incident.

Fallo:
WHEREFORE, we DENY the petition. We AFFIRM the Decision dated 5
November 2001 and the Resolution dated 14 March 2002 of the Court of Appeals.

VIVAR, 4
CRIMINAL LAW DIGESTS

Mala Prohibita <<<

Jorge Navarra v. People


G.R. No. 224943, March 20, 2017
PERLAS-BERNABE J:

Doctrine:
In mala prohibita, the defenses of good faith and lack of criminal intent are immaterial.
Criminal intent is not, as a rule, necessary, it being sufficient that the offender has the
intent to perpetrate the act prohibited by the special law. It is punishable because the
prohibited act is so injurious to the public welfare that it is the crime itself.

Facts:

An information dated 18 January 2001 was filed before the RTC charging petitioner
Jorge B. Navarra, President and Chairman of the Board of Directors of the Far East
Network of Integrated Circuits Subcontractors (FENICS) Corporation, a covered member
of the Social Security System (SSS), of violation of Section 22 (a), in relation to Section
28 (h) and (f), of RA 8282.

Pending preliminary investigation proceedings, in petitioner’s letter dated October 25,


2000, he offered SSS to pay in installments FENICS' delinquent remittances from July
1997 to September 2000, attaching thereto two (2) postdated checks in the amount of
P500,000.00 each and payable to SSS as payment, and promising to pay the remaining
balance via 48 equal monthly installments. While the first check was encashed, the
second was dishonored for being drawn against a closed account. The petitioner also
failed to follow through with the monthly installments. Later on and while the case
was pending trial, petitioner sent another letter dated April 25, 2003 to the SSS,
proposing a restructuring of FENICS's account, but the SSS rejected such proposal. In
defense, petitioner averred that he never had custody of the employees' SSS
contributions, as it was the Human Resources Department task.

Further, he asserted that during the period when the alleged delinquencies were
incurred, FENICS had already shut down. That (a) from 1995-1996, FENICS diligently
remitted the employees' SSS contributions; (b) beginning 1997, its business started to
decline due to the pull-out of one of its biggest customers eventually leading to its shut
down; and (c) since FENICS was already non-operational, its employees were unable
to work, and naturally, there could have been no wages/salaries from which the SSS
contributions could be sourced.

RTC: The petitioner was found guilty beyond reasonable doubt of the crime charged and,
accordingly, sentenced him to suffer the penalty of imprisonment for the
indeterminate period of four (4) years and two (2) months of prision correccional, as
minimum, to twenty (20) years of reclusion temporal, as maximum, and ordered him
to pay the SSS the unpaid obligation of P9,577,656.24[24] plus three percent (3%)
monthly interest reckoned from July 1997 until fully paid.

CA: Affirmed petitioner's conviction in toto.

Issue:

Whether or not good faith and lack of criminal intent is a defense in crimes mala
prohibita.

Ruling:

No, As a rule, prompt remittance of SSS contributions under Sec 22 of RA 8282 is


mandatory. Any divergence from this rule subjects the employer not only to monetary
sanctions, i.e. the payment of penalty of three percent (3%) per month, but also to
criminal prosecution if the employer fails to: (a) register its employees with the SSS;
(b) deduct monthly contributions from the salaries/wages of its employees; or (c)
remit to

VIVAR, 4
CRIMINAL LAW DIGESTS

the SSS its employees' SSS contributions and/or loan payments after deducting the
same from their respective salaries/wages. In this regard, Section 28 (f) of RA 8282
explicitly provides that "[i]f the act or omission penalized by this Act be committed by
an association, partnership, corporation or any other institution, its managing head,
directors or partners shall be liable to the penalties provided in this Act for the offense."
Notably, the aforesaid punishable acts are considered mala prohibita and, thus, the
defenses of good faith and lack of criminal intent are rendered immaterial.

Fallo:

WHEREFORE, the petition is DENIED. The Decision dated October 29, 2015 and the
Resolution dated May 19, 2016 of the Court of Appeals in CA-G.R. CR No. 35855, which
affirmed the Decision dated March 13, 2013 of the Regional Trial Court of Muntinlupa
City, Branch 206 in Crim. Case No. 01-303 finding petitioner Jorge B. Navarra GUILTY
beyond reasonable doubt of the crime of violation of Section 22 (a), in relation to Section
28 (h) and (f), of Republic Act No. 8282 is hereby AFFIRMED. Accordingly, petitioner
Jorge B. Navarra is sentenced to suffer the penalty of imprisonment for the
indeterminate period of four (4) years and two (2) months of prision correccional, as
minimum, to twenty (20) years of reclusion temporal, as maximum, and is ordered to
pay the SSS the unpaid obligation of P9,577,656.24 plus three percent (3%) monthly
interest reckoned from July 1997 until fully paid.

VIVAR, 4
CRIMINAL LAW DIGESTS

Mala Prohibita <<<

Dungo v. People
G.R. No. 209464, July 1, 2015
MENDOZA J:

Doctrine:
In mala prohibita, the defenses of good faith and lack of criminal intent are immaterial.
Criminal intent is not, as a rule, necessary, it being sufficient that the offender has the
intent to perpetrate the act prohibited by the special law. It is punishable because the
prohibited act is so injurious to the public welfare that it is the crime itself.

Facts:
On January 14, 2006, at Villa Novaliches Resort, Brgy. Pansol, Calamba, Laguna, the
petitioners Dandy L. Dungo (Dungo) and Gregorio A. Sibal, Jr. (Sibal), during a planned
initiation rite and being then officers and members of Alpha Phi Omega fraternity and
present thereat, in conspiracy with more or less twenty other members and officers,
whose identity is not yet known, did then and there willfully, unlawfully and
feloniously assault and use personal violence upon one Marlon Villanueva y Mejilla, a
neophyte thereof and as condition for his admission to the fraternity, thereby
subjecting him to physical harm, resulting to his death, to the damage and prejudice of
the heirs of the victim.

The testimonies from the witnesses presented by the prosecution placed both Dungo
and Sibal in Villa Novaliches at the time the hazing took place and various doctors also
gave their statements affirming that Villanueva’s injuries were hazing-related.

On the other hand, witnesses of the defense argued that Dungo was not in Villa
Novaliches when the initiation rite took place and that he only arrived after receiving a
call from Sibal at around 2:00 o’clock in the early morning of January 14, 2006. As for
Sibal, Gilbert Gopez testified that Villanuze already had a bruise on the left side of his
face prior to their departure going to Villa Novaliches.

RTC: Found Dungo and Sibal guilty beyond reasonable doubt of the crime of violating
Section 4 of R.A. No. 8049, and sentenced them to suffer the penalty of reclusion
perpetua.
The petitioners filed a notice of appeal on the ground that the prosecution failed to
establish their guilt beyond reasonable doubt and that Section 4 of R.A. No. 8049 in
unconstitutional.
CA: Denied the appeal made by the petitioners and affirmed the decision of the RTC.

Issue:
Whether or not violation of Anti-Hazing law is mala in se or mala prohibita?

Ruling:
Violation of Anti-Hazing Law is mala prohibita. The crime of hazing under R.A. No.
8049 is malum prohibitum. The Senate deliberations would show that the lawmakers
intended the anti-hazing statute to be malum prohibitum. The Congress created a
special law on hazing, founded upon the principle of mala prohibita.

In Vedana v. Valencia, the Court noted that in our nation’s very recent history, the
people had spoken, through the Congress, to deem conduct constitutive of hazing, an
act previously considered harmless by custom, as criminal. The act of hazing itself is
not inherently immoral, but the law deems the same to be against public policy and
must be prohibited. Accordingly, the existence of criminal intent is immaterial in the
crime of hazing. Also, the defense of good faith cannot be raised in its prosecution.

Fallo:
WHEREFORE, the petition is DENIED. The April 26, 2013 Decision and the October 8,
2013 Resolution of the Court of Appeals in CA-G.R. CR-H.C. No. 05046 are hereby
AFFIRMED in toto.

VIVAR, 4
CRIMINAL LAW DIGESTS

Criminal Intent <<<

Calimutan vs. People


GR No. 152133, February 9, 2006
CHICO-NAZARIO J:

Doctrine:
The act cannot be criminal where the mind is not criminal. Actus non facit reum, nisi
mens sit rea. The felony characterized by dolo, but not a felony resulting from culpa.
This maxim is not an absolute one because it is not applied to culpable felonies, or
those that result from negligence.

Facts:
 February 4, 1996 around 10 am: Cantre and witness Sañ ano, together with two
other companions, had a drinking spree at a videoke bar but as they were headed
home, they crossed paths with Calimutan and Michael Bulalacao.
 Cantre, 26 years old and 5 ft. 9 inches, had a grudge against Bulalacao, a 15 year-
old boy of 5ft. for suspecting that he threw stones at the his house on a previous
night so he punched him
 Seeking to protect Bulalacao and to stop Cantre, Calimutan picked a stone, as big
as a man’s fist and hitting Cantre at the left side of his back not noticing that
Bulalacao was already able to ran away.
 Cantre stopped for a moment and held his back and Calimutan desisted from any
other act of violence
 Witness Sañ ano then brought Cantre home where he complained of backache and
also of stomach ache and was unable to eat
 By night time, he felt cold then warm then he was sweating profusely and his entire
body felt numb
 Having no vehicle, they could not bring him to a doctor so his mother just continue
to wipe him with a piece of cloth and brought him some food when he asked.
 After eating a little, he vomited.
 Shortly after complaining again of his backache and stomach ache, he died.
 The Post-Mortem Examination Report and Certification of Death, issued and signed
by Dr. Ulanday, stated that the cause of death of victim Cantre was cardio-
respiratory arrest due to suspected food poisoning
 With the help of the Lingkod Bayan-Circulo de Abogadas of the ABS-CBN
Foundation, an autopsy was done by Dr. Ronaldo B. Mendez which showed that
there was internal hemorrhage and massive accumulation of blood in his abdominal
cavity due to his lacerated spleen caused by a blunt object like a stone.
 RTC issued a warrant of arrest and during arraignment Calimutan pleaded not
guilty to the crime of homicide
RTC: Essentially adopting the prosecution’s account of the incident, held that
Calimutan was guilty beyond reasonable doubt of homicide with a penalty of
imprisonment from 8 years of Prision Mayor as minimum, to 12 years and 1 day of
Reclusion Temporal as maximum, and to indemnify the heirs of Philip Cantre the
sum of P50,000 as compensatory damages and the sum of P50,000 as moral
damages.
o NOT defense of stranger, because after the boxing Bulalacao, he was able to run
thereby the unlawful aggression by Cantre ceased.
o The act of throwing a stone from behind which hit the victim at his back on the
left side was a treacherous
o Criminally liable for all the direct and natural consequences of this unlawful act
even if the ultimate result had not been intended
 CA: Affirmed RTC
 Calimutan filed a petition for review on certiorari contending that the dissimilar
findings on the cause of death constituted reasonable doubt.

Issue:
Whether or not Calimutan is guilty beyond reasonable doubt of homicide?

VIVAR, 5
CRIMINAL LAW DIGESTS

Ruling:
While the Supreme Court is in accord with the factual findings of the RTC and the CA
and affirms that there is ample evidence proving that the death of the victim Cantre was
caused by his lacerated spleen which is the result by the stone thrown at him by
petitioner Calimutan, it nonetheless, is at variance with the RTC and the CA as to the
determination of the appropriate crime or offense for which the petitioner should have
been convicted for.

Article 3 of the Revised Penal Code classifies felonies according to the means by
which they are committed, in particular: (1) intentional felonies, and (2) culpable felonies.
These two types of felonies are distinguished from each other by the existence or absence
of malicious intent of the offender.

In intentional felonies, the act or omission of the offender is malicious. In the language
of Art. 3, the act is performed with deliberate intent (with malice). The offender,
in performing the act or in incurring the omission, has the intention to cause an injury
to another. In culpable felonies, the act or omission of the offender is not malicious.
The injury caused by the offender to another person is "unintentional, it being simply
the incident of another act performed without malice." (People vs. Sara, 55 Phil. 939).
As stated in Art. 3, the wrongful act results from imprudence, negligence, lack of
foresight or lack of skill.
In the Petition at bar, this Court cannot, in good conscience, attribute to
petitioner any malicious intent to injure, much less to kill, the victim Cantre; and in the
absence of such intent, this Court cannot sustain the conviction of petitioner Calimutan
for the intentional crime of homicide, as rendered by the RTC and affirmed by
the Court of Appeals. Instead, this Court finds petitioner Calimutan guilty beyond
reasonable doubt of the culpable felony of reckless imprudence resulting in
homicide under Article 365 of the Revised Penal Code. The prosecution did not
establish that petitioner Calimutan threw the stone at the victim Cantre with the
specific intent of killing, or at the very least, of harming the victim Cantre. What is
obvious to this Court was petitioner Calimutan’s intention to drive away the attacker
who was, at that point, the victim Cantre, and to protect his helper Bulalacao
who was, as earlier described, much younger and smaller in built than the victim
Cantre.

Fallo:

WHEREFORE, the assailed Decision of the Court of Appeals in CA-G.R. CR No. 23306,
dated 29 August 2001, affirming the Decision of the RTC in Criminal Case No. 8184,
dated 19 November 1998, is hereby MODIFIED. Petitioner Calimutan is found GUILTY
beyond reasonable doubt of reckless imprudence resulting in homicide, under Article
365 of the Revised Penal Code, and is accordingly sentenced to imprisonment for a
minimum period of 4 months of arresto mayor to a maximum period of two years and
one day of prision correccional. Petitioner Calimutan is further ORDERED to pay the
heirs of the victim Cantre the amount of ₱50,000.00 as civil indemnity for the latter’s
death and ₱50,000.00 as moral damages.

VIVAR, 5
CRIMINAL LAW DIGESTS

Mala In Se <<<

Estrada vs. Sandiganbayan


GR No. 148560, November 19, 2011
BELLOSILLO J:

Doctrine:
The act cannot be criminal where the mind is not criminal. Actus non facit reum, nisi
mens sit rea. The felony characterized by dolo, but not a felony resulting from culpa.
This maxim is not an absolute one because it is not applied to culpable felonies, or
those that result from negligence.

Facts:
On April 25, 2001, the Sandiganbayan issued a resolution in Criminal Case No. 26558,
finding probable cause that petitioner Joseph Ejercito Estrada, then the President of
the Philippines has committed the offense of plunder, and that he be prosecuted under
RA 7080 (An Act Defining and Penalizing the Crime of Plunder).

The petitioner wishes to impress upon the Court that the assailed law is so defectively
fashioned that it crosses that thin but distinct line which divides the valid from the
constitutionally infirm. His contentions are mainly based on the effects of the said law
that it suffers from the (1) vice of vagueness; (2) it dispenses with the "reasonable doubt"
standard in criminal prosecutions; and (3) it abolishes the element of mens rea in
crimes already punishable under The Revised Penal Code saying that it violates the
fundamental rights of the accused.

The focal point of the case is the alleged “vagueness” of the law in the terms it uses.
Particularly, this terms are: combination, series and unwarranted. Because of this, the
petitioner uses the facial challenge on the validity of the mentioned law.

Issue:

Whether or not plunder, a crime punishable under a special penal law, mala in se
which requires the element of mensrea??

Ruling:

Yes. The legislative declaration in R.A. No. 7659 that plunder is a heinous offense
implies that it is a malum in se. For when the acts punished are inherently immoral or
inherently wrong, they are mala in se and it does not matter that such acts are
punished in a special law, especially since in the case of plunder where the predicate
crimes are mainly mala in se. Indeed, it would be absurd to treat prosecutions for
plunder as though they are mere prosecutions for violations of the Bouncing Check
Law (BP 22) or of an ordinance against jaywalking, without regard to the inherent
wrongness of the acts.

Fallo:

PREMISES CONSIDERED, this Court holds that RA 7080 otherwise known as the
Plunder Law, as amended by RA 7659, is CONSTITUTIONAL. Consequently, the
petition to declare the law unconstitutional is DISMISSED for lack of merit.

VIVAR, 5
CRIMINAL LAW DIGESTS

Intentional Felony- Wrongful Act Different From That Intended <<<

Garcia v. People
G.R. No. 171951, August 28, 2009
QUISUMBING J:

Doctrine:
A person committing a felony is responsible for all the natural and logical
consequences resulting from it although the unlawful act performed is different from
the one he intended.

Facts:
Amado Garcia and Fidel Foz Jr., had a drinking spree in the morning of September 1999
that lasted the until the evening of that day. Because of the blaring noise of the videoke
machine that the two were enjoying, Manuel Chy, told the group to quiet down. Two
days after, they met again on a wedding and again, Chy told the two to stop singing. On
the next day, the two, now with a friend, decided to have a drinking session and later
moved to Punta.

On their way to Punta, they saw Chy. Chy was about to come out of his house and
upon being summoned, Garcia suddenly punched him. Chy continued to parry the
blows. Petitioner reached for a bottle of beer, and with it, struck the lower back portion
of Chy's head causing Chy to fall. When Chy found an opportunity to escape, he ran
home and phoned his wife to call the police regarding the mauling. He also complained
of difficulty in breathing. He was found later unconscious on the kitchen floor,
salivating. He was pronounced dead on arrival at the hospital. The autopsy
confirmed that Chy died of myocardial infarction.

Garcia pleaded not guilty to the crime of homicide. The autopsy doctor confirms that
the boxing and the striking of the bottle beer on the victim could not have caused any
direct physical effect to cause the heart attack if the victim’s heart is healthy . What
could have caused said heart attack is the victims emotions concerning the violence
inflicted upon him.

Issue:
Whether or not the circumstance of having no intention to commit so grave a wrong as
that committed should be appreciated.

Ruling:
Yes. The circumstance that the petitioner did not intend so grave an evil as the death of
the victim does not exempt him from criminal liability. Since he deliberately
committed an act prohibited by law, said condition simply mitigates his guilt in
accordance with Article 13(3) of the Revised Penal Code. Nevertheless, said
circumstance must be appreciated in favour of the petitioner. The fact that the physical
injuries he inflicted on the victim could not have naturally and logically caused the
actual death of the victim, if the latter’s heart is in good condition.

Considering this mitigating circumstance, imposable penalty should be in the minimum


period, that is, reclusion temporal in its minimum period. Applying the Indeterminate
Sentence Law, the trial court properly imposed upon petitioner an indeterminate penalty
of ten (10) years of prision mayor, as minimum, to fourteen (14) years and eight (8)
months of reclusion temporal as maximum.

Fallo:
WHEREFORE, the Decision dated December 20, 2005 and the Resolution dated March
13, 2006 of the Court of Appeals in CA-G.R.-CR No. 27544 are AFFIRMED with
MODIFICATION in that the award of moral damages is reduced to P50,000. Petitioner
is further ordered to indemnify the heirs of Manuel K. Chy P50,000 as civil indemnity;
P200,000, representing expenses for the wake and burial; and P1,229,600 as loss of
earning capacity.

VIVAR, 5
CRIMINAL LAW DIGESTS

Intentional Felony- Wrongful Act Different From That Intended <<<

People v. Jenny Likiran


G.R. No. 201858, June 4, 2014
REYES J:

Doctrine:
A person committing a felony is responsible for all the natural and logical
consequences resulting from it although the unlawful act performed is different from
the one he intended.

Facts:
On March 19, 2000, during the eve of town fiesta in Barangay Bugca-on, Lantapon,
Bukidnon, a dance was held in a basketball court where Jerome Likiran, brother of
Jenny Likiran, suddenly punched Prescado Mercado in the mouth. Constancio Goloceno,
the friend of the latter, tried to assist him, however the former and his brother, Jenny
Likiran, were armed by short firearm and a hunting knife so he backed off.

The other colleagues of Mercado, who are Rolando Sareno, Sr. and Celso Dagangon,
were outside the dance area heard the commotion. Dagangon saw Jerome Likiran
approached Sareno and shot him several times. As Sareno tumbled, Jenny Likiran
stabbed him at the back. He brought Sareno to the hospital but he was already dead.
He suffered multiple shots and a stab wound at the left scapular area.

Jenny Likiran was convicted for the crime of Murder in the Regional Trial Court,
Malaybalay, City Branch 8. Jerome was not impleaded in the Information and the Court
did not acquire any jurisdiction over him.

The Court of Appeals affirmed the decision of RTC. The accused-appellant insisted his
innocence.

Issue:
Whether or not the circumstance of having no intention to commit so grave a wrong as
that committed should be appreciated.

Ruling:
Yes. The accused-appellant, therefore, is bound by his admission of Sareno's cause of
death.

More importantly, the accused-appellant is criminally liable for the natural and logical
consequence resulting from his act of stabbing Sareno. It may be that he was not the
shooter, it is nevertheless true that the stab wound he inflicted on Sareno contributed
to the latter's death. In Quinto v. Andres, 30 the Court stated that:
If a person inflicts a wound with a deadly weapon in such a manner as to put life
in jeopardy and death follows as a consequence of their felonious act, it does not alter its
nature or diminish its criminality to prove that other causes cooperated in producing the
factual result. The offender is criminally liable for the death of the victim if his delictual
act caused, accelerated or contributed to the death of the victim.

Fallo:
WHEREFORE, the Decision dated July 27, 2011 of the Court of Appeals in CA-G.R. CR-
HC No. 00484 is MODIFIED in that accused-appellant Jenny Likiran alias "Loloy" is
hereby found guilty of the lesser crime of HOMICIDE, and is sentenced to suffer the
indeterminate penalty of ten (10) years of prision mayor medium, as minimum, to
fourteen (14) years, eight (8) months and one (1) day of reclusion temporal medium,
as maximum. Further, the award of attorney's fees is hereby DELETED.
Interest at the rate of six percent (6%) per annum shall be imposed on all the damages
awarded, to earn from the date of the finality of this judgment until fully paid. In all
other respects, the Court of Appeals decision is AFFIRMED.

VIVAR, 5
CRIMINAL LAW DIGESTS

Proximate Cause <<<

People v. Ulep
G.R. No. L-36858, June 20, 1988
GANCAYCO J:

Doctrine:
“He who is the cause of the cause is the cause of the evil caused.” This is the rationale
in Article 4 of the Revised Penal Code.

Facts:

Macario Ulep, accused, elbowed her wife on her breast, upon being drunk and uttering
indecent words. Subsequently, the victim vomited and then went to bed. The accused
then left for the fields and upon returning home, found his wife dead. He immediately
reported this death to their barrio captain. Medical reports show that the victim’s
cause of death is due to cardiac arrest and primary shock.

Issue:

Whether or not the accused is criminally liable for the death of his wife.

Ruling:

Yes. Even though a blow with the fist or a kick does not cause any external wound, it
may easily produce inflammation of the spleen and peritonitis and cause death, and
even though the victim may have been previously affected by some internal malady,
yet if the blow with the fist or foot accelerated death, he who caused such acceleration
is responsible for the death as the result of an injury willfully and unlawfully inflicted.

There is that clear and categorical showing that on the appellant fell the blame for
these in human acts on his wife. He should answer for her tragic death.

Though, the appellant presented a witness to prove that sometime, his wife was
pinned down by a sack of rice and the side portion of a bull cart and was attended to
by a town quack doctor called an arbularyo. This witness said that two (2) ribs on each
side of the chest were fractured, without stating which particular ribs were so affected.
However, it was being held that even if the victim is suffering from an internal ailment,
liver or heart disease, or tuberculosis, if the blow delivered by the accused —
(a) is the efficient cause of death; or
(b) accelerated his death; or
(c) is the proximate cause of death; then there is criminal liability.

Apropos to all these is that time-respected doctrine: "He who is the cause of the cause
is the cause of the evil caused." This is the rationale in Article 4 of the Revised Penal
Code which provides that "criminal liability shall be incurred by a person committing a
felony (delito) although the wrongful act done be different from that which he intended.

Fallo:

WHEREFORE, with the above modification as to indemnity, the judgment appealed


from is hereby AFFIRMED in all other respects.

VIVAR, 5
CRIMINAL LAW DIGESTS

Proximate Cause <<<

Belbis Jr. v. People


G.R. No. 181052, November 14, 2012
PERALTA J:

Doctrine:
Proximate cause is that cause, which, in natural and continuous sequence, unbroken
by any efficient intervening cause, produces the injury, and without which the result
would not have occurred.

Facts:

On December 9, 1997, Veronica Dacir heard his live-in partner, Jose Bahillo, shouting
and calling her name. She saw blood at the back of Jose’s shorts. It was there that he
told her that he was held by petitioner Alberto while Rodolfo Belbis stabbed him four
times. He was taken to the hospital and was given immediate treatment and
antibiotics. Although his wounds were not completely healed, Jose failed to return to
the hospital due to financial constraints. Veronica brought Jose back to the hospital on
January 1, 1998, because he was complaining of urinary retention and pains in his left
and lumbar regions. He was diagnosed with having advanced Pyelonephritis, wherein his
kidney was inflamed and with pus formation and scarring. Subsequently, Joes died.

The RTC convicted both accused of the crime of homicide, but appreciated the
mitigating circumstance of incomplete self-defense. The CA affirmed the decision.

On appeal, the petitioner argue that the CA erred in ruling that the statements issued
by the victim in the presence of witnesses Veronica Dacir and SPO1 Bataller before he
died are dying declarations within the contemplation of the law as the victim still lived
for one month after the said dying declaration was made.

Issue:

Whether or not the accused is criminally liable for the death of Joes.

Ruling:

Yes. The stabbed wounds were the proximate cause of the death. Had it not for the
wounds, the victim will not suffer multiple organ failure due to exposure to infections.

Moreover, the fact that there is a lapse of time from the incident and the death of the
victim is not controlling since what really needs to be proven in a case when the victim
dies is the proximate cause of his death. It can be concluded from the doctors’
testimonies that without the stab wounds, the victim could not have been afflicted with
an infection which later on caused multiple organ failure that caused his death. The
offender is criminally liable for the death of the victim if his delictual act caused,
accelerated or contributed to the death of the victim. The petitioners are found guilty of
homicide.

Fallo:

WHEREFORE, the Petition for Review on Certiorari under Rule 45, dated February 22,
2008, of Rodolfo Belbis, Jr. and Alberto Brucales, is hereby DENIED. Consequently, the
Decision of the Court of Appeals, dated August 17, 2007, and its Resolution dated
January 4, 2008, affirming with modification the Decision dated December 23, 2004 of
the Regional Trial Court, Tabaco City, Albay, Branch 17, finding petitioners guilty
beyond reasonable doubt of the crime of Homicide are hereby AFFIRMED.

VIVAR, 5
CRIMINAL LAW DIGESTS

Criminal Liability- Wrongful Act Different From That Intended- praeter intentionem <<<

People v. Ortega
G.R. No. 116736, July 24, 1997
PANGANIBAN J:

Doctrine:
The liability of appellant Ortega; he is liable for the direct and natural consequence of
his felonious act, even if the resulting offense is worse than that intended.

Facts:

 October 15, 1992 5:30 pm: Andre Mar Masangkay (courting Raquel Ortega), Ariel
Caranto, Romeo Ortega, Roberto San Andres, Searfin, Boyet and Diosdado Quitlong
were having a drinking spree with gin and finger foods.
 October 15, 1992 11:00 pm: Benjamin Ortega, Jr. and Manuel Garcia who were
already drank joined them.
 October 16, 1992 midnight: Andre answering a call of nature went to the back
portion of the house and Benjamin followed him. Suddenly, they heard a shout
from Andre “Don’t, help me!” (Huwag, tulungan ninyo ako!)
 Diosdado and Ariel ran and saw Benjamin on top of Andre who was lying down
being stabbed. Ariel got Benjamin Ortega, Sr., Benjamin’s father while Diosdado
called Romeo to pacify his brother. Romeo, Benjamin and Manuel lifted Andre from
the canal and dropped him in the well. They dropped stones to Andre’s body to
weigh the body down. Romeo warned Diosdado not to tell anybody what he saw.
He agreed so he was allowed to go home. But, his conscience bothered him so he
told his mother, reported it to the police and accompanied them to the crime
scene.
 NBI Medico Legal Officer Dr. Ludivico J. Lagat:
o cause of death is drowning with multiple stab wounds, contributory
o 13 stab wounds
o stab wound on the upper left shoulder, near the upper left armpit and left
chest wall- front
o stab wound on the back left side of the body and the stab wound on the
back right portion of the body – back
 Manuel Garcia alibi
o He was asked to go home by his wife to fetched his mother-in-law who
performed a ritual called “tawas” on his sick daughter and stayed home
after
 Benjamin Ortega, Jr. story
o After Masangkay left, he left to urinate and he saw Andre peeking
through the room of his sister Raquel. Then, Andre approached him to
ask where his sister was. When he answered he didn’t know, Andre
punched him so he bled and fell to the ground. Andre drew a knife and
stabbed him, hitting him on the left arm, thereby immobilizing him.
Andre then gripped his neck with his left arm and threatened to kill him.
Unable to move, Ortega shouted for help. Quitlong came, seized the knife
and stabbed Andre 10 times with it. Andre then ran towards the
direction of the well. Then, he tended his wound in the lips and armpit
and slept.

 RTC: Benjamin and Manuel through conspiracy and the taking advantage of
superior strength committed murder.

Issue:

Whether or not Benjamin Ortega should be liable for murder.

VIVAR, 5
CRIMINAL LAW DIGESTS

Ruling:

Yes. Article 4, par. 1, of the Revised Penal Code states that criminal liability shall be
incurred by "any person committing a felony (delito) although the wrongful act done
be different from that which he intended." The essential requisites for the application
of this provision are that (a) the intended act is felonious; (b) the resulting act is
likewise a felony; and (c) the unintended albeit graver wrong was primarily caused by
the actor's wrongful acts. In assisting Appellant Ortega. Jr. carry the body of
Masangkay to the well, Appellant Garcia was committing a felony. The offense was that
of concealing the body of the crime to prevent its discovery, i.e. that of being an
accessory in the crime of homicide. Although Appellant Garcia may have been unaware
that the victim was still alive when he assisted Ortega in throwing the body into the
well, he is still liable for the direct and natural consequence of his felonious act, even if
the resulting offense is worse than that intended. True, Appellant Garcia merely
assisted in concealing the body of the victim. But the autopsy conducted by the NBI
medico-legal officer showed that the victim at that time was still alive, and that he died
subsequently of drowning. That drowning was the immediate cause of death was
medically demonstrated by the muddy particles found in the victim's airway, lungs
and stomach. The drowning was direct, natural and logical consequence of the felony
that Appellant Garcia had intended to commit; it exemplifies praeter intentionem
covered by Article 4, par. 1, of the Revised Penal Code. Under this paragraph, a person
may be convicted of homicide although he had no original intent to kill.

Fallo:

WHEREFORE, premises considered, the joint appeal is PARTLY GRANTED. Appellant


Benjamin Ortega, Jr. is found GUILTY of homicide and sentenced to ten (10) years of
prision mayor medium, as minimum, to fourteen (14) years, eight (8) months and one
(1) day of reclusion temporal medium, as maximum. Appellant Ortega, Jr. is also
ORDERED to pay the heirs of the victim P50,000.00 as indemnity and P31,790.00 as
actual damages. Appellant Manuel Garcia is ACQUITTED. His immediate release from
confinement is ORDERED unless he is detained for some other valid cause.

VIVAR, 5
CRIMINAL LAW DIGESTS

Efficient intervening cause <<<

Urbano v. IAC
G.R. No. 72964, January 7,
1988 GUTIERREZ, JR. J:

Doctrine:
Proximate cause is that cause, which, in natural and continuous sequence, unbroken
by any efficient intervening cause, produces the injury, and without which the result
would not have occurred.

Facts:

On October 23, 1980, petitioner Filomeno Urbano went to his ricefield and found the
place where he stored his palay flooded with water coming from the irrigation canal
nearby which had overflowed. Urbano saw Javier and the latter admitted that he was
the one responsible. A quarrel between them ensued. Urbano unsheathed his bolo
hacked Javier hitting him on the right palm of his hand, which was used in parrying
the bolo hack. Urbano was also hacked Javier on the left leg with the back portion of
said bolo, causing a swelling on said leg.

Upon the intercession of Councilman, Urbano and Javier agreed to settle their
differences. Urbano promised to pay P700.00 for the medical expenses of Javier.
Hence, on October 27, 1980, the two accompanied by Solis appeared before the San
Fabian Police to formalize their amicable settlement.

However, on November 14, 1980, Javier was rushed to the hospital with a lockjaw and
was having convulsions. The doctor found that it was caused by tetanus toxin. He
noticed the presence of a healing wound in Javier's palm which could have been
infected by tetanus.

On November 15, 1980 Javier died in the hospital. An information for homicide was filed
against Urbano. The RTC found Urbano guilty of the crime charged.

Issue:

Whether or not there was an efficient intervening cause from the time Javier was
wounded until his death which would exculpate Urbano from any liability for Javier's
death.

Ruling:

Yes. The evidence on record does not clearly show that the wound inflicted by Urbano
was infected with tetanus at the time of the infliction of the wound.

We look into the nature of tetanus-

The incubation period of tetanus, i.e., the time between injury and the appearance of
unmistakable symptoms, ranges from 2 to 56 days. However, over 80 percent of
patients become symptomatic within 14 days. A short incubation period indicates severe
disease, and when symptoms occur within 2 or 3 days of injury the mortality rate
approaches 100 percent.

Therefore, medically speaking, the reaction to tetanus found inside a man's body
depends on the incubation period of the disease.

In the case at bar, Javier suffered a 2-inch incised wound on his right palm when he
parried the bolo which Urbano used in hacking him. This incident took place on
October 23, 1980. After 22 days, or on November 14, 1980, he suffered the symptoms
of tetanus, like lockjaw and muscle spasms. The following day, November 15, 1980, he
died.

VIVAR, 5
CRIMINAL LAW DIGESTS

If, therefore, the wound of Javier inflicted by the appellant was already infected by
tetanus germs at the time, it is more medically probable that Javier should have been
infected with only a mild cause of tetanus because the symptoms of tetanus appeared
on the 22nd day after the hacking incident or more than 14 days after the infliction of
the wound. Therefore, the onset time should have been more than six days. Javier,
however, died on the second day from the onset time. The more credible conclusion is
that at the time Javier's wound was inflicted by the appellant, the severe form of
tetanus that killed him was not yet present. Consequently, Javier's wound could have
been infected with tetanus after the hacking incident. Considering the circumstance
surrounding Javier's death, his wound could have been infected by tetanus 2 or 3 or a
few but not 20 to 22 days before he died.

The rule is that the death of the victim must be the direct, natural, and logical
consequence of the wounds inflicted upon him by the accused. And since we are
dealing with a criminal conviction, the proof that the accused caused the victim's death
must convince a rational mind beyond reasonable doubt. The medical findings,
however, lead us to a distinct possibility that the infection of the wound by tetanus
was an efficient intervening cause later or between the time Javier was wounded to
the time of his death. The infection was, therefore, distinct and foreign to the crime.

Fallo:

WHEREFORE, the instant petition is hereby GRANTED. The questioned decision of the
then Intermediate Appellate Court, now Court of Appeals, is REVERSED and SET
ASIDE. The petitioner is ACQUITTED of the crime of homicide. Costs de oficio.

VIVAR, 6
CRIMINAL LAW DIGESTS

Efficient intervening cause <<<

People v. Villacorta
G.R. No. 186412, September 7, 2011
LEONARDO-DE CASTRO J:

Doctrine:
Proximate cause is that cause, which, in natural and continuous sequence, unbroken
by any efficient intervening cause, produces the injury, and without which the result
would not have occurred.

Facts:
In the early morning of January, while Danilo Cruz was buying bread from the store of
Cristina Mendeja located in Navotasm, Villacorta suddenly appeared out of nowhere
and stabbed Cruz in the left side of his body using a sharpened. Mendeja chased
Villacorta but did not succeed to caught him. When Mendeja returned to the store, her
neighbor Aron was already tending to the wounds of Cruz, removing the bamboo stick
out of his body.

Cruz was brought to the hospital on January 23, 2002 where he was treated as an out-
patient. Twenty-three days later, on February 14, Cruz was once again rushed to the
hospital because of the symptoms of tetanus infection. He later died on the next day.

RTC found accused Orlito Villacorta guilty beyond reasonable doubt of the crime of
murder and was sentenced to suffer the penalty of reclusion perpetua.
CA further affirmed the decision of the RTC.
Cruz appealed to SC and argued that he is only liable for the crime of slight physical
injuries as stabbing of Cruz is not the direct cause of his death.

Issue:
Whether or not there was an efficient intervening cause from the time Cruz was wounded
until his death which would exculpate Villacorta from any liability for Cruz's death.

Ruling:
Yes. The proximate cause of Cruz’s death is the tetanus infection and not the stab
wound.

In the event he is found to have indeed stabbed Cruz, he should only be held liable for
slight physical injuries for the stab wound he inflicted upon Cruz.

If Cruz acquired severe tetanus infection from the stabbing, then the symptoms would
have appeared a lot sooner than 22 days later. Ultimately, we can only deduce that
Cruz’s stab wound was merely the remote cause, and its subsequent infection with
tetanus might have been the proximate cause of Cruz's death. The infection of Cruz’s
stab wound by tetanus was an efficient intervening cause later or between the time Cruz
was stabbed to the time of his death.

The rule is that the death of the victim must be the direct, natural, and logical
consequence of the wounds inflicted upon him by the accused. And since we are
dealing with a criminal conviction, the proof that the accused caused the victim's death
must convince a rational mind beyond reasonable doubt. The medical findings,
however, lead us to a distinct possibility that the infection of the wound by tetanus
was an efficient intervening cause later or between the time Javier was wounded to
the time of his death. The infection was, therefore, distinct and foreign to the crime.

Fallo:
WHEREFORE, the Decision dated July 30, 2008 of the Court of Appeals in CA-G.R. CR.-
H.C. No. 02550, affirming the Decision dated September 22, 2006 of the Regional Trial
Court, Branch 170, of Malabon, in Criminal Case No. 27039-MN, is REVERSED and
SET ASIDE. A new judgment is entered finding Villacorta GUILTY beyond reasonable
doubt of the crime of slight physical injuries, as defined and punished by

VIVAR, 6
CRIMINAL LAW DIGESTS

Article 266 of the Revised Penal Code, and sentenced to suffer the penalty of thirty
(30) days arresto menor. Considering that Villacorta has been incarcerated well
beyond the period of the penalty herein imposed, the Director of the Bureau of Prisons
is ordered to cause Villacorta's immediate release, unless Villacorta is being lawfully
held for another cause, and to inform this Court, within five (5) days from receipt of
this Decision, of the compliance with such order. Villacorta is ordered to pay the heirs
of the late Danilo Cruz moral damages in the sum of Five Thousand Pesos (P5,000.00).

VIVAR, 6
CRIMINAL LAW DIGESTS

Proximate Cause <<<

People v. Acuram
G.R. No. 117954, April 27, 2000
QUISUMBING J:

Doctrine:
Proximate cause is that cause, which, in natural and continuous sequence, unbroken
by any efficient intervening cause, produces the injury, and without which the result
would not have occurred.

Facts:
Orlando Manabat and several other persons were on the highway in El Salvador,
Misamis Oriental waiting for a ride. They flagged down a passenger jeepney which
swerved dangerously towards them but did not stop. Appellant, a police officer, was then
a passenger of the jeep. Manabat shouted damning curses at the jeep and immediately
thereafter two gunshots coming from the vehicle were heard. Appellant ignored the
incident and went his way. Shot, Manabat was brought to Cagayan de Oro Medical
Center but was transferred to another hospital where he expired after undergoing an
operation. Charged with murder, appellant, who was surrendered by his commanding
officer after the issuance of an arrest warrant, admitted having been on board the said
jeep but denied ever firing his service firearm. No other passenger carried a firearm
except appellant. The trial court rendered judgment finding appellant guilty as charged
with the qualifying circumstance of treachery. Hence, this appeal.

The Court ruled that the mitigating circumstance of voluntary surrender cannot be
appreciated where appellant did not give himself up and submit himself
unconditionally to the authorities but was, on the contrary, surrendered after the
issuance of a warrant of arrest; that treachery cannot be appreciated where the
shooting was done at the spur of the moment rather than from a deliberate act of the
will; that conviction of an accused may be had on the basis of circumstantial evidence
which in the case at bar constitutes an unbroken chain pointing to the culpability of
the appellant; that anyone inflicting injuries to another is responsible for all the
consequences of his criminal act; and that in the absence of any qualifying
circumstance, the crime committed is homicide, not murder.

Issue:
Whether or not the death of Rolando Manabat’s was caused by an efficient
intervening cause of lack of prompt medical attention.

Ruling:
No. The perceived delay in giving medical treatment to the victim does not break at all
the causal connection between the wrongful act of the appellant and the injuries
sustained by the victim. It does not constitute efficient intervening cause. The
proximate cause of the death of the deceased is the shooting by the appellant. It is
settled that anyone inflicting injuries is responsible for all the consequences of his
criminal act such as death that supervenes in consequence of the injuries. The fact that
the injured did not receive proper medical attendance would not affect appellant's
criminal responsibility. The rule is founded on the practical policy of closing to the
wrongdoer a convenient avenue of escape from the just consequences of his wrongful
act. If the rule were otherwise, many criminals could avoid just accounting for their
acts by merely establishing a doubt as to the immediate cause of death.

Fallo:
WHEREFORE, the assailed DECISION of the Regional Trial Court of Cagayan de Oro City, Branch
22, in Criminal Case No. 91-1161, is hereby MODIFIED. Appellant Orlando Acuram is hereby
found GUILTY of HOMICIDE and sentenced to suffer a prison term of 10 years of the medium
period of prison mayor, as minimum, to 15 years and 10 months and 1 day of the medium period
of reclusion temporal, as maximum, with accessory penalties provided by law, to indemnify the
heirs of the deceased Rolando Manabat in the amount of P50,000.00, without subsidiary
imprisonment in case of insolvency, and to pay the costs.

VIVAR, 6
CRIMINAL LAW DIGESTS

ABERRATIO ICTUS (mistake in the blow) <<<

People v. Talampas
G.R. No. 180219, November 23, 2011
BERSAMIN, J:

Doctrine:
Aberratio Ictus is a situation wherein the offender directed a blow at his intended
victim but because of poor aim, the blow landed on another victim.

Facts:
1. Jose Sevillo (Jose) testified that on July 5, 1995 at about 7:00 in the evening, he
together with Eduardo Matic (Eduardo) and victim Ernesto Matic (Ernesto) were
in front of his house in Wawa, Malaban, Biñ an, Laguna when he noticed Virgilio
Talampas (Talampas -appellant) who was riding on a bicycle passed by and
stopped.
2. Talampas alighted the bicycle three meters away from the witness, brought out
a revolver gun, poked and fired the same to Eduardo hitting him. Eduardo then
took refuge behind Ernesto. The appellant again fired his gun three (3) times,
one shot hitting Ernesto at the right portion of his back causing him (Ernesto).
Thereafter, the appellant ran away, while he (Jose) and his neighbors brought
the victims to the hospital, wherein victim Ernesto died.
3. Talampas pleaded that the act was done in self-defense. The courts however
found that Talampas was guilty beyond reasonable doubt of the crime of
Homicide, with one mitigating circumstance of voluntary surrender, and was
sentence to suffer an indeterminate penalty of IMPRISONMENT ranging from TEN
(10) years and One (1) day of prision mayor, as minimum, to FOURTEEN (14)
years and EIGHT (8) months of reclusion temporal, as maximum.

Issue:
Whether or not the trial court erred the attack against another interpose was a self-
defense as a justifying circumstance?

Ruling:
NO, one who initiates the attack against another cannot interpose self-defense as a
justifying circumstance . The fact that the target of Talampas' assault was Eduardo, not
Ernesto, did not excuse his hitting and killing of Ernesto. The fatal hitting of Ernesto
was the natural and direct consequence of Talampas' felonious deadly assault against
Eduardo. Talampas' poor aim amounted to aberration ictus, or mistake in the blow,
a circumstance that neither exempted him from criminal responsibility nor mitigated
his criminal liability. Lo que es causa de la causa, es causa del mal causado (what is
the cause of the cause is the cause of the evil caused). Under Article 4 of the Revised
Penal Code, 14 criminal liability is incurred by any person committing a felony although
the wrongful act done be different from that which he intended.

Fallo:
WHEREFORE, the Court AFFIRMS the decision promulgated on August 16, 2007
finding VIRGILIO TALAMPAS y MATIC guilty beyond reasonable doubt of the crime of
homicide, and IMPOSES the indeterminate sentence of 10 years of prision mayor, as
minimum, to 14 years, eight months, and one day of reclusion temporal, as maximum.

VIVAR, 6
CRIMINAL LAW DIGESTS

ABERRATIO ICTUS (mistake in the blow) <<<

People v. Flora
G.R. No. 125909, June 23, 2000
QUISUMBING, J:

Doctrine:
Aberratio Ictus is a situation wherein the offender directed a blow at his intended
victim but because of poor aim, the blow landed on another victim.

Facts:
The Regional Trial Court, Branch 26, Sta. Cruz, Laguna convicted appellants
Hermogenes Flora alias "Bodoy" and Edwin Flora alias "Boboy" for the crimes of
double murder and attempted murder. During trial, it was established that days before
the incident Ireneo Gallarte pacified Hermogenes Mora and Ireneo's nephew, Oscar
Villanueva. On January 9, 1993, there was a dance party in celebration of the birthday
of Jeng-jeng Malubago. The dancing went on until past midnight but at about 1:30 in
the morning, Edwin Flora signaled Hermogenes Flora. Hermogenes then started firing
his .38 caliber revolver. The first shot razed the right shoulder of Flor Espinas, then hit
Emerita Roma below her shoulder. The second shot hit Ireneo Gallarte who slumped
onto the floor. Witness Rosalie Roma was shocked and she uttered " Si Boboy, si
Boboy." Edwin approached her, poked a knife at her neck and threatened to kill her.
Thereafter, Edwin Hermogenes fled from the scene. Emerita and Ireneo died as a
result of the said incident. Contrarily, Hermogenes and Edwin interposed alibi as a
defense. Hence, this appeal.

This Court ruled that the defense of alibi and the usual corroboration thereof are
disfavored in law since both could be very easily contrived. In the present case,
appellants' alibi is patently self-serving. Although Edwin's testimony was corroborated
by his common-law wife, it is ineffectual against the positive testimonies of eyewitnesses
and surviving victims who contradicted his alibi. Moreover, appellants did not present
any proof of improper motive on the part of the eyewitnesses in pointing to the Flora
brothers as the perpetrators of the crime. There is no history of animosity between them.
Emerita Roma and Flora Espinas were merely innocent bystanders when hit by gunfire .
Where eyewitnesses had no grudge against the accused, their testimony is credible. In
the absence of ulterior motive, mere relationship of witnesses to the victim does not
discredit their testimony. However, we cannot find Edwin Flora similarly responsible for
the death of Emerita Roma and the injury of Flor Espinas. The evidence only shows
conspiracy to kill Ireneo Gallarte and no one else. For acts done outside the
contemplation of the conspirators only the actual perpetrators are liable. To conclude,
appellant Edwin Flora is guilty beyond reasonable doubt only of the murder of Ireneo
Gallarte. He has no liability for the death of Emerita Roma nor for the injuries of Flor
Espinas caused by his co-accused Hermogenes Flora.

Issue:
Whether or not treachery may be appreciated in cases of aberration ictus.

Ruling:
YES, [W]hen Hermogenes Flora first fired his gun at Ireneo, but missed, and hit
Emerita Roma and Flor Espinas instead, he became liable for Emerita's death and
Flor's injuries. Hermogenes cannot escape culpability on the basis of aberratio ictus
principle. Criminal liability is incurred by any person committing a felony, although the
wrongful act be different from that which he intended.

Fallo:
To conclude, appellant Edwin Flora is guilty beyond reasonable doubt only of the murder
of Ireneo Gallarte. He has no liability for the death of Emerita Roma nor for the injuries
of Flor Espinas caused by his co-accused Hermogenes Flora.

VIVAR, 6
CRIMINAL LAW DIGESTS

ABERRATIO ICTUS (mistake in the blow) <<<

People v. Adriano
G.R. No. 205228, July 15, 2015
PEREZ, J:

Doctrine:
Aberratio Ictus is a situation wherein the offender directed a blow at his intended
victim but because of poor aim, the blow landed on another victim.

Facts:
The accused-appellant, Adriano, with other accused and Joe Does, while traversing the
Olongapo-Gapan National Road overtook the vehicle of the victim as well that of Police
Offers Santos and Garabiles. When the accused started shooting the latter swerved and
fall in the road embankment. The accused alighted from the vehicle and started
shooting at the driver, Cabiede. A by stander, Bulanan, who was standing near the
road embankment was hit by a stray bullet. The four (4) armed men hurriedly left the
crime scene. The police officers followed the accused but lost track of them. Later,
both Cabiedes and Bulanan die from fatal gunshot wounds. Adriano was arrested
when the vehicle was traced as rented. Adriano was identified by the policies as one
of the four assailants who alighted from the vehicle and shot Cabiedes. An examination
of the crime scene established several bullets.

The Regional Trial Court (RTC) convicted of murder for the death of Cabieded and
homicide for the death of Bulanan. The RTC gave more credence to the testimonies of
the police officers who positively identified the former as opposed to his alibi which
was not supported by clear and convincing evidence.
On appeal, the Court of Appeals affirmed the decision of the RTC and ordered Adriano
to pay the heirs of Cabiedes and Bulanas, moral and actual damages.

Issue:
Whether or not petitioner was guilty of Article 4 on the basis of aberration ictus.

Ruling:
Yes, The death of Cabiedes was one of murder by ambush which exemplifies the nature
treachery under paragraph 16 , Article 14 of the Revised Penal Code. As for Bulanan,
who was merely a bystander, was killed by a stray bullet. Stray bullets, obviously, kill
indiscriminately and often without warning, precluding the unknowing victim from
repelling the attack or defending himself. At the outset, Adriano had no intention to kill
Bulanan, much less, employ any particular means of attack.

Logically, Bulanan’s death was random and unintentional and the method used to kill
her, as she was killed by a stray a bullet, was, by no means, deliberate. Nonetheless,
Adriano is guilty of the death of Bulanan under Article 4 of the Revised Penal Code.

Pursuant to the doctrine of aberratio ictus, which imposes criminal liability for the acts
committed in violation of law and for all the natural and logical consequences resulting
therefrom. While it may not have been Adriano’s intention to shoot Bulanan, this fact
will not exculpate him. Bulanan’s death caused by the bullet fired by Adriano was the
natural and direct consequence of Adriano’s felonious deadly assault against Cabiedes.
1. By any person committing a felony (delito) although the wrongful act done be different
from that which he intended. As we already held in People v. Herrera citing People v.
Hilario, “the fact that accused killed a person other than their intended victim is of no
moment.” Evidently, Adriano’s original intent was to kill Cabiedes. However, during
the commission of the crime of murder, a stray bullet hit and killed Bulanan. Adriano is
responsible for the consequences of his act of shooting Cabiedes.

Fallo:
The appeal is DISMISSED with MODIFICATIONS on the ruling of the Court of Appeals.
Accused-appellant Adriano is guilty of murder for the deaths of Cabiedes and Bulanan.

VIVAR, 6
CRIMINAL LAW DIGESTS

ABERRATIO ICTUS (mistake in the blow) <<<

People v. Violin
G.R. Nos. 114003-06, January 14, 1997
BELLOSILLO, J:

Doctrine:
Aberratio Ictus is a situation wherein the offender directed a blow at his intended
victim but because of poor aim, the blow landed on another victim.

Facts:
December 31, 1985
 6:00pm - Cesar Allego (Barangay Captain of Daram) sought help to settle a
dispute with a Mrs. Agas from Dioscoro Astorga (Commander of Police Station
Daram), while Dioscoro, who was visiting his sister, together with his brother,
Darmo Astorga
 9:00 pm - Allego and the Astorga brothers left for San Jose on a boat owned by
Violin, after stopping at the wharf and drinking 3 bottles of beer each. They
were joined Violin, Cherriguene and Yazar.
 Violin got off at Pait and promised to meet Allego at the public dance. At 11:30
pm, the rest arrived at San Jose and followed Allego to his house. Yazar and
Cherriguene soon left after promising to meet Allego at the public dance.
 Mrs. Agas was still fishing out at sea, so the Astorgas wanted to go back to
Catbalogan, but Allego refused due to lack of transportation. The brothers had
no choice but to spend the night at Allego’s house. Allego left the brothers and
went to the public dance.
January 1, 1986
 4:00 am - Allego woke the Astorga brothers up and invited them to eat and drink.
The others were already drinking.
 Allego asked Violin to buy beer. Cherriguene and Yazar followed Violin.
 Dioscoro wanted to urinate, but was told to go outside because Allego will be
using the bathroom.
 Darmo, who was left in the kitchen, heard several gunshots. He ran to the door,
but was met by the dying Dioscoro and was told to hide so he could avoid
getting shot by Violin, Cherriguene, and Yazar. So, Darmo crawled under a small
table, saw the 3 assailants and heard them say:
o Violin - “Tapos ka, ayos na ang singkwenta mil pesos na bayad ni Cata.”
o Yazar - ”Kapitan, waray na kamo kuntra hit election, waray na kuntra
iton mga Figueroa”
 When the 3 assailants left, Darmo came out of hiding, looked for his brother’s
firearm but did not find it, went back inside the house and found Allego sprawled
and crying on the bathroom floor.
 The barangay councilor took Darmo back to Catbalogan, where he received
treatment at a hospital
 Dr. Honorata Gabon ascertained that Dioscoro died of cardiorespiratory failure
secondary to severe hemorrhage resulting from multiple gunshot wounds.

RTC:
1. Accused are guilty of murder qualified by treachery aggravated by craft and abuse
of superior strength. (against Dioscoro)

2. Accused are guilty of frustrated murder (against Darmo Astorga)

SC:
1. Modified – murder qualified by treachery. (against Dioscoro)
o No craft because there is no establishment of the use of intellectual trickery or
cunning.
o There is treachery because victim had no opportunity to defend himself and
was without any inkling about the assault
2. Modified – slight physical injuries (against Darmo Astorga)

VIVAR, 6
CRIMINAL LAW DIGESTS

o Violin fired at Dioscoro and not at Darmo.


o There was no indication that Violin knew that Darmo was hiding.
o Darmo attested that he grazed by a stray bullet. The wound is superficial.

Issue:
Whether or not petitioner was guilty of Article 4 on the basis of aberration ictus.

Ruling:
YES, The crime of slight physical injuries, not frustrated murder, was committed
against Darmo Astorga. Antonio Violin fired at Dioscoro Astorga Jr. and not at Darmo.
There is not the slightest indication that at that time Violin knew that Darmo was
hiding under a table. Darmo himself admitted that he was injured by a stray bullet
which grazed the right parietal region of his head. The wound was diagnosed as
superficial and required treatment only for three (3) days.

Fallo:
WHEREFORE, the decision of the court a quo dated 24 November 1993 in Crim.
Case No. 3030 finding ANTONIO VIOLIN, EUTIQUIO CHERRIGUENE and REMEGIO
YAZAR
guilty of murder qualified by treachery and aggravated by craft and abuse of superior
strength is MODIFIED. The crime committed by the accused was murder qualified by
treachery. They are therefore sentenced to suffer the penalty of reclusion perpetua
and to indemnify the heirs of Dioscoro Astorga Jr. the sum of P50,000.00, and to pay
the costs. The decision in Crim. Case No. 3031 for frustrated murder is likewise
MODIFIED. Accused ANTONIO VIOLIN is found guilty only of the crime of slight
physical injuries and is accordingly sentenced to suffer a straight prison term of ten
(10) days of arresto menor, and to pay the costs.

VIVAR, 6
CRIMINAL LAW DIGESTS

Proximate Cause <<<

People v. Sales
G.R. No. 177218, October 3, 2011
DEL CASTILLO, J:

Doctrine:
In order that a person may be criminally liable for a felony different from that which
he intended to commit, it is indispensible (a) that a felony was committed and (b) that
the wrong done to the aggrieved person be the direct consequence of the crime
committed by the perpetrator.

Facts:
 In the night of September 2002 in Camarines Sur, appellant Noel T. Sales beaten
his two sons Noel Jr. and Noelmar because they failed to return home after joining
the fluvial procession of Our Lady of Penafracia. Sales whipped his son with a
piece of wood approximately one meter in length and one and a half inches in
diameter. After he was finished beating his sons, his wife, Maria, noticed that
there was a crack in the head of Noemar so they brought him to a quack doctor,
who said that Noemar was already dead.
 Appellant held Noemar while on their way to the crossroad and observed his
difficulty in breathing. The pupils of Noemar’s eyes were also moving up and down.
Appellant heard him say that he wanted to sleep and saw him pointing to his
chest in pain. However, they waited in vain since a vehicle never came. It was
then that Noemar died. Appellant thus decided to just bring Noemar back to
their house.
 The wake of the child lasted only for a day and his body was never examined by
a physician.
 Sales surrendered to the police the day after.
 RTC found Sales guilty beyond reasonable doubt of the crime of parricide and
sentenced to suffer the penalty of reclusion perpetua. Sales appealed to CA but
the CA affirmed the decision of RTC.
 Appellant raised the argument that it was at this moment that Noemar died, not
during his whipping. To substantiate his claim, appellant presented his wife,
Maria, who testified that Noemar indeed suffered seizures, but this was due to
epilepsy.

Issue:
Whether or not Sales was responsible for the death of Noemar.

Ruling:
YES, Appellant attempts to evade criminal culpability by arguing that he merely
intended to discipline Noemar and not to kill him. However, the relevant portion of
Article 4 of the Revised Penal Code states:

Art. 4. Criminal liability. – Criminal liability shall be incurred:


By any person committing a felony (delito) although the wrongful act done be different
from that which he intended.
xxxx

In order that a person may be criminally liable for a felony different from that which
he intended to commit, it is indispensible (a) that a felony was committed and (b) that
the wrong done to the aggrieved person be the direct consequence of the crime
committed by the perpetrator.

Here, there is no doubt appellant in beating his son Noemar and inflicting upon him
physical injuries, committed a felony. As a direct consequence of the beating suffered
by the child, he expired. Appellant’s criminal liability for the death of his son, Noemar,
is thus clear.

VIVAR, 6
CRIMINAL LAW DIGESTS

Fallo:
WHEREFORE, the appeal is DENIED. The Decision of the Court of Appeals in CA-G.R.
CR-H.C. No. 01627 that affirmed the Joint Decision of the Regional Trial Court, Branch
63 of Calabanga, Camarines Sur in Criminal Case Nos. RTC'03-782 and RTC'03-789,
convicting Noel T. Sales of the crimes of parricide and slight physical injuries is
AFFIRMED with MODIFICATIONS that the award of exemplary damages is increased
to P30,000.00. In addition, an interest of 6% is imposed on all monetary awards from
date of finality of this Decision until fully paid.

VIVAR, 7
CRIMINAL LAW DIGESTS

Impossible Crime (Legal Impossibility) <<<

Intod v. CA
G.R. No. 103119, October 21, 1992
CAMPOS, JR J:

Case Summary:
Petitioner and 4 friends met at one of their houses said that he wanted to kill
Bernandina Palangpangan because of a land dispute. Later that same day, the
Petitioner along with his friends went to the house of Palangpangan, loaded with
firearms and started shooting at her room. However, during their assault, their
intended victim was not home. Petitioner was convicted for attempted murder, and
appealed saying that his acts were impossible crimes. The CA affirmed the decision of
the RTC.

Doctrine:
An Impossible Crime is committed by any person performing an act which would be an
offense against persons or property, were it not for the inherent impossibility of its
accomplishment or an account of the employment of inadequate or ineffectual means

Facts:
 In the morning of February 4, 1979, Sulpicio Intod, Jorge Pangasian, Santos
Tubio and Avelino Daligdig went to Salvador Mandaya's house... and asked him
to go with them to the house of Bernardina Palangpangan.
 Intod, Pangasian, Tubio and Daligdig had a meeting with Aniceto Dumalagan. He
told Mandaya that he wanted Palangpangan to be killed because of a land
dispute between them and that Mandaya should accompany the four (4) men,
otherwise, he would also be killed.
 At about 10:00 o'clock in the evening of the same day,... Mandaya, Pangasian,
Tubio and Daligdig, all armed with firearms, arrived at Palangpangan's house
 At the instance of his companions, Mandaya pointed the location... of
Palangpangan's bedroom. Thereafter, Petitioner, Pangasian, Tubio and Daligdig
fired at said room. It turned out; however, that Palangpangan was in another
City and her home was then occupied by her son-in-law and his family.
 No one was in the room when the accused fired the... shots. No one was hit by
the gun fire.
 After trial, the Regional Trial Court convicted Intod of attempted murder.
 Petitioner seeks from this Court a modification of the judgment by holding him
liable only for an impossible crime.

Issue:
Whether or not Intod is guilty attempted murder since it is an impossible crime under
Art. 4 (2).

Ruling:
YES
2. By any person performing an act which would be an offense against persons or
property, were it not for the inherent impossibility of its accomplishment or on
account of the employment of inadequate or ineffectual means.

Petitioner contends that, Palangpangan's absence from her room on the night he and
his companions riddled it with bullets made the crime inherently impossible.
 The Revised Penal Code, inspired by the Positivist School, recognizes in the
offender his formidability to punish criminal tendencies in Art. 4(2)
 Legal impossibility occurs where the intended acts, even if completed, would not
amount to a crime
 Legal impossibility would apply to those circumstances where
1. the motive, desire and expectation is to perform an act in violation of the
law
2. there is intention to perform the physical act
3. there is a performance of the intended physical act

VIVAR, 7
CRIMINAL LAW DIGESTS

4. the consequence resulting from the intended act does not amount to a
crime

Ex: The impossibility of killing a person already dead

 Factual impossibility occurs when extraneous circumstances unknown to the


actor or beyond his control prevent the consummation of the intended crime –
this case
Ex: man who puts his hand in the coat pocket of another with the intention to
steal the latter's wallet and finds the pocket empty

 United States: where the offense sought to be committed is factually impossible


or accomplishment - attempt to commit a crime; legally impossible of
accomplishment - cannot be held liable for any crime.

Fallo:
WHEREFORE, PREMISES CONSIDERED, the petition is hereby GRANTED, the decision
of respondent Court of Appeals holding Petitioner guilty of Attempted Murder is
hereby MODIFIED. WE hereby hold Petitioner guilty of an impossible crime as defined
and penalized in Articles 4, paragraph 2, and 59 of the Revised Penal Code,
respectively. Having in mind the social danger and degree of criminality shown by
Petitioner, this Court sentences him to suffer the penalty of six (6) months of arresto
mayor, together with the accessory penalties provided by the law, and to pay the
costs.

VIVAR, 7
CRIMINAL LAW DIGESTS

Impossible Crime (Factual Impossibility) <<<

Jacinto v. People
G.R. No. 162540, July 13, 2009
PERALTA J:

Case Summary:
Petitioner accepted a check worth P10,000 as an employee of Megafoam and tried to
deposit it in her friend’s husband’s account. However, the check was postdated and
bounced. This resulted into the Bank reporting the case to NBI where after entrapment
procedures filed a case for theft against the petitioner, her friend and her husband.

Doctrine:
An Impossible Crime is committed by any person performing an act which would be an
offense against persons or property, were it not for the inherent impossibility of its
accomplishment or an account of the employment of inadequate or ineffectual means

Facts:
 In June 1997, Baby Aquino, handed petitioner -collector (Valencia) of Mega
Foam, a post dated checked worth P10,000 as payment for Baby’s purchases
from Mega Foam International, Inc. The said check was deposited to the account
of Jacqueline Capitle’s husband-Generoso. Rowena Recablanca, another
employee of Mega Foam, received a phone call from an employee of Land Bank,
who was looking for Generoso to inform Capitle that the BDO check deposited
had been dishonored.
 Ricablanca then phoned accused Anita Valencia, a former employee/collector of
Mega Foam, asking the latter to inform Jacqueline Capitle about the phone call
from Land Bank regarding the bounced check.
 Valencia then told Ricablanca that the check came from Baby Aquino, and
instructed Ricablanca to ask Baby Aquino to replace the check with cash.
Valencia also told Ricablanca of a plan to take the cash and divide it equally into
four: for herself, Ricablanca, petitioner Jacinto and Jacqueline Capitle.
Ricablanca, upon the advise of Mega Foams accountant, reported the matter to
the owner of Mega Foam, Joseph Dyhengco.
 Thereafter, Joseph Dyhenga talked to Baby to tell that the BDO Check bounced.
However, Baby said that she had already paid Mega Foam P10,000 cash in
August 1997 as replacement for the dishonored check.
 Dyhengco filed a compliant with the National Bureau of Investigation (NBI) and
worked out an entrapment operation with its agents. Thereafter, petitioner and
Valencia were arrested. The NBI filed a criminal case for qualified theft against
the two (2) and Jacqueline Capitle.
 RTC rendered a decision that Gemma, Anita and Jacqueline GUILTY beyond
reasonable doubt of the crime of QUALIFIED THEFT and each of the sentenced
to suffer imprisonment of Five (5) years, Five (5) months and Eleven (11) days to
Six (6) years, Eight (8) months and Twenty (20) days.

Issue:
Whether or not the crime committed falls the definition of Impossible Crime.

Ruling:
YES
In this case, petitioner unlawfully took the postdated check belonging to Mega Foam,
but the same was apparently without value, as it was subsequently dishonored. Thus,
the question arises on whether the crime of qualified theft was actually produced.

The Court must resolve the issue in the negative.

The requisites of an impossible crime are: (1) that the act performed would be an offense
against persons or property; (2) that the act was done with evil intent; and (3) that its
accomplishment was inherently impossible, or the means employed was either
inadequate or ineffectual.

VIVAR, 7
CRIMINAL LAW DIGESTS

The fact that petitioner was later entrapped receiving the P5,000.00 marked money,
which she thought was the cash replacement for the dishonored check, is of no moment.
The Court held in Valenzuela v. People that under the definition of theft in Article 308
of the Revised Penal Code, there is only one operative act of execution by the actor
involved in theft ─ the taking of personal property of another.

There can be no question that as of the time that petitioner took possession of the
check meant for Mega Foam, she had performed all the acts to consummate the crime
of theft, had it not been impossible of accomplishment in this case. The
circumstance of petitioner receiving the P5,000.00 cash as supposed replacement for
the dishonored check was no longer necessary for the consummation of the crime of
qualified theft.

 Since the crime of theft is not a continuing offense, petitioner’s act of receiving
the cash replacement should not be considered as a continuation of the theft. At
most, the fact that petitioner was caught receiving the marked money was
merely corroborating evidence to strengthen proof of her intent to gain.

Fallo:
IN VIEW OF THE FOREGOING, the petition is GRANTED. The Decision of the Court of
Appeals, dated December 16, 2003, and its Resolution dated March 5, 2004, are
MODIFIED. Petitioner Gemma T. Jacinto is found GUILTY of an IMPOSSIBLE CRIME
as defined and penalized in Articles 4, paragraph 2, and 59 of the Revised Penal Code,
respectively. Petitioner is sentenced to suffer the penalty of six (6) months of arresto
mayor, and to pay the costs.

VIVAR, 7
CRIMINAL LAW DIGESTS

Impossible Crime <<<

People v. Tan
G.R. No. 95322, March 1,
1993 PERALTA J:

Doctrine:
An Impossible Crime is committed by any person performing an act which would be an
offense against persons or property, were it not for the inherent impossibility of its
accomplishment or an account of the employment of inadequate or ineffectual means.

Facts:
 March 11, 1982 morning: While Enrico was walking with Tirso Ferreras, his
classmate, along Roque street in the Poblacion of Lopez, Quezon, he was
approached by Pablito Domasian who requested his assistance in getting his
father's signature on a medical certificate. Enrico agreed to help and rode with
the man in a tricycle to Calantipayan, where he waited outside while the man
went into a building to get the certificate. Enrico became apprehensive and
started to cry when, instead of taking him to the hospital, the man flagged a
minibus and forced him inside, holding him firmly all the while. The man told
him to stop crying or he would not be returned to his father. When they alighted
at Gumaca, they took another tricycle, this time bound for the municipal building
from where they walked to the market. Here the man talked to a jeepney driver
and handed him an envelope addressed to Dr. Enrique Agra, the boy's father. The
two then boarded a tricycle headed for San Vicente. As Enrico was crying and
being firmly held, Alexander Grate, the tricycle driver became suspicious and
asked Domasian about his relationship with the boy who told him they were
brothers. Their physical differences and the wide gap between their ages made
Grate doubt so he immediately reported the matter to two barangay tanods when
his passengers alighted from the tricycle. Grate and the tanods went after the
two and saw the man dragging the boy. Noticing that they were being pursued,
Domasian was able to escape, leaving Enrico behind. Enrico was on his way home
in a passenger jeep when he met his parents, who were riding in the hospital
ambulance and already looking for him.
 March 11, 1982 1:45 pm: of the same day, after Enrico's return, Agra received
an envelope containing a ransom note demanding P1 million otherwise Enrico
will be killed. Agra thought the handwriting in the note was familiar so he
referred it to the NBI for examination and it turned out to be Dr. Samson Tan’s
signature.
 Domasian and Tan were subsequently charged with the crime of kidnapping
with serious illegal detention in the Regional Trial Court of Quezon
o Domasian’s alibi: at the time of the incident he was watching a mahjong
game in a friend's house and later went to an optical clinic with his wife
for the refraction of his eyeglasses.
o Dr. Tan’s alibi: he was in Manila
 Enrico, Tirso Ferreras and Grate all pointed Domasian.
 RTC: Domasian and Tan guilty as charged and sentenced them to suffer the
penalty of reclusion perpetua and all accessory penalties.
 Appealed

Issue:
Whether or not the crime committed falls the definition of Impossible Crime.

Ruling:
NO
Art. 4. Criminal liability. — Criminal liability shall be incurred:
1. By any person committing a felony (delito) although the wrongful act done be
different from that which he intended.
 Even before the ransom note was received, the crime of kidnapping with
serious illegal detention had already been committed. The act cannot be
considered an impossible crime because there was no inherent

VIVAR, 7
CRIMINAL LAW DIGESTS

improbability of its accomplishment or the employment of inadequate or


ineffective means. The sending of the ransom note would have had the
effect only of increasing the penalty to death under the last paragraph of
Article 267 although this too would not have been possible under the
new Constitution.

Fallo:
WHEREFORE, the appealed decision is AFFIRMED, with costs against the accused-
appellants.

VIVAR, 7
CRIMINAL LAW DIGESTS

Frustrated Felony <<<

People v. Gaborne
G.R. No. 210710. July 27, 2016
PEREZ, J:

Doctrine:
A felony is frustrated when the offender performs all the acts of execution which
would produce the felony as a consequence but which, nevertheless, do not produce it
by reason of causes independent of the will of the perpetrator.

Facts:
Rey Perfecto De Luna (De Luna) and Sixto Elizan (Elizan) entered a videoke bar at
Barangay Mugdo, Hinabangan, Samar. Noli Abayan (Abayan), appellant and Joselito
Bardelas (Bardelas) followed five minutes thereafter. While Elizan and De Luna were
drinking, singing and merely having fun, four successive gunshots were fired through
the window. Elizan and De Luna were hit from behind. De Luna and Marialinisa Pasana
(Pasana) saw appellant, who was then wearing a black t-shirt and a black cap, holding
a gun aimed at their location. Pasana also saw accused-appellant and Bardelas escape
after the incident. Elizan and De Luna were brought to St. Paul's Hospital at Tacloban
City. Unfortunately, Elizan was pronounced dead upon arrival while De Luna survived.

Appellant’s alibi: According to him, he and his companions ordered for bottles of beer.
However, when they tried to order for more bottles, the waitress refused to give them
their order unless they pay for their previous orders first. While Abayan was
explaining to the father of the owner of the videoke bar, appellant and Bardelas went
out to urinate, however, the waitress locked the front door. While standing outside, he
heard the waitress utter the words, "If you will not pay, I [will] have you killed, all of
you, right this moment. He also consistently contend that it was a man wearing black
shirt and camouflage pants who fired shots to the videoke bar, not him.

The following day, appellant and Bardelas were arrested and underwent paraffin test.

RTC found him guilty of the two 2 charges of Murder with the use of Unlicensed
Firearm and Frustrated Murder. Abayan and Bardelas were acquitted. CA affirmed the
decision of the RTC.

Issue:
Whether or not the degree of proof required in criminal cases has been met in the case
at bar.

Ruling:
Yes
The elements of murder are: (1) that a person was killed; (2) that the accused killed him or
her; (3) that the killing was attended by any of the qualifying circumstances mentioned in
Article 248 of the RPC; and (4) that the killing is not parricide or infanticide.

ARTICLE 248. Murder. - Any person who, not falling within the provisions of Article
246 shall kill another, shall be guilty of murder and shall be punished by reclusion
temporal in its maximum period to death, if committed with any of the following
attendant circumstances:
1. With treachery, taking advantage of superior strength, with the aid of armed men,
or employing means to weaken the defense or of means or persons to insure or
afford impunity.
2. In consideration of a price, reward or promise.
3. By means of inundation, fire, poison, explosion, shipwreck, stranding of a vessel,
derailment or assault upon a street car or locomotive, fall of an airship, by
means of motor vehicles, or with the use of any other means involving great
waste and ruin.
4. On occasion of any of the calamities enumerated in the preceding paragraph, or
of an earthquake, eruption of a volcano, destructive cyclone, epidemic, or any
other public calamity.

VIVAR, 7
CRIMINAL LAW DIGESTS

5. With evident premeditation.


6. With cruelty, by deliberately and inhumanly augmenting the suffering of the
victim, or outraging or scoffing at his person or corpse.

The requisites of treachery are: (1) The employment of means method, or manner of
execution which will ensure the safety of the malefactor from defensive or retaliating acts
on the part of the victim, no opportunity being given to the latter to defend himself or to
retaliate; and (2) Deliberate or conscious adoption of such means, method, or manner of
execution. In this case, the hapless victims were merely drinking and singing in-front of
the videoke machine when shot by the appellant. The firing was so sudden and swift that
they had no opportunity to defend themselves or to retaliate. Furthermore, appellant's
acts of using a gun and even going out of the videoke bar evidently show that he consciously
adopted means to ensure the execution of the crime.

In addition, the lower courts appropriately found appellant liable for the crime of Frustrated
Murder.

A felony is frustrated when the offender performs all the acts of execution which would
produce the felony as a consequence but which, nevertheless, do not produce it by reason
of causes independent of the will of the perpetrator.

Intent is not synonymous with motive. Motive alone is not a proof and is hardly ever an
essential element of a crime. As a general rule, proof of motive for the commission of the
offense charged does not show guilt and absence of proof of such motive does not establish
the innocence of accused for the crime charged such as murder.

Evidently, accused-appellant's intent to kill was established beyond reasonable doubt. This
can be seen from his act of shooting Elizan and De Luna from behind with a firearm while
they were innocently singing and drinking.

In view of the amendments introduced by R.A. No. 8294 and R.A. No. 10591, to Presidential
Decree No. 1866, separate prosecutions for homicide and illegal possession are no longer
in order. Instead, illegal possession of firearm is merely to be taken as an aggravating
circumstance in the crime of murder. It is clear from the foregoing that where murder
results from the use of an unlicensed firearm, the crime is not qualified illegal possession
but, murder. In such a case, the use of the unlicensed firearm is not considered as a
separate crime but shall be appreciated as a mere aggravating circumstance. Thus, where
murder was committed, the penalty for illegal possession of firearms is no longer
imposable since it becomes merely a special aggravating circumstance. The intent of
Congress is to treat the offense of illegal possession of firearm and the commission of
homicide or murder with the use of unlicensed firearm as a single offense.

In the case at hand, since it was proven that accused-appellant was not a licensed firearm
holder, and that he was positively identified by the witnesses as the one who fired shots
against the victims, the use of an unlicensed firearm in the commission of the crimes of
Murder and Frustrated Murder should be considered as an aggravating circumstance
thereof.

Fallo:
WHEREFORE, the 29 July 2013 Decision of the Court of Appeals in CA G.R. CR-H.C. No.
01183 is AFFIRMED with MODIFICATIONS. Appellant LUISITO GABORNE Y CINCO is found
GUILTY beyond reasonable doubt of the crime of Murder with the use of Unlicensed Firearm
and shall suffer a penalty of Reclusion Perpetua, without eligibility for parole and shall pay the
Heirs of Sixto Elizan y Herrera P100,000.00 as civil indemnity, P100,000.00 as moral damages,
and P100,000.00 as exemplary damages; and of the crime of Frustrated Murder and is hereby
sentenced to suffer the indeterminate penalty ranging from eleven (11) years of Prision Mayor
as minimum, to
eighteen (18) years of Reclusion Temporal as maximum and shall pay P75,000.00 as civil
indemnity, P75,000.00 as moral damages, and P75,000.00 as exemplary damages.
All monetary awards for damages shall earn interest at the legal rate of six percent (6%)
per annum from the date of finality of this judgment until fully paid.
In the service of his sentence, appellant, who is a detention prisoner, shall be credited
with the entire period of his preventive imprisonment.

VIVAR, 7
CRIMINAL LAW DIGESTS

Attempted tresspass to dwelling instead of attempted robbery <<<

People v. Lamahang
G.R. No. 43530. August 3,
1935 RECTO, J:

Case Summary:
The case is about the error in judgement of the lower court convicting the offender of
attempted robbery. An appeal was filed by the defendant, Aurelio Lamahang, with the
SC. The defendant was caught on March 2, 1935 by Jose Tomambing, a policeman. He
was caught making an opening with an iron bar on the wall of a store of cheap goods.

Doctrine:

Facts:
SUBSTANTIVE
 The defendant Aurelio Lamahang is on appeal from a decision finding him guilty
of attempted robbery.
 At early dawn on March 2, 1935, policeman Jose Tomambing, who was
patrolling his beat on Delgado and C.R. Fuentes streets of the City of Iloilo,
caught the accused in the act of making an opening with an iron bar on the
wall of a store of cheap goods located on the last named street.
 At that time the owner of the store, Tan Yu, was sleeping inside with another
Chinaman.
 The accused had only succeeded in breaking one board and in unfastening
another from the wall, when the policeman showed up, who instantly arrested
him and placed him under custody.

PROCEDURAL
 The Court of First Instance of Iloilo convicted Lamahang of attempted robbery
and sentenced him to suffer two years and four months of prision correccional
and to an additional penalty of ten years and one day of prision mayor for being
a habitual delinquent, with the accessory penalties of the law, and to pay the
costs of the proceeding.
 Aurelio Lamahang appealed the decision of the lower court.
 Supreme Court revoked the appealed sentence and changed the charge to
attempted trespass to dwelling.

Issue:
Whether or not the accused was erroneously declared guilty of attempted robbery (YES).

Ruling:
 It is necessary to prove that said beginning of execution, if carried to its
complete termination following its natural course, without being frustrated by
external obstacles nor by the voluntary desistance of the perpetrator, will
logically and necessarily ripen into a concrete offense. In the case of robbery, it
must be shown that the offender clearly intended to take possession, for the
purpose of gain, of some personal property belonging to another. In the instant
case, it may only be inferred as a logical conclusion that his evident intention
was to enter by means of force said store against the will of its owner. That his
final objective, once he succeeded in entering the store, was to rob, to cause
physical injury to the inmates, or to commit any other offense, there is nothing
in the record to justify a concrete finding.
 It must be borne in mind (I Groizard, p. 99) that in offenses not consummated,
as the material damage is wanting, the nature of the action intended (accion fin)
cannot exactly be ascertained, but the same must be inferred from the nature of
the acts executed (accion medio). The relation existing between the facts
submitted for appreciation and the offense which said facts are supposed to

VIVAR, 7
CRIMINAL LAW DIGESTS

produce must be direct; the intention must be ascertained from the facts and
therefore it is necessary, in order to avoid regrettable instances of injustice.
 Under article 280 of the Revised Penal Code, the Court is of the opinion that the
fact under consideration does not constitute attempted robbery but attempted
trespass to dwelling. Against the accused must be taken into consideration the
aggravating and mitigating circumstances.
 Aggravating circumstances: Act committed at nighttime and former convictions
(several final judgments for robbery and theft).
 Mitigating circumstance: lack of instruction
 Pursuant to article 29 of the same Code, the accused is not entitled to credit for
one-half of his preventive imprisonment.

Fallo:
WHEREFORE, the sentence appealed from is revoked and the accused is hereby held
guilty of attempted trespass to dwelling, committed by means of force, with the aforesaid
aggravating and mitigating circumstances and sentenced to three months and one day
of arresto mayor, with the accessory penalties thereof and to pay the costs.

VIVAR, 8
CRIMINAL LAW DIGESTS

Light Coercion as opposed to attempted rape <<<

Baleros v. People
G.R. No. 138033. February 22, 2006
GARCIA, J:

Doctrine:
In the crime of rape, penetration is an essential act of execution to produce the felony.
Thus, for there to be an attempted rape, the accused must have commenced the act of
penetrating his sexual organ to the vagina of the victim but for some cause or accident
other than his own spontaneous desistance, the penetration, however, slight, is not
completed.

Facts:
In early morning of December 1991, Renato Baleros went to the dormitory room of
Martina Lourdes T. Albano (Malou), placed himself on top of her, and pressed a
handkerchief soaked in chloroform. Malou struggled to free herself in the hands of
Baleros and succeeded by grabbing his sexual organ and squeezing it.

The morning after, the police said to the tenants of the dormitory to grab the things
that are theirs. The room was left with an unclaimed bag which Christian, one of the
tenants, knew right away that was Renato’s. Among the contents of the bag was a
handkerchief with a volatile substance (Chloroform).

They later found out that Renato was a suitor of Malou which she rejected a week ago.

The petitoner was convicted of attempted rape by the RTC. Aggrieved, petitioner went to
the CA whereat his appellate recourse was docketed.

Petitioner moved for reconsideration, but his motion was denied by the CA. Petitioner
is now with this Court, on the contention that the CA erred –

1. In not finding that it is improbable for petitioner to have committed the attempted
rape imputed to him, absent sufficient, competent and convincing evidence to
prove the offense charged.
2. In convicting petitioner of attempted rape on the basis merely of circumstantial
evidence since the prosecution failed to satisfy all the requisites for conviction
based thereon.
3. In not finding that the circumstances it relied on to convict the petitioner are
unreliable, inconclusive and contradictory.
4. In not finding that proof of motive is miserably wanting in his case

Issue:
Whether or not the petitioner is guilty of attempted rape.

Ruling:
No, he is not. There is absolutely no dispute about the absence of sexual intercourse or
carnal knowledge in the present case. The next question that thus comes to the fore is
whether or not the act of the petitioner, i.e., the pressing of a chemical-soaked cloth
while on top of Malou, constitutes an overt act of rape.

Overt or external act has been defined as some physical activity or deed, indicating the
intention to commit a particular crime, more than a mere planning or preparation,
which if carried out to its complete termination following its natural course, without
being frustrated by external obstacles nor by the voluntary desistance of the perpetrator,
will logically and necessarily ripen into a concrete offense.

Harmonizing the above definition to the facts of this case, it would be too strained to
construe petitioner's act of pressing a chemical-soaked cloth in the mouth of Malou
which would induce her to sleep as an overt act that will logically and necessarily
ripen into rape. As it were, petitioner did not commence at all the performance of
any act

VIVAR, 8
CRIMINAL LAW DIGESTS

indicative of an intent or attempt to rape Malou. It cannot be overemphasized that


petitioner was fully clothed and that there was no attempt on his part to undress Malou,
let alone touch her private part. For what reason petitioner wanted the complainant
unconscious, if that was really his immediate intention, is anybody’s guess.
Fallo:
WHEREFORE, the assailed Decision of the Court of Appeals affirming that of the
Regional Trial Court of Manila, is hereby REVERSED and SET ASIDE and a new one
entered ACQUITTING petitioner Renato D. Baleros, Jr. of the charge for attempted
rape. Petitioner, however, is adjudged GUILTY of light coercion and is accordingly
sentenced to 30 days of arresto menor and to pay a fine of P200.00, with the accessory
penalties thereof and to pay the costs.

VIVAR, 8
CRIMINAL LAW DIGESTS

Attempted Rape <<<

People v. Lizada
G.R. Nos. 143468-71. January 24, 2003
CALLEJO, SR., J:

Doctrine:
In the crime of rape, penetration is an essential act of execution to produce the felony.
Thus, for there to be an attempted rape, the accused must have commenced the act of
penetrating his sexual organ to the vagina of the victim but for some cause or accident
other than his own spontaneous desistance, the penetration, however, slight, is not
completed.

Facts:
Rose Orillosa had three children and one of them was Analia, who was the victim. In
1994, Rose met the accused-appellant, wherein they eventually decided to live
together. Whenever Rose was out of the house, Analia and Rossel, her sibling, would
take turns to tend the video shop as their business.

Sometime in 1996, Analia was in her room when accused-appellant entered and laid
on top of her to take off her shirt and underwear. He proceeded to insert his finger in
her vagina then, later on, inserted his penis. Analia felt a sticky substance coming
out of his penis then pain in her vagina afterward. The accused removed himself on
top of Analia then threatened to kill her if he was exposed.

Consequently, the same happened in August 1997, the accused entered Analia’s room
then placed himself on top of her and captured her legs and arms. He proceeded to
insert his finger into Analia’s vagina then left the room after he was finished. The
sexual abuse happened two times a week from 1996 to 1998.

In November 1998, Analia went to her room after studying when the accused-
appellant also went inside Analia’s room. Then again, the accused placed himself on
top of her and touched her breasts. Analia struggled to free herself as the accused-
appellant removed her panty to touch her vagina. Momentarily, he inserted his penis
into her organ then ejaculated afterward. When Rossel suddenly passed by Analia’s
room, the accused dismounted from her.

After a heated argument between the accused and Analia occurred, Rose and Analia
left the house. Analia then told her mother that the accused had been touching her.
The two proceeded to file an Affidavit-Complaint to the Police District and submitted
herself to genitalia examination.

The trial court rendered its judgment finding that the accused-appellant is
guilty beyond reasonable doubt of four counts of rape and is meted of the death
penalty in each count.

Issue:
Whether or not the accused-appellant is guilty of attempted rape under Article 335 of
the Revised Penal Code

Ruling:
Yes, the accused-appellant is guilty of attempted rape. Pursuant to Article 6 of the
Revised Penal Code, it states that there is an attempt when the offender commences
the commission of a felony directly by overt acts and does not perform all the acts of
execution which should produce the felony by reason of some cause or accident other
than his own spontaneous desistance. The elements of an attempted felony include
the following:
1. The offender commences the commission of the felony directly by overt acts;
2. He does not perform all the acts of execution which should produce the felony;
3. The offender's act be not stopped by his own spontaneous desistance;

VIVAR, 8
CRIMINAL LAW DIGESTS

4. The non-performance of all acts of execution was due to cause or accident other
than his spontaneous desistance.

The Supreme Court affirmed the facts given by the prosecution, finding that
the accused-appellant intended to have carnal knowledge of private complainant. The
accused-appellant had commenced the execution of rape, which if not for the
spontaneous desistance, will ripen into the crime of rape. Even though the accused-
appellant desisted from executing the act, his desistance, however, was not
spontaneous as he was impelled to do so because of a sudden arrival of Rossel. Hence,
the accused- appellant is guilty of attempted and is hereby meted an indeterminate
penalty of from six years of prision correccional in its maximum period, as minimum to
ten years of prision mayor in its medium period, as maximum.

Fallo:
IN LIGHT OF ALL THE FOREGOING, the Decision of the Regional Trial Court of Manila,
Branch 54, is SET ASIDE. Another judgment is hereby rendered as follows:

1. In Criminal Case No. 99-171390, accused-appellant is hereby found guilty


beyond reasonable doubt of simple rape under Article 335 of the Revised Penal
Code as amended and is hereby meted the penalty of reclusion perpetua.
Accusedappellant is also hereby ordered to pay private complainant Analia
Orillosa the amounts of P50,000.00 by way of civil indemnity and P50,000.00 by
way of moral damages;
2. In Criminal Case No. 99-171391, accused-appellant is hereby found guilty of
attempted rape under Article 335 of the Revised Penal Code as amended in
relation to Article 6 of the said Code and is hereby meted an indeterminate
penalty of from six years of prision correccional in its maximum period, as
minimum to ten years of prision mayor in its medium period, as maximum.
Accused-appellant is hereby ordered to pay private complainant Analia Orillosa
the amount of P25,000.00 by way of moral damages; and,

3. In Criminal Cases Nos. 99-171392 and 99-171393, accusedappellant is hereby


found guilty beyond reasonable doubt of two counts of simple rape, defined in
Article 335 of the Revised Penal Code as amended and is hereby meted the
penalty of reclusion perpetua for each count. Accusedappellant is hereby
ordered to pay to private complainant Analia Orillosa the amount of P50,000.00
by way of civil indemnity and the amount of P50,000.00 by way of moral
damages for each count, or a total amount of P200,000.00.

VIVAR, 8
CRIMINAL LAW DIGESTS

Frustrated Homicide <<<

FE ABELLA y PERPETUA v. PEOPLE OF THE PHILIPPINES


G.R. No. 198400. October 7, 2013
REYES, J:

Doctrine:
In cases of frustrated homicide, the main element is the accused's intent to take his
victim's life. Intent to kill may be proved by evidence of: (a) motive; (b) the nature or
number of weapons used in the commission of the crime; (c) the nature and number of
wounds inflicted on the victim; (d) the manner the crime was committed; and (e) the
words uttered by the offender at the time the injuries are inflicted by him on the victim.

Facts:
 On October 7, 1998, the petitioner, who at times worked as a farmer, baker and
trisicad driver, was charged with frustrated homicide in an Information which
reads:

That on or about September 6, 1998, at 11:00 o’clock in the evening, more or


less, at Sitio Puli, Canitoan, Cagayan de Oro City, Philippines and within the
jurisdiction of this Honorable Court, the above-named accused, without any
justifiable cause, did then and there willfully, unlawfully and feloniously and
with intent to kill, attack, assault, harm and hack one, BENIGNO ABELLA y
PERPETUA, with the use of a scythe, hitting the latter’s neck, thereby inflicting
the injury described below, to wit:
o hacking wound left lateral aspect neck; and
o incised wound left hand dorsal aspect thus performing all the acts of
execution which would produce the crime of homicide as a consequence,
but nevertheless, did not produce it by reason of some cause or causes
independent of the will of the accused, that is the timely and able
intervention of the medical attendance rendered to the said victim.

 The Prosecution evidence established that on September 6, 1998, at around


11:00 p.m., Benigno was watching television in his house. A certain Roger
Laranjo arrived and asked Benigno to pacify the petitioner, who was stirring
trouble in a nearby store.
 Benigno and Amelita found the petitioner fighting with Alejandro and a certain
Dionisio Ybañ es (Dionisio). Benigno was able to convince the petitioner to go
home. Benigno and Amelita followed suit and along the way, they dropped by the
houses of Alejandro and Dionisio to apologize for the petitioner’s conduct.
 Benigno and Amelita were in Alejandro’s house when the petitioner arrived
bringing with him two scythes, one in each of his hands. Benigno instructed
Alejandro and Dionisio to run away and the latter two complied. The petitioner
wanted to enter Alejandro’s house, but Benigno blocked his way and asked him
not to proceed. The petitioner then pointed the scythe, which he held in his left
hand, in the direction of Benigno’s stomach, while the scythe in the right hand
was used to hack the latter’s neck once. Benigno fell to the ground and was
immediately taken to the hospital while the petitioner ran to chase Alejandro.
 Dr. Ardiente testified that Benigno sustained: (a) a "hacking wound left lateral
aspect neck 11 cm"; and (b) an "incised wound left hand dorsal aspect 4 cm".
Benigno was initially confined in the hospital on September 6, 1998 and was
discharged on September 23, 1998. From Dr. Ardiente’s recollection, since the
scythe used in the hacking was not sterile, complications and infections could
have developed from the big and open wounds sustained by Benigno, but
fortunately did not.
 The petitioner relied on denial and alibi as defenses. He claimed that from
September 2, 1998 to October 2002, he and his family resided in Buenavista,
Agusan del Norte. Sitio Puli, Canitoan, Cagayan de Oro City, where the hacking
incident occurred, is about four (4) hours drive away. Fernando testified that on
September 6, 1998, he saw the petitioner gathering woods to make a hut. Later

VIVAR, 8
CRIMINAL LAW DIGESTS

in the evening, at around 5:00 p.m., Urbano spotted the petitioner drinking tuba
in the store of Clarita Perpetua.

RTC: On July 13, 2006 convicted the petitioner of Frustrated Homicide. The RTC found
the petitioner’s defenses of alibi and denial as weak. No disinterested witnesses were
presented to corroborate the petitioner’s claim that he was nowhere at the scene of
the hacking incident on September 6, 1998. Fernando and Urbano’s testimonies were
riddled with inconsistencies.

CA: On October 26, 2010, the CA affirmed the petitioner’s conviction for the crime of
frustrated homicide ratiocinating that:
Intent to kill may be proved by evidence of:
(a) motive;
(b) the nature or number of weapons used in the commission of the crime;
(c) the nature and number of wounds inflicted on the victim;
(d) the manner the crime was committed; and
(e) the words uttered by the offender at the time the injuries are inflicted by him on the
victim.

Here, the intent to kill was sufficiently proven by the Prosecution. The petitioner
attacked Benigno with deadly weapons, two scythes. The petitioner’s blow was directed
to the neck of Benigno. The attack on the unarmed and unsuspecting Benigno was
swift and sudden. The latter had no means, and no time, to defend himself.

Dr. Roberto Ardiente, Jr., who attended and issued the Medical Certificate, testified that
Benigno suffered from a hack wound on the left neck, and an incised wound on the left
hand palm. He said that the wounds might have been caused by a sharp, pointed and
sharp-edged instrument, and may have resulted to death without proper medical
attendance. Benigno was hospitalized for about a month because of the injuries. The
location of the wound (on the neck) shows the nature and seriousness of the wound
suffered by Benigno. It would have caused his death, had it not been for the timely
intervention of medical science.

Issue:
Whether or not the RTC and CA erred in holding the petitioner guilty of Frustrated
Homicide.

Ruling:
No
Even if this Court were to be exceptionally liberal and allow a review of factual issues,
still, the instant petition is susceptible to denial.

To successfully prosecute the crime of homicide, the following elements must be


proved beyond reasonable doubt:
(1) that a person was killed;
(2) that the accused killed that person without any justifying circumstance;
(3) that the accused had the intention to kill, which is presumed; and
(4) that the killing was not attended by any of the qualifying circumstances of murder,
or by that of parricide or infanticide.

Moreover, the offender is said to have performed all the acts of execution if the wound
inflicted on the victim is mortal and could cause the death of the victim without medical
intervention or attendance.

In cases of frustrated homicide, the main element is the accused’s intent to take his
victim’s life. The prosecution has to prove this clearly and convincingly to exclude
every possible doubt regarding homicidal intent. And the intent to kill is often inferred
from, among other things, the means the offender used and the nature, location, and
number of wounds he inflicted on his victim.

VIVAR, 8
CRIMINAL LAW DIGESTS

In Benigno’s case, he sustained an 11-centimeter long hacking wound in the neck and
a 4-cm long incised wound in his left hand caused by the unsterile scythe used by the
petitioner. Dr. Ardiente testified that "it is possible to have complications resulting
from these injuries because the wounds were extensive and they were big and they were
open wounds, so there is a possibility of infections resulting from these kinds of
wounds, and the instrument used was not a sterile instrument contaminated with
other things." No complications developed from Benigno’s wounds which could have
caused his death, but he was confined in the hospital for a period of 17 days from
September 6, 1998 to September 23, 1998.

From the foregoing, this Court concludes and thus agrees with the CA that the use of a
scythe against Benigno’s neck was determinative of the petitioner’s homicidal intent
when the hacking blow was delivered. It does not require imagination to figure out
that a single hacking blow in the neck with the use of a scythe could be enough to
decapitate a person and leave him dead. While no complications actually developed
from the gaping wounds in Benigno’s neck and left hand, it perplexes logic to conclude
that the injuries he sustained were potentially not fatal considering the period of his
confinement in the hospital. A mere grazing injury would have necessitated a lesser
degree of medical attention.

This Court likewise finds wanting in merit the petitioner’s claim that an intent to kill is
negated by the fact that he pursued Alejandro instead and refrained from further
hacking Benigno. What could have been a fatal blow was already delivered and there
was no more desistance to speak of. Benigno did not die from the hacking incident by
reason of a timely medical intervention provided to him, which is a cause independent
of the petitioner’s will.

Fallo:
WHEREFORE, the instant petition is DENIED. The Decision and Resolution, dated
October 26, 2010 and August 11, 2011, respectively, of the Court of Appeals in CA-G.R.
CR No. 00336-MIN are AFFIRMED with MODIFICATIONS. The petitioner, Fe Abella y
Perpetua is ORDERED TO PAY the offended party moral damages in the amount of
P25,000.00 and temperate damages in the amount of P25,000.00. Further, the
monetary awards for damages shall be subject to interest at the legal rate of six
percent (6%) per annum from the date of finality of this Decision until fully paid.

VIVAR, 8
CRIMINAL LAW DIGESTS

Attempted Murder not Frustrated Murder <<<

People v. Labiaga
G.R. No. 202867. July 15, 2013
CARPIO, J:

Doctrine:
In frustrated murder, there must be evidence showing that the wound would have
been fatal were it not for timely medical intervention. If the evidence fails to convince the
court that the wound sustained would have caused the victim’s death without timely
medical attention, the accused should be convicted of attempted murder and not
frustrated murder.

Facts:
 Labiaga (appellant) together with Barcenas and Demapanag was charged with
murder with the use of unlicensed firearm. As read in the information, in Ajuy,
Iloilo, the three men, conspiring, confederating and helping one another, armed
with unlicensed firearm, with deliberate intent and decided purpose to kill, by
means of treachery and with evident premeditation.
 Barcenas remained at large, while both Labiaga and Demapang pleaded not guilty
in both cases.

Prosecution:
 According to the prosecution, Gregorio Conde and his two daughters, Judy and
Glenelyn were in their home in Ajuy, Iloilo. Gregorio stepped outside while
Glenelyn was in their store, which is a part of their house. Labiaga, 5 meters away
from Gregorio, shot him. Judy called for help. When Judy and Gledelyn rushed to
their father, Labiaga shot Judy in the abdomen. Barcenas and Demapang were
standing behind Labiaga and said that Gregorio’s dead and thereafter fled.
 Gregorio and Judy were rushed to the hospital. Judy was pronounced dead on
arrival, while Gregorio made a full recovery. An autopsy was conducted and it
was found that Judy’s death was caused by cardiopulmonary arrest secondary to
cardiac tamponade due to the gunshot wound. On the other hand, Gregorio was
examined and found that he sustained a gunshot wound measuring in one
centimeter diameter in his right forearm and abrasion wounds hematoma
formation in his right shoulder.

Defendant:
 Labiaga admitted that he was present during the incident. However, he claimed
that he acted in self-defense, because Gregorio, armed with a shotgun, challenged
him to a fight. Gregorio attempted to shoot him but the shotgun jammed. Labiaga
tried to wrest the shotgun from Gregorio, and during the struggle, the shotgun
fired. He claimed that he did not know if anyone was hit.
 Demapanag on the other hand claimed that at the time of the shooting he was in
D&D Ricemill,14 km away from the crime scene. This was corroborated by his
brother.

Lower Court(s):
 RTC acquitted Demapanag due to insufficiency of evidence. Labiaga, however, was
convicted of murder and frustrated murder. This was also upheld by the CA-Cebu.
 Hence, this appeal.

Issue:
Whether or not Labiaga is guilty of frustrated murder.

Ruling:
NO. Labiaga is guilty of attempted murder and not frustrated murder on trying to kill
Gregorio and crime of murder for killing Judy. Article 6 of the RPC defines the stages in
the commission of felonies. There is an attempt when the offender commences the
commission of a felony directly by overt acts, and does not perform all the acts of
execution which should produce the felony by reason of some cause or accident other

VIVAR, 8
CRIMINAL LAW DIGESTS

than his own spontaneous desistance. In frustrated murder, there must be evidence
showing that the wound would have been fatal were it not for timely medical intervention.
If the evidence fails to convince the court that the wound sustained would have caused
the victim’s death without timely medical attention, the accused should be convicted of
attempted murder and not frustrated murder. In the instant case, it does not appear
that the wound sustained by Gregorio was mortal. Thus, the court holds that appellant
should be convicted of attempted murder and not frustrated murder.

Fallo:
WHEREFORE, we AFFIRM the 18 October 2011 Decision of the Court of Appeals-Cebu
in CA-G.R. CEB CR-HC No. 01000 with MODIFICATIONS. In Criminal Case No. 2002-
1777, we find that appellant Regie Labiaga is GUILTY of Attempted Murder and shall
suffer an indeterminate sentence ranging from two (2) years, four (4) months and one
(1) day of prision correccional as minimum, to eight (8) years and one (1) day of prision
mayor as maximum, and pay P40,000.00 as moral damages and P30,000.00 as
exemplary damages. In Criminal Case No. 2001-1555, appellant shall pay P75,000.00
as civil indemnity, P50,000.00 as moral damages, and P30,000.00 as exemplary
damages.

VIVAR, 8
CRIMINAL LAW DIGESTS

No frustrated theft, only attempted or consummated <<<

Valenzuela v. People
G.R. No. 160188. June 21, 2007
TINGA, J:

Doctrine:
Indeed, we have, after all, held that unlawful taking, or apoderamiento, is deemed
complete from the moment the offender gains possession of the thing, even if he has
no opportunity to dispose of the same.

Facts:
 Valenzuela (petitioner) and Calderon were charged with the crime of theft. They were
seen outside the Super Sale Club, a supermarket within SM North EDSA, by Lago
(security guard). Lago saw Valenzuela wearing an id with the mark “receiving
dispatching unit (rdu)”, hauling a push cart with cases of tide detergent. Valenzuela
unloaded the cases in an open parking space where Calderon was waiting.
Valenzuela returned to the supermarket and emerged with more cartons of Tide
Ultramatic and unloaded in the same open parking space. Valenzuela then hailed a
taxi and directed it towards where Calderon was. Calderon loaded the Tide
Ultramatic inside the taxi, then boarded the vehicle. Lago stopped the taxi as it was
leaving the open parking and asked Valenzuela for a receipt of the merchandise.
Calderon fled on foot while Lago fired a warning shot to alert his fellow security
guards. Valenzuela and Calderon were apprehended at the scene and the stolen
merchandise were recovered.
 Valenzuela and Calderon were first brought to the SM security office then transferred
to the Baler Station II of PNP QC. It appears in the investigation that 4 persons were
apprehended by the security guards but only Valenzuela and Calderon were charged
by the prosecutor.

Defense:
 Valenzuela and Calderon both claimed to have been innocent bystanders. Calderon
alleged that he was there to withdraw from the ATM with his neighbor. Due to the
long queue, he decided to buy snacks inside the supermarket. They were eating
when they heard a gunshot, leading them to head out of the building to see what
was transpiring. As they were outside, a security guard suddenly grabbed them and
commenced their detention.
 Valenzuela on the other hand, was in the parking lot walking and headed to ride a
tricycle. The gunshot caused him and others to run, at which he was apprehended
and brought to the security office.

Lower Court(s):
 The RTC of Quezon City convicted both Valenzuela and Calderon of the crime of
consummated theft. Both of them filed their respective notice of appeal but only
Valenzuela filed a brief.
 Valenzuela argued that he should only be convicted of frustrated theft since at the
time he was apprehended, he was never placed in a position to freely dispose of the
stolen articles.
 The CA rejected this contention and affirmed his conviction. Hence, this petition for
review, which expressly seeks that petitioner’s conviction be modified to only
frustrated theft.

Issue:
Whether or not Valencia shall only be convicted of frustrated theft.

Ruling:
NO. Article 6 provides those 3 stages, namely the consummated, frustrated and
attempted felonies. It also provides that a felony is consummated when all the
elements necessary for its execution and accomplishment are present. In the crime of
theft, the following elements should be present: (1) that there be taking of personal
property; (2)

VIVAR, 9
CRIMINAL LAW DIGESTS

that said property belongs to another; (3) that the taking be done with intent to gain;
(4) that the taking be done without the consent of the owner; and (5) that the taking
be accomplished without the use of violence against of intimidating of persons or force
upon things. The court held that theft is produced when there is deprivation of
personal property by one with intent to gain. Thus, it is immaterial that the offender is
able or unable to freely dispose of the property stolen since he has already committed
all the acts of execution and the deprivation from the owner has already ensued from
such acts. Therefore, theft cannot have a frustrated stage, and can only be attempted
or consummated.

Fallo:
WHEREFORE, the petition is DENIED. Costs against petitioner.

VIVAR, 9
CRIMINAL LAW DIGESTS

No frustrated theft <<<

Canceran v. People
G.R. No. 206442. July 1, 2015
MENDOZA, J:

Doctrine:
There is no language in Article 308 that expressly or impliedly allows that the "free
disposition of the items stolen" is in any way determinative of whether the crime of Theft
has been produced. The Supreme Court thus concluded that, under the RPC, there is
no crime of frustrated Theft.

Facts:
 Canceran together with Vequizo and Diaz, was charged with frustrated theft.
 In Ororama Mega Center Grocery Dept., CDO, Canceran, conspiring, confederating
together and mutually helping one another with his co-accused Vequizo, URC
merchandiser and Diaz, a Unilever merchandiser, with intent to gain and without
knowledge and consent of the owner, wilfully, unlawfully and feloniously take, steal
and carry away 14 cartons Ponds White Beauty Cream valued at P28,627.20. By
reason of some cause independent of the accused’s will, they were discovered by
the employees of Ororama who prevented them from further carrying away the
said 14 cartons of Ponds.

Prosecution:
 The prosecution presented Ompoc (security guard), and Arcenio (customer relation
officer) of Ororama as its witnesses. Ompoc saw Canceran approach one counter
while pushing a cart which contained two boxes of Magic Flakes for which he paid.
Ompoc went to the packer and asked if the boxes had been checked. Upon
inspection, they found that the contents of the boxes were not biscuits but
Ponds. Canceran hurriedly left and a chase ensued. Canceran stumbled as he
attempted to ride a jeepney. After being questioned, he tried to settle with the
guards and even offered his personal effects to pay for the items he tried to take.

Defendant:
 Canceran vehemently denied the charges against him. He claimed that he was a
promo merchandiser and that he was in Ororama to buy medicine for his wife. On
his way out, a male around 20 years of age requested him to pay for the items in his
cart and gave him money. He didn’t know the contents of the boxes. When he was
to take a jeepney, 3 persons ran after him and he was caught and even took
his personal belongings.

 Cenceran further claimed that an earlier information for theft was already filed,
which was eventually dismissed.

Lower Court(s):
 The RTC found Canceran guilty beyond reasonable doubt of consummated theft.
Canceran appealed on the ground of double jeopardy but was denied by the CA, as
well as the MR that he subsequently filed.
 Hence, this petition. Canceran argues that there was no taking of the ponds cream
considering that the information in such case admits that the act of the petitioner
did not produce the crime of theft. Thus, absent the element of taking, the felony of
theft was never proved.

Issue:
Whether or not Canceran shall be convicted of consummated theft?

Ruling:
Yes
 As stated earlier, there is no crime of Frustrated Theft. The Information can never
be read to charge Canceran of consummated Theft because the indictment itself
stated

VIVAR, 9
CRIMINAL LAW DIGESTS

that the crime was never produced. Instead, the Information should be construed to
mean that Canceran was being charged with theft in its attempted stage only.
Necessarily, Canceran may only be convicted of the lesser crime of Attempted Theft.
 An accused cannot be convicted of a higher offense than that with which he was
charged in the complaint or information and on which he was tried. It matters not
how conclusive and convincing the evidence of guilt may be, an accused cannot be
convicted in the courts of any offense, unless it is charged in the complaint or
information on which he is tried, or necessarily included therein. He has a right to
be informed as to the nature of the offense with which he is charged before he is
put on trial, and to convict him of an offense higher than that charged in the
complaint or information on which he is tried would be an unauthorized denial of
that right.

Fallo:
WHEREFORE, the petition is PARTIALLY GRANTED . The August 10, 2012 Decision
and the March 7, 2013 Resolution of the Court of Appeals in CA-G.R. CR No. 00559 are
hereby MODIFIED, in that, the Court finds accused Jovito Canceran guilty beyond
reasonable doubt of the crime of Attempted Theft.

Accordingly, the Court sentences the accused to suffer the indeterminate prison term
ranging from Four (4) Months of Arresto Mayor, as minimum, to Two (2) Years, Four (4)
Months of Prision Correccional, as maximum.

VIVAR, 9
CRIMINAL LAW DIGESTS

Consummated Rape <<<

People v. Quinanola
G.R. No. 126148. May 5,
1999 VITUG, J:

Doctrine:
In the crime of rape, from the moment the offender has carnal knowledge of his victim,
he actually attains his purpose and, from that moment also all the essential elements
of the offense have been accomplished. Nothing more is left to be done by the offender,
because he has performed the last act necessary to produce the crime. Thus, the felony
is consummated.

Facts:
In the evening of March 1994, Catalina Carciler (Catalina), with her two companions,
attended a party in Cebu. An hour later, they decided to go home. On their way home,
Agapito Quianola (Agapito) and Eduardo Escuadro (Eduardo), beamed a light to the
three. When the light flashed in the face of Catalina, Agapito said that they were
members of NPA.

Eduardo brought Catalina’s companions outside the waiting shed. Meanwhile, Agapito
forcibly brought Catalina in a nearby school. Pointing a gun at her, undressed her. He
then unzipped his pants and laid on top of her while Eduardo was holding her legs. She
felt his organ on the lips of her vagina. When Agapito satisfied his lust, Eduardo took
his turn.

When the two were finished, they left Catalina who was only wearing Tshirt and bra.
She managed to run and go home. After recovering from a state of shock, she told his
mother that she was raped by Agapito and Eduardo.

They reported the crime to the police filed a criminal case against the two. Agapito denied
the allegation stating that he was on a day-off and he helped in the construction of
their house. Eduardo also denied the allegation by insisting that the charge was result
of a mistaken identity.

RTC finds them guilty beyond reasonable doubt principals by direct participation and
indispensable cooperation of the frustrated rape. CA affirmed the decision. Petitioners
raised the case to the SC averring that they should be charged of frustrated rape only.

Issue:
Whether or not the petitioners are guilty of consummated rape.

Ruling:
YES. Petition DENIED.
While the evidence may not show full penetration on both occasions of rape, the slightest
penetration is enough to consummate the offense. In fact, there was vulva penetration
in both cases. The fact that the hymen was intact upon examination does not belie rape
for a broken hymen is not an essential element of rape; nor does the fact that the
victim has remained a virgin negate the crime. What is fundamental is that the
entrance, or at least the introduction, of the male organ into the labia of the
pudendum is proved.

Jurisprudence is well-settled to the effect that for rape to be consummated, rupture of


the hymen is not necessary, nor is it necessary that the vagina sustained a laceration
especially if the complainant is a young girl.

In its recent holding in People vs. Echegaray,[43] the Court has declared that a mere
knocking at the doors of the pudenda, so to speak, by the accused’s penis suffices to
constitute the crime of rape as full entry into the victims vagina is not required to
sustain a conviction.

VIVAR, 9
CRIMINAL LAW DIGESTS

Clearly, in the crime of rape, from the moment the offender has carnal knowledge of
his victim, he actually attains his purpose and, from that moment also all the essential
elements of the offense have been accomplished. Nothing more is left to be done by the
offender, because he has performed the last act necessary to produce the crime. Thus,
the felony is consummated.

Until Congress sees it fit to define the term frustrated rape and thereby penalize it, the
Court will see its continued usage in the statute book as being merely a persistent
lapse in language.

Apellants Agapito Quianola y Escuadro and Eduardo Escuadro y Floro are each found
guilty beyond reasonable doubt of two (2) counts of consummated rape.

Fallo:
WHEREFORE, appellants Agapito Quiñ anola y Escuadro and Eduardo Escuadro y Floro
are each found guilty beyond reasonable doubt of two (2) counts of consummated
rape and, accordingly, sentenced to the penalty of reclusion perpetua in each case. Said
appellants are ordered to pay, jointly and severally, Catalina Carciller the sum of
P100,000.00 by way of indemnity ex delictu for the two counts of consummated rape
plus P60,000.00 moral damages. Costs against appellants.

VIVAR, 9
CRIMINAL LAW DIGESTS

No Frustrated Rape <<<

People v. Orande
G.R. Nos. 141724-27. November 12, 2003
CORONA, J:

Doctrine:
The Revised Penal Code presently stands that there is no such crime as frustrated rape.

Facts:
In April 1994 in Paco, Manila, Jessica Dela Cruz (Jessica) was raped by Arnulfo Chavez
(Arnulfo), her stepfather, while her biological mother, Girlie, was in a fish market. The
incident happened three more times until November of 1996. The following year, Mrs.
Mojica, Jessica’s teacher noticed the unusual treatment of Arnulfo to his step-daughter.
Mrs. Mojica asked Jessica about the incident and later confessed that she was raped
several times by his step-father. Mrs. Mojica narrated to Jessica’s aunt what she’s
confessed and later accompanied her to the nearest police station. Medical examiner
found that there was a healed hymenal tear in her vagina.

Arnulfo denied the allegation stating that Jessica has a motive of falsely accusing him
of raping her because she disapproved his relationship with his mother.

RTC found Arnulfo guilty beyond reasonable doubt of the crime of rape (three counts)
and frustrated rape. CA affirmed the decision, hence the petition. Petitioner contends
the court erred in convicting the accused of frustrated rape.

Issue:
Whether or not accused is guilty of frustrated rape.

Ruling:
NO. Let it be said once again that, as the Revised Penal Code presently so stands, there
is no such crime as frustrated rape. In People vs. Orita, the Court has explicitly
pronounced:

Clearly, in the crime of rape, from the moment the offender has carnal knowledge of
his victim, he actually attains his purpose and, from that moment also all the essential
elements of the offense have been accomplished. Nothing more is left to be done by the
offender, because he has performed the last act necessary to produce the crime. Thus,
the felony is consummated.

Thus, it was error for the trial court to convict appellant of frustrated rape. Besides,
after a careful review of the records, we find that the rape was in fact consummated.
Jessica initially testified that, although appellant did not succeed in inserting his penis
in her vagina, she felt his sex organ touch hers and she saw and felt semen come out of
his penis and smear her vagina.

Fallo:
WHEREFORE, the decision of the Regional Trial Court of Manila, Branch 18, in Criminal Case
Nos. 97-159184 to 87 is AFFIRMED with the following MODIFICATIONS:

1. In Criminal Case No. 97-159184, appellant is convicted of simple rape under Article 335
of the Revised Penal Code and sentenced to suffer the penalty of reclusion perpetua.
2. In Criminal Case No. 97-159185, appellant is convicted of statutory rape under Article
335 of the Revised Penal Code and sentenced to suffer the penalty of reclusion perpetua.
3. In Criminal Case No. 97-159186, appellant is convicted of statutory rape under Article
335 of the Revised Penal Code and sentenced to suffer the penalty of reclusion perpetua.
4. In Criminal Case No. 97-159187, appellant is convicted of simple rape under Article 335
of the Revised Penal Code and sentenced to suffer the penalty of reclusion perpetua.

For each count of rape, appellant is ordered to pay complainant Jessica Castro P50,000 as moral
damages, P50,000 as civil indemnity and P25,000 as exemplary damages, or a total of
P500,000. Costs against appellant.

VIVAR, 9
CRIMINAL LAW DIGESTS

Acts of Lasciviousness instead of attempted rape <<<

People v. Pareja
G.R. No. 202122. January 15,
2014 LEONARDO-DE CASTRO, J:

Doctrine:
When the "touching" of the vagina by the penis is coupled with the intent to penetrate,
attempted rape is committed; otherwise, the crime committed is merely acts of
lasciviousness.

Facts:
 The prosecution charged the appellant before the RTC of Mandaluyong of the crime
of rape committed against AAA, 13 years of age, sister of the common-law spouse of
appellant.

Prosecution:
 At around 3:30 a.m. of June 16, 2003, AAA was sleeping beside her two-year old
nephew, BBB, on the floor of her sister's room, when the appellant hugged her and
kissed her nape and neck. AAA cried, but the appellant covered her and BBB with a
blanket. The appellant removed AAA's clothes, short pants, and underwear; he then
took off his short pants and briefs. The appellant went on top of AAA, and held her
hands. AAA resisted, but the appellant parted her legs using his own legs, and then
tried to insert his penis into her vagina. The appellant stopped when AAA's cry got
louder; AAA kicked the appellant's upper thigh as the latter was about to stand up.
The appellant put his clothes back on, and threatened to kill AAA if she disclosed
the incident to anyone. Immediately after, the appellant left the room. AAA covered
herself with a blanket and cried.
 At around 6:00 a.m. of the same day, AAA's brother, CCC, went to her room and
asked her why she was lying on the floor and crying. AAA did not answer, and
instead hurled invectives at CCC. AAA went to the house of her other brother, but
the latter was not in his house. AAA proceeded to the house of her older sister,
DDD, at Block 19, Welfareville Compound, and narrated to her what had happened.
Afterwards, AAA and her two (2) siblings went to the Women and Children's Desk
of the Mandaluyong City Police Station and reported the incident.

Defense:
 For his defense, the appellant declared on the witness stand that he hauled "filling
materials" at his house, located at Block 38, Fabella Compound, on the evening of
June 15, 2003. At around10:00 p.m., he went to his room and slept. On the next
day, the appellant, accompanied by his mother and brother-in-law, went to the
municipal hall to ask for financial assistance for his wife who was confined in the
hospital. Upon arrival at the hospital, the doctor told him that his wife needed
blood. Immediately after, the appellant and his companions went to Pasig City to
find blood donors.
 On the evening of June 16, 2003, and while the appellant was folding the clothes of
his son, two policemen entered his house and informed him that a complaint for
attempted rape had been filed against him. The police brought him to the Criminal
Investigation and Detection Group, forced him to admit the crime, mauled him, and
then placed him in a detention cell. The appellant added that he filed a complaint
before the Office of the Ombudsman against the police officers who beat him up.

Lower Courts:
 RTC: The RTC convicted the appellant of rape.
 CA: The CA affirmed the RTC decision. It explained that a slight penetration of the
labia by the male organ is sufficient to constitute rape, and held that a slight
penetration took place when the appellant's penis touched AAA's vagina as he was
trying to insert it.
 The appellate court further ruled that the presence of people in the other room did
not make it impossible for the appellant to have raped the victim, because lust is no

VIVAR, 9
CRIMINAL LAW DIGESTS

respecter of time and place. It also held that the victim's lack of tenacity in resisting
the appellant's sexual aggression did not amount to consent or voluntary
submission to the criminal act.

Issue:
1. Whether or not appellant should be convicted of the crime of rape.
2. Whether or not appellant should be convicted of attempted rape.

Ruling:
1. NO.
We find that the prosecution failed to prove the appellant's guilt beyond
reasonable doubt of the crime of consummated rape.

By definition, rape is committed by having carnal knowledge of a woman with


the use of force, threat or intimidation, or when she is deprived of reason or
otherwise unconscious, or when she is under 12years of age or is demented.
"Carnal knowledge is defined as the act of a man having sexual intercourse or
sexual bodily connections with a woman." Carnal knowledge of the victim by the
accused must be proven beyond reasonable doubt, considering that it is the
central element in the crime of rape.

In her testimony, AAA recounted the alleged rape as follows:


Q: And when he was able to part your legs, what happened next?
A: He tried to insert his sexual organ but he was not able to do so, ma'am.
Q: How did you know that he was trying to insert his sexual organ?
A: "Naidikit po niya sa ari ko."
Q: Which part of your body was he able to touch his sexual organ? (sic)
A: On my sexual organ, ma'am.xxx xxx xxx
Q: You mentioned earlier that he was not able to penetrate your private part,
[AAA]?
A: Yes, ma'am.

From the foregoing, we find it clear that the appellant's penis did not penetrate,
but merely 'touched'(i.e., "naidikit"), AAA's private part. In fact, the victim
confirmed on cross-examination that the appellant did not succeed in inserting
his penis into her vagina. Significantly, AAA's Sinumpaang Salaysay also
disclosed that the appellant was holding the victim's hand when he was trying
to insert his penis in her vagina. This circumstance 4 coupled with the victim's
declaration that she was resisting the appellant's attempt to insert his penis into
her vagina 4 makes penile penetration highly difficult, if not improbable.
Significantly, nothing in the records supports the CA's conclusion that the
appellant's penis penetrated, however slightly, the victim's female organ.

2. YES.
Rape is consummated by the slightest penile penetration of the labia majora or
pudendum of the female organ. Without any showing of such penetration, there
can be no consummated rape; at most, it can only be attempted rape [or] acts of
lasciviousness. As earlier discussed, the prosecution failed to present sufficient
and convincing evidence to establish the required penile penetration. AAA's
testimony did not establish that the appellant's penis touched the labias or slid
into her private part. Aside from AAA's testimony, no other evidence on record,
such as a medico-legal report, could confirm whether there indeed had been
penetration, however slight, of the victim's labias. In the absence of testimonial
or physical evidence to establish penile penetration, the appellant cannot be
convicted of consummated rape. Article 6 of the Revised Penal Code, as
amended, states that there is an attempt when the offender commenced the
commission of the crime directly by overt acts but does not perform all the acts
of execution by reason of some cause or accident other than his own
spontaneous desistance. In People v. Publico, we ruled that when the "touching"
of the vagina by the penis is coupled with the intent to penetrate, attempted
rape is committed; otherwise, the crime committed is merely acts of
lasciviousness.

VIVAR, 9
CRIMINAL LAW DIGESTS

In the present case, the appellant commenced the commission of rape by the
following overt acts: kissing AAA's nape and neck; undressing her; removing his
clothes and briefs; lying on top of her; holding her hands and parting her legs;
and trying to insert his penis into her vagina. The appellant, however, failed to
perform all the acts of execution which should produce the crime of rape by
reason of a cause other than his own spontaneous desistance, i.e., the victim's
loud cries and resistance. The totality of the appellant's acts demonstrated the
unmistakable objective to insert his penis into the victim's private parts.

Fallo:
WHEREFORE, premises considered, the Decision of the Court of Appeals in CA-G.R.
CR.-H.C. No. 03794 is hereby AFFIRMED with MODIFICATION. We find accused-
appellant Bernabe Pareja y Cruz GUILTY of two counts of Acts of Lasciviousness,
defined and penalized under Article 336 of the Revised Penal Code, as amended. He is
sentenced to two (2) indeterminate prison terms of 6 months of arresto mayor, as
minimum, to 4 years and 2 months of prisió n correccional, as maximum; and is
ORDERED to pay the victim, AAA, P20,000.00 as civil indemnity, P30,000.00 as
moral damages, and P10,000.00 as exemplary damages, for each count of acts of
lasciviousness, all with interest at the rate of 6% per annum from the date of finality
of this judgment.

VIVAR, 9
CRIMINAL LAW DIGESTS

No frustrated stage for the crime of rape; Difference of attempted rape and
acts of lasciviousness. <<<

Cruz v. People
G.R. No. 166441. October 8, 2014
BERSAMIN, J:

Doctrine:
No frustrated stage for the crime of rape; Difference of attempted rape and acts of
lasciviousness.

Facts:
Prosecution’s version:
 On December 20, 1993, Norberto Bartolome and Belinda employed AAA and BBB
to help them in selling their plastic and glass wares in Bangar, La Union which was
then celebrating its fiesta. AAA and BBB boarded a passenger jeepney owned by
Norberto. They were accompanied by Norberto, Belinda, Ruben Rodriguez (driver)
and a sales boy named Jess.
 At 8 PM of December 20, they reached Bangar, La Union. They parked in front of
Maroon enterprises. They brought out all the goods and wares for display. Two
tents were fixed in order that they will have a place to sleep. Belinda and the driver
proceeded to Manila to get more goods to be sold.
 AAA and BBB went to sleep. AAA was then awakened when Norberto was mashing
her breast and touching her private parts. She realised that she was totally naked.
Norberto ordered her not to scream or she’ll be killed. AAA tried to push Norberto
away then kicked Norberto twice. The latter offered her money and told her not to
tell the incident to her mother, otherwise she will be killed. AAA looked for Jess but
he didn’t wake up.
 When AAA returned to their tent, she saw Norberto touching the private parts of
BBB. The latter was awake but her hands were shaking. After AAA entered the tent,
Norberto left and went outside.
 In the evening of Dec. 21, AAA and BBB went straight to the municipal hall and told
a policeman the sexual advances made by Norberto. Norberto was summoned to
the police station. Belinda arrived at the police station and an argument ensued
between them.
 On Dec 22, at around 2:20 AM, they ordered the complainants to return at 6 AM.
Norberto and Belinda were still able to bring AAA and BBB home with them and
worked for them until Dec30, after which they were sent back to Lingayen,
Pangasinan.
 On January 10, 1994, AAA and BBB executed their respective sworn statements
against Norberto upon going back to La Union.

Defense:
 The accused maintains that it was not possible for him to commit the crimes hurled
against him. On the date of the alleged incident, there were many people around
who were preparing for the "simbang gabi". Considering the location of the tents,
which were near the road and the municipal hall, he could not possibly do the
dastardly acts out in the open, not to mention the fact that once AAA and BBB
would scream, the policemen in the municipal hall could hear them. He believes
that the reason why the complainants filed these cases against him was solely for
the purpose of extorting money from him.

RTC:
The petitioner guilty beyond reasonable doubt of attempted rape and acts of
lasciviousness.

CA:
Affirmed the conviction of the petitioner for attempted rape, but acquitted Norberto of
the acts of lasciviousness due to the insufficiency of the evidence (BBB did not testify).

VIVAR, 1
CRIMINAL LAW DIGESTS

Defense (upon raising the issue to the SC):


 The petitioner assails the behavior and credibility of AAA. He argues that AAA still
continued working for him and his wife until December 30, despite the alleged
attempted rape, there by belying his commission of the crime against her; that he
could not have undressed her without rousing her if she had gone to sleep only an
hour before, because her bra was locked at her back; that her testimony about his
having been on top of her for nearly an hour while they struggled was also
inconceivable unless she either consented to his act and yielded to his lust, or the
incident did not happen at all, being the product only of her fertile imagination; that
the record does not indicate if he himself was also naked, or that his penis was
poised to penetrate her; and that she and her mother demanded from him
P80,000.00 as settlement, under threat that she would file a case against him.
 On the second issue, the petitioner assails the glaring inconsistencies in the
testimony of AAA that cast doubt on her veracity.

Issue:
Whether or not Norberto committed attempted rape.

Ruling:
No
Judicial experience has shown, indeed, that the trial courts are in the best position to
decide issues of credibility of witnesses, having themselves heard and seen the witnesses
and observed firsthand their demeanor and deportment and the manner of testifying
under exacting examination. As such, the contentions of the petitioner on the
credibility of AAA as a witness for the State cannot be entertained. The review focuses
only on determining the question of law of whether or not the petitioner's climbing on
top of the undressed AAA such that they faced each other, with him mashing her
breasts and touching her genitalia with his hands, constituted attempted rape. There is
an attempt, according to Article 6 of the RPC, when the offender commences the
commission of a felony directly by overt acts, and does not perform all the acts of
execution which should produce the felony by reason of some cause or accident other
than this own spontaneous desistance. To ascertain whether the acts performed by the
petitioner constituted attempted rape, we have to determine the law on rape in effect
on December 21, when the petitioner committed the crime he was convicted of. The
basic element of rape then and now is carnal knowledge of a female. Carnal knowledge
is defined simply as "the act of a man having sexual bodily connections with a woman,"
which explains why the slightest penetration of the female genitalia consummates the
rape. In other words, rape is consummated once the penis capable of consummating
the sexual act touches the external genitalia of the female.

In People v. Campuhan, the SC defined the extent of “touching” by the penis in rape:
Touching when applied to rape cases does not simply mean mere epidermal contact,
stroking or grazing of organs..

There must be sufficient and convincing proof that the penis indeed touched the labias or
slid into the female organ, and not merely stroked the external surface thereof, for an
accused to be convicted of consummated rape. As the , which are required to be
"touched" by the penis, are by their natural situs or location beneath the mons pubis or
the vaginal surface, to touch them with the penis is to attain some degree of penetration
beneath the surface, hence, the conclusion that touching the labia majora or the labia
minora pudendum constitutes consummated rape.

Thus, a grazing of the surface of the female organ or touching the mons pubis of the
pudendum is not sufficient to constitute consummated rape. Absent any showing of the
slightest penetration of the female organ, ie, touching of either labia of the pudendum by
the penis, there can be no consummated rape; at most, itcan only be attempted rape, if
not acts of lasciviousness.

The SC further explained why there is no frustrated rape. It is a physical impossibility,


considering that the requisites of a frustrated felony under Article 6 of the RPC are that:
(1) the offender has performed all the acts of execution which would produce the felony;

VIVAR, 1
CRIMINAL LAW DIGESTS

and (2) that the felony is not produced due to causes independent of the perpetrator's
will. The ruling in People v. Eriña where the offender was declared guilty of frustrated
rape because of lack of conclusive evidence was a stray decision.

In attempted rape, therefore, the concrete felony is rape, but the offender DOES NOT
perform all the acts of execution of having carnal knowledge. The State, to establish
attempted rape, must show that his overt acts, should his criminal intent be carried to
its complete termination without being thwarted by extraneous matters, would ripen
into rape, for, as succinctly put in People v. Dominguez, Jr.: The gauge in determining
whether the crime of attempted rape had been committed is the commencement of the
act of sexual intercourse, i.e., penetration of the penis into the vagina, before the
interruption.

The petitioner climbed on top of the naked victim, and was already touching her
genitalia with his hands and mashing her breasts when she freed herself from his
clutches and effectively ended his designs on her. Yet, inferring from such
circumstances that rape, and no other, was his intended felony would be highly
unwarranted. This was so, despite his lust for and lewd designs towards her being
fully manifest. Such circumstances remained equivocal, or "susceptible of double
interpretation," as Justice Recto put in People v Lamahang. Verily, his felony would not
exclusively be rape had he been allowed by her to continue, and to have sexual
congress with her, for some other felony like simple seduction (if he should employ
deceit to have her yield to him) could also be ultimate felony. We clarify that the direct
overt acts of the petitioner that would have produced attempted rape did not include
equivocal preparatory acts. The former would have related to his acts directly
connected to rape as the intended crime, but the latter, whether external or internal,
had no connection with rape as the intended crime. Perforce, his perpetration of the
preparatory acts would not render him guilty of an attempt to commit such felony.

If the acts of the petitioner did not constitute attempted rape, did they constitute acts
of lasciviousness? It is obvious that the fundamental difference between attempted
rape and acts of lasciviousness is the offender's intent to lie with the female. In rape,
intent to lie with the female is indispensable, but this element is not required in acts of
lasciviousness. The information charged that the petitioner "remove[d] her panty and
underwear and la[id] on top of said AAA embracing and touching her vagina and
breast." With such allegation of the information being competently and satisfactorily
proven beyond a reasonable doubt, he was guilty only of acts of lasciviousness, not
attempted rape. His embracing her and touching her vagina and breasts did not
directly manifest his intent to lie with her. The lack of evidence showing his erectile
penis being in the position to penetrate her when he was on top of her deterred any
inference about his intent to lie with her. At most, his acts reflected lewdness and lust
for her. The intent to commit rape should not easily be inferred against the petitioner,
even from his own declaration of it, if any, unless he committed overt acts directly
leading to rape.

Fallo:
WHEREFORE, the Court FINDS and PRONOUNCES petitioner NORBERTO CRUZ y
BARTOLOME guilty of ACTS OF LASCIVIOUSNESS, and, ACCORDINGLY, PENALIZES
him with the indeterminate sentence of three (3) months of arresto mayor, as the
minimum, to two (2) years, four (4) months and one day of prision correccional, as the
maximum; ORDERS him to pay moral damages of P30,000.00 and civil indemnity of
P20,000.00 to the complainant, with interest of 6% per annum on such awards
reckoned from the finality of this decision until full payment; and DIRECTS him to pay
the costs of suit.

VIVAR, 1
CRIMINAL LAW DIGESTS

Continued Crime <<<

Defensor Santiago v. Justice Garchitorena


G.R. No. 109266. December 2, 1993
QUIASON, J:

Doctrine:
For delito continuado to exist there should be a plurality of acts performed during a
period of time; unity of penal provision violated; and unity of criminal intent or
purpose, which means that two or more violations of the same penal provisions are
united in one and the same intent or resolution leading to the perpetration of the same
criminal purpose or aim. A delito continuado consists of several crimes but in
reality there is only one crime in the mind of the perpetrator.

Facts:
Petitioner Miriam Defensor-Santiago, the then the Commission of Immigration and
Deportation (CID) Commissioner, was charged in Criminal Case No. 16698 of the
Sandiganbayan with violation of Section 3(e) of R.A. No. 3019, as amended, otherwise
known as the Anti-Graft and Corrupt Practices Act, allegedly committed by her favoring
"unqualified" aliens with the benefits of the Alien Legalization Program wherein Santiago,
approved the application for legalization of the stay of about 32 aliens who arrived in
the Philippines in violation of Executive Order No. 324 which does not allow the
legalization of the same, thereby causing undue injury to the government and giving
unwarranted benefits and advantages to said aliens in the discharge of the official and
administrative functions of said accused.

She filed a petition for certiorari and prohibition to enjoin the Sandiganbayan from
proceeding with Criminal Case No. 16698 on the ground that said case was intended
solely to harass her as she was then a presidential candidate. She also moved to inhibit
Sandiganbayan Presiding Justice Garchitorena from the case and to defer her
arraignment pending action on her motion to inhibit. Her motion was denied by the
Sandiganbayan.

Santiago filed a motion for a bill of particulars stating that while the information
alleged that she had approved the application for legalization of "aliens" and gave
them indirect benefits and advantages it lacked a list of the favored aliens. According
to her, unless she was furnished with the names and identities of the aliens, she could
not properly plead and prepare for trial.

She contended in this case that the public prosecutors filed 32 Amended Informations
against her, after manifesting to the Sandiganbayan that they would only file one
amended information. She also questioned in her opposition to the motion to admit
the 32 Amended Informations, the splitting of the original information.

She even claimed that the Amended Informations filed against her did not charge any
offense punishable under Section 3 (e) of R.A. No. 3019 because the official acts
complained of therein were authorized under EO 324 and that the Board of
Commissioners of the Bureau of Investigation adopted the policy of approving
applications for legalization of spouses and unmarried, minor children of "qualified
aliens" even though they had arrived in the Philippines after December 31, 1983. She
concludes that the Sandiganbayan erred in not granting her motion to quash the
informations.

Issue:
Whether or not there was only one crime that was committed in Santiago’s case and
hence, there should only be one information to be filed against her. (YES)

Ruling:
Technically, there was only one crime that was committed in petitioner Santiago's case,
and hence, there should only be one information to be filed against her.

VIVAR, 1
CRIMINAL LAW DIGESTS

The 32 Amended Informations charge what is known as delito continuado or


"continued crime" and sometimes referred to as "continuous crime."

Where only one single criminal act of approving the application for legalization of
32 aliens was committed on the same period of time, the 32 informations should
be consolidated into only one. Under the following circumstances, the 32 informations
filed by the prosecution should be consolidated into only one information. In the case
at bench, the original information charged petitioner Santiago with performing a single
criminal act — that of her approving the application for legalization of aliens not
qualified under the law to enjoy such privilege. The original information also averred
that the criminal act: (i) committed by petitioner was in violation of a law—Executive
Order No. 324 dated April 13, 1988, (ii) caused an undue injury to one offended party,
the Government, and (iii) was done on a single day, i.e., on or about October 17, 1988.
The 32 Amended Informations reproduced verbatim the allegation of the original
information, except that instead of the word “aliens” in the original information each
amended information states the name of the individual whose stay was legalized.

The 32 Amended Informations aver that the offenses were committed on the same period
of time, i.e., on or about October 17, 1988. The strong probability even exists that the
approval of the application for the legalization of the stay of the 32 aliens was done by
a single stroke of the pen, as when the approval was embodied in the same document.

For delito continuado to exist there should be a plurality of acts performed during a
period of time; unity of penal provision violated; and unity of criminal intent or
purpose, which means that two or more violations of the same penal provisions are
united in one and the same intent or resolution leading to the perpetration of the same
criminal purpose or aim. A delito continuado consists of several crimes but in
reality there is only one crime in the mind of the perpetrator.

The concept of delito continuado, although an outcrop of the Spanish Penal Code, has
been applied to crimes penalized under special laws. Under Article 10 of the Revised
Penal Code, the Code shall be supplementary to special laws, unless the latter provide
the contrary. Hence, legal principles developed from the Penal Code may be applied in
a supplementary capacity to crimes punished under special laws. The question
surrounding the concept of delito continuado is that whether a series of criminal acts
over a period of time creates a single offense or separate offenses.

At the hearing of the motion for a bill of particulars, the public prosecutors manifested
that they would file only one amended information embodying the legalization of stay of
the 32 aliens.

Hence, in this case, the Office of the Special Prosecutor of the Office of the Ombudsman
is directed to consolidate the 32 Amended Informations (Criminal Cases Nos.
18371 to 18402) into one information charging only one offense under the original
case number, i.e., No. 16698.

Fallo:
WHEREFORE, the Resolution dated March 3, 1993 in Criminal Case No. 16698 of the
Sandiganbayan (First Division) is AFFIRMED and its Resolution dated March 11, 1993
in Criminal Case No. 16698 is MODIFIED in the sense that the Office of the Special
Prosecutor of the Office of the Ombudsman is directed to consolidate the 32 Amended
Informations (Criminal Cases Nos. 18371 to 18402) into one information charging
only one offense under the original case number, i.e., No. 16698. The temporary
restraining order issued by this Court on March 25, 1993 is LIFTED insofar as to the
disqualification of Presiding Justice Francis Garchitorena is concerned.

VIVAR, 1
CRIMINAL LAW DIGESTS

Consummated Rape- No Frustrated Rape <<<

People v. Orita
G.R. No. 88724. April 3,
1990 MEDIALDEA, J:

Doctrine:
ART. 335 When and how rape is committed - Rape is committed by having carnal
knowledge of a woman under any of the following circumstances:
1. By using force or intimidation;
2. When the woman is deprived of reason or otherwise unconscious; and
3. When the woman is under twelve years of age, even though neither of the
circumstances mentioned in the two next preceding paragraphs shall be present.

Facts:
 Orita, a Philippine Constabulary Officer soldier (accused) was charged with the crime
rape before the RTC of Borongan, Eastern Samar.
 Inside a boarding house, Orita, with lewd designs and by the use of a Batangas
knife, conveniently provided himself for the purpose and with threats and
intimidation, willfully, unlawfully and feloniously lay with and succeeded in having
sexual intercourse with Abayan (19y/o freshman student), against her will and
without her consent.

Prosecution:
 Abayan came from a party. She knocked at the door of her boarding house when all
of a sudden, Orita held her and poked a knife to her neck. She recognized Orita as
a frequent visitor of another boarder. Orita ordered her to enter her room, take off
her clothes, lie down on the floor and mount her. Orita could not fully penetrate
her. In a different position where only a small part of his penis was inserted in her
vagina, Abaya thought of escaping. She dashed out to the next room, to another
until she reached the municipal building naked and the policemen found her.

Lower Court(s):
 The RTC charged Orita with the crime of rape.
 On appeal, Orita alleged that the trial court erred in declaring that the crime of
frustrated rape was committed by him.

Issue:
Whether or not Orita shall be convicted for frustrated rape?

Ruling:
NO.
 Correlating Art. 6 and Art. 335 of the RPC, there is no debate that the attempted
and consummated stages apply only to the crime of rape.
 The requisites of a frustrated felony are: (1) that the offender has performed all the
acts of execution which would produce the felony and (2) that the felony is not
produced due to cases independent of the perpetrator’s will.
 In an attempted crime the purpose of the offender must be thwarted by a foreign
force or agency which intervenes and compels him to stop prior to the moment
when he has performed all of the acts which should produce the crime as a
consequence, which acts it is his intention to perform. If he has performed all of the
acts which should result in the consummation of the crime and voluntarily desists
from proceeding further, it can not be an attempt.
 In the crime of rape, from the moment the offender has carnal knowledge of his
victim he actually attains his purpose and, from that moment also all the essential
elements of the offense have been accomplished. Any penetration of the female organ
by the male organ is sufficient. Entry of the labia or lips of the female organ,
without rupture of the hymen or laceration of the vagina is sufficient to warrant
conviction. Necessarily, rape is attempted if there is no penetration of the female
organ.

VIVAR, 1
CRIMINAL LAW DIGESTS

Fallo:
ACCORDINGLY, the decision of the Regional Trial Court is hereby MODIFIED. The
accused Ceilito Orita is hereby found guilty beyond reasonable doubt of the crime of
rape and sentenced to reclusion perpetua as well as to indemnify the victim in the
amount of P30,000.00.

VIVAR, 1
CRIMINAL LAW DIGESTS

Attempted Rape <<<

People v. Campuhan
G.R. No. 129433. March 30, 2000
BELLOSILLO, J:

Doctrine:
Rape is attempted when the offender commences the commission of rape directly by
overt acts, and does not perform all the acts of execution which should produce the
crime of rape by reason of some cause or accident other than his own spontaneous
desistance.

Facts:
 At around 4pm, Corazon, the mother of 4 y/o Crsythel, went down from the 2nd
floor of their house to prepare milo for her 2 children. At the ground floor, he met
Campuhan (the helper ofhis brother), who was busy filling plastic bags with water
to be frozen into ice. As Corazon was busy preparing the drinks, she heard one of
her daughters cry “ayoko, ayoko!” which prompted her to rush upstairs. She saw
Campuhan inside Crysthel’s room kneeling before her child whose pajamas and
panty were already removed, while his shorts were down to his knees. According
to Corazon, he was forcing his penis into Crysthel’s vagina. She cursed Camphuan
and boxed him several times. She shouted for help when Campuhan ran out,
prompting her brother, cousin and uncle to chase Campuhan and was
apprehended. They held Campuhan until the brgy officials detained him.
 Physical examination of the victim yielded negative results. No evident sign of
extra- genital physical injury was noted.

Defendant:
 Campuhan maintained his innocence and assailed the charge as a mere scheme of
Corazon’s ill will against him for his refusal to run an errand for her.
 He asserted that Chrystel was in a playing mood and wanted to ride him on the
back. She suddenly pulled him down causing them to fall down on the floor.

Lower Court(s):
 Campuhan was convicted of the crime of statutory rape and was sentenced to the
extreme penalty of death. Hence, the automatic review.

Issue:
Whether or not Campuhan shall be guilty of consummated statutory rape?

Ruling:
NO.
 Campuhan is guilty of attempted rape. Under Art. 6, in relation to Art. 335, of the
Revised Penal Code, rape is attempted when the offender commences the
commission of rape directly by overt acts, and does not perform all the acts of
execution which should produce the crime of rape by reason of some cause or
accident other than his own spontaneous desistance. All the elements of attempted
rape 4 and only of attempted rape 4 are present in the instant case, hence, the
accused should be punished only for it.
 Primo's kneeling position rendered an unbridled observation impossible. Crysthel
made a categorical statement denying penetration but her vocabulary is yet as
under developed. Corazon narrated that Primo had to hold his penis with his right
hand, thus showing that he had yet to attain an erection to be able to penetrate his
victim.
 The possibility of Primo's penis having breached Crysthel's vagina is belied by the
child's own assertion that she resisted Primo's advances by putting her legs close
together and that she did not feel any intense pain but just felt "not happy" about
what Primo did to her. Thus, she only shouted "Ayo'ko, ayo'ko!" not "Aray ko, aray
ko!
 There is no medical basis to hold that there was sexual contact between the
accused and the victim.

VIVAR, 1
CRIMINAL LAW DIGESTS

Fallo:

WHEREFORE, the Decision of the courta quo finding accused PRIMO "SONNY"
CAMPUHAN Y BELLO guilty of statutory rape and sentencing him to death and to pay
damages is MODIFIED. He is instead found guilty of ATTEMPTED RAPE and sentenced
to an indeterminate prison term of eight (8) years four (4) months and ten (10) days of
prision mayor medium as minimum, to fourteen (14) years ten (10) months and twenty
(20) days of reclusion temporal medium as maximum. Costs de oficio.

VIVAR, 1
CRIMINAL LAW DIGESTS

Three Stages of Felony <<<


People v. Listerio
G.R. No. 122099. July 5, 2000
YNARES-SANTIAGO, YNARES-SANTIAGO, J:

Doctrine:
It is not the gravity of the wounds inflicted which determines whether a felony is
attempted or frustrated but whether or not the subjective phase in the commission of
an offense has been passed.

Case Summary:
Marlon and Jeonito were collecting money. They were blocked by a group of people
(w/ accused Listerio). Jeonito was stabbed from behind, and died. Marlon was hit by a
lead pipe on is head. RTC convicted Listerio for Attempted Homicide for the injuries
sustained by Marlon. SC ruled that Listerio should be guilty of Frustrated Homicide.

Facts:
 Listerio was charged with Murder and frustrated Murder for the killing of Jeonito
Aranque.
 Jeonito and his brother Marlon were collecting money from a certain Tino. They were
not able to collect the money so they went home. On their way home (near Tramo),
Listerio and 2 others blocked their path and attacked them with lead pipes and
bladed weapons. Listerio stabbed Jeonito from behind. Marlon was struck with a
lead pipe on his head so he lost consciousness. When he woke up, Jeonito was
alredy dead.
 Listerio contends that he went home and slept. He was awoken by a friend when
there was a commotion near their place (Tramo). Policemen passing by his house
invited him to the police station and was implicated for the death of Jeonito.
 There were doctors who testified to the wounds sustained by both Jeonito and
Marlon.
 Dr. Manimtim testified that the wounds sustained by Marlon were not fatal.
 RTC RULING: Convicted Listerio of Murder for the death of Jeonito, and Attempted
Homicide for the injury sustained by Marlon. The 2 others escaped prison.

Issue:
Whether or not Listerio is guilty of attempted homicide for the injuries sustained by
Marlon – NO, he is guilty of frustrated homicide.

Ruling:
RTC convicted Listerio of Attempted Homicide only on the basis of Dr. Manimtim’s
testimony that none of the wounds sustained by Marlon Araque were fatal.
 The reasoning of the lower court on this point is flawed because it is not the
gravity of the wounds inflicted which determines whether a felony is
attempted or frustrated but whether or not the subjective phase in the
commission of an offense has been passed.

Subjective phase = That portion of the acts constituting the crime included between
the act which begins the commission of the crime and the last act performed by the
offender which, with the prior acts, should result in the consummated crime.
 From that time forward, the phase is objective.
 It may also be said to be that period occupied by the acts of the offender over
which he has control—that period between the point where he begins and the
point where he voluntarily desists.
 If between these two points the offender is stopped by reason of any cause outside
of his own voluntary desistance, the subjective phase has not been passed and it
is an attempt.
 If he is not so stopped but continues until he performs the last act, it is frustrated.

IN THIS CASE, Listerio and others stopped hitting Marlon and fled the scene because
they thought that Marlon was already dead after he was hit twice in the head.

VIVAR, 1
CRIMINAL LAW DIGESTS

IT IS FRUSTRATED WHEN:
 the offender has performed all the acts of execution which would
produce the felony;
 the felony is not produced due to causes independent of the perpetrator’s
will.

IT IS ATTEMPTED WHEN:
 the offender commits overt acts to commence the perpetration of the crime;
 he is not able to perform all the acts of execution which should produce
the felony; and
 his failure to perform all the acts of execution was due to some cause or
accident other than his spontaneous desistance

The essential element which distinguishes attempted from frustrated felony is that, in
the latter (frustrated), there is no intervention of a foreign or extraneous cause or agency
between the beginning of the commission of the crime and the moment when all the
acts have been performed which should result in the consummated crime.

In relation to the foregoing, it bears stressing that intent to kill determines whether
the infliction of injuries should be punished as attempted or frustrated murder,
homicide, parricide or consummated physical injuries.

 Homicidal intent must be evidenced by acts which at the time of their execution
are unmistakably calculated to produce the death of the victim by adequate
means. Suffice it to state that the intent to kill of the malefactors herein who were
armed with bladed weapons and lead pipes can hardly be doubted given the
prevailing facts of the case.

On the issue that Marlon’s testimony is uncorroborated: It is well settled that witnesses
are to be weighed, not numbered, such that the testimony of a single, trustworthy and
credible witness could be sufficient to convict an accused. In here, Marlon was seen as
a credible witness. His testimony was candid and straightforward.

 The trial judge is the best and the most competent person who can weigh and
evaluate the testimony of witnesses.
 Marlon’s credibility cannot be doubted in this case because as a victim himself
and an eyewitness to the incident, it can be clearly gleaned from the foregoing
excerpts of his testimony that he remembered with a high degree of reliability
the identity of the malefactors.

Fallo:
WHEREFORE, the appealed decision is AFFIRMED with the following MODIFICATIONS:

1. the award of P5,000.00 to Marlon Araque by way of moral damages in Criminal


Case No. 91-5843 is DELETED;
2. Accused-Appellant is found GUILTY beyond reasonable doubt in Criminal Case
No. 91-5843 of Frustrated Homicide and is sentenced to suffer an indeterminate
penalty of Six (6) Years of Prision Correccional, as minimum to Ten (10) Years
and One (1) Day of Prision Mayor, as maximum.

After finality of this Decision, the records shall be remanded to the Regional Trial Court
of Makati City, which is directed to render judgment based on the evidence against
Samson dela Torre y Esquela.

VIVAR, 1
CRIMINAL LAW DIGESTS

Conspiracy <<<
People v. Pagalasan
G.R. Nos. 131926 & 138991. June 18,
2003 CALLEJO, SR., J:

Doctrine:
Conspirators are necessarily liable for the acts of another conspirator even though
such act differs radically and substantively from that which they intended to commit.

Facts:
Spouses George and Desiree Lim and their three young children, one of whom was 10-
year-old Christopher Neal Lim, resided at Villa Consuelo Subdivision, General Santos
City. The spouses hired a security guard, Ferdinand Cortez. When all of their family
was at home, Michael Pagalasan together with 3 other men wearing bonnets over their
faces and armed with handguns and grenades ransacked their house getting cash and
valuables. With them was Ferdinand Cortez, whose hands were tied behind his back.
The masked men kidnapped George and Christopher using George’s Nissan Car, with
Pagalasan as the driver. Before leaving, they gave Desiree a handwritten note (First
Handwritten Letter) warning them not to cooperate with the military, not to take any
action concerning the kidnapping without their consent and made it clear to the couple
that only those communications, whether by letter or by telephone, bearing the name
MR. MUBARAK II or 2 came from them. While in transit, Christopher was transferred
to another place. In the meantime, the police was informed of the kidnapping incident.
George was then rescued during the police check point.

In the custodial investigation, Michael made an extra judicial confession that upon
orders of a certain Ronnie Cabalo, he, Ferdinand Cortez, Boy and Aladin, kidnapped
Christopher. In the light of Michael’s confession, farmer Hadji Aladin Malang Cabalo,
Ronie Puntuan and Fernando Quizon were arrested and detained at Camp Fermin Lira
Barracks, General Santos City. In the meantime, on September 6, 1994, George
received the second handwritten letter, ordering the release of Michael and Ronie
Puntuan because they were innocent, and demanding P3,000,000 for Christophers
release. On Sept. 9, 1994, George received the third handwritten letter, informing him
and his wife that the kidnappers did not want the military to be involved nor innocent
people to be prejudiced. The spouses were also warned that their son would not be
released alive unless Ronie Puntuan was freed in three days. Fortunately, Christopher
was rescued by the policemen without any ransom being paid.

MTC: Found probable cause against Michael, together with the other accused, with
kidnapping for ransom and violation of PD 1866 before the Municipal Trial Court
(MTC) of General Santos City.

RTC: The Trial court rendered judgment acquitting Ferdinand Cortez and convicting
Michael of kidnapping for ransom.

Defense: Michael averred that was forced at gunpoint by Boy and Aladin to barge into
the Lim residence and drive the latter’s car. Ferdinand Cortez denied kidnapping
George and Christopher.

Issue:
1. WON there is a conspiracy committed between Pagalasan and his cohorts
(Article 8 of the RPC)
2. WON Pagalasan is guilty of kidnapping George and Christopher Lim under
Article 267 of the Revised Penal Code (main issue)

Ruling:
1. Yes, conspiracy between the accused and his cohorts exists.
On conspiracy:

VIVAR, 1
CRIMINAL LAW DIGESTS

 There is conspiracy when two or more persons agree to commit a felony and
decide to commit it. Conspiracy as a mode of incurring criminal liability must be
proven separately from and with the same quantum of proof as the crime itself.
Conspiracy need not be proven by direct evidence. After all, secrecy and
concealment are essential features of a successful conspiracy. Settled as a rule
of law is that the conspiracy continues until the object is attained, unless in the
meantime the conspirator abandons the conspiracy or is arrested. The loner a
conspiracy is deemed to continue, the greater the chances that additional
persons will be found to have joined it.
 Each conspirator is liable is liable for everything that is done by his
confederates which follows incidentally in the execution of a common design as
one of its probable and natural consequence even though it was not intended as
part of the original design.
 In this case, the collective, concerted and synchronized sets of Pagalasan with
his three cohorts before, during and after the kidnapping constitute concrete
proof that he and his companions conspired with each other to attain a common
objective; to kidnap George and Christopher and detain them illegally.
Pagalasan was a principal by his direct participation in the kidnapping of the
two victims.

2. Yes. He is guilty of kidnapping (with no ransom) under Article 267 and guilty of
slight illegal detention of George under Article 268 of the Revised Penal Code.
Guilty of Kidnapping:
 For Christopher (Article 267 Paragraph 4, kidnapping): Pagalasan and others
conspired to kidnap George and Christopher and detained them illegally but
prosecution failed to prove that they intended to extort ransom. (see the 3
letters below). Of the 3 letters only the second letter is asking for ransom and
it is not signed by MUBARAK II or 2. It is possible that it did not come from
the kidnappers or others are acting independently to benefit from the
situation. Even if the letter asking for ransom came from the kidnappers,
Pagalasan‘s conspiracy with them already ended at the time of his arrest.
There is no proof that what is contained in the second and third letters is with
the knowledge and concurrence of Pagalasan.
 For George (Article 268, slight illegal detention): George had been
kidnapped and detained illegally by the appellant and his allies, but only for
less than a day. George regained his freedom after the appellant had been
arrested at the intersection of the national highway and Espina Road. There
is no evidence that the appellant and his allies kidnapped George for the
purpose of extorting ransom for his release. There is likewise no evidence
that they inflicted any serious physical injuries on George, or simulated
public authority, or threatened to kill him. Furthermore, there is no evidence
that the appellant and his allies intended to detain the victim for more than
three days. The appellant is not entitled to the privileged mitigating
circumstance under the second paragraph of Article 268 of the Revised
Penal Code because he did not voluntarily release George within three days
from the kidnapping.

Fallo:
IN LIGHT OF ALL THE FOREGOING, the Decision dated September 27, 1997 of the Regional
Trial Court of General Santos City, Branch 35, is AFFIRMED WITH MODIFICATIONS. Appellant
Michael Pagalasan alias "Mike" is found guilty of kidnapping under Article 267, paragraph 4 of
the Revised Penal Code and there being no modifying circumstances in the commission of the
crime is hereby sentenced to suffer the penalty of reclusion perpetua. Appellant Michael
Pagalasan alias "Mike" is found guilty beyond reasonable doubt of the crime of slight illegal
detention under Article 268 of the Revised Penal Code and there being no modifying
circumstances in the commission of the crime is hereby sentenced to suffer an indeterminate
penalty of from nine years and four months of prision mayor in its medium period as minimum
to sixteen years and five months of reclusion temporal in its medium period as maximum. The
said appellant is ordered to pay to Spouses George and Desiree Lim the total amount of
P150,000 as moral damages; and P50,000 as exemplary damages in the two cases. Costs de
oficio.

VIVAR, 1
CRIMINAL LAW DIGESTS

Conspiracy <<<
People v. Larrañaga
G.R. No. 138874-75. July 13,
2015 SERENO, C.J:

Case Summary:
These cases involve the kidnapping and illegal detention of a college beauty queen
along with her comely and courageous sister. An intriguing tale of ribaldry and gang-
rape was followed by the murder of the beauty queen. She was thrown off a cliff into a
deep forested ravine where she was left to die. Her sister was subjected to heartless
indignities before she was also gang-raped in the aftermath of the kidnapping and rape,
the sister was made to disappear. Where she is and what further crimes were inflicted
upon her remain unknown and unsolved up to the present.

Doctrine:
Conspiracy may be deduced from the mode and manner by which the offense was
perpetrated, or may be inferred from the acts of the accused themselves, when such
point to a joint design and community of interest.

Facts:
On the night of July 16, 1997, victims Marijoy and Jacqueline Chiong failed to come
home on the expected time. Two days after, a young woman was found dead at the foot
of a cliff. Her pants were torn, her t-shirt was raised up to her breast and her bra was
pulled down. Her face and neck were covered with masking tape and attached to her left
wrist was a handcuff. The woman was identified as Marijoy. After almost ten months,
accused Davidson Rusia surfaced and admitted before the police having participated in
the abduction of the sisters. He identified appellants Francisco Juan Larrañ aga, Josman
Aznar, Rowen Adlawan, Alberto Cañ o, Ariel Balansag, James Anthony Uy, and James
Andrew Uy as co-perpetrators in the crime. Rusia provided the following before the trial
court:

1. That at 10:30 in the evening of July 16, 1997, he met Rowen and Josman and
told him to ride with them in a white car. Following them were Larrañ aga,
James Anthony and James Andrew who were in a red car. Josman stopped the
white car in front of the waiting shed where the sisters Marijoy and Jacqueline
were standing and forced them to ride the car. Rusia taped their mouths while
Rowen handcuffed them jointly.
2. That after stopping by a safe house, the group thereafter headed to the South
Bus Terminal where they met Alberto and Ariel, and hired the white van driven
by the former. They traveled towards south of Cebu City, leaving the red car at
the South Bus Terminal.
3. That after parking their vehicles near a precipice, they drank and had a pot
session. Later, they started to rape Marijoy inside the vehicle, and thereafter
raped Jaqueline.
4. That Josman instructed Rowen and Ariel to bring Marijoy to the cliff and push
her into the ravine.

The claims of Rusia were supported by other witnesses. He was discharged as an


accused and became a state witness. Still, the body of Jacqueline was never found. The
trial court found the other appellants guilty of two crimes of kidnapping and serious
illegal detention and sentenced each of them to suffer the penalties of two (2)
reclusion perpetua. The appellants assailed the said decision, arguing inter alia, that
court erred in finding that there was conspiracy. James Anthony was also claimed to be
only 16 years old when the crimes were committed.

Issue:
1. Whether or not there was conspiracy.
2. Whether or not the trial court erred in characterizing the crime.
3. Whether or not the trial court erred imposing the correct penalty

VIVAR, 1
CRIMINAL LAW DIGESTS

Ruling:
1. Yes. Conspiracy may be deduced from the mode and manner by which the offense
was perpetrated, or may be inferred from the acts of the accused themselves, when
such point to a joint design and community of interest. The appellants’ actions
showed that they had the same objective to kidnap and detain the Chiong sisters.
The Court affirmed the trial court’s finding that the appellants indeed conspired in
the commission of the crimes charged.

2. Yes. The rule is that when the law provides a single penalty for two or more
component offenses, the resulting crime is called a special complex crime. Article
267 of the Revised Penal Code, as amended by Section 8 of R.A. 7659, provides that
in the crime of kidnapping and serious illegal detention, when the victim is killed or
dies as a consequence of the detention, or is raped or is subjected to torture or
dehumanizing acts, the maximum penalty shall be imposed. Thus, the resulting
crime will change from complex crime to special complex crime. In the present
case, the victims were raped and subjected to dehumanizing acts. Thus, the Court
held that all the appellants were guilty of the special complex crime of kidnapping
and serious illegal detention with homicide and rape in the case where Marijoy is
the victim; and simple kidnapping and serious illegal detention in the case of
Jacqueline.

3. Yes. Article 68 of the Revised Penal Code provides that by reason of minority, the
imposable penalty to the offender is one degree lower than the statutory penalty.
James Anthony was only 16 years old when the crimes were committed. As penalty
for the special complex crime of kidnapping and serious illegal detention with
homicide and rape is death, the correct penalty to be imposed should be reclusion
perpetua. On the other hand, the penalty for simple kidnapping and serious illegal
detention is reclusion perpetua to death. One degree lower from the said penalty is
reclusion temporal. There being no aggravating and mitigating circumstance, the
penalty to be imposed on him should be reclusion temporal in its medium period.
Applying the Indeterminate Sentence Law, he should be sentenced to suffer the
penalty of twelve (12) years of prision mayor in its maximum period, as minimum,
to seventeen (17) years of reclusion temporal in its medium period, as maximum.
With regard to the rest of the appellants, the statutory penalty as provided above
should be imposed. Therefore, trial court erred in merely imposing “two (2) reclusion
perpetua”.

Fallo:
WHEREFORE, the Decision of the Regional Trial Court, Branch 7, Cebu City in Criminal
Cases Nos. CBU-45303 and 45304 is AFFIRMED with the following MODIFICATIONS:
1. In Criminal Case No. CBU-45303, appellants FRANCISCO JUAN LARRAÑ AGA alias
"PACO;" JOSMAN AZNAR; ROWEN ADLAWAN alias "WESLEY;" ALBERTO CAÑ O
alias "ALLAN PAHAK;" ARIEL BALANSAG; and JAMES ANDREW UY alias "MM," are
found guilty beyond reasonable doubt of the special complex crime of kidnapping
and serious illegal detention with homicide and rape and are sentenced to suffer
the penalty of DEATH by lethal injection;
2. In Criminal Case No. CBU-45304, appellants FRANCISCO JUAN LARRAÑ AGA alias
"PACO;" JOSMAN AZNAR; ROWEN ADLAWAN alias "WESLEY;" ALBERTO CAÑ O
alias "ALLAN PAHAK;" ARIEL BALANSAG; and JAMES ANDREW UY alias "MM," are
found guilty beyond reasonable doubt of simple kidnapping and serious illegal
detention and are sentenced to suffer the penalty of RECLUSION PERPETUA;
3. In Criminal Case No. CBU-45303, appellant JAMES ANTHONY UY, who was a minor
at the time the crime was committed, is likewise found guilty beyond reasonable
doubt of the special complex crime of kidnapping and serious illegal detention with
homicide and rape and is hereby sentenced to suffer the penalty of RECLUSION
PERPETUA; in Criminal Case No. CBU-45304, he is declared guilty of simple
kidnapping and serious illegal detention and is sentenced to suffer the penalty of
twelve (12) years of prision mayor in its maximum period, as MINIMUM, to seventeen
(17) years of reclusion temporal in its medium period, as MAXIMUM.
4. Appellants are ordered to pay jointly and severally the heirs of Marijoy and
Jacqueline, in each case, the amounts of (a) P100,000.00 as civil indemnity, (b)
P25,000.00 as temperate damages, (c) P150,000.00 as moral damages, and (d)

VIVAR, 1
CRIMINAL LAW DIGESTS

P100,000.00 as exemplary damages. Three (3) Justices of the Court maintain their
position that RA 7659 is unconstitutional insofar as it prescribes the death penalty;
nevertheless, they submit to the ruling of the majority that the law is constitutional
and the death penalty can be lawfully imposed in the case at bar.

In accordance with Article 83 of The Revised Penal Code, as amended by Section 25 of


RA No. 7659, upon the finality of this Decision let the records of this case be forthwith
forwarded to the Office of the President for the possible exercise of Her Excellency's
pardoning power.

VIVAR, 1
CRIMINAL LAW DIGESTS

Conspiracy <<<
People v. Carandang
G.R. No. 175926. July 6, 2011
LEONARDO-DE CASTRO, J:

Doctrine:
Unlike evident premeditation, there is no requirement for conspiracy to exist that
there be a sufficient period of time to elapse to afford full opportunity for meditation
and reflection. Instead, conspiracy arises on the very moment the plotters agree,
expressly or impliedly, to commit the subject felony.

Facts:
In the afternoon of April 2001, La Loma Police Station received a request from the
sister of accused Milan that there is a drug trade that is happening on their house in
Quezon City. At around 4:00 PM, SPO2 Wilfredo Pilar (Red) along with Police Officer 2
Dionisio Alonzo, SPO1 Estores, and SPO1 Montecalvo went to Milan’s house and
surrounded the area. The door of the house was open, enabling the police officers to
see Carandang, Milan, and Chua inside.

PO2 Alonzo and SPO2 Red pushed the door open, and shouted “Walang gagalaw!”.
They are instantly shot and failed to return fire causing their instant death. SPO1
Montecalvo fell on the ground, SPO1 Estores heard Chua say to Milan “Sugurin mo na!”.
Milan lunged Montecalvo but failed to maul him because the police officer was able to
fire his gun to Milan. Thereafter, Estores went inside the house to pull Montecalvo
out.

RTC found the three guilty beyond reasonable doubt of the crime of murder. Likewise,
they were also found guilty of attempted murder in Relation to Article 6 par 2, having
been acted in conspiracy. CA affirmed the decision.

Accused appealed to SC, arguing that the court a quo erred in holding that there is
conspiracy among the appellants.

Issue:
Whether or not accused-appelants are guilty of conspiracy to commit murder against
SPO1 Montefalco.

Ruling:
YES. Conspiracy exists when two or more persons come to an agreement concerning
the commission of a felony and decide to commit it. Evidence need not establish the
actual agreement among the conspirators showing a preconceived plan or motive for the
commission of the crime. Proof of concerted action before, during and after the crime, which
demonstrates their unity of design and objective, is sufficient. When conspiracy is
established, the act of one is the act of all regardless of the degree of participation of
each.

In the case at bar, the conclusion that Milan and Chua conspired with Carandang was
established by their acts (1) before Carandang shot the victims (Milans closing the
door when the police officers introduced themselves, allowing Carandang to wait in
ambush), and (2) after the shooting (Chuas directive to Milan to attack SPO1
Montecalvo and Milans following such instruction). These facts are convincing
circumstantial evidence of the unity of purpose in the minds of the three. As co-
conspirators, all three are considered principals by direct participation.

Neither can the rapid turn of events be considered to negate a finding of conspiracy.
Unlike evident premeditation, there is no requirement for conspiracy to exist that
there be a sufficient period of time to elapse to afford full opportunity for meditation
and reflection. Instead, conspiracy arises on the very moment the plotters agree,
expressly or impliedly, to commit the subject felony.

Fallo:

VIVAR, 1
CRIMINAL LAW DIGESTS

Conspiracy <<<
People v. Octa
G.R. No. 195196. July 13, 2015
SERENO, C.J:

Doctrine:
While Octa’s receipt of the ransom money was not a material element of the crime, it
was nevertheless part of the grand plan and was in fact the main reason for
kidnapping the victims. The taking of the ransom money was an overt act made in
pursuance or furtherance of the complicity making Octa liable not just as a mere
accomplice but a principal for being a co-conspirator in the crime of Kidnapping for
Ransom.

Facts:
 On September 25, 2003, around 6:40 A.M Johnny Corpuz and Mike Batuigas while
travelling on Buenos Aires St., Sampaloc, Manila on board a silver Honda Civic Car
was blocked by a red-orange Mitsubishi box type Lancer car.
 The four (4) armed occupants of the Lancer car alighted and one of them fired his
pistol thus compelling Johnny to open the locked door of the car. The armed men
went inside the car, then handcuffed, blindfolded and even boxed Johnny.
 The kidnappers communicated with Johnny’s wife Ana Marie Corpuz giving the
information that they have in their custody her husband Johnny and her brother
Mike. Ana sought the assistance of the PACER [Police Anti-Crime and Emergency
Response].
 The kidnappers initially demanded ₱20 million but were considerably reduced to
₱538,000.00.
 On September 30, 2003, the kidnappers asked Ana to deliver the money to a man
wearing a redcap at the nearby Caltex Auto Supply in Fairview.
 Ana described the man wearing a red cap to be good looking, lightly built, in his
early 20s, around 5’4" in height and with dimples, which she later identified in
court as accused Estanly Octa. On October 1, 2003, after 6 days of detention, Johnny
and Mike were released by their captors.
 On 4 December 2003, Octa was arrested in connection with another kidnap for
ransom incident. He was identified by prosecution witness Ana from a police line-up
as the person who had received the ransom money from her.
 Consequently, Information was filed against Octa charging him with the crime of
kidnapping for ransom. Octa, for his defense, interposed an alibi that on September
25 he was still in Camarines Norte and that he himself was a victim of abduction.

CRIME CHARGED: Kidnapping for ransom

RTC: Finds the accused guilty of the crime charged. The trial court viewed the act of
receiving ransom money as sufficient evidence to establish Octa’s conspiratorial act in
the kidnapping for ransom of the victims. With respect to the defense of denial and alibi,
the RTC found them to be weak as opposed to the straightforward testimony of Ana.

CA: Affirmed RTC’s decision. The CA ruled that accused-appellant had been rightly
found to be a co-conspirator in this case. At the time he received the ransom money,
the crime of kidnapping was still continuing, Octa’s act of taking the ransom money
was an overt act made in pursuance or furtherance of the complicity.

Hence this present petition. Octa claims that he cannot be considered as a conspirator
since his receipt of the ransom money transpired only after the kidnapping had been
consummated and was not an essential element of the crime.

Issue:
Whether or not Octa is a conspirator to the crime of kidnapping for ransom.

VIVAR, 1
CRIMINAL LAW DIGESTS

Ruling:
Yes. The CA is correct in its observation that at the time the accused-appellant received
the ransom money, the crime of kidnapping was still continuing, since both victims
were still being illegally detained by the kidnappers. While Octa’s receipt of the ransom
money was not a material element of the crime, it was nevertheless part of the grand
plan and was in fact the main reason for kidnapping the victims. Ransom is money,
price or consideration paid or demanded for the redemption of a captured person or
persons; or payment that releases them from captivity. Without ransom money, the
freedom of the detained victims cannot be achieved. Evidently, to hold an accused
guilty as a co- principal by reason of conspiracy, he must be shown to have performed
an overt act in pursuance or furtherance of the complicity. There must be intentional
participation in the transaction with a view to the furtherance of the common design
and purpose. The commonality of purpose of the acts of Octa together with the other
accused can no longer be denied. Such acts have the common design or purpose to
commit the felony of kidnapping for ransom. Thus, the accused-appellants' argument
that he is a mere accomplice must fail. He is liable as a principal for being a co-
conspirator in the crime of Kidnapping for Ransom under Art. 267 of the RPC, as
amended by R.A. 7659.

Fallo:
WHEREFORE, the appeal is hereby DISMISSED. The assailed Decision of the Court of
Appeals in CA-G.R. CR.-HC No. 03490 is AFFIRMED WITH MODIFICATION. Accused-
appellant is hereby sentenced to suffer the penalty of reclusion perpetua and ordered
to pay P538,000 as actual damages, P100,000 as moral damages, and P100,000 as
exemplary damages.

VIVAR, 1
CRIMINAL LAW DIGESTS

Conspiracy <<<
People v. Feliciano
G.R. No. 196735. May 5,
2014 LEONEN, J:

Doctrine:
The moment it is established that the malefactors conspired and confederated
in the commission of the felony proved, collective liability of the accused conspirators
attaches by reason of the conspiracy, and the court shall not speculate nor even
investigate as to the actual degree of participation of each of the perpetrators present
at the scene of the crime.

Facts:
 On December 8, 1994, seven (7) members of the Sigma Rho fraternity were eating
lunch at the Beach House Canteen of the UP Diliman, when they were attacked by
several masked men carrying baseball bats and lead pipes. Some of them sustained
injuries that required hospitalization. One of the Sigma Rho members, Dennis
Venturina, died from his injuries. An information for murder was filed against
several members of the Scintilla Juris fraternity, who wore masks and/or other
forms of disguise, conspiring, confederating with other persons whose true names,
identities and whereabouts have not as yet been ascertained, and mutually helping
one another, with intent to kill, qualified with treachery, and with evident
premeditation, taking advantage of superior strength, armed with baseball bats,
lead pipes, and cutters, did then and there willfully, unlawfully and feloniously
attack, assault and employ personal violence upon the person of Dennis Venturina.
Separate informations were also filed against them for the attempted murder and
frustrated murder of Sigma Rho fraternity members.

 One of the members (Arnel Fortes) ran but when he looked back, he saw Danilo
Feliciano Jr hitting Venturina. He was also able to see Warren Zipangan and George
Morano at the scene.

 The Sigma Rho counsel, Atty. Frank Chavez, told the UP Police that the victims will
give their statements to the NBI. Meanwhile, Venturina was transferred from the
UP Infirmary to St.Luke’s Hospital. He died after two days.

Defense: The accused provided different alibis, each claiming that they were not at the
scene of the crime.

RTC:
 “Although each victim had a very strong motive to place his fraternity rivals
permanently behind bars, not one of them testified against all of them. If the
prosecution eyewitnesses, who were all Sigma Rhoans, were simply bent on
convicting Scintilla Juris members for that matter, they could have easily tagged
each and every single accused as a participant.. each eye witness named only one or
two and some were candid enough to say that they did not see who delivered the
blows against them.”
 All of the accused-appellants were found guilty of the murder of Dennis Venturina
and the attempted murder of 5 victims.
 Because one of the penalties meted out was reclusion perpetua, the case was
brought to this court on automatic appeal. However, due to the amendment of the
Rules on Appeal, the case was remanded to the Court of Appeals.

CA: Affirmed the RTC decision, but modified the liabilities and found that the accused-
appellants were guilty of attempted murder only against 2 members (Natalicio and
Flores), and NOT against the other 3 (Mangrobang, Lachica, and Gaston). It is the
appellate court's reasoning that because Lachica and Mangrobang "were no longer
chased by the attackers," it concluded that accused-appellants "voluntary desisted
from pursuing them and from inflicting harm to them, which shows that they did not
have the intent to do more than to make them suffer pain by slightly injuring them."
It also

VIVAR, 1
CRIMINAL LAW DIGESTS

pointed out that the wound inflicted on Gaston "was too shallow to have been done
with an intent to kill." Thus, it concluded that the accused-appellants would have been
guilty only of slight physical injuries.

Issue:
Whether or not the accussed-appellants should be treated as co-conspirators.

Ruling:
YES. Once an express or implied conspiracy is proved, all of the conspirators are liable
as co-principals regardless of the extent and character of their respective
active participation in the commission of the crime or crimes perpetrated
in furtherance of the conspiracy because in contemplation of law the act of one
is the act of all. Verily, the moment it is established that the malefactors conspired and
confederated in the commission of the felony proved, collective liability of the accused
conspirators attaches by reason of the conspiracy, and the court shall not speculate
nor even investigate as to the actual degree of participation of each of the perpetrators
present at the scene of the crime. The appellate court, therefore, erred in finding the
accused-appellants guilty only of slight physical injuries. It would be illogical to
presume that despite the swiftness and suddenness of the attack, the attackers
intended to kill only Venturina, Natalicio, and Fortes, and only intended to injure
Lachica, Mangrobang, and Gaston. Since the intent to kill was evident from the moment
the accused-appellants took their first swing, all of them were liable for that intent to
kill. Accused-appellants should be liable for the murder of Dennis Venturina and the
attempted murder of Mervin Natalicio, Cesar Mangrobang, Jr., Leandro Lachica, Arnel
Fortes, and Cristobal Gaston, Jr.

Fallo:
WHEREFORE, the decision of the Court of Appeals in CA-G.R. CR No. 01158 dated
November 26, 2010 is AFFIRMED insofar as the accusedappellants Danilo Feliciano, Jr.,
Julius Victor Medalla, Christopher Soliva, Warren L. Zingapan, and Robert Michael
Beltran Alvir are found GUILTY beyond reasonable doubt of Murder in Criminal Case
No. Q95-61133 with the MODIFICATION that they be found GUILTY beyond
reasonable doubt of Attempted Murder in Criminal Case Nos. Q95-61136, Q95-61135,
Q95- 61134, Q95-61138, and Q95-61137.

VIVAR, 1
CRIMINAL LAW DIGESTS

Conspiracy <<<
People vs. Bokingco
G.R. No. 187536. August 10, 2011
PEREZ, J:

Doctrine:
Conspiracy exists when two or more persons come to an agreement to commit an
unlawful act. It may be inferred from the conduct of the accused before, during, and
after the commission of the crime. Conspiracy may be deduced from the mode and
manner in which the offense was perpetrated or inferred from the acts of the accused
evincing a joint or common purpose and design, concerted action, and community of
interest. Unity of purpose and unity in the execution of the unlawful objective are
essential to establish the existence of conspiracy.

Facts:
The victim, Noli Pasion (Pasion) and his wife, Elsa, owned a pawnshop, which formed
part of his house. He also maintained two (2) rows of apartment units at the back of
his house. A certain Vitalicio was spin-drying his clothes and he heard a commotion
from Apartment No. 3. He headed to said unit to check and saw Bokingco hitting
something on the floor. Upon seeing Vitalicio, Bokingco allegedly attacked him with a
hammer in his hand. Vitalicio was hit several times. Vitalicio bit Bokingco’s neck and
managed to push him away. Bokingco tried to chase Vitalicio but was eventually
subdued by a co- worker. Pasion was found dead in the kitchen of Apartment No. 3.

Elsa testified that she was in the master’s bedroom on the second floor of the house
when she heard banging sounds and her husband’s moans. She immediately went
down. Before reaching the kitchen, Col blocked her way. Col suddenly sprayed tear gas
on her eyes and poked a sharp object under her chin. Col then instructed her to open
the vault of the pawnshop but Elsa informed him that she does not know the
combination. Col dragged her towards the back door by holding her neck. Before they
reached the door, Elsa saw Bokingco open the screen door and heard him tell Col: "tara,
patay na siya."Col immediately let her go and ran away with Bokingco.

Bokingco testified that he was sleeping when he was awakened by Pasion who
appeared to be intoxicated. The latter wanted to know why he did not see Bokingco at
the construction site. When Bokingco replied that he just stayed at the apartment,
Pasion suddenly hit him in the head. This prompted Bokingco to take a hammer and
hit Pasion repeatedly. Col alleged that he went home to Cainta, Rizal, where he was
apprehended and brought to Camp Olivas. Bokingco pointed to him as the person who
killed Pasion. He insisted that he doesn’t know Bokingco very well. On preliminary
investigation, Bokingco admitted that he conspired with Col to kill Pasion and that
they planned the killing several days before because they got "fed up" with Pasion.

Lower Court(s):
RTC: the accused MICHAEL BOKINGO and REYNANTE COL guilty beyond reasonable
doubt of the crime of MURDER, and there being the two aggravating circumstances of
nighttime and abuse of confidence to be considered against both accused and the
mitigating circumstance of voluntary plea of guilty in favor of accused Bokingo only,
hereby sentences each of them to suffer the penalty of DEATH.

CA: affirmed the findings of the trial court but reduced the penalty to reclusion perpetua.

Issue:
Whether or not appellant Col is guilty beyond reasonable doubt as a co-conspirator.

Ruling:
No. In order to convict Col as a principal by direct participation in the case before us, it
is necessary that conspiracy between him and Bokingco be proved. Conspiracy exists
when two or more persons come to an agreement to commit an unlawful act. It may be
inferred from the conduct of the accused before, during, and after the commission of

VIVAR, 1
CRIMINAL LAW DIGESTS

the crime. Conspiracy may be deduced from the mode and manner in which the offense
was perpetrated or inferred from the acts of the accused evincing a joint or common
purpose and design, concerted action, and community of interest. Unity of purpose and
unity in the execution of the unlawful objective are essential to establish the existence
of conspiracy.

As a rule, conspiracy must be established with the same quantum of proof as the crime
itself and must be shown as clearly as the commission of the crime.

The finding of conspiracy was premised on Elsa's testimony that appellants fled
together after killing her husband and the extrajudicial confession of Bokingco.
Nobody witnessed the commencement of the attack. Col was not seen at the apartment
where Pasion was being attacked by Bokingco. In fact, he was at Elsa's house and
allegedly ordering her to open the pawnshop vault.

Based on these acts alone, it cannot be logically inferred that Col conspired with
Bokingco in killing Pasion. At the most, Col's actuations can be equated to attempted
robbery, which was actually the initial information filed against appellants before it
was amended, on motion of the prosecution, for murder.

Elsa testified that she heard Bokingco call out to Col that Pasion had been killed and
that they had to leave the place. This does not prove that they acted in concert towards
the consummation of the crime. It only proves, at best, that there were two crimes
committed simultaneously and they were united in their efforts to escape from the
crimes they separately committed.

Their acts did not reveal a unity of purpose that is to kill Pasion. Bokingco had already
killed Pasion even before he sought Col. Their moves were not coordinated because while
Bokingco was killing Pasion because of his pent-up anger, Col was attempting to rob
the pawnshop.

In as much as Bokingco's extrajudicial confession is inadmissible against him, it is


likewise inadmissible against Col, specifically where he implicated the latter as a
cohort.

Fallo:
WHEREFORE, the appeal is GRANTED. The Decision of the Court of Appeals in CA-G.R.
CR-H.C. No. 00658 is REVERSED and SET ASIDE. Appellant Reynante Col is
ACQUITTED on ground of reasonable doubt. The Bureau of Corrections is ordered to
cause the immediate release of accused-appellant, unless he is being lawfully held for
another cause, and to inform this Court of action taken within ten (10) days from
notice.

Appellant Michael Bokingco is found GUILTY beyond reasonable doubt of the crime of
Homicide. He is hereby sentenced to suffer the penalty of six years (6) and one (1) day
of prision mayor as minimum to 14 years, eight (8) months and one (1) day of reclusion
temporal, as maximum Appellant is further ordered to indemnify the heirs of Noli Pasion
in the amount of Seventy five thousand pesos (P75,000.00); Fifty thousand pesos
(P50,000.00) as moral damages; Twenty five thousand pesos (P25,000.00) as temperate
damages; Fifteen thousand pesos (P15,000.00) as attorney's fees; and to pay the costs.

VIVAR, 1
CRIMINAL LAW DIGESTS

Conspiracy <<<
People v. Castillo
G.R. No. 187536. August 10, 2011
PER CURIAM:

Doctrine:
Conspiracy is established by the presence of two factors: (1) singularity of intent; and
(2) unity in execution of an unlawful objective. The two must concur. Performance of an
act that contributes to the goal of another is not enough.

Facts:
On March 1, 1995, an Evangeline Padayhag fetched Rocky De Guzman from his home
at BF Homes, Paranaque from his maid, Rosanna. Evangeline brought Rocky to a
nearby and thereat, Elizabeth Castillo and proceeded to a house in Caloocan.

The maid was made to believe that Rocky will attend to a birthday party. Later in the
afternoon, spouses De Guzman found out that their son was kidnapped when a
Elizabeth called Luiz De Guzman, Rocky’s father, and demanded his ATM card in
exchange of his son. The day after, Elizabeth changed her mind and demanded 1
million pesos . Evangeline and Luis agreed to half the amount as ransom money. Luis
was then instructed to drop the bag containing the ransom money on a Sabadista
Chruch in Obando, Bulacan. On March 5, Rocky was released and retured home via
tricycle.

Police investigation ensued and later discovered that Elizabeth was already in Dipolig
City. RTC charged Evangline Padayhag and Elizabeth Castillo guilty beyond reasonable
doubt of kidnapping and was sentened to suffer the extreme penalty of death. CA
affirmed the decision.

Issue:
Whether or not there was a conspiracy between Elizabeth and Evangelince to extort
money.

Ruling:
NO. There must be positive and conclusive evidence that Padayhag acted in concert
with Castillo to commit the same criminal act. To hold an accused guilty as a co-
principal by conspiracy, there must be a sufficient and unbroken chain of events that
directly and definitely links the accused to the commission of the crime without any
space for baseless suppositions or frenzied theories to filter through. Indeed,
conspiracy must be proven as clearly as the commission of the crime itself.

Conspiracy is established by the presence of two factors: (1) singularity of intent; and
(2) unity in execution of an unlawful objective. The two must concur. Performance of an
act that contributes to the goal of another is not enough.

The act must be motivated by the same unlawful intent. Neither joint nor simultaneous
action is per se sufficient indicium of conspiracy, unless proved to have been
motivated by a common design.

Padayhags act of fetching Rocky is not conclusive proof of her complicity with Castillos
plan, a plan Padayhag did not even know. Both appellants testified that Padayhag met
Castillo only because Castillo told Padayhag that Padayhags boyfriend was sick. It was
precisely on the pretext that they were to visit Padayhags boyfriend that the two met.
When they met, Padayhag realized that Castillo had deceived her.

Neither is there any hint that Castillo told Padayhag to abduct the boy, or to
misrepresent herself or use means that would have led Padayhag to suspect that Castillo
had some criminal design. Nor was there any proof that Padayhag knew that Castillo
had no permission from the boys parents. The appearance of the boy itself, newly bathed
and dressed for a stroll, would have led Padayhag to believe whatever story Castillo
contrived to ask her in fetching the boy.

VIVAR, 1
CRIMINAL LAW DIGESTS

Padayhags acts before, during and after the crime all point to the conclusion that she
was no more than an unwitting tool of Castillo. Castillo misled her into a meeting.
Castillo again misled her into fetching Rocky. Castillo never met or contacted her after
the day of Rockys abduction.

Nothing links Padayhag to the demand for ransom. She never received any part of the
ransom, precisely because she did not even know it existed.

Decision convicting appellant Evangeline Padayhag is REVERSED. We ACQUIT


Evangeline Padayhag and order her immediate RELEASE.

Fallo:
WHEREFORE, the Decision of the Regional Trial Court of Parañ aque, Branch 260,
National Capital Judicial Region, in Criminal Case No. 95-86 convicting appellant
Elizabeth Castillo is AFFIRMED with MODIFICATION. Appellant Elizabeth Castillo is
sentenced, to suffer the penalty of DEATH and to pay the victim P100,000 as moral
damages. The award for exemplary damages is deleted for lack of legal basis. The trial
court's Decision convicting appellant Evangeline Padayhag is REVERSED. We ACQUIT
Evangeline Padayhag and order her immediate RELEASE from confinement unless
held for another lawful cause. The Director of the Bureau of Corrections is ordered to
report to the Court, within five days from notice, compliance with this Decision.

In accordance with Article 83 of the Revised Penal Code, as amended by Section 25 of


Republic Act No. 7659, upon finality of this decision, let certified true copies of the
records of this case be forwarded to the President of the Philippines for the possible
exercise of the pardoning power.

VIVAR, 1
CRIMINAL LAW DIGESTS

Conspiracy <<<
Fernan Jr, et al vs. People
G.R. No. 145927. August 24, 2007
VELASCO, JR., J:

Doctrine:
Where the acts of each of the accused constitute an essential link in a chain and the
desistance of even one of them would prevent the chain from being completed, then no
conspiracy could result as its consummation would then be impossible or aborted . But
when each and everyone of the accused in the instant cases performed their assigned
tasks and roles with martinet-like precision and accuracy, by individually performing
essential overt acts, so much so that the common objective is attained, which is to
secure the illegal release of public funds under the guise of fake or simulated public
documents, then each and everyone of said accused are equally liable as co-principals
under the well-established and universally-accepted principle that, once a conspiracy
is directly or impliedly proven, the act of one is the act of all and such liability exists
notwithstanding no-participation in every detail in the execution of the offense.

Facts:
 86 million highway scam; 119 criminal cases filed with the Sandiganbayan
involving no less than 36 former officials and employees of the then Ministry of
Public Highways and several suppliers of construction material for defalcation of
public funds arising from numerous transactions in the Cebu First Highway
Engineering District.
 COA Regional Director directed its auditors to verify and submit a report on sub-
allotment advice issued to various highway engineering districts in Cebu City, Cebu
1st, Cebu 2nd and Mandaue City Highway Engineering Districts.
 In their findings, they confirmed the issuance of fake letters of Advice of Allotments
(LAA)and discovered that 2 sets were received by the districts. One set were duly
signed and the other set consists of fake LAAs.
 Due to the irregularities, then President Marcos created a special cabinet
committee “Ghost Projects Anomalies”, to conduct a wider and more extended
investigation in all 15 highway engineering districts. The questionable
disbursements of which are the subject matter.
 Accused Mangubat, Preagido, Sayson and Cruz came up with a plan to steal large
sums of money from the government treasury. Mangubat had found a way to
withdraw government money through the use of fake LAAs, vouchers and other
documents, and to conceal traces thereof with the connivance of other government
officials and employees.
 The four formed the nucleus of the nefarious conspiracy. Other government
employees, tempted by the prospect of earning big money, allowed their names to be
used and signed spurious documents.
 On the other hand, petitioners were both Civil Engineers of the MPH assigned to the
Cebu First Highway Engineering District. Petitioner Fernan, Jr. was included among
the accused, allegedly for having signed six (6) tally sheets or statements of deliveries
of materials, used as bases for the preparation of the corresponding number of
general vouchers. Fund releases were made to the suppliers, contractors, and
payees based on these general vouchers.
 On the other hand, petitioner Torrevillas was one of the accused in 9 criminal cases.

Sandiganbayan Ruling:
 Petitioners were charged with the complex crime of estafa through falsification of
public documents. The anti-graft court was fully convinced of the guilt of petitioner
Fernan, Jr. Petitioner Torrevillas suffered the same fate and was convicted in the
nine (9) criminal cases.

Issue:
Whether or not petitioners are liable as co-conspirators of the crimes committed?

VIVAR, 1
CRIMINAL LAW DIGESTS

Ruling:
 YES. Petitioners’ guilt was established beyond reasonable doubt.
 On the part of petitioners, they readily admitted that they either signed the tally
sheets and/or delivery receipts, reports of inspection, requests for supplies and
materials, and other related documents which became part of the supporting
documents that led to the issuance of general vouchers and eventually the
disbursement of public funds. The tally sheets are statements of delivery that
purportedly indicate the specified quantities of materials for the construction and
maintenance of roads that have been delivered on supposed project sites on given
dates at specific places.
 The Court finds no reason to disturb the findings of the court that all the essential
elements of the crime of estafa through falsification of public documents were
present. There is no question that petitioners, at the time of the commission of the
crime, were public officers and civil engineers assigned to the MPH. Their signing of
tally sheets and related documents pertaining to the alleged deliveries of supplies
for road repair and construction constitutes intervention and/or taking advantage
of their official positions, especially considering that they had the duty to inspect
the purported deliveries and ascertain the veracity of the documents and the
statements contained in them.
 Indeed, the burden of proving the allegation of conspiracy falls to the shoulders of
the prosecution. Considering, however, the difficulty in establishing the existence of
conspiracy, settled jurisprudence finds no need to prove it by direct evidence. The
Court finds that the conspiracy in the instant cases resembles the wheel conspiracy.
The 36 disparate persons who constituted the massive conspiracy to defraud the
government were controlled by a single hub, namely: Rolando Mangubat (Chief
Accountant), Delia Preagido (Accountant III), Jose Sayson (Budget Examiner), and
Edgardo Cruz (Clerk II), who controlled the separate spokes of the conspiracy.
Petitioners were among the many spokes of the wheel After a close re-examination
of the records, the Court finds no reason to disturb the finding of the anti-graft
court that petitioners are co-conspirators of the other accused, headed by Chief
Accountant Rolando Mangubat, who were similarly convicted in practically all the
119 counts of estafa. Undisturbed is the rule that this Court is not a trier of facts
and in the absence of strong and compelling reasons or justifications, it will accord
finality to the findings of facts of the SB. The feeble defense of petitioners that they
were not aware of the ingenuous plan of the group of accused Mangubat and the
indispensable acts to defraud the government does not merit any consideration.
The State is not tasked to adduce direct proof of the agreement by petitioners with
the other accused, for such requirement, in many cases, would border on near
impossibility. The State needs to adduce proof only when the accused committed
acts that constitute a vital connection to the chain of conspiracy or in furtherance of
the objective of the conspiracy.
 In the case at bench, the signing of the fake tally sheets and/or delivery receipts,
reports of inspection, and requests for supplies and materials by petitioners on
separate occasions is vital to the success of the Mangubat Group in siphoning off
government funds. Without such fabricated documents, the general vouchers
covering the supply of materials cannot be properly accomplished and submitted
to the disbursing officer for the preparation of checks.
 In sum, the required quantum of proof has been adduced by the State on the
conspiracy among the accused including petitioners. The conviction of petitioners
must be sustained.

Fallo:
WHEREFORE, we DENY the petition and AFFIRM the December 4, 1997 Decision of the
SB in the consolidated criminal cases subject of this petition. No costs.

VIVAR, 1
CRIMINAL LAW DIGESTS

Conspiracy in BP 22 <<<

Ladonga vs. People


GR 141066, February 17, 2005
AUSTRIA-MARTINEZ, J:

Doctrine:
B.P. Blg. 22 does not provide any prohibition regarding the applicability in a suppletory
character of the provisions of the Revised Penal Code to it.

Facts:
Accused Evangeline Ladonga was charged with her husband Adronico Ladonga violation
of BP 22 for issuing checks on closed accounts.

Conspiring and knowing fully well that they did not have sufficient funds deposited with
the United Coconut Planters Bank (UCPB), drew and issue UCPB Check No. 284743
post dated July 7, 1990 in the amount of P9,075.55), payable to Alfredo Oculam, and
thereafter, without informing the latter that they did not have sufficient funds deposited
with the bank to cover up the amount of the check, did then and there willfully,
unlawfully and feloniously pass on, indorse, give and deliver the said check to Alfredo
by way of rediscounting of the aforementioned checks; however, upon presentation of
the check to the drawee bank for encashment, the same was dishonored for the reason
that the account of the accused had already been closed, to the damage and prejudice
of Alfredo.

RTC found her husband guilty and the accused as a co-conspirator of the said crime.
Accused appeal tothe CA contending that BP 22 has no provision on conspiracy. CA
affirmed the decision of the RTC stating that as the BP 22 do not prohibit the
application of RPC in supplementary.

Conspiracy to commit violation of BP 22 is valid. Hence this case.

Issue:
Whether or not conspiracy is applicable in violations of Batas Pambansa Bilang 22, by
invoking art.10 of RPC?

Ruling:
Yes. Some provisions of the Revised Penal Code, especially with the addition of the
second sentence in Article 10, are applicable to special laws. It submits that B.P. Blg.
22 does not provide any prohibition regarding the applicability in a suppletory character
of the provisions of the Revised PenalCode to it.

Article 10 of the RPC reads as follows: ART. 10. Offenses not subject to the provisions of
this Code. 3Offenses which are or in the future may be punishable under special laws are
not subject to the provisions of this Code. This Code shall be supplementary to such laws,
unless the latter should specially provide the contrary.

The article is composed of two clauses. The first provides that offenses which in the
future are made punishable under special laws are not subject to the provisions of the
RPC, while the second makes the RPC supplementary to such laws.

Fallo:
WHEREFORE, the instant petition is GRANTED. The assailed Decision, dated May 17,
1999, of the Court of Appeals in CA-G.R. CR No. 20443 affirming the Decision, dated
August 24, 1996, of the Regional Trial Court (Branch 3), Bohol, in Criminal Case Nos.
7068, 7069 and 7070 convicting the petitioner of violation of B.P. Blg. 22 is hereby
REVERSED and SET ASIDE. Petitioner Evangeline Ladonga is ACQUITTED of the
charges against her under B.P. Blg. 22 for failure of the prosecution to prove her guilt
beyond reasonable doubt. No pronouncement as to costs.

VIVAR, 1
CRIMINAL LAW DIGESTS

Dangerous Drugs Act and RPC <<<

People vs. Simon


G.R. No. 93028. July 29, 1994
REGALADO, J:

Doctrine:
Dangerous Drugs Act and RPC

Facts:
Accused Martin Simon was charged with a violation of Section 4, Article II of Republic
Act No. 6425 or the Dangerous Drugs Act of 1972. He sold tea bags of marijuana to a
Narcotics Command (NARCOM) poseur-buyer. The confiscated 4 tea bags, weighing a
total of 3.8 grams, when subjected to laboratory examination, were found positive for
marijuana.

Simon denied the accusation against him, claiming that on the day of question, he was
picked up by the police at their house while watching TV. He was told that he was a
pusher so he attempted to alight from the jeep but he was handcuffed instead. When
they finally reached the camp, he was ordered to sign some papers and, when he refused,
he was boxed in the stomach eight or nine times by Sgt. Pejoro. He was then compelled
to affix his signature and fingerprints on the documents presented to him. He denied
knowledge of the marked money or the 4 teabags of dried marijuana leaves and insisted
that the marked money came from the pocket of Pejoro. Moreover, the reason why he
vomited blood was because of the blows he suffered at the hands of Pejoro.

Dr. Evelyn Gomez-Aguas, a resident physician of Romana Pangan District Hospital,


declared that she treated appellant for three days due to abdominal pain, but her
examination revealed that the cause for this ailment was appellant’s peptic ulcer. She
did not see any sign of slight or serious external injury, abrasion or contusion on his
body.

Simon was sentenced to suffer the penalty of life imprisonment, to pay a fine of twenty
thousand pesos and to pay the costs.

Simon then seek the reversal of the judgement.

Issue:
1. Whether or not the conviction of Simon correct?

Ruling:
YES.
To sustain a conviction for selling prohibited drugs, the sale must be clearly and
unmistakably established. To sell means to give, whether for money or any other
material consideration. It must, therefore, be established beyond doubt that appellant
actually sold and delivered two tea bags of marijuana dried leaves to Sgt. Lopez, who
acted as the poseur-buyer, in exchange for two twenty-peso bills.

After careful review, the Court held that there were 2 tea bags of marijuana that was
sold and there were 2 other tea bags of marijuana confiscated. Thus, Simon should be
charged of selling for the 2 tea bags of marijuana only.

However, there is an overlapping error in the provisions on the penalty of reclusion


perpetua by reason of its dual imposition, that is, as the maximum of the penalty where
the marijuana is less than 750 grams, and also as the minimum of the penalty where
the marijuana involved is 750 grams or more. The same error has been committed with
respect to the other prohibited and regulated drugs provided in said Section 20. To
harmonize such conflicting provisions in order to give effect to the whole law, the court
hereby hold that the penalty to be imposed where the quantity of the drugs involved is
less than the quantities stated in the first paragraph shall range from prision
correccional to reclusion temporal, and not reclusion perpetua. This is also concordant

VIVAR, 1
CRIMINAL LAW DIGESTS

with the fundamental rule in criminal law that all doubts should be construed in a
manner favorable to the accused.

The court held that Republic Act No. 6425, as now amended by Republic Act No. 7659,
has unqualifiedly adopted the penalties under the Revised Penal Code in their
technical terms, hence with their technical signification and effects. In fact, for
purposes of determining the maximum of said sentence, the court have applied the
provisions of the amended Section 20 of said law to arrive at prision correccional and
Article 64 of the Code to impose the same in the medium period. Such offense,
although provided for

Fallo:
ACCORDINGLY, under all the foregoing premises, the judgment of conviction rendered
by the court a quo against accused-appellant Martin Simon y Sunga is AFFIRMED, but
with the MODIFICATION that he should be, as he hereby is, sentenced to serve an
indeterminate penalty of six (6) months of arresto mayor, as the minimum, to six (6)
months ofarresto mayor, as the minimum, to six (6) years of prision correccional, as
the maximum thereof.

VIVAR, 1
CRIMINAL LAW DIGESTS

VAWC and Conspiracy <<<

Go-Tan vs. Tan


GR 168852, September 30, 2008
AUSTRIA-MARTINEZ, J:

Doctrine:
Parents-in-law may be held liable for VAWC applying the provision of RPC on conspiracy.

Facts:
Sharica (go-tan/petitioner) and steven (tan) were married. After 6 years of marriage,
sharica filed a petition for Temporary protective order against Steven and her parents- in-
law before the RTC. She alleged that Steven, in conspiracy with respondents, were causing
verbal, psychological and economic abuses upon her in violation of Section 5, paragraphs
(e)(2)(3)(4), (h)(5), and (i) 7 of Republic Act (R.A.) No. 9262 otherwise known as the “Anti
Violence against Women and Their Children Act of 2004.” RTC granted the TPO.
Respondents filed an MTD with opposition to the issuance of TPO contending that RTC had
no jurisdiction over their person as parent-in-laws because they are not covered by RA
9262. RTC granted the MTD following the principle of “expressio unius est exclusio
alterius.”

MR was filed by petitioner sharica ontending that the doctrine of necessary


implication should be applied inthe broader interests of substantial justice and due
process. Respondents answered and argued that petitioner’s liberal construction
unduly broadened the provisions of R.A. No. 9262 since the relationship between the
offender and the alleged victim was an essential condition for the application of R.A.
No. 9262. RTC denied MR.

Hence this appeal to the SC. Petitioner sharica contends that R.A. No. 9262 must be
understood in the light of the provisions of Section 47 of R.A. No. 9262 which explicitly
provides for the suppletory application of the Revised Penal Code (RPC) and, accordingly,
the provision on “conspiracy” under Article 8 of the RPC can be suppletorily applied to
R.A. No. 9262.

On the other hand, respondents submit that they are not covered by R.A. No. 9262
since Section 3 thereof explicitly provides that the offender should be related to the
victim only by marriage, a former marriage, or a dating or sexual relationship.

Issue:
Whether or not respondents-spouses perfecto & juanita, parents in law of sharica, may
be included in the petition for the issuance of a protective order, in accordance with
republic act no. 9262, otherwise known as the “antiviolence against women and their
children act of 2004.”

Ruling:
Yes. The Court rules in favor of the petitioner.
Section 3 of R.A. No. 9262 defines ‘‘[v]iolence against women and their children’’ as
“any act or a series of acts committed by any person against a woman who is his wife,
former wife, or against a woman with whom the person has or had a sexual or dating
relationship, or with whom he has a common child, or against her child whether
legitimate or illegitimate, within or without the family abode, which result in or is
likely to result in physical, sexual, psychological harm or suffering, or economic abuse
including threats of such acts, battery, assault, coercion, harassment or arbitrary
deprivation of liberty.” While the said provision provides that the offender be related
or connected to the victim by marriage, former marriage, or a sexual or dating
relationship, it does not preclude the application of the principle of conspiracy under
the RPC. Indeed, Section 47 of R.A. No. 9262 expressly provides for the suppletory
application of the RPC, thus: “SEC. 47. Suppletory Application.—For purposes of this
Act, the Revised Penal Code and other applicable laws, shall have suppletory
application.” (Emphasis supplied)

VIVAR, 1
CRIMINAL LAW DIGESTS

Hence, legal principles developed from the Penal Code may be applied in a
supplementary capacity to crimes punished under special laws, such as R.A. No. 9262,
in which the special law is silent on a particular matter.

FURTHER, it must be noted that Section 5 of R.A. No. 9262 expressly recognizes that
the acts of violence against women and their children may be committed by an
offender through another, thus: “SEC. 5. Acts of Violence Against Women and Their
Children. — The crime of violence against women and their children is committed
through any of the following acts: x x x (h) Engaging in purposeful, knowing, or
reckless conduct, personally or through another, that alarms or causes substantial
emotional or psychological distress to the woman or her child.

Fallo:
WHEREFORE, the instant petition is GRANTED. The assailed Resolutions dated March
7, 2005 and July 11, 2005 of the Regional Trial Court, Branch 94, Quezon City in Civil
Case No. Q-05-54536 are hereby PARTLY REVERSED and SET ASIDE insofar as the
dismissal of the petition against respondents is concerned.

VIVAR, 1
CRIMINAL LAW DIGESTS

Absence of Unlawful Aggression; Battered Wife Syndrome <<<

People v. Genosa
G.R. No. 135981, January 15,
2004 PANGANIBAN, J:

Doctrine:
Unlawful aggression presupposes actual, sudden and unexpected attack or an
imminent danger (See RPC Art. 11); Existence of the syndrome in a relationship does
not in itself establish the legal right of the woman to kill her abusive partner.

Case Summary:
Marivic Genosa (Marivic) attacked and wounded his husband (Ben), which ultimately
led to his death. According to Marivic, she did not provoke her husband when she got
home that night it was her husband who began the provocation. Also, that she was
frightened that her husband would hurt her and she wanted to make sure she would
deliver her baby safely. In fact, Marivic had to be admitted later at the Rizal Medical
Centre as she was suffering from eclampsia and hypertension, and the baby was born
prematurely. During their marriage she had tried to leave her husband at least five (5)
times, but that Ben would always follow her and they would reconcile. Marivic alleges
that the reason why Ben was violent and abusive towards her that night was because
'he was crazy about his recent girlfriend, Lulu Rubillos. After being interviewed by
specialists, Marivic has been shown to be suffering from Battered Woman Syndrome.
Marivic, with a plea of self-defense admitted the killing of her husband, was found
guilty of Parricide, with the aggravating circumstance of treachery, for the husband
was attacked while asleep. The Supreme Court affirmed her conviction.

Facts:
A. Facts Leading to the Automatic Review:
a. Before the killing of Ben Genosa, by his wife Marivic Genosa, Marivic. Marivic and
Ben lived happily in their first year of marriage but Ben changed and the couple
would always quarrel and sometimes their quarrels became violent.
b. Marivic testified that every time her husband came home drunk, he would
provoke her and sometimes beat her. Whenever beaten by her husband, she
consulted medical doctors who testified during the trial.
c. On the night of the killing, Marivic and the victim were quarreled and the victim
beat the Marivic. However, Marivic was able to run to another room. Marivic
admitted having killed the victim with the use of a gun. The information for
parricide against Marivic, however, alleged that the cause of death of the victim
was by beating through the use of a lead pipe.
d. Marivic invoked self defense and defense of her unborn child. After trial, the
Regional Trial Court found Marivic guilty beyond reasonable doubt of the crime
of parricide with an aggravating circumstance of treachery and imposed the
penalty of death.
e. On automatic review before the Supreme Court, Marivic filed an URGENT
OMNIBUS MOTION praying that the Honorable Court allow (1) the exhumation
of Ben Genosa and the re-examination of the cause of his death; (2) the
examination of Marivic Genosa by qualified psychologists and psychiatrists to
determine her state of mind at the time she killed her husband; and finally, (3)
the inclusion of the said experts’ reports in the records of the case for purposes
of the automatic review or, in the alternative, a partial re-opening of the case a
quo to take the testimony of said psychologists and psychiatrists.
f. Supreme Court partly granted the URGENT OMNIBUS MOTION of Marivic. It
remanded the case to the trial court for reception of expert psychological
and/or psychiatric opinion on the “buttered woman syndrome” plea.
Testimonies of two expert witnesses on the “battered woman syndrome”, Dra.
Dayan and Dr. Pajarillo, were presented and admitted by the trial court and
subsequently submitted to the Supreme Court as part of the records.

B. Ruling of the Trial Court

VIVAR, 1
CRIMINAL LAW DIGESTS

Finding the theory of self-defense untenable, RTC gave credence to prosecution


evidence that Marivic had killed the deceased while he was in bed sleeping.
Further, RTC appreciated the generic aggravating circumstance of treachery,
because Ben Genosa was supposedly defenseless when he was killed lying in
bed asleep when Marivic smashed him with a pipe at the back of his head. The
capital penalty having been imposed, the case was elevated to this Court for
automatic review.

Issue:
Whether or not Marivic Genosa acted in self-defense – No

Ruling:

Self-Defense and Defense of a Fetus


Marivic admits killing Ben Genosa but, to avoid criminal liability, invokes self-defense
and/or defense of her unborn child. When the accused admits killing the victim, it is
incumbent upon her to prove any claimed justifying circumstance by clear and
convincing evidence. Self-defense (and similarly, defense of a stranger or third person)
shifts the burden of proof from the prosecution to the defense.

BWS as Self-Defense
Existence of the syndrome in a relationship does not in itself establish the legal
right of the woman to kill her abusive partner. Evidence must still be considered in
the context of self-defense. The Court reckons further that crucial to the BWS defense
is the state of mind of the battered woman at the time of the offense she must have
actually feared imminent harm from her batterer and honestly believed in the need to
kill him in order to save her life.

The rule is that the one who resorts to self-defense must face a real threat on one’s life;
and the peril sought to be avoided must be imminent and actual, not merely
imaginary. Thus, the Revised Penal Code provides the following requisites and effect of
self-defense:

Art. 11. Justifying circumstances. The following do not incur any criminal liability: (1)
Anyone who acts in defense of his person or rights, provided that the following
circumstances concur: First. Unlawful aggression; Second. Reasonable necessity of the
means employed to prevent or repel it; Third. Lack of sufficient provocation on the part
of the person defending himself .

Unlawful aggression presupposes actual, sudden and unexpected attack or an


imminent danger. In the present case, there was a sufficient time interval between the
unlawful aggression of Ben and her fatal attack upon him. She had already been able
to withdraw from his violent behavior and escape to their children’s bedroom. During
that time, he apparently ceased his attack and went to bed. The reality or even the
imminence of the danger he posed had ended altogether. He was no longer in a
position that presented an actual threat on her life or safety. Had Ben still been
awaiting Marivic when she came out of their children’s bedroom and based on past
violent incidents, there was a great probability that he would still have pursued her
and inflicted graver harm then, the imminence of the real threat upon her life would
not have ceased yet.

Where Self-defense may be appreciated (considering circumstances and existence of BWS)


Where the brutalized person is already suffering from BWS, further evidence of actual
physical assault at the time of the killing is not required. Incidents of domestic battery
usually have a predictable pattern. Torequire the battered person to await an obvious,
deadly attack before she can defend her life would amount to sentencing her to murder
by installment. Still, impending danger (based on the conduct of the victim in previous
battering episodes) prior to the defendant’s use of deadly force must be shown.
Threatening behavior or communication can satisfy the required imminence of danger.

Considering such circumstances and the existence of BWS, self-defense may be

VIVAR, 1
CRIMINAL LAW DIGESTS

appreciated. We reiterate the principle that aggression, if not continuous, does not

VIVAR, 1
CRIMINAL LAW DIGESTS

warrant self-defense. In the absence of such aggression, there can be no self-defense


complete or incomplete on the part of the victim.
.
Thus Marivic’s killing of Ben was not completely justified under the circumstances.

Mitigating Circumstances Present


The mitigating factors of psychological paralysis and passion and obfuscation were,
however, taken in favor of Marivic. It should be clarified that these two circumstances
-
- psychological paralysis as well as passion and obfuscation -- did not arise from the
same set of facts. The first circumstance arose from the cyclical nature and the severity
of the battery inflicted by the batterer-spouse upon Marivic.

That is, the repeated beatings over a period of time resulted in her psychological
paralysis, which was analogous to an illness diminishing the exercise of her will power
without depriving her of consciousness of her acts.

As to the extenuating circumstance of having acted upon an impulse so powerful as to


have naturally produced passion and obfuscation, it has been held that this state of
mind is present when a crime is committed as a result of an uncontrollable burst of
passion provoked by prior unjust or improper acts or by a legitimate stimulus so
powerful as to overcome reason. To appreciate this circumstance, the following
requisites should concur: (1) there is an act, both unlawful and sufficient to produce
such a condition of mind; and (2) this act is not far removed from the commission of
the crime by a considerable length of time, during which the accused might recover
her normal equanimity.

Fallo:
WHEREFORE, the conviction of Marivic Genosa for parricide is hereby AFFIRMED.
However, there being two(2) mitigating circumstances and no aggravating circumstance
attending her commission of the offense, her penalty is REDUCED.

Inasmuch as Marivic has been detained for more than the minimum penalty hereby
imposed upon her, the director of the Bureau of Corrections may immediately RELEASE
her from custody upon due determination that she is eligible for parole, unless she is
being held for some other lawful cause.

VIVAR, 1
CRIMINAL LAW DIGESTS

Absence of Unlawful Aggression <<<

People v. Regalario
G.R. No. 174483 March 31, 2009
LEONARDO-DE CASTRO, J:

Doctrine:
The moment the inceptive unlawful aggression ceases to exist, the person defending
himself must not kill or wound the aggressor. Self-defense does not justify the
unnecessary killing of an aggressor who is retreating from the fray.
Retaliation is not a justifying circumstance.

Facts:
On the night of February 22, 1997, a public dance and singing contest was held in
Ligao, Albay. There was a commotion in the area assigned to accused Ramon
Regalario. When he approached the group where the disturbance was taking place,
Rolando Sevilla suddenly emerged from the group and fired a shot at him.
Instinctively, and in order to disable Sevilla from firing more shots, he struck his
assailant with his nightstick and hit him at the back of his head.

Sotero arrived and Ramon told him that Rolando still had the gun. So, Sotero plunged
at Rolando and they wrestled on the ground for the possession of the gun. Ramon
knocked the gun off his hand and it fell near the place where Jose Poblete was
standing. Poblete just arrived at the scene along with Marciano Regalario. Poblete
picked up the gun. He was instructed by Marciano to keep it until it is turned over to
the authorities.

Bienvenido Regalario, the barangay tanod, was instructed by Marciano, the barangay
captain to effect the arrest of Rolando Sevilla for the crime of shooting Ramon. So, he
tied the hands and feet of Rolando Sevilla for fear that he might be able to escape.

On the early morning of February 23, a team of policemen went to Natasan and found
the dead body of Rolando Sevilla.

For automatic review is the decision of the CA which affirmed with modification, an
earlier decision of the RTC, finding accused-appellants Ramon, Marciano, Sotero,
Bienvenido and Noel, all surnamed Regalario guilty of murder.

Issue:
Whether or not the lower court erred in not finding that the deceased was killed in self-
defense and/or defense of relative.

Ruling:
When self-defense is invoked by an accused charged with murder or homicide he
necessarily owns up to the killing but may escape criminal liability by proving that it
was justified and that he incurred no criminal liability therefor. Hence, the three (3)
elements of self-defense, namely: (a) unlawful aggression on the part of the victim; (b)
reasonable necessity of the means employed to prevent or repel the aggression; and (c)
lack of sufficient provocation on the part of the person defending himself, must be
proved by clear and convincing evidence. However, without unlawful aggression, there
can be no self-defense, either complete or incomplete.

By Ramon’s own account, after he was shot, he hit the victim at the back of the latter’s
head and he continued hitting the victim who retreated backward. From that moment,
the inceptive unlawful aggression on the part of the victim ceased to exist and the
continuation of the offensive stance of Ramon put him in the place of an aggressor.
There was clearly no longer any danger, but still Ramon went beyond the call of self-
preservation. In People v. Cajurao, SC held:

…The settled rule in jurisprudence is that when unlawful aggression ceases, the defender
no longer has the right to kill or even wound the former aggressor. Retaliation is not a
justifying circumstance. Upon the cessation of the unlawful aggression and the danger or

VIVAR, 1
CRIMINAL LAW DIGESTS

risk to life and limb, the necessity for the person invoking self-defense to attack his
adversary ceases. If he persists in attacking his adversary, he can no longer invoke the
justifying circumstance of self-defense. Self-defense does not justify the unnecessary
killing of an aggressor who is retreating from the fray.

Ramon’s claim of self-defense is further belied by the presence of two (2) stab wounds
on the neck, four (4) lacerated wounds on the head, as well as multiple abrasions and
contusions on different parts of the victim’s body. Indeed, even if it were true that the
victim fired a gun at Ramon, the number, nature and severity of the injuries suffered by
the victim indicated that the force used against him by Ramon and his co-accused was
not only to disarm the victim or prevent him from doing harm to others.

Considering the foregoing, as well as the manner in which the attack against Rolando
was carried out, and the testimonies of the prosecution witnesses positively
identifying the accused-appellants as the assailants, SC concur in the rulings of the CA,
affirming those of the trial court, in (a) disregarding Ramon Regalario’s declaration
that he attacked the victim in self-defense and (b) holding that all the accused-
appellants acted in concert and killed Rolando.

Fallo:
WHEREFORE, the decision of the Court of Appeals dated May 31, 2006 in CA-G.R. CR
No. 01556 is hereby AFFIRMED with the following modifications: (1) the penalty of death
imposed on accused-appellants is lowered to reclusion perpetua without eligibility for
parole; (2) the monetary awards to be paid jointly and severally by accused-appellants
are as follows: P75,000.00 as civil indemnity, P75,000.00 as moral damages and
P30,000.00 as exemplary damages; and (3) interest on all the damages awarded at the
legal rate of 6% from this date until fully paid is imposed.

VIVAR, 1
CRIMINAL LAW DIGESTS

Absence of Unlawful Aggression <<<

People v. Dulin
G.R. No. 171284 June 29, 2015
BERSAMIN, J:

Doctrine:
Retaliation is not the same as self-defense. In retaliation, the aggression that the victim
started already ceased when the accused attacked him, but in self-defense, the
aggression was still continuing when the accused injured the aggressor.

Case Summary:
Batulan initially stabbed Dulin but after grappling, Dulin was able to gain possession
of the weapon. Batulan still pursued the attack and this eventually led to his death due
to various laceration wounds. Dulin claims that what he did was due to self-defense.
The court did not agree because the unlawful aggression, a key element in self-defense,
had already ended when the act of killing Batulan was done.

Facts:
Statement of the Case: That on or about August 22, 1990, in the Municipality of
Tuguegarao, Province of Cagayan, and within the jurisdiction of this Honorable Court,
the said accused, Alfredo Dulin y Narag alias Freddie, armed with a sharp blade(d)
instrument, with intent to kill, with evident premeditation and with treachery did then
and there willfully, unlawfully and feloniously attack, assault and stab one, Francisco
Batulan, inflicting upon him several stab wounds on the different parts of his body
which caused his death.

 Tamayao was on Tamayao Street in Atulayan Norte, Tuguegarao at about 10:00


o’clock in the evening of August 22, 1990 when a young man came running from
the house of Vicente Danao towards the house of Batulan, shouting that his Uncle
Totoy (Batulan) had been stabbed. Tamayao rushed towards Danao’s house, which
was about 30 meters from his own house, and there he saw Dulin stabbing Batulan
who was already prostrate face down. Dulin was on top of Batulan, as if kneeling
with his left foot touching the ground. Dulin was holding Batulan by the hair with
his left hand, and thrusting the knife at the latter with his right hand. Seeing this,
Tamayao ran towards Batulan’s house to inform Estelita Batulan, the victim’s wife
who was his aunt, about the incident. He went home afterwards.

 There has been a long standing grudge between Batulan and Dulin, and of seeing
them fighting in April 1990. He recalled Dulin uttering on two occasions: He will
soon have his day and I will kill him.

 Cabalza, a barangay tanod, was in his house around 10:00 o’clock in the evening of
August 22, 1990 when he heard the commotion in Danao’s house which was facing
his house. It was Carolina, Danao’s daughter, screaming for help. He thus sought
out a fellow barangay tanod. On his return to the scene, he found Batulan at the
door of Danao’s house, with Dulin wielding a sharp pointed instrument, about 6-7
inches long. Fearing for his safety, he rushed to the Barangay Hall to seek the
assistance of Edwin Cabalza and Nanding Buenaflor to bring Batulan to the
Provincial Hospital in Carig, Tuguegarao.

 At the hospital, she was told that her husband had sustained two wounds in the
back and several stab wounds in the front, and was being attended to at the
hospital’s intensive care unit (ICU) before he expired.

 Batulan was attended to at the Cagayan Valley Regional Hospital on August 22, 1990
by Dr. Macaraniag, who said that the victim was in a state of shock from his 12 stab
wounds (lacerations). He died eventually.

 Defenders Side: He was attacked first.

VIVAR, 1
CRIMINAL LAW DIGESTS

 RTC rendered its decision convicting Dulin of murder.


 CA affirmed decision.

Issue:
1. Whether or not Dulin was actual in the act of self-defense. NO
2. Whether or not Dulin’s actions may be considered as an incomplete form of self-
defense? NO

Ruling:
1. NO.
 The CA observed that although Batulan had initiated the attack against Dulin
the unlawful aggression from Batulan effectively ceased once Dulin had
wrested the weapon from the latter.
 Batulan, albeit the initial aggressor against Dulin, ceased to be the aggressor
as soon as Dulin had dispossessed him of the weapon. Even if Batulan still
went after Dulin despite the latter going inside the house of Danao, where
they again grappled for control of the weapon, the grappling for the weapon
did not amount to aggression from Batulan for it was still Dulin who held
control of the weapon at that point.
 Whatever Dulin did thereafter – like stabbing Batulan with the weapon –
constituted retaliation against Batulan. In this regard, retaliation was not the
same as self-defense. In retaliation, the aggression that the victim started
already ceased when the accused attacked him, but in self-defense, the
aggression was still continuing when the accused injured the aggressor. As
such, there was no unlawful aggression on the part of Batulan to justify his
fatal stabbing by Dulin.

2. NO
Like in complete self-defense, Dulin should prove the elements of incomplete
self- defense by first credibly establishing that the victim had committed
unlawful aggression against him. With Batulan’s aggression having already
ceased from the moment that Dulin divested Batulan of the weapon, there
would not be any incomplete self-defense. Moreover, as borne out by his
stabbing of Batulan several times, Dulin did not act in order to defend himself or
to repel any attack, but instead to inflict injury on Batulan.

Fallo:
WHEREFORE, the Court MODIFIES the judgment promulgated on August 26, 2005 by
finding ALFREDO DULIN YNARAG guilty beyond reasonable doubt of HOMICIDE, and
SENTENCES him to suffer the indeterminate sentence of EIGHT YEARS AND ONE DAY
OF PRISION MAYOR, AS THE MINIMUM, TO 14 YEARS, EIGHT MONTHS AND ONE
DAY OF RECLUSION TEMPORAL, with full credit of his preventive imprisonment;
ORDERS him to pay to the heirs of Francisco Batulan ₱50,000.00 as civil indemnity,
₱50,000.00 as moral damages, and ₱25,000.00 as temperate damages, plus interest of
6% per annum on each item reckoned from the finality of this decision until full
payment; and DIRECTS him to pay the costs of suit.

NOTES:
The test for the presence of unlawful aggression under the circumstances are:
(a) there must be a physical or material attack or assault;
(b) the attack or assault must be actual, or, at least, imminent; and
(c) the attack or assault must be unlawful.

Unlawful aggression is of two kinds:


(a) actual or material unlawful aggression; and
(b) imminent unlawful aggression

VIVAR, 1
CRIMINAL LAW DIGESTS

Absence of reasonable necessity of means employed <<<

People v. Fontanilla
G.R. No. 177743 January 25,
2012 BERSAMIN, J:

Doctrine:
Elements of unlawful aggression are: 1. There must be physical or material attack or
assault; 2. The attack or assault must be actual or at least imminent; 3. The attack or
assault must be unlawful.

Unlawful aggression is of two kinds: 1. Actual or material unlawful aggression – an


attack with physical force or with a weapon, an offensive act that positively
determines the intent of the aggressor to cause the injury; and 2. Imminent unlawful
aggression – an attack that is impending or at the point of happening; it must not
consist in a mere threatening attitude, nor must it be merely imaginary, but must be
offensive and positively strong.

Facts:
Jose Olais was walking along the road when Alfonso Fontanilla suddenly struck
him in the head with a piece of wood called bellang. Olais fell facedown to the ground,
but Fontanilla hit him again in the head with a piece of stone. He only desisted from
hitting Olais a third time because Joel Marquez and Tirso Abunan, the sons-in-law of
Olais, shouted at him, causing him to run away. Marquez and Abunan rushed their
father-in law to a medical clinic, where Olais was pronounced dead on arrival.
Fontanilla was charged with murder, consequently.
At the trial, Fontanilla claimed self-defense and that Olais was drunk.
Prosecution presented the physician who conducted the autopsy on the cadaver of
Olais. She attested that her post-mortem examination showed that Olais had suffered a
fracture on the skull more than once, causing his death.
The RTC rejected Fontanilla’s plea of self-defense by observing that he had “ no
necessity to employ a big stone, inflicting upon the victim a mortal wound causing his
death” due to the victim attacking him only with bare hands. It noted that Fontanilla
did not suffer any injury despite his claim that the victim had mauled him; that
Fontanilla did not receive any treatment, and no medical certificate attested to any
injury he might have suffered, having been immediately released from the hospital.

Issue:
Whether or not alleging by the accused of self-defense may be given credence amidst
gravity of the injury sustained by the victim.

Ruling:
NO
In order for self-defense to be appreciated, he had to prove by clear and convincing
evidence the following elements: (a) unlawful aggression on the part of the victim; (b)
reasonable necessity of the means employed to prevent or repel it; and (c) lack of
sufficient provocation on the part of the person defending himself. Unlawful
aggression is the indispensable element of self-defense, for if no unlawful aggression
attributed to the victim is established, self-defense is unavailing, for there is nothing
to repel.
The plea of self-defense was thus belied, for the weapons used by Fontanilla and
the location and number of wounds he inflicted on Olais revealed his intent to kill, not
merely an effort to prevent or repel an attack from Olais. We consider to be significant
that the gravity of the wounds manifested the determined effort of the accused to kill
his victim, not just to defend himself.

Fallo:
WHEREFORE, we AFFIRM the decision promulgated on June 29, 2006 by the Court of Appeals, subject to
the MODIFICATION of the civil damages, by ordering accused Alfonso Fontanilla y Obaldo to pay to the
heirs of Jose Olais P25,000.00 as temperate damages and P30,000.00 as exemplary damages in addition
to the P50,000.00 as death indemnity and the P50,000.00 as moral damages, plus interest of 6% per
annum on such amounts from the finality of the judgment. The accused shall pay the costs of suit.

VIVAR, 1
CRIMINAL LAW DIGESTS

Absence of Unlawful Aggression <<<

Toledo v. People
G.R. No. 158057 September 24, 2004
CALLEJO, SR., J:

Doctrine:
There is no accidental self-defense, as self-defense is inconsistent with accident.
Because in self-defense, it is direct and positive overt act in the name of self-
preservation. It is direct and positive. It cannot be done out of accident.

Self-defense necessarily implies a deliberate and positive overt act of the accused. The
accused has freedom of action. He is aware of the consequences of his deliberate acts.
On the other hand, the basis of accident is the lack of negligence and intent on the part
of the accused.

Facts:
This is a petition for review of the Decision of the Court of Appeals (CA) affirming on
appeal, the Decision of the Regional Trial Court (RTC) of Odiongan, Romblon, Branch
82 convicting the petitioner of homicide.

On September 16, 1995, Noe Toledo y Tamboong (Noe) went to a black-smith who
made the design of his bolo. When he went home to Tuburan, Odiongan, Romblon late
in the afternoon, he saw the group of Lani Famero, Michael Fosana, Rex Cortez and
Ricky Guarte drinking gin at the house of the Spouses Manuel and Eliza Guarte, Ricky’s
parents. His house is about five (5) meters away from the house of Spouses Guarte.

He requested the group of Ricky to refrain from making any noise and proceeded inside
his house and went to sleep. Around 9:00 p.m., Gerardo Faminia, Eliza Guarte’s brother
arrived at the Guarte house and asked for any left-over food. Eliza prepared dinner for
him and after Gerardo finished eating, he went home accompanied by Ricky. Gerardo’s
home is about 12 meters away from the Guarte home. Minutes later, Ricky came back
and together with Lani, Rex and Michael, went to sleep at the Guarte house. They had
not laid down for long when they heard stones being hurled at the roof of the house.
The stoning was made three (3) times. Ricky rose from bed and peeped through a window.
He saw Noe stoning their house. Ricky went out of the house and proceeded to Noe’s
house. Ricky asked Noe, his uncle, why he was stoning their house. Noe did not answer
but met Ricky at the doorstep of his house and, without any warning, stabbed Ricky on
the abdomen with a bolo. Eliza had followed his son Ricky and upon seeing that Ricky
was stabbed, shouted for help. Lani heard Eliza’s cry for help and immediately rushed
outside the house. Lani saw Ricky leaning on the ground and supporting his body with
his hands. Lani helped Ricky stand up and brought him to the main road. Lani asked
Ricky who stabbed him and Ricky replied that it was Noe who stabbed him. Then
Docloy Cortez arrived at the scene on board his tricycle. Accordingly, Ricky was put
on the tricycle and taken to the Romblon Provincial Hospital.

At the Romblon Provincial Hospital, Dr. Fetalvero operated on Ricky that very night.
Ricky had sustained one (1) stab wound but due to massive blood loss, he died while
being operated on. Dr. Fetalvero issued a Medico-Legal Certificate showing the injuries
sustained by Ricky wherein there is a stab wound on the left chest with gastric &
transverse colon evisceration measuring 6 cms. long, irregular-edged at 8th ICS, left
penetrating.

RTC rendered judgment finding the petitioner guilty as charged.


The CA rendered judgment affirming the assailed decision with modifications. The CA
also denied the petitioner’s motion for reconsideration thereof. The appellate court ruled
that the petitioner failed to prove that he acted in self-defense.

The Office of the Solicitor General asserts that the petitioner failed to prove self-defense
with clear and convincing evidence. Hence, the decision of the CA affirming, on appeal,
the decision of the RTC is correct.

VIVAR, 1
CRIMINAL LAW DIGESTS

Aggrieved, the petitioner filed the instant petition for review, contending that the CA
erred in not finding that he acted in self-defense when he stabbed the victim by accident
and prays that he be acquitted of the crime charged.

Issue:
Whether or not accused-appellant is exempted from criminal liable invoking the par. 4,
art. 12 of the RPC.

Ruling:
No. The accused-appellant is not exempted from criminal liability because he failed to
prove that the victim was killed by accident, without fault or intention on his part to
cause it. The petitioner was burdened to prove with clear and convincing evidence, the
essential requisites for the exempting circumstance under Article 12, paragraph 4, viz:

1. A person is performing a lawful act;


2. With due care;
3. He causes an injury to another by mere accident;
4. Without fault or intention of causing it

Petitioner Noe testified that his bolo hit the victim accidentally.

He asserted in the RTC and in the CA that he is exempt from criminal liability for the
death of the victim under Article 12, paragraph 4 of the Revised Penal Code which
reads:

4. Any person who, while performing a lawful act with due care, causes an injury
by mere accident without fault or intention of causing it.

As such, he contends, he is not criminally liable under Article 11, paragraph 1 of the
Revised Penal Code which reads:

Art. 11. Justifying circumstances. – The following do not incur any criminal
liability:
1. Anyone who acts in defense of his person or rights, provided that
the following circumstances concur:
First. Unlawful aggression;
Second. Reasonable necessity of the means employed to prevent
or repel it:
Third. Lack of sufficient provocation on the part of the person
defending himself.

The petitioner also avers that he was able to prove the essential elements of
complete self-defense.
The essential requisites of self-defense are:
1) unlawful aggression on the part of the victim;
2) reasonable scrutiny of the means employed to prevent or repel it; and
3) lack of sufficient provocation on the part of the person defending himself.

However, the petitioner also claims that his bolo accidentally hit the stomach of the
victim. It is a matter of law that when a party adopts a particular theory and the case is
tried and decided upon that theory in the court below, he will not be permitted to change
his theory on appeal. The petitioner is proscribed from changing in this Court, his
theory of defense which he adopted in the trial court and foisted in the CA – by
claiming that he stabbed and killed the victim in complete self-defense. He relied on
Article 12, paragraph 4 of the Revised Penal Code in the trial and appellate courts, but
adopted in this Court two divergent theories – (1) that he killed the victim to defend
himself against his unlawful aggression; hence, is justified under Article 11,
paragraph 1 of the Revised Penal Code; (2) that his bolo accidentally hit the victim
and is, thus, exempt from criminal liability under Article 12, paragraph 4 of the
Revised Penal Code.

VIVAR, 1
CRIMINAL LAW DIGESTS

It is an aberration for the petitioner to invoke the two defenses at the same time
because the said defenses are intrinsically antithetical. There is no such defense as
accidental self-defense in the realm of criminal law.

Self-defense under Article 11, paragraph 1 of the Revised Penal Code necessarily
implies a deliberate and positive overt act of the accused to prevent or repel an
unlawful aggression of another with the use of reasonable means. The accused has
freedom of action. He is aware of the consequences of his deliberate acts. The defense
is based on necessity which is the supreme and irresistible master of men of all human
affairs, and of the law. From necessity, and limited by it, proceeds the right of self-
defense. The right begins when necessity does, and ends where it ends. Although the
accused, in fact, injures or kills the victim, however, his act is in accordance with law so
much so that the accused is deemed not to have transgressed the law and is free from
both criminal and civil liabilities.

On the other hand, the basis of exempting circumstances under Article 12 of the
Revised Penal Code is the complete absence of intelligence, freedom of action, or intent,
or the absence of negligence on the part of the accused. The basis of the exemption in
Article 12, paragraph 4 of the Revised Penal Code is lack of negligence and intent. The
accused does not commit either an intentional or culpable felony. The accused
commits a crime but there is no criminal liability because of the complete absence of
any of the conditions which constitute free will or voluntariness of the act. An accident
is a fortuitous circumstance, event or happening; an event happening wholly or partly
through human agency, an event which under the circumstances is unusual or
unexpected by the person to whom it happens.

Self-defense, under Article 11, paragraph 1, and accident, under Article 12, paragraph
4 of the Revised Penal Code, are affirmative defenses which the accused is burdened to
prove, with clear and convincing evidence.The petitioner failed to prove that the
victim was killed by accident, without fault or intention on his part to cause it. The
petitioner was burdened to prove with clear and convincing evidence, the essential
requisites for the exempting circumstance under Article 12, paragraph 4, viz:

1) A person is performing a lawful act;


2) With due care;
3) He causes an injury to another by mere accident;
4) Without fault or intention of causing it.

Unlawful aggression is a condition sine qua non for the justifying circumstances of
self- defense, whether complete or incomplete. It presupposes an actual, sudden, and
unexpected attack, or imminent danger thereof, and not merely a threatening or
intimidating attitude. Supreme Court agrees with the ruling of the CA that the
petitioner failed to prove self defense, whether complete or incomplete.

Fallo:
IN LIGHT OF ALL THE FOREGOING, the petition is DENIED. The assailed decision of
the Court of Appeals is AFFIRMED. Costs against the petitioner.

VIVAR, 1
CRIMINAL LAW DIGESTS

Defense of Honor <<<

US vs. Ramos
G.R. No. 7020. March 15,
1912 TRENT, J:

Doctrine:
A woman in defense of her honor is justified in inflicting wounds on her assailant with
a bolo which she happens to be carrying, even though her cry for assistance might
have been heard by people near by.

Facts:
Damian Santa Ana and Dorotea Ramos were husband and wife and were charged with
crime of lesiones menos graves in the Court of First Instances of the Province of Rizal.
Damian was acquitted and Ramos was found guilty and was charged and sentenced to
one month and one day of arresto mayor, to indemnify Santos in the sum of P42.00.
Ramos appealed to the decision stating that hers was an act of self-defense against
Santos.

Issue:
Whether or not Dorotea Ramos was actually in the act of self-defense when she strikes
Santos with a bolo and shall be acquitted. Yes.

Ruling:
The court decided that Ramos was entitled to an absolute acquittal upon the ground of
self-defense. Ramos used the bolo for the protection of her honor and to repel Santos’
attack.

Fallo:
The judgment appealed from is therefore reversed and the appellant ordered released
from custody forthwith, with costs de oficio.

VIVAR, 1
CRIMINAL LAW DIGESTS

Incomplete Defense; Defense of Property; Lack of Reasonable Means Employed


<<<

People vs. Narvaez


G.R. Nos. L-33466-67, April 20, 1983
MAKASIAR, J:

Doctrine:
Justifying Circumstances - Unlawful Aggression against property rights: Defense of
property can be invoked as a justifying circumstance only when it is coupled with an
attack on the person of one entrusted with said property.

*It must be noted that there can be no self-defense, whether complete or incomplete,
unless the victim had committed unlawful aggression against the person who resorted
to self-defense.

*Also, the party should have been the subject of a real and imminent threat, which
represents the unlawful aggression made upon him. There must also be
reasonableness in his use of a knife or any other weapon as his means to defend
himself. And finally, there should be no provocation on his part that caused his
aggressor to harm him.

Facts:
 Mamerto Narvaez has been convicted of murder (qualified by treachery) of
David Fleischer and Flaviano Rubia. On August 22, 1968, Narvaez shot Fleischer
and Rubia during the time the two were constructing a fence that would
prevent Narvaez from getting into his house and rice mill.

 The defendant was taking a nap when he heard sounds of construction and
found fence being made. He addressed the group and asked them to stop
destroying his house and asking if they could talk things over. Fleischer
responded with "No, gadamit, proceed, go ahead." Defendant lost his
"equilibrium," and shot Fleisher with his shotgun.

 He also shot Rubia who was running towards the jeep where the deceased's gun
was placed. Prior to the shooting, Fleischer and Co. (the company of Fleischer's
family) was involved in a legal battle with the defendant and other land settlers
of Cotabato over certain pieces of property.

 At the time of the shooting, the civil case was still pending for annulment (settlers
wanted granting of property to Fleisher and Co. to be annulled). At time of the
shooting, defendant had leased his property from Fleisher (though case pending
and ownership uncertain) to avoid trouble.

 On June 25, defendant received letter terminating contract because he allegedly


didn't pay rent. He was given 6 months to remove his house from the land.
Shooting was barely 2 months after letter.

 Defendant claims he killed in defense of his person and property. CFI ruled that
Narvaez was guilty. Aggravating circumstances of evident premeditation offset by
the mitigating circumstance of voluntary surrender. For both murders, CFI
sentenced him to reclusion perpetua, to indemnify the heirs, and to pay for moral
damages.

Issue:
1. Whether or not CFI erred in convicting defendant-appellant despite the fact that
he acted in defense of his person.
2. Whether or not the court erred in convicting defendant-appellant although he
acted in defense of his rights.
3. Whether or not he should be liable for subsidiary imprisonment since he is
unable to pay the civil indemnity due to the offended party.

VIVAR, 1
CRIMINAL LAW DIGESTS

Ruling:
1. No. The courts concurred that the fencing and chiselling of the walls of the
house of the defendant was indeed a form of aggression on the part of the
victim. However, this aggression was not done on the person of the victim but
rather on his rights to property. On the first issue, the courts did not err.
However, in consideration of the violation of property rights, the courts
referred to Art. 30 of the civil code recognizing the right of owners to close and
fence their land.

Although is not in dispute, the victim was not in the position to subscribe to the
article because his ownership of the land being awarded by the government was
still pending, therefore putting ownership into question. It is accepted that the
victim was the original aggressor.

2. Yes. However, the argument of the justifying circumstance of self-defense is


applicable only if the 3 requirements are fulfilled. Art. 11(1) RPC enumerates
these requisites:
 Unlawful aggression. In the case at bar, there was unlawful aggression
towards appellant's property rights. Fleisher had given Narvaez 6
months and he should have left him in peace before time was up, instead
of chiseling Narvaez's house and putting up fence. Art. 536 of the Civil
Code also provides that possession may not be acquired through force or
intimidation; while Art. 539 provides that every possessor has the right
to be respected in his possession.
 Reasonable necessity of means employed to prevent or repel attack. In the
case, killing was disproportionate to the attack.
 Lack of sufficient provocation on part of person defending himself. Here,
there was no provocation at all since he was asleep.

Since not all requisites present, defendant is credited with the special mitigating
circumstance of incomplete defense, pursuant to Art. 13(6) RPC. These
mitigating circumstances are: voluntary surrender and passion and obfuscation
(read p. 405 explanation) Crime is homicide (2 counts) not murder because
treachery is not applicable on account of provocation by the deceased. Also,
assault was not deliberately chosen with view to kill since slayer acted
instantaneously. There was also no direct evidence of planning or preparation
to kill. Art. 249 RPC: Penalty for homicide is reclusion temporal. However, due
to mitigating circumstances and incomplete defense, it can be lowered three
degrees (Art. 64) to arrestomayor.

3. No. He is not liable to be subsidiarily imprisoned for nonpayment of civil


indemnity. RA 5465 made the provisions of Art. 39 applicable to fines only and
not to reparation of damage caused, indemnification of consequential damages
and costs of proceedings. Although it was enacted only after its conviction,
considering that RA 5465 is favorable to the accused who is not a habitual
delinquent, it may be given retroactive effect pursuant to Art. 22 of the RPC.

Fallo:
Defendant guilty of homicide but w/ mitigating circumstances and extenuating
circumstance of incomplete self-defense. Penalty is 4 months arresto mayor and to
indemnify each group of heirs 4,000 w/o subsidiary imprisonment and w/o award for
moral damages. Appellant has already been detained 14 years so his immediate
release is ordered.

Gutierrez, dissenting. Defense of property can only be invoked when coupled with
form of attack on person defending property. In the case at bar, this was not so.
Appellant should then be sentenced to prision mayor. However, since he has served
more than that, he should be released.

VIVAR, 1
CRIMINAL LAW DIGESTS

Presence of all elements of self defense <<<

Nacnac vs. People


GR 191913, March 21, 2012
MAKASIAR, J:

Doctrine:
The lone wound inflicted on the victim supports the argument that petitioner feared for
his life and only shot the victim to defend himself. The lone gunshot was a reasonable
means chosen by petitioner in defending himself in view of the proximity of the armed
victim, his drunken state, disobedience of an unlawful order, and failure to stand down
despite a warning shot.

Facts:
 On February 20, 2003, SPO2 Nacnac (accused-appellant), the SPO1 Doddie
Espejo (victim) and several other police officers were on duty. Nacnac, being the
highest-ranking officer during the shift, was designated the officer-of-the-day.
 Shortly before 10:00 in the evening, Espejo, together with then SPO1 Basilio,
took the patrol tricycle from the station grounds. When Nacnac saw this, he
stopped the victim and his colleague from using the tricycle.
 Espejo told Nacnac that he needed it to go to Laoag City to settle a previous
disagreement with a security of a local bar. Nacnac still refused. He told Espejo
that he is needed at the station and, at any rate, he should stay at the station
because he was drunk.
 This was not received well by Espejo. He told Nacnac in Ilocano: “Iyot ni inam
kapi” (Coitus of your mother, cousin!). Espejo alighted from the tricycle. SPO1
Basilio did the same, went inside the office, and left Nacnac and Espejo alone.
 Espejo took a few steps and drew his .45 caliber gun which was tucked in a
holster on the right side of his chest. Nacnac then fired his M-16 armalite
upward as a warning shot. Undaunted, Espejo still drew his gun. Nacnac then
shot the victim on the head, which caused the Espejo’s instantaneous death.
 Nacnac later surrendered to the stations Chief of Police.
 The RTC found Nacnac guilty beyond reasonable doubt of the crime of homicide.
It held that the claim of self-defense by Nacnac was unavailing due to the absence
of unlawful aggression on the part of Espejo.
 On appeal, CA affirmed the findings of RTC. It held that the essential and
primary element of unlawful aggression was lacking. It gave credence to the
finding of the trial court that no one else saw the victim drawing his weapon
and pointing it at accused Nacnac.
 In its comment dated April 27, 2011, the OSG avers that petitioner is entitled to
an acquittal, or at the very least, not one but two mitigating circumstances.

Issue:
Whether or not the justifying circumstances of the petitioner’s acts constitutes a valid
self-defense.

Ruling:
YES.
LEGAL BASIS
Article 11 of the Revised Penal Code states that anyone who acts in defense of his
person or rights do not incur any criminal liability, provided that the following
circumstances concur:
(i) unlawful aggression;
(ii) reasonable necessity of the means employed to prevent or repel it; and
(iii) lack of sufficient provocation on the part of the person defending himself.

APPLICATION OF LEGAL BASIS


Unlawful aggression is an indispensable element of self-defense; and ordinarily, there
is a difference between the act of drawing one’s gun and the act of pointing one’s gun
at a target in determining the presence of unlawful aggression. The former cannot be
said to

VIVAR, 1
CRIMINAL LAW DIGESTS

be an unlawful aggression on the part of the victim, while the latter is generally
considered unlawful aggression. Here, a warning shot fired by fellow police officer
(petitioner) was left unheeded as the victim reached for his own firearm and pointed it
at petitioner. Petitioner was justified in defending himself from an inebriated and
disobedient colleague.

As to the second circumstance above, the nature and number of wounds inflicted by
the accused are constantly and unremittingly considered as important indicia of the
means employed by the accused, which must be reasonably commensurate to the
nature and the extent of the attack sought to be averted. Here, the lone gunshot was a
reasonable means chosen by petitioner in defending himself in view of the proximity
of the armed victim, his drunken state, disobedience of an unlawful order, and failure
to stand down despite a warning shot.

There is also lack of sufficient provocation on the part of the person defending himself
or herself in this case. Petitioner gave the victim a lawful order and fired a warning
shot before shooting the armed and drunk victim. There was no evidence on petitioner
sufficiently provoking the victim prior to the shooting. Petitioner was only defending
himself on the night he shot his fellow policer.

Fallo:
WHEREFORE, petitioner's Motion for Reconsideration is GRANTED. The CA Decision
dated July 20, 2009 in CA-G.R. CR-H.C. No. 30907 is REVERSED and SET ASIDE.
Petitioner SPO2 Lolito T. Nacnac is ACQUITTED of homicide on reasonable doubt. The
Director of the Bureau of Prisons is ordered to immediately RELEASE petitioner from
custody, unless he is being held for some other lawful cause, and to INFORM this Court
within five (5) days from receipt of this Decision of the date petitioner was actually
released from confinement.

VIVAR, 1
CRIMINAL LAW DIGESTS

Privileged Mitigating Circumstance- Child in Conflict with the Law <<<

People v. Sarcia
G.R. No. 169641, September 10, 2009
LEONARDO-DE CASTRO, J:

Doctrine:
The fact of minority of the offender at the time of the commission of the offense has no
bearing on the gravity and extent of injury caused to the victim and her family,
particularly considering the circumstances attending this case. Here, the accused-
appelant could have been eighteen at the time of the commission of the rape. He was
accorded the benefit of the privileged mitigating circumstance of minority because of a
lack of proof regarding his actual age and the date of the rape rather than a moral or
evidentiary certainty of his minority.

Case Summary:
The accused-appellant Richard O. Sarcia alias Nogi is guilty beyond reasonable
doubt of the crime of rape committed against AAA, and sentenced him to suffer the
penalty of Reclusion Perpetua and to pay the amount of P50,000.00 as civil indemnity,
P50,000.00 as moral damages, and the cost of the suit. However, the CA modified the
penalties imposed by the RTC by imposing the death penalty, increasing the award of
civil indemnity to P75,000.00, and awarding P25,000.00 as exemplary damages, aside
from the P50,000.00 for moral damages.

The crime of rape was allegedly committed sometime in 1996 against AAA, a five
(5) year old girl. After almost four (4) years, AAA’s father filed a complaint for acts of
lasciviousness against herein accused-appellant on July 7, 2000.

Facts:
 On December 16, 1996, five-year-old [AAA], together with her [cousin and two
other playmates], was playing in the yard of Saling Crisologo when the accused-
appellant invited her to the backyard of the house and raped her. AAA’s cousin
witnessed what happened.
 A complaint for acts of lasciviousness was filed against the accused-appellant. Upon
review, his charges were upgraded to rape.
 The court found the accused-appellant guilty of the crime and imposed the penalty.
 July 15, 2004, accused-appellant filed an appeal.
 The accused denied the allegations on grounds of:
o Testimonies of AAA and cousin are inconsistent.
o AAA’s inability to recall the crime.
o Delay in filling
o Absence of proof of force or intimidation
o Negative lacerations based on medical report.
 The RTC found accused-appellant guilty and imposed the penalty of reclusion
perpetua as well as civil indemnity of P50,000.00 and moral damages of
P50,000.00.
 Case was forwarded to CA, its decision of July 14, 2005, in CA-G.R. CR-H.C. No.
000717, affirmed with modification the judgment of conviction pronounced by the
trial court.
 The case was elevated to the SC for further review.
 RA 9344 took effect while the case was pending before the SC

Issue:
1. Whether or not accused-appellant was guilty beyond reasonable doubt.
2. Whether or not (If the accused-appellant is guilty), the penalty imposed was
proper.
3. Whether or not accused-appellant can avail of the retroactive effect of RA 9344.

Ruling:
1. Yes. Accused-appellant is guilty beyond reasonable doubt. The claim of the
accused-appellant regarding inconsistent in testimonies does not affect the

VIVAR, 1
CRIMINAL LAW DIGESTS

veracity of the testimonies. The discrepancies regarding date and time, does not
make the information defective. The medical report finding the victim with no
laceration is not indispensable in the crime of rape. The rape victim's delay or
hesitation in reporting the crime does not destroy the truth of the charge nor is
it an indication of deceit.

2. Under Art. 335 of the RPC, the imposable penalty for statutory rape is death.
However, accused-appellant is entitled to privileged mitigating circumstance of
minority because he was 18 years old at the time of the commission of the offense.
Since the prosecution was not able to prove the exact date and time when the
rape was committed, it is not certain that the crime of rape was committed on
or after he reached 18 years of age in 1996.

3. Yes, RA9344 provides for its retroactive application as follows:


“Sec. 68. Children Who Have Been Convicted and are Serving Sentence. —
Persons who have been convicted and are serving sentence at the time of
the effectivity of this Act, and who were below the age of eighteen (18) years
at the time of the commission of the offense for which they were
convicted and are serving sentence, shall likewise benefit from the
retroactive application of this Act. . . “

RATIO: The legislative intent, to apply to heinous crimes the automatic suspension of
sentence of a child in conflict with the law can be gleaned from the Senate deliberations
50 on Senate Bill No. 1402 (Juvenile Justice and Delinquency Prevention Act of
2005), the pertinent portion of which is quoted below:
If a mature minor, maybe 16 years old to below 18 years old is charged, accused
with, or may have committed a serious offense, and may have acted with
discernment, then the child could be recommended by the Department of Social
Welfare and Development (DSWD), by the Local Council for the Protection of
Children (LCPC), or by my proposed Office of Juvenile Welfare and Restoration
to go through a judicial proceeding; but the welfare, best interests, and
restoration of the child should still be a primordial or primary consideration.
Even in heinous crimes, the intention should still be the child's restoration,
rehabilitation and reintegration. . .

Fallo:
WHEREFORE, the decision of the CA dated July 14, 2005 in CA-G.R. CR-H.C. No. 00717
is hereby AFFIRMED with the following MODIFICATIONS:(1) the penalty of death
imposed on accused-appellant is reduced to reclusion perpetua; and (2) accused-
appellant is ordered to pay the victim the amount of P75,000.00 and P30,000.00 as
moral damages and exemplary damages, respectively. The award of civil indemnity in
the amount of P75,000.00 is maintained. However, the case shall be REMANDED to the
court a quo for appropriate disposition in accordance with Sec. 51 of R.A. 9344.|||
(People v. Sarcia, G.R. No. 169641, [September 10, 2009], 615 PHIL 97-131)

VIVAR, 1
CRIMINAL LAW DIGESTS

Elements of Accident <<<

People v. Victoriano Dela Cruz


G.R. No. 187683, February 11, 2010
NACHURA, J:

Doctrine:
For an accident to become an exempting circumstance, the act that causes the injury
has to be lawful. Victoriano’s act of physically maltreating his spouse is definitely not a
lawful act. To say otherwise would be a travesty—a gross affront to our existing laws
on violence against women.

Facts:
 Before this court is an Appeal, seeking the reversal of the CA decision dated October
31, 2008 which affirmed with modification the decision of the RTC of Malolos,
Bulacan, dated August 15, 2005, convicting appellant Victoriano Dela Cruz y
Lorenzo of the crime of Parricide.
 On August 18, 2002, in Malolos, Bulacan, the accused, with intent to kill his wife
Anna Liza Caparas-Dela Cruz, with whom he was married to, used personal
violence and stabbed the said victim, thereby causing serious physical injuries that
directly caused her death.
 Upon arraignment, Victoriano pleaded not guilty.
 According to the prosecution’s witness, Joel Song, testified that he witnessed
Victoriano kicking and punching his wife in front of their house. Victoriano was
said to have dragged his wife by the hair into the house and further heard
altercations. Suddenly, as they came out of a room, he saw blood spurting out of the
wife’s mouth and Victoriano sought Joel’s help to bring her to the hospital.
However, was pronounced dead on arrival.
 According to the defense of herein respondent, Victoriano testified that his wife
was nagging him about being intoxicated and was worried about their daughter
seeing his father like that, then, as the nagging went on, Victoriano pushed her
aside so he could go out of the house, however, she fell on a broken window, and
noticed that her back was punctured by a shattered glass from the broken window.
And then after, called Joel for help, whom had accompanied them to the hospital.
Victoriano further asserted that he does not usually drink and that he had
consumed hard liquor at the time of the incident, and that he loved his wife
and that he did not intentionally hurt her.
 On August 15, 2005, the RTC rendered a decision finding Victoriano guilty beyond
reasonable doubt, sentencing him to reclusion perpetua. The CA affirmed the
decision.
 Hence, this appeal to SC

Issue:
Whether or not there is a exempting circumstance?

Ruling:
NO.
Victoriano’s claim that the injury sustained by his wife was caused by an accident,
without fault or intention of causing it, it is clear that Victoriano was not performing a
lawful act at the time of the incident. Before an accused may be exempted from criminal
liability by the invocation of Article 12 (paragraph 4) of the RPC, the following elements
must concur: (1) a person is performing a lawful act (2) with due care, and (3) he causes
an injury to another by mere accident and (4) without any fault or intention of causing
it. For an accident to become an exempting circumstance, the act that causes the injury
has to be lawful. Victoriano’s act of physically maltreating his spouse is definitely not a
lawful act. To say otherwise would be a travesty—a gross affront to our existing laws on
violence against women. Thus, we fully agree with the apt findings of the CA, to wit:

“With the foregoing avowal, We find that the death of appellant’s wife was not
caused by mere accident. An accident is an occurrence that “happens outside the

VIVAR, 1
CRIMINAL LAW DIGESTS

sway of our will, and although it comes about through some act of our will, lies
beyond the bounds of humanly foreseeable consequences.” It connotes the
absence of criminal intent. Intent is a mental state, the existence of which is
shown by a person’s overt acts.

In the case at bench, evidence disclosed that appellant started beating his wife
outside their house and was even the one who dragged her inside. This, to Our
mind, contradicts his theory that he only pushed her so as to go out of the house
to avoid any further quarrel. Such incongruity whittles down appellant’s
defense that he did not deliberately kill his wife.”

Supreme Court Ruling:


The instant appeal is bereft of merit.

Victoriano claims that Joel's testimony coincides with his own, which refers to the
slapping incident that occurred outside their house. It does not at all point to him as
the actual perpetrator of the crime. Thus, Victoriano submits that Joel’s testimony is
merely circumstantial.

But circumstantial evidence is sufficient for conviction, as we ruled in People v.


Castillo: Direct evidence of the commission of the offense is not the only matrix
wherefrom a trial court may draw its conclusions and finding of guilt. Conviction can be
had on the basis of circumstantial evidence provided that: (1) there is more than one
circumstance; (2) the facts from which the inferences are derived are proven; and (3) the
combination of all the circumstances is such as to produce a conviction beyond
reasonable doubt.

In this case, we note the presence of the requisites for circumstantial evidence to
sustain a conviction. First, immediately preceding the killing, Victoriano physically
maltreated his wife, not merely by slapping her as he claimed, but by repeatedly
punching and kicking her. Second, it was Victoriano who violently dragged the victim
inside their house, by pulling her hair. Third, in Dr. Viray's Report, Anna sustained
injuries in different parts of her body due to Victoriano's acts of physical abuse.
Fourth, the location and extent of the wound indicated Victoriano's intent to kill the
victim. The Report revealed that the victim sustained a fatal stab wound, lacerating the
upper lobe of her right lung, a vital organ. The extent of the physical injury inflicted on
the deceased manifests Victoriano's intention to extinguish life. Fifth, as found by both
the RTC and the CA, only Victoriano and Anna were inside the house, other than their
young daughter. Thus, it can be said with certitude that Victoriano was the lone
assailant. Sixth, we have held that the act of carrying the body of a wounded victim and
bringing her to the hospital ― as Victoriano did ― does not manifest innocence. It
could merely be an indication of repentance or contrition on his part.

The foregoing circumstances are proven facts, and the Court finds no reason to
discredit Joel’s testimony and Dr. Viray's Report. Besides, well-entrenched is the
rule that the trial court's assessment of the credibility of witnesses is accorded great
respect and will not be disturbed on appeal, in as much as the court below was in a
position to observe the demeanor of the witnesses while testifying. The Court does not
find any arbitrariness or error on the part of the RTC as would warrant a deviation from
this well-entrenched rule.

Even if, for the sake of argument, we consider Victoriano’s claim that the injury
sustained by his wife was caused by an accident, without fault or intention of
causing it, it is clear that Victoriano was not performing a lawful act at the time
of the incident. Before an accused may be exempted from criminal liability by the
invocation of Article 12 (paragraph 4) of the RPC, the following elements must concur:
(1) a person is performing a lawful act (2) with due care, and (3) he causes an injury to
another by mere accident and (4) without any fault or intention of causing it. For an
accident to become an exempting circumstance, the act that causes the injury has to
be lawful Victoriano's act of physically maltreating his spouse is definitely not a lawful
act. To say otherwise would be a travesty -- a gross affront to our existing laws on
violence against women.

VIVAR, 1
CRIMINAL LAW DIGESTS

Fallo:
WHEREFORE, the Decision of the Court of Appeals in CA-G.R. CR HC No. 01575,
finding appellant, Victoriano dela Cruz y Lorenzo, guilty beyond reasonable doubt of
the crime of PARRICIDE, is hereby AFFIRMED WITH MODIFICATION. Appellant is
sentenced to suffer the penalty of reclusion perpetua and to pay the heirs of the victim,
Anna Liza Caparas-dela Cruz, the amounts of ₱50,000.00 as civil indemnity,
₱50,000.00 as moral damages, and ₱30,000.00 as exemplary damages. No costs.

VIVAR, 1
CRIMINAL LAW DIGESTS

Uncontrollable Fear <<<

Ty v. People
G.R. No. 149275, September 27, 2004
TINGA, J:

Doctrine:
Exempting Circumstances; Defense of Uncontrollable Fear; Requisites for the defense
of acting under an uncontrollable fear to be invoked.—The only question of law raised
—whether the defense of uncontrollable fear is tenable to warrant her exemption
from criminal liability—has to be resolved in the negative. For this exempting
circumstance to be invoked successfully, the following requisites must concur: (1)
existence of an uncontrollable fear; (2) the fear must be real and imminent; and (3) the
fear of an injury is greater than or at least equal to that committed.

Case Summary:
Ty issued 7 checks to cover her mother’s and sister’s hospital fees. The checks bounced
and a case for BP 22 was filed against her. Court did not give credence to the defense of
uncontrollable fear and found her guilty.

Facts:
 This case stemmed from the filing of seven (7) Informations for violation of B.P. 22
against Ty before the RTC of Manila.
 Accused did then and there willfully, unlawfully and feloniously make or draw and
issue to Manila Doctors’ Hospital to apply on account or for value to Editha L.
Vecino ,payable to Manila Doctors Hospital in the amount of ₱30,000.00, said
accused well knowing that at the time of issue she did not have sufficient funds in
or credit with the drawee bank for payment of such check in full upon its
presentment, which check when presented for payment within ninety (90) days
from the date hereof, was subsequently dishonored by the drawee bank for
"Account Closed" and despite receipt of notice of such dishonor, said accused failed
to pay said Manila Doctors Hospital the amount of the check or to make
arrangement for full payment of the same within five (5) banking days after
receiving said notice.
 Ty’s mother, Chua Lo So Un, was confined in Manila Doctors – accused
"Acknowledgment of Responsibility for Payment" in the Contract of Admission dated
30 October 1990. As of 4 June 1992, the Statement of Account shows the total
liability of her mother.
 Ty’s sister, Judy Chua, was also confined at the hospital. The total hospital bills of
the two patients amounted to ₱1,075,592.95.
 Ty executed a promissory note wherein she assumed payment of the obligation in
installments. To assure payment of the obligation, she drew several postdated checks
against Metrobank payable to the hospital. The seven (7) checks, each covering the
amount of ₱30,000.00, were all deposited on their due dates. But they were all
dishonored by the drawee bank and returned unpaid to the hospital due to
insufficiency of funds, with the "Account Closed" advice.
 Soon thereafter, the complainant hospital sent demand letters to Ty by registered
mail. As the demand letters were not heeded, complainant filed the seven (7)
Informations subject of the instant case.
 Ty in her defense that she issued the checks "under the impulse of an
uncontrollable fear of a greater injury or in avoidance of a greater evil or injury."
She was forced to issue the checks to obtain release for her mother whom the
hospital inhumanely and harshly treated and would not discharge unless the
hospital bills are paid.
 RTC Ruling:
Trial court found that Ty issued the checks subject of the case in payment of the
hospital bills of her mother and rejected the theory of the defense. Thus, on 21
April 1997, the trial court rendered a Decision finding Ty guilty of seven (7) counts
of violation of B.P. 22 and sentencing her to a prison term. The dispositive part
of the Decision reads:

VIVAR, 1
CRIMINAL LAW DIGESTS

CONSEQUENTLY, the accused Vicky C. Ty, for her acts of issuing seven (7)
checks in payment of a valid obligation, which turned unfounded on their
respective dates of maturity, is found guilty of seven (7) counts of violations of
Batas Pambansa Blg. 22, and is hereby sentenced to suffer the penalty of
imprisonment of SIX MONTHS per count or a total of forty-two (42) months.

 CA Ruling: affirmed the judgment of the trial court with modification. It set aside the
penalty of imprisonment and instead sentenced Ty "to pay a fine of sixty thousand
pesos (₱60,000.00) equivalent to double the amount of the check, in each case.
 the Court of Appeals rejected Ty’s defenses of involuntariness in the issuance of
the checks and the hospital’s knowledge of her checking account’s lack of funds.
It held that B.P. 22 makes the mere act of issuing a worthless check punishable
as a special offense, it being a malum prohibitum. What the law punishes is the
issuance of a bouncing check and not the purpose for which it was issued nor
the terms and conditions relating to its issuance.
 the Court of Appeals is also not convinced that there was no valuable
consideration for the issuance of the checks as they were issued in payment of
the hospital bills of Ty’s mother.
 In sentencing Ty to pay a fine instead of a prison term, the appellate court applied
the case of Vaca v. Court of Appeals, wherein this Court declared that in
determining the penalty imposed for violation of B.P. 22, the philosophy
underlying the Indeterminate Sentence Law should be observed, i.e., redeeming
valuable human material and preventing unnecessary deprivation of personal
liberty and economic usefulness, with due regard to the protection of the social
order.

Issue:
Whether or not the defense of uncontrollable fear is tenable to warrant her exemption
from criminal liability.

Ruling:
NO.
 In this case, far from it, the fear, if any, harbored by Ty was not real and imminent.
Ty claims that she was compelled to issue the checks—a condition the hospital
allegedly demanded of her before her mother could be discharged--for fear that her
mother’s health might deteriorate further due to the inhumane treatment of the
hospital or worse, her mother might commit suicide. This is speculative fear; it is
not the uncontrollable fear contemplated by law.
1. There was no showing that the mother’s illness was so life-threatening such that
her continued stay in the hospital suffering all its alleged unethical treatment
would induce a well-grounded apprehension of her death.

Uncontrollable fear - For this exempting circumstance to be invoked successfully, the


following requisites must concur: (1) existence of an uncontrollable fear; (2) the fear
must be real and imminent; and (3) the fear of an injury is greater than or at least
equal to that committed.

In the instant case, the evil sought to be avoided is merely expected or anticipated. If
the evil sought to be avoided is merely expected or anticipated or may happen in the
future, this defense is not applicable.

It must appear that the threat that caused the uncontrollable fear is of such gravity and
imminence that the ordinary man would have succumbed to it. It should be based on a
real, imminent or reasonable fear for one’s life or limb. A mere threat of a future injury
is not enough. It should not be speculative, fanciful, or remote. A person invoking
uncontrollable fear must show therefore that the compulsion was such that it reduced
him to a mere instrument acting not only without will but against his will as well. It
must be of such character as to leave no opportunity to the accused for escape.

VIVAR, 1
CRIMINAL LAW DIGESTS

Speculative fear
The fear harbored by Ty was not real and imminent. Ty claims that she was compelled
to issue the checks, a condition the hospital allegedly demanded of her before her
mother could be discharged, for fear that her mother’s health might deteriorate
further due to the inhumane treatment of the hospital or worse, her mother might
commit suicide. This is speculative fear; it is not the uncontrollable fear contemplated
by law.

Ty likewise suggests in the prefatory statement of her Petition and Memorandum that
the justifying circumstance of state of necessity under par. 4, Art. 11 of the Revised
Penal Code may find application in this case.

We do not agree. The law prescribes the presence of three requisites to exempt the actor
from liability under this paragraph: (1) that the evil sought to be avoided actually exists;
(2) that the injury feared be greater than the one done to avoid it; (3) that there be no
other practical and less harmful means of preventing it.

In the instant case, the evil sought to be avoided is merely expected or anticipated. If
the evil sought to be avoided is merely expected or anticipated or may happen in the
future, this defense is not applicable. Ty could have taken advantage of an available
option to avoid committing a crime. By her own admission, she had the choice to give
jewelry or other forms of security instead of postdated checks to secure her obligation.

Fallo:
WHEREFORE, the instant Petition is DENIED and the assailed Decision of the Court of
Appeals, dated 31 July 2001, finding petitioner Vicky C. Ty GUILTY of violating Batas
Pambansa Bilang 22 is AFFIRMED with MODIFICATIONS. Petitioner Vicky C. Ty is
ORDERED to pay a FINE equivalent to double the amount of each dishonored check
subject of the seven cases at bar with subsidiary imprisonment in case of insolvency in
accordance with Article 39 of the Revised Penal Code. She is also ordered to pay
private complainant, Manila Doctors’ Hospital, the amount of Two Hundred Ten
Thousand Pesos (₱210,000.00) representing the total amount of the dishonored
checks. Costs against the petitioner.

VIVAR, 1
CRIMINAL LAW DIGESTS

Elements of Accident <<<

People v. Castillo
G.R. No. 172695, June 29, 2007
YNARES-SANTIAGO, J:

Doctrine:
Exempting Circumstances; Requisites; By no stretch of imagination could playing with
or using a deadly sling and arrow be considered as performing a “lawful act.”—“Accident”
is an affirmative defense which the accused is burdened to prove, with clear and
convincing evidence. The defense miserably failed to discharge its burden of proof. The
essential requisites for this exempting circumstance, are: 1. A person is performing a
lawful act; 2. With due care; 3. He causes an injury to another by mere accident; 4.
Without fault or intention of causing it. By no stretch of imagination could playing with
or using a deadly sling and arrow be considered as performing a “lawful act.” Thus, on
this ground alone, appellant’s defense of accident must be struck down because he was
performing an unlawful act during the incident.

Facts:
 In the evening of November 5, 1993, between 9:00 o'clock to 10:00 o'clock, the
accused came home drunk and was in an angry mood. The accused kicked the door
and table, and then threw the electric fan away. He was prevailed upon by
Guillermo to take a rest. But the accused did not heed the advice of Guillermo as he
took instead his sling and arrow from the house ceiling where he was keeping
them. Dejectedly, Guillermo transferred to the adjacent house of her . . . daughter
[in-law] Yolanda. From there, Guillermo heard the victim crying and, afterwards,
shouting at the accused. Guillermo concernedly ordered Yolanda to see what was
happening inside the house of Consorcia, and Yolanda obeyed. On her way, Yolanda
met the accused carrying the bloodied body of Consorcia. Guillermo, the accused,
and Yolanda brought Consorcia to the hospital but to no avail. According to Dr.
Solita P. Plastina, Municipal Health Officer of Calamba, Laguna, who conducted the
autopsy on the victim's body, the fatal weapon could have been a "pointed
instrument like a nail". There is no dispute likewise that the accused shot with a
dart from a rubber sling, his wife hitting her at the neck and causing her
instantaneous death.
 The trial court: finds accused Isaias Castillo GUILTY beyond reasonable doubt of
the crime of PARRICIDE.
 Court of Appeals: denied.
 Hence, the present petition. Appellant claimed that it was not established that he
was the one who shot his wife with a deadly arrow considering that at the time of
the incident, he and his drinking buddies were all engaged in target shooting using
the sling and arrow. Hence, he surmised that any one of them could have shot the
victim. At any rate, even assuming that he was the one who killed his wife, the same
was accidental and not intentional.

Issue:
1. Whether or not the fatal injury inflicted on the victim was accidental?
2. Whether or not the exempting circumstance of accident applicable in the
instant case?

Ruling:
1. The essential requisites for this exempting circumstance are:
a. person is performing a lawful act
b. with due care;
c. he causes and injury to another by mere accident
d. without fault or intention of causing it.

The mere possession of sling and arrow is punishable under the law. In penalizing
the act, the consideration of the deadly weapon was used for no legal purpose,
but to inflict injury. Also, the fact that the accused-appellant disappeared while
his wife was in the hospital is unbecoming of a husband with a dying wife. Due

VIVAR, 1
CRIMINAL LAW DIGESTS

to the weakness of the defense’s evidence, the claim that the act was accidental
cannot be appreciated in favor of the accused.

Wherefore, the accused was found guilty of the crime of Parricide wherein the
court imposed the penalty of reclusion perpetua.

2. With regards to the Second Issue:


No. Article 12, par. 4 of the Revised Penal Code, provides:
Article. 12: Circumstances which exempt from criminal liability. — The
following are exempt from criminal liability: Any person who, while performing
a lawful act with due care, causes an injury by mere accident without fault or
intention of causing it.

"Accident" is an affirmative defense which the accused is burdened to prove,


with clear and convincing evidence. The defense miserably failed to discharge
its burden of proof. The essential requisites for this exempting circumstance,
are:
i. A person is performing a lawful act;
ii. With due care;
iii. He causes an injury to another by mere accident;
iv. Without fault or intention of causing it.

By no stretch of imagination could playing with or using a deadly sling and


arrow be considered as performing a "lawful act." Thus, on this ground alone,
appellant's defense of accident must be struck down because he was performing
an unlawful act during the incident. Also, the fact that the accused-appellant
disappeared while his wife was in the hospital is unbecoming of a husband with
a dying wife. Accused was found guilty of the crime of Parricide wherein the
court imposed the penalty of reclusion perpetua.

Fallo:
WHEREFORE, the petition is DENIED. The Decision of the Court of Appeals dated
February 28, 2005 which affirmed with modification the judgment of the Regional
Trial Court of Biñ an, Laguna, Branch 24, finding appellant Isaias Castillo y Completo
guilty of parricide and sentencing him to suffer the penalty of reclusion perpetua and
ordering him to pay the heirs of his victim P50,000.00 as moral damages and
P50,000.00 as civil indemnity, is AFFIRMED. With costs.

VIVAR, 1
CRIMINAL LAW DIGESTS

Elements of Accident <<<

People v. Abrazaldo
G.R. No. 124392, February 7, 2003
SANDOVAL-GUTIERREZ, J:

Doctrine:
The exemption from punishment is based on the complete absence of intelligence,
freedom of action, or intent, or on the absence of negligence on the part of the accused.

Facts:
July 1995, at about 10:00 o'clock in the evening, accused-appellant Abrazaldo, then
intoxicated, attempted to hack his uncle, Bernabe Quinto, but instead hit the post of
the latter's house. The incident was reported to the barangay authorities, prompting
barangay tanods victim Guban, Fajardo, and Loceste to rush to the scene. Upon
reaching the place, Fajardo heard accused-appellant shouting at his uncle, "I will kill
you!" Thereafter, he saw accused- appellant coming out of Quinto's house with blood
oozing from his forehead. At that time, the place was well lighted by a fluorescent
lamp. Guban tried to assist accused-appellant.

However, for unknown reason, accused-appellant and Guban shouted at each other
and grappled "face to face." Accused-appellant pulled out his knife, stabbed Guban at
the abdomen and ran away. When Fajardo got hold of Guban, the latter said, "I was
stabbed by Feding Abrazaldo." Guban was rushed to the hospital but died after a few
hours. The victim's father testified that he was the one who spent for his son’s funeral
expense.

During the trial, the accused-appellant invoked self-defense saying that it was Guban
who, armed with a knife, was the one who attacked him. According to him, it was
Guban who accidentally stabbed himself. The accused-appellant’s very own sister
however testified against him. The trial court appreciated treachery and consequently
found Abrazaldo guilty of murder with the aggravating circumstances that the crime
was committed while the public authorities were engaged in the discharge of their
duties and was committed at nighttime. He was sentenced to suffer death and pay the
heirs of Guban actual expenses.

Issue:
WHETHER OR NOT the justifying circumstance of self-defense or the exempting
circumstance can be invoked by the accused-appellant.

Ruling:
NO. Ingrained in jurisprudence is the Doctrine that the plea of self-defense cannot be
justifiably entertained where it is not only uncorroborated by separate and competent
evidence but in itself is extremely doubtful.

In the present case, accused-appellant showed ambivalence (having or showing


simultaneous and contradictory attitudes or feelings toward something or someone) in
invoking self-defense. While he admitted the commission of the crime in order to
preserve his own life, he maintained that Guban accidentally stabbed himself.

Moreover, the exempting circumstances Under Article 12 of the Revised Penal Code ,
of the accident cannot be appreciated considering accused-appellant’s flight from the
crime scene and his failure to inform the authorities of the incident. More so, the
testimony of the accused was contradicted by his own witness who happened to be his
sister. Standing alone against the testimonies of the prosecution witnesses, accused-
appellant’s own account of the killing must necessarily fail.

Basis, the exemption from punishment is based on the complete absence of


intelligence, freedom of action, or intent, or on the absence of negligence on the
part of the accused.

VIVAR, 1
CRIMINAL LAW DIGESTS

Furthermore, that he did not surrender the knife to the authorities is inconsistent with
a clean conscience and, instead, indicates his culpability of the crime charged.

Fallo:
WHEREFORE, the assailed judgment in Criminal Case No. 95-01052-D is AFFIRMED
with MODIFICATION. Accused-appellant Federico Abrazaldo is declared guilty beyond
reasonable doubt of homicide defined and penalized under Article 249 of the Revised
Penal Code and is sentenced to suffer an indeterminate penalty of six (6) years and 1
day of prision mayor, as minimum, to fourteen (14) years, eight (8) months and one
(1) day of reclusion temporal in its medium period, as maximum. He is ordered to pay
the heirs of the late Delfin Guban P50,000.00 as indemnity and P25,000.00 as
temperate damages. Costs de oficio.

VIVAR, 1
CRIMINAL LAW DIGESTS

Accident- Intent to kill <<<

People v. Latosa
G.R. No. 186128, June 23, 2010
VILLARAMA, JR., J:

Doctrine:
Criminal Law; Parricide; Exempting Circumstances; Accident; Elements.—The basis of
appellant’s defense of accidental shooting is Article 12, paragraph 4 of the Revised
Penal Code, as amended, which provides: ART. 12. Circumstances which exempt from
criminal liability.—The following are exempt from criminal liability: x x x x 4. Any person
who, while performing a lawful act with due care, causes an injury by mere accident
without fault or intention of causing it. Thus, it was incumbent upon appellant to prove
with clear and convincing evidence, the following essential requisites for the
exempting circumstance of accident, to wit: 1. She was performing a lawful act; 2.
With due care;
3. She caused the injury to her husband by mere accident; 4. Without fault or intention
of causing it.

Facts:
 On February 5, 2002, at around 2:00 in the afternoon, Susan Latosa, herein
appellant, together with his husband Major Felixberto Sr. and two children
Sassymae and Michael, were in their house in Fort Bonifacio, Taguig. While Major
Felixberto Sr. was asleep, Sassymae saw her mother take Felixberto Sr.'s gun and
leave. She asked her mother where she was going and if she could come along,
appellant refused. Moments later, appellant returned and told Sassymae to buy ice
cream. After Sassymae left, appellant instructed Michael to join his sister, but he
refused. Appellant thereafter turned up the volume of the television and radio to
full. Shorty after that, appellant gave her son money to buy food.

 After buying his food, Michael went back to their house and thereupon saw his
friend Mac-Mac who told him that he saw appellant running away from their
house. Moments later, a certain Sgt. Ramos arrived and asked if something had
happened in their house. Michael replied in the negative then entered their house.
At that point, he saw his father lying on the bed with a hole in the left portion of his
head and a gun at his left hand.

 Michael immediately went outside and informed Sgt. Ramos about what happened.
Sgt. Ramos told him that appellant had reported the shooting incident to the Provost
Marshall office. Then, Sassymae arrived and saw her father with a bullet wound on
his head and a gun near his left hand.

 Appellant claimed that the killing was an accident, that when Felixberto, Sr. woke
up, he asked her to get his service pistol from the cabinet adjacent to their bed. As
she was handing the pistol to him it suddenly fired, hitting Felixberto, Sr. who was
still lying down.
o The RTC found appellant guilty beyond reasonable doubt for the crime of
parricide. The RTC, in finding appellant guilty, considered the following
circumstantial evidence established by the prosecution: (1) shortly before
the shooting, appellant asked her two (2) children to do errands for her
which were not usually asked of them; (2) at the time of the shooting, only
the appellant and Felixberto, Sr. were in the house; (3) appellant was seen
running away from the house immediately after the shooting; (4) when
Michael went inside their house, he found his father with a hole in the
head and a gun in his left hand; (5) the medico-legal report showed that
the cause of death was intracranial hemorrhage due to the gunshot wound
on the head with the point of entry at the left temporal region; (6) the
Firearms Identification Report concluded that appellant fired two (2) shots;
(7) Felixberto, Sr. was right-handed and the gun was found near his left
hand; (8) Sassymae testified that she heard Sta. Inez tell appellant "bakit
mo inamin. Sana pinahawak mo kay Major iyong baril saka mo pinutok";
(9)appellant's children testified that they were informed by Felixberto,

VIVAR, 1
CRIMINAL LAW DIGESTS

Sr.

VIVAR, 1
CRIMINAL LAW DIGESTS

regarding the threat of appellant's paramour, Sta. Inez, to the whole family;
and (10) Francisco Latosa presented a memorandum showing that
appellant was terminated from her teaching job by reason of immorality.

o The CA upheld the decision of the RTC. The CA held that since appellant
admitted having killed her husband albeit allegedly by accident, she has
the burden of proving the presence of the exempting circumstance of
accident to relieve herself of criminal responsibility. She must rely on the
strength of her own evidence and not on the weakness of the
prosecution, for even if this be weak, it cannot be disbelieved after the
appellant has admitted the killing.

Issue:
Whether or not appellant has strongly established the exempting circumstance of
accident to relieve him from criminal liability.

Ruling:
NO.
SC held that it was incumbent upon appellant to prove with clear and convincing
evidence, the following essential requisites for the exempting circumstance of
accident. To prove the circumstance she must rely on the strength of her own evidence
and not on the weakness of that of the prosecution, for even if this be weak, it can not
be disbelieved after the accused has admitted the killing.

SC find no merit in appellant's contention that the prosecution failed to prove by


circumstantial evidence her motive in killing her husband. Intent to kill and not motive
is the essential element of the offense on which her conviction rests.

The following circumstantial evidence considered by the RTC and affirmed by the CA
satisfactorily established appellant's intent to kill her husband and sustained her
conviction for the crime.

Fallo:
WHEREFORE, the appeal of Susan Latosa y Chico is DISMISSED. The April 23, 2008
Decision of the Court of Appeals in CA-G.R. CR-H.C. No. 02192 is hereby AFFIRMED
with MODIFICATION. The amount of exemplary damages is increased to P30,000.00.
With costs against the accused-appellant.

VIVAR, 1
CRIMINAL LAW DIGESTS

Grave wrong; not passion or obfuscation <<<

Del Poso v. People


G.R. No. 210810, December 7, 2016
PERALTA, J:

Doctrine:
Lack of Intention to Commit So Grave a Wrong; It is a hornbook doctrine that the
mitigating circumstance of lack of intention to commit so grave a wrong can be taken
into account only when the facts proven show that there is a notable and evident
disproportion between the means employed to execute the criminal act and its
consequences.

Facts:
The victim, VVV was given by her biological mother to the petitioner, when she was 7
years old and the latter then acted as her guardian. On September 10, 2005, when VVV
was 9 years old, petitioner ordered her to attend to petitioner's photocopying
business. While attending the business, VVV fell asleep. When petitioner saw VVV
asleep, the former became furious and laid VVV on top of an ironing board and placed
a heated flat iron on her. When VVV tried to evade the heat emanating from the flat iron,
her forehead, right elbow, left cheek, left buttock and back got burned. Thereafter,
petitioner got her down from the ironing board and ordered her to sleep. The
following morning, petitioner's wife saw the burns on VVV and told petitioner not to
do it again. Later on, VVV went to her Lola Ma. Luisa who saw her burns and several
other people, saw the burns prompting Lola Ma. Luisa to bring VVV to the Barangay
Hall where the incident was put on blotter. Thereafter, VVV was brought to the
hospital and then to the police station. Hence, an Information was filed against
petitioner, which reads as follows “That on or about September 10, 2005, in the City of
Manila, Philippines, the said accused, did then and there willfully, unlawfully, and
knowingly commit cruelty and abusive acts upon VVV, a minor, 9 years old, by then
and there injuring the said minor on the forehead, right cheek, abdomen and at her
right forearm with a hot flat iron, inflicting upon her multiple 1st degree burns, which
debases and demeans the intrinsic worth and dignity of said VVV as a human being, an
act prejudicial to her normal growth and development, to her damage and prejudice.”

Petitioner, on the other hand, claimed that the incident happened accidentally.
According to him, on that particular day, he just came from work when he saw VVV
playing under a table and to teach her a lesson, he tried to scare her with a hot flat
iron. Petitioner was then not aware that VVV was hurt as there were no marks on her.
The marks only became evident the following morning. Petitioner claimed that he
applied medication on VVV's burns.

The RTC found petitioner guilty beyond reasonable doubt of violation of Section 10 (a)
of R.A No. 7610 in its Decision dated July 1, 2011, the dispositive portion of which
reads as follows: WHEREFORE, premises considered, the Court finds that the
prosecution has proven the guilt of the accused beyond reasonable doubt from the
crime of violation of Section 10 (a) of RA 7610, "The Special Protection of Children
Against Child Abuse, Exploitation and Discrimination Act" and hereby sentences
Ricardo Del Poso y Cerna to suffer the penalty of four (4) years, nine (9) months and
eleven (11) days of prision correccional, as minimum, to six (6) years, eight (8) months
and one (1) day of prision mayor, as maximum.

The petitioner filed his appeal with the CA and the latter court, in its Decision dated
July 22, 2013, dismissed the same appeal and affirmed the Decision of the RTC.

Hence, the present petition. Petitioner filed an appeal for Petition for Review on
Certiorari with the CA and sought the reversal of the Decision dated July 22, 2013 of
the latter court. Petitioner insists that the CA erred in convicting him when the minor
admitted that she sustained the burns when she tried to evade the heated iron that he
was holding over her while lying on the ironing board just to scare her as a way of
chastening her. He also claims that assuming the CA is correct, it still erred in refusing

VIVAR, 1
CRIMINAL LAW DIGESTS

to appreciate the mitigating circumstances of no intention to commit so grave a wrong


as that committed and passion and/or obfuscation, thus, also erring in not modifying
his sentence to another degree lower.

Issue:
Whether or not the court erred when it refused to appreciate in favor of the petitioner
the mitigating circumstances of (1) no intention to commit so grave a wrong as that
committed and (2) passion and /or obfuscation.

Ruling:
No, the court did commit error when it did not appreciate the mitigating
circumstances of lack of intent to commit so grave a wrong and passion and/or
obfuscation from Art 13 (Par 3, 6). The court finds the petition unmeritorious. It is a
hornbook doctrine that this mitigating circumstance can be taken into account only
when the facts proven show that there is a notable and evident disproportion between
the means employed to execute the criminal act and its consequences. The facts
found by the trial court and the CA show that petitioner intended the natural
consequence of his act. The observation of the OSG that petitioner's intention of
inflicting such harm should be judged in accordance with his previous acts of abusing
the victim, of regarding VVV as a mere adoptive child who is not his blood relative and
petitioner's evident superiority of physique as a fully grown man inflicting harm upon
a 9-year-old victim, and thus, when petitioner pressed the hot iron upon the body of
the victim, it must be presumed that his intention was to physically abuse her since
such act was sufficient to produce the evil which resulted from such act is also worth
noting.

Applying the same set of facts, petitioner is also not entitled to the application of the
mitigating circumstance of passion and/or obfuscation. The mitigating circumstance of
passion or obfuscation only applies if (1) the act of the victim is both unlawful and
sufficient to produce such condition of mind and (2) the act that produced the
obfuscation was not far removed from the commission of the crime by a considerable
length of time, during which the perpetrator might recover his normal equanimity. These
elements were not present in the case. A child who fell asleep while attending to a
business establishment is not an offense at all and not sufficient to produce passion
and raging anger. It could not give rise to an impulse sufficiently powerful to naturally
produce a justified diminution of an adult's self-control, even to a disciplinarian foster
parent. Hence, even to a disciplinarian foster parent.

Hence, the trial court and the CA correctly imposed the penalty by not considering the
mitigating circumstances claimed by petitioner. Section 10 (a) of R.A. No. 7610
imposes the penalty of prision mayor in its minimum period. Applying the
Indeterminate Sentence Law, the trial court did not err when it imposed the penalty of
4 years, 9 months and 11 days of prision correccional, as minimum, to 6 years, 8
months and 1 day of prision mayor, as maximum.

Fallo:
WHEREFORE, the Petition for Review on Certiorari under Rule 45 dated January 28,
2014 of Ricardo Del Poso y Dela Cerna is DENIED for lack merit and the Decision dated
July 22, 2013, dismissing petitioner's appeal and affirming the Decision dated July 1,
2011 of the Regional Trial Court, Branch 38, Manila in Criminal Case No. 05-239429,
convicting petitioner of violation of Section 10 (a) of R.A No. 7610 and imposing upon
petitioner the indeterminate penalty of imprisonment of four (4) years, nine (9) months
and eleven (11) days of prision correccional, as minimum, to six (6) years, eight (8)
months and one (1) day of prision mayor, as maximum, is AFFIRMED.

VIVAR, 1
CRIMINAL LAW DIGESTS

No intention to commit so grave a wrong; sufficient provocation <<<

Urbano v. People
G.R. No. 182750, January 20,
2009 VELASCO, JR., J:

Doctrine:
The third requisite of self-defense is present: (1) when no provocation at all was given to
the aggressor; (2) when, even if provocation was given, it was not sufficient; (3) when
even if the provocation was sufficient, it was not given by the person defending himself;
or (4) when even if a provocation was given by the person defending himself, it was not
proximate and immediate to the act of aggression; The victim’s insulting remarks
directed at the accused, and uttered immediately before the fist fight constitute
sufficient provocation.

While intent to kill may be presumed from the fact of the death of the victim, this
mitigating factor may still be considered when attendant facts and circumstances so
warrant.

Facts:
On September 28, 1993, at around 8:00 p.m., the victim, Brigido Tomelden, and
petitioner, Rodel Urbano, were at the compound of the Lingayen Water District
(LIWAD) in Lingayen, Pangasinan, having just arrived from a picnic in the nearby town
of Bugallon, Pangasinan, where, with some other co-workers, they drank beer in a
restaurant. While inside the compound, the two had a heated altercation, in the course
of which, Tomelden hurled insulting remarks at petitioner. Reacting, petitioner asked
why Tomelden, when drunk, has the penchant of insulting petitioner.

The exchange of words led to an exchange of blows. Cooler heads succeeded in


breaking up the fight, but only for a brief moment as the protagonists refused to be
pacified and continued throwing fist blows at each other. Then petitioner delivered a
"lucky punch", as described by eyewitness Orje Salazar, on Tomelden's face, which
made Tomelden topple down. Tomelden was on the verge of hitting his head on the
ground had their companions not caught him and prevented the fall. The blow,
however, caused Tomelden's nose to bleed and rendered him unconscious.

Petitioner and his other co-workers brought Tomelden to the office of the LIWAD general
manager where he spent the night. He remained in the compound until the following
day, September 29, 1993. Upon arriving home at around 6:00 p.m. of that day,
Tomelden informed his wife, Rosario, of the fight the previous night and of his having
been rendered unconscious. He complained of pain in his nape, head, and ear which
impelled Rosario to immediately bring him to the Lingayen Community Hospital where
Dr. Daisy Arellano examined him and treated his lacerated left index finger, contusions,
and hematoma at the right cerebrum.

On October 2 and 7, 1993, Tomelden went back to the hospital complaining of


dizziness, headache, and other pains. The attending doctors observed the patient to be
in a state of drowsiness and frequent vomiting. On October 8, 1993, Rosario brought
Tomelden to the Sison Memorial Provincial Hospital in Dagupan City, where the
attending physician, Dr. Ramon Ramos, diagnosed Tomelden suffering from "brain
injury, secondary to mauling to consider cerebral hemorrhage".

Tomelden was confined in the provincial hospital until 3:00 p.m. of October 10, 1993,
and, due to financial constraints, was thereafter discharged despite signs negating
physical condition improvement. Upon reaching their house, however, Tomelden again
complained of extreme head pain, prompting his wife to bring him back to the Lingayen
Community Hospital where Dr. Arellano again attended to him. This time, things turned
for the worst, the doctor noting that Tomelden appeared to be semi-conscious, sleepy,
uncooperative, and not responding to any stimulant. Tomelden died at 9:00 p.m. of that
day due, per Dr. Arellano, to "cardiorespiratory arrest secondary to cerebral concussion
with resultant cerebral hemorrhage due to mauling incident".

VIVAR, 1
CRIMINAL LAW DIGESTS

The defense presented petitioner who denied having any intention to kill, asserting
that hypertension, for which Tomelden was receiving treatment, was the cause of the
latter's death.

On April 30, 2001, the RTC rendered judgment finding petitioner guilty of Homicide.
On January 25, 2008, the CA rendered a decision, affirming the conviction of
petitioner, but awarding moral damages to the heirs of Tomelden.

Issue:
Whether or not the court erred in not appreciating the mitigating circumstances of
sufficient provocation on the part of the victim and lack of intent to commit so grave a
wrong in favor of the petitioner.

Ruling:
Yes, the court erred in not appreciating the mitigating circumstances. Petitioner guilty
of homicide.

1. Sufficient Provocation
When the law speaks of provocation either as a mitigating circumstance or as an
essential element of self-defense, the reference is to an unjust or improper conduct
of the offended party capable of exciting, inciting, or irritating anyone; it is not
enough that the provocative act be unreasonable or annoying; the provocation
must be sufficient to excite one to commit the wrongful act and should immediately
precede the act. This third requisite of self-defense is present: (1) when no
provocation at all was given to the aggressor; (2) when, even if provocation was given,
it was not sufficient; (3) when even if the provocation was sufficient, it was not
given by the person defending himself; or (4) when even if a provocation was given
by the person defending himself, it was not proximate and immediate to the act of
aggression.

In the instant case, Tomelden’s insulting remarks directed at petitioner and


uttered immediately before the fist fight constituted sufficient provocation.
This is not to mention other irritating statements made by the deceased
while they were having beer in Bugallon. Petitioner was the one provoked
and challenged to a fist fight.

Tomelden insulted petitioner, telling the latter he had no business stopping him
from further drinking. Petitioner returned to his table to report to Navarro. At that
time Tomelden had already consumed 17 bottles of beer. Tomelden allegedly
slapped and hurled insults at him, calling him "sipsip" just to maintain his
employment as Navarro’s tricycle driver. Tomelden allegedly then delivered
several fist and kick blows at petitioner, a couple of which hit him despite his
evasive actions. Petitioner maintained that he only boxed the victim in retaliation,
landing that lucky punch in the course of parrying the latter’s blows.

2. No intention to commit so grave a wrong as that committed.


While intent to kill may be presumed from the fact of the death of the victim, this
mitigating factor may still be considered when attendant facts and circumstances
so warrant, as in the instant case. Consider: Petitioner tried to avoid the fight, being
very much smaller than Tomelden. He tried to parry the blows of Tomelden , albeit
he was able, during the scuffle, to connect a lucky punch that ended the fight. He
even helped carry his unconscious co-worker to the office of the LIWAD’s general
manager. Surely, such gesture cannot reasonably be expected from one intending
to commit so grave a wrong as killing the victim. A bare-knuckle fight as a means to
parry the challenge issued by Tomelden was commensurate to the potential
violence petitioner was facing. It was just unfortunate that Tomelden died from
that lucky punch, an eventuality that could have possibly been averted had he had
the financial means to get the proper medical attention.

VIVAR, 1
CRIMINAL LAW DIGESTS

Fallo:
WHEREFORE, the CA Decision dated January 25, 2008 in CA-G.R. CR No. 25371 is, in
the light of the presence and the appreciation of two mitigating circumstances in favor
of petitioner, hereby MODIFIED by decreasing the term of imprisonment. As thus
modified, petitioner Rodel Urbano is hereby sentenced to serve an indeterminate
prison term of from two (2) years and four (4) months of prision correccional, as
minimum, to eight (8) years and one (1) day of prision mayor, as maximum, with
whatever imprisonment he has already served fully credited in the service of this
sentence. The rest of the judgment is hereby AFFIRMED.

VIVAR, 1
CRIMINAL LAW DIGESTS

Immediate vindication; passion or obfuscation, voluntary surrender <<<

People v. Ignas
G.R. No. 140514-15, September 30, 2003
QUISUMBING, J:

Doctrine:
The word “immediate” in the English text is not the correct translation of the controlling
Spanish text of the Revised Penal Code, which uses the word “proxima”—the Spanish
text, on this point, allows a lapse of time between the grave offense and the actual
vindication; The lapse of two (2) weeks between the discovery by the accused of his wife’s
infidelity and the killing of her supposed paramour could no longer be considered
proximate.

It is not enough that the special aggravating circumstance of use of unlicensed firearm
be alleged in the information, the matter must be proven with the same quantum of
proof as the killing itself—the prosecution must prove (1) the existence of the subject
firearm, and, (2) the fact that the accused who owned or possessed it does not have the
corresponding license or permit to own or possess the same.

Facts:
 Appellant June Ignas is an elementary school graduate and operated a bakery.
He is married to Wilma, and they have a minor son. Wilma confided to her close
friend (Romenda) that she was having an affair with one Nemesio Lopate.
 From Benguet, Wilma, the close friend, and Nemesio went to Manila. They were
sending Wilma off at NAIA as she was leaving for Taiwan to work as a Domestic
Helper. The trio checked in at Dangwa hotel and Nemesio and Wilma shared a
room.
 Thereafter, Romenda received from Taiwan 4 letters written by Wilma on
various dates. Although all the letters were addressed to Romenda, two of them
were meant to be read by her paramour, Nemesio. In the other two letters,
Wilma instructed Romenda to reveal to appellant her affair with Nemesio.
 In February 1996, Romenda informed appellant about the extramarital affair.
Romenda informed him about the hotel incident. Appellant became furious and
declared in Ilocano (There will be a day for that Nemesio. I will kill that
Nemesio). Appellant then got all the letters of Wilma from Romenda.
 That same week Alfred Mayamnes had a talk with appellant. Mayamnes was an
elder of the Kankanaey tribe to which appellant and Nemesio belonged to. He
wanted to confirm whether Nemesio was having an affair with appellant’s
spouse. Talk apparently had reached the tribal elders and they wanted the
problem resolved as soon as possible. A visibly angry appellant confirmed the
gossip. Mayamnes also testified that he advised Nemesio to stay at the Mountain
Trail Kankanaey community until things had cooled down.
 After their talk, appellant closed down his bakeshop and offered his equipment
for sale. Mayamnes saw appellant load his bakery equipment on board a hired
truck and depart for Nueva Vizcaya.
 A witness said she was at the unloading area at the Trading Post, Benguet, when
suddenly 2 gunshots shattered the quiet evening. She turned towards the place
where the sound came from where she saw a person falling to the ground.
Standing behind the fallen individual, some 16 inches away, was another person
who tucked a handgun into his waistband and casually walked away. She caught
a glimpse of the face = it was the appellant. She was 5 meters away from the
scene and the taillight of a parked jeepney, plus roof lights from the bagsakan
shed, helped her vision.
 Also at the bagsakan area was another witness who testified that on hearing
gunshots, he saw people converging on a spot where a bloodied figure was lying
on the ground. He saw that the fallen victim was Nemesio then saw another
person, some 25 meters away, hastily walking away and identified the latter as
his close friend & neighbor June.

VIVAR, 1
CRIMINAL LAW DIGESTS

 Mona, a bakery worker, testified that at the night after the incident, appellant
came to her residence at La Trinidad. After being served refreshments, appellant
took out a handgun from his jacket and removed the empty shells from the
chamber. Appellant then told her to throw the empty cartridges out of the window.
She also said that appellant disclosed to her that he had just shot his wife’s
paramour. Appellant stayed there for 9 hours and left in the morning.
 According to witnesses on the scene, responding policemen immediately
brought the victim Nemesio Lopate, to the Benguet General Hospital where he
was pronounced dead on arrival.
 Police investigators, accompanied by one of appellant’s brothers, as well as
prosecution witness Julio went to Nueva Vizcaya, to invite appellant to shed
light on the slaying of Nemesio. The law enforcers found appellant selling bread
at Kayapa and brought him back to La Trinidad. Julio testified that shortly after
they arrived from Kayapa, appellant disclosed that he shot and killed Nemesio.
Prosecution witness Pauline Gumpic, the victim’s sister, testified that she and
appellant had a private talk, while the latter was in police custody, and
appellant admitted to her that he killed her brother. SPO4 Arthur Bomagao of
the La Trinidad police, who headed the team that investigated the fatal shooting
of Nemesio, declared on the stand that appellant voluntarily admitted to him
that he shot the victim with a .38 caliber handgun. Bomagao further testified
that appellant surrendered to him the letters of Wilma Grace, wherein the latter
admitted her affair with Nemesio.
 RTC (GUILTY of MURDER with aggravating circumstance of the use of
unlicensed firearm)
 SC (GUILTY of HOMICIDE only without any aggravating nor mitigating
circumstance)

Issue:
1. Whether or not the Trial Court committed reversible error when it appreciated the
alleged use of an unlicensed .38 calibre firearm as an aggravating circumstance in
the commission of the crime of murder without any factual and legal basis.
2. Whether or not the Trial Court committed reversible error when it did not appreciate
in favour of the Accused-Appellant the Mitigating Circumstances of (a) immediate
vindication of a grave offense, (b) passion and obfuscation and (c) voluntary
surrender.

Ruling:
1. In the first issue raised, the Supreme Court ruled that:
“We find merit in the appellants contentions. It is not enough that the special
aggravating circumstance of use of unlicensed firearm be alleged in the
information, the matter must be proven with the same quantum of proof as the
killing itself... The records do not show that the prosecution presented any
evidence to prove that appellant is not a duly licensed holder of a calibre .38
firearm… Absent the proper evidentiary proof, this Court cannot validly declare
that the special aggravating circumstance of use of unlicensed firearm was
satisfactorily established by the Prosecution. Hence, such special circumstance
cannot be considered for purposes of imposing the penalty in its maximum
period.”

2. As for the second issue, however, the Supreme Court ruled that:
a. “The Solicitor General counters that there was literally no immediate
vindication to speak of in this case. Appellant had sufficient time to recover
his serenity following the discovery of his wife’s infidelity.
…We agree with the Solicitor General that the lapse of two (2) weeks
between his discovery of his wife’s infidelity and the killing of her supposed
paramour could no longer be considered proximate. The passage of a
fortnight is more than sufficient time for appellant to have recovered his
composure and assuaged the ease of his mind. The established rule is
that there can be no immediate vindication of a grave offense when
the accused had sufficient time to recover his serenity. Thus, in this
case, we hold that the mitigating circumstance of immediate vindication

VIVAR, 1
CRIMINAL LAW DIGESTS

of a grave offense cannot be considered in appellants favor.”

VIVAR, 1
CRIMINAL LAW DIGESTS

b. “We likewise find the alleged mitigating circumstance of passion and


obfuscation inexistent. The rule is that the mitigating circumstances of
vindication of a grave offense and passion and obfuscation cannot be
claimed at the same time, if they arise from the same facts or motive. In
other words, if appellant attacked his victim in proximate vindication of a
grave offense, he could no longer claim in the same breath that passion
and obfuscation also blinded him.”

c. “On this point, the following requirements must be satisfied: (1) the
offender has not actually been arrested; (2) the offender surrendered
himself to a person in authority; and (3) the surrender was voluntary.
Records show, however, that leaflets and posters were circulated for
information to bring the killer of Nemesio to justice. A team of police
investigators from La Trinidad, Benguet then went to Kayapa, Nueva
Vizcaya to invite appellant for questioning. Only then did he return to
Benguet. But he denied the charge of killing the victim. Clearly, appellants
claimed surrender was neither spontaneous nor voluntary.”

Fallo:
WHEREFORE, the judgment of the Regional Trial Court of La Trinidad, Benguet, Branch 8, in Criminal
Case No. 96-CR-2522 is MODIFIED.
Appellant June Ignas y Sanggino is found GUILTY beyond reasonable doubt of the crime of HOMICIDE as
defined and penalized under Article 249 of the Revised Penal Code, as amended. There being neither
aggravating nor mitigating circumstance, he is hereby sentenced to suffer an indeterminate penalty of ten
(10) years and one (1) day of prision mayor as minimum, to fourteen (14) years, eight (8) months, and one
(1) day of reclusion temporal as maximum.

VIVAR, 1
CRIMINAL LAW DIGESTS

Voluntary surrender <<<

Nizurtado v. Sandiganbayan
G.R. No. 107838, December 7, 1994
VITUG, J:

Doctrine:
The presence of the third mitigating circumstance of praeter intentionem (lack of
intention to commit so grave a wrong as that committed) would result in imposing a
period the court may deem applicable. Considering, however, that the penalty has to be
imposed in the maximum period, the only effect of this additional mitigating
circumstance is to impose only the minimum portion of that maximum period, that is,
from eight years, eight months and one day to nine years, six months and ten days,
from which range the maximum of the indeterminate sentence shall be taken.

Facts:
It appears from the evidence, testimonial and documentary, as well as from the
stipulations of the parties that accused Felix V. Nizurtado was the Barangay Captain of
Barangay Panghulo, Malabon, Metro Manila from 1983 to 1988. Nizurtado and Manuel
P. Romero, Barangay Treasurer of Panghulo, attended a seminar at the University of
Life, Pasig, Metro Manila. The seminar was about the Barangay Livelihood Program of
the Ministry of Human Settlements (MHS), the Metro Manila Commission (MMC), and
the Kilusang Kabuhayan at Kaunlaran (KKK). Under the program, the barangays in
Metro Manila could avail of loans of P10,000.00 per barangay to finance viable livelihood
projects which the Barangay Councils would identify from the modules developed by the
KKK Secretariat or which, in the absence of such modules, the Councils would choose
subject to the evaluation/validation of the Secretariat.

After the seminar, Nizurtado received a check for P10,000.00 intended for Barangay
Panghulo and issued in his name. The check, however, could be encashed only upon
submission to the Secretariat of a resolution approved by the Barangay Council
identifying the livelihood project in which the loan would be invested. He entrusted
the check to Romero for safekeeping. In one of its regular sessions, which was on the
second Saturday of each month, the Barangay Council of Panghulo discussed the
project in which to invest the P10,000.00. Among the proposals was that of Romero
that a barangay service center be established. But the meeting ended without the
Councilmen agreeing on any livelihood project.

A few days after the meeting, Nizurtado got back the check from Romero, saying that he
would return it because, as admitted by Nizurtado during the trial, the Councilmen
could not agree on any livelihood project. Nizurtado signed a receipt dated August 4,
1983, for the check "to be returned to the Metro Manila Commission." After a few more
days, Nizurtado asked Romero to sign an unaccomplished resolution in mimeograph
form. All the blank spaces in the form were unfilled-up, except those at the bottom
which were intended for the names of the Barangay Councilmen, Secretary, and
Captain, which were already filled-up and signed by Councilmen Marcelo Sandel, Jose
Bautista, Alfredo Aguilar, Alfredo Dalmacio, F.A. Manalang (the alleged Barangay
Secretary), and Nizurtado. In asking Romero to sign, Nizurtado said that the MMC was
hurrying up the matter and that the livelihood project to be stated in the resolution
was that proposed by Romero — barangay service center. Trusting Nizurtado, Romero
affixed his signature above his typewritten name. When he did so, the blank resolution
did not yet bear the signatures of Councilmen Santos Gomez and Ceferino Roldan.

Romero and Gomez made inquiries. They learned that the check for P10,000.00 was
indeed encashed by Nizurtado and that the blank resolution which they had signed
was filled-up to make it appear that in a Council meeting where all councilmen were
present on August 25, 1983, T-shirt manufacturing was adopted as the livelihood
project of Panghulo. But no such meeting occurred on that day or on any other day.
Neither was Nizurtado authorized by the Council to submit T-shirt Manufacturing as
the livelihood project of Panghulo.

VIVAR, 1
CRIMINAL LAW DIGESTS

A case was filed and the Sandiganbayan convicted Nizurtado guilty beyond reasonable
doubt of the complex crime of malversation of public funds committed through
falsification of public document.

On appeal, Nizurtado sought to justify the questioned act in that it was only when the
members of the Barangay Council had realized that P10,000.00 was not enough to
support the T-shirt manufacturing project, that they decided to distribute the money
in the form of loans to themselves.

Issue:
Whether or not the accused is guilty of complex crime of malversation of public funds
through falsification of public document.

Ruling:
Yes.
Nizurtado was a public officer, having been the Barangay Captain of Panghulo,
Malabon, Metro Manila, from 1983 to 1988; in that capacity, he received and later
encashed a check for P10,000.00, specifically intended by way of a loan to the
barangay for its livelihood program; and the funds had come from the Ministry of
Human Settlements, the Metro Manila Commission and "Kilusang Kabuhayan at
Kaunlaran."

Petitioner was able to encash the check on 18 October 1988 on the basis of a resolution
of the Barangay Council, submitted to the KKK Secretariat, to the effect that a livelihood
project, i.e., "T-shirt manufacturing," had already been identified by the council. The
money, however, instead of its being used for the project, was later lent to, along with
petitioner, the members of the Barangay Council. Undoubtedly, the act constituted
"misappropriation" within the meaning of the law.

Also, art. 171. Falsification by public officer, employee or notary or ecclesiastic minister.
—The penalty of prision mayor and a fine not to exceed 5,000 pesos shall be imposed
upon any public officer, employee, or notary who, taking advantage of his official
position, shall falsify a document by committing any of the following acts:

2. Causing it to appear that persons have participated in any act or proceeding


when they did not in fact so participate; In falsification under the above-quoted
paragraph, the document need not be an authentic official paper since its
simulation, in fact, is the essence of falsification. So, also, the signatures
appearing thereon need not necessarily be forged.

In concluding that the Barangay Council resolution, Exhibit "D," was a falsified
document for which petitioner should be held responsible, the Sandiganbayan gave
credence to the testimonies of Barangay Councilman Santos A. Gomez and Barangay
Treasurer Manuel P. Romero. The two testified that no meeting had actually taken
place on 25 August 1983, the date when "T-shirt manufacturing" was allegedly decided
to be the barangay livelihood project. The Sandiganbayan concluded that Nizurtado
had induced Romero and Gomez to sign the blank resolution, Exhibit "J" 13 on the
representation that Romero's proposal to build a barangay service center would so
later be indicated in that resolution as the barangay livelihood project.

The supreme court said restitution is akin to voluntary surrender. So aside from the
real voluntary surrender, another one shall be appreciated because he returned the
funds. Such restitution is akin to voluntary surrender and as such, shall be treated
as another mitigating circumstance.

Fallo:
WHEREFORE, the decision of the Sandiganbayan convicting Nizurtado for
malversation of public funds through falsification of public document is AFFIRMED but
the sentence, given the circumstances here obtaining, is MODIFIED by imposing on
petitioner a reduced indeterminate sentence of from two years, four months and one
day to eight years, eight months and one day, perpetual special disqualification and a
fine of P2,000.00. SO ORDERED.

VIVAR, 1
CRIMINAL LAW DIGESTS

Passion or obfuscation; Illness <<<

People v. Genosa
G.R. No. 135981, January 15,
2004 PANGANIBAN, J:

Doctrine:
In addition, we also find in favor of appellant the extenuating circumstance of having
acted upon an impulse so powerful as to have naturally produced passion and
obfuscation. It has been held that this state of mind is present when a crime is
committed as a result of an uncontrollable burst of passion provoked by prior unjust
or improper acts or by a legitimate stimulus so powerful as to overcome reason. To
appreciate this circumstance, the following requisites should concur: (1) there is an
act, both unlawful and sufficient to produce such a condition of mind; and (2) this act
is not far removed from the commission of the crime by a considerable length of time,
during which the accused might recover her normal equanimity.

Case Summary:
Marivic Genosa (Marivic) attacked and wounded his husband (Ben), which ultimately
led to his death. According to Marivic, she did not provoke her husband when she got
home that night it was her husband who began the provocation. Also, that she was
frightened that her husband would hurt her and she wanted to make sure she would
deliver her baby safely. In fact, Marivic had to be admitted later at the Rizal Medical
Centre as she was suffering from eclampsia and hypertension, and the baby was born
prematurely. During their marriage she had tried to leave her husband at least five (5)
times, but that Ben would always follow her and they would reconcile. Marivic alleges
that the reason why Ben was violent and abusive towards her that night was because
'he was crazy about his recent girlfriend, Lulu Rubillos. After being interviewed by
specialists, Marivic has been shown to be suffering from Battered Woman Syndrome.
Marivic, with a plea of self-defense admitted the killing of her husband, was found
guilty of Parricide, with the aggravating circumstance of treachery, for the husband
was attacked while asleep. The Supreme Court affirmed her conviction.

Facts:
1. Facts Leading to the Automatic Review:
a. Before the killing of Ben Genosa, by his wife Marivic Genosa, Marivic. Marivic and
Ben lived happily in their first year of marriage but Ben changed and the couple
would always quarrel and sometimes their quarrels became violent.
b. Marivic testified that every time her husband came home drunk, he would
provoke her and sometimes beat her. Whenever beaten by her husband, she
consulted medical doctors who testified during the trial.
c. On the night of the killing, Marivic and the victim were quarreled and the victim
beat the Marivic. However, Marivic was able to run to another room. Marivic
admitted having killed the victim with the use of a gun. The information for
parricide against Marivic, however, alleged that the cause of death of the victim
was by beating through the use of a lead pipe.
d. Marivic invoked self defense and defense of her unborn child. After trial, the
Regional Trial Court found Marivic guilty beyond reasonable doubt of the crime
of parricide with an aggravating circumstance of treachery and imposed the
penalty of death.
e. On automatic review before the Supreme Court, Marivic filed an URGENT
OMNIBUS MOTION praying that the Honorable Court allow (1) the exhumation
of Ben Genosa and the re-examination of the cause of his death; (2) the
examination of Marivic Genosa by qualified psychologists and psychiatrists to
determine her state of mind at the time she killed her husband; and finally, (3)
the inclusion of the said experts’ reports in the records of the case for purposes
of the automatic review or, in the alternative, a partial re-opening of the case a
quo to take the testimony of said psychologists and psychiatrists.
f. Supreme Court partly granted the URGENT OMNIBUS MOTION of Marivic. It
remanded the case to the trial court for reception of expert psychological
and/or psychiatric opinion on the “buttered woman syndrome” plea.
Testimonies of two

VIVAR, 1
CRIMINAL LAW DIGESTS

expert witnesses on the “battered woman syndrome”, Dra. Dayan and Dr.
Pajarillo, were presented and admitted by the trial court and subsequently
submitted to the Supreme Court as part of the records.

2. Ruling of the Trial Court


Finding the theory of self-defense untenable, RTC gave credence to prosecution
evidence that Marivic had killed the deceased while he was in bed sleeping.
Further, RTC appreciated the generic aggravating circumstance of treachery,
because Ben Genosa was supposedly defenseless when he was killed lying in
bed asleep when Marivic smashed him with a pipe at the back of his head. The
capital penalty having been imposed, the case was elevated to this Court for
automatic review.

Issue:
Whether or not appellant herein can validly invoke the “battered woman syndrome” as
constituting self- defense – No

Ruling:

Self-Defense and Defense of a Fetus


Marivic admits killing Ben Genosa but, to avoid criminal liability, invokes self-defense
and/or defense of her unborn child. When the accused admits killing the victim, it is
incumbent upon her to prove any claimed justifying circumstance by clear and
convincing evidence. Self-defense (and similarly, defense of a stranger or third person)
shifts the burden of proof from the prosecution to the defense.

BWS as Self-Defense
Existence of the syndrome in a relationship does not in itself establish the legal
right of the woman to kill her abusive partner. Evidence must still be considered in
the context of self-defense. The Court reckons further that crucial to the BWS defense
is the state of mind of the battered woman at the time of the offense she must have
actually feared imminent harm from her batterer and honestly believed in the need to
kill him in order to save her life.

The rule is that the one who resorts to self-defense must face a real threat on one’s life;
and the peril sought to be avoided must be imminent and actual, not merely
imaginary. Thus, the Revised Penal Code provides the following requisites and effect of
self-defense:

Art. 11. Justifying circumstances. The following do not incur any criminal liability: (1)
Anyone who acts in defense of his person or rights, provided that the following
circumstances concur: First. Unlawful aggression; Second. Reasonable necessity of the
means employed to prevent or repel it; Third. Lack of sufficient provocation on the part
of the person defending himself .

Unlawful aggression presupposes actual, sudden and unexpected attack or an


imminent danger. In the present case, there was a sufficient time interval between the
unlawful aggression of Ben and her fatal attack upon him. She had already been able
to withdraw from his violent behavior and escape to their children’s bedroom. During
that time, he apparently ceased his attack and went to bed. The reality or even the
imminence of the danger he posed had ended altogether. He was no longer in a
position that presented an actual threat on her life or safety. Had Ben still been
awaiting Marivic when she came out of their children’s bedroom and based on past
violent incidents, there was a great probability that he would still have pursued her
and inflicted graver harm then, the imminence of the real threat upon her life would
not have ceased yet.

Where Self-defense may be appreciated (considering circumstances and existence of BWS)


Where the brutalized person is already suffering from BWS, further evidence of actual
physical assault at the time of the killing is not required. Incidents of domestic battery

VIVAR, 1
CRIMINAL LAW DIGESTS

usually have a predictable pattern. Torequire the battered person to await an obvious,
deadly attack before she can defend her life would amount to sentencing her to murder
by installment. Still, impending danger (based on the conduct of the victim in previous
battering episodes) prior to the defendant’s use of deadly force must be shown.
Threatening behavior or communication can satisfy the required imminence of danger.

Considering such circumstances and the existence of BWS, self-defense may be


appreciated. We reiterate the principle that aggression, if not continuous, does not
warrant self-defense. In the absence of such aggression, there can be no self-defense
complete or incomplete on the part of the victim.
.
Thus Marivic’s killing of Ben was not completely justified under the circumstances.

Mitigating Circumstances Present


The mitigating factors of psychological paralysis and passion and obfuscation
were, however, taken in favor of Marivic. It should be clarified that these two
circumstances -
- psychological paralysis as well as passion and obfuscation -- did not arise from the
same set of facts. The first circumstance arose from the cyclical nature and the severity
of the battery inflicted by the batterer-spouse upon Marivic.

That is, the repeated beatings over a period of time resulted in her psychological
paralysis, which was analogous to an illness diminishing the exercise of her will
power without depriving her of consciousness of her acts.

As to the extenuating circumstance of having acted upon an impulse so powerful as to


have naturally produced passion and obfuscation, it has been held that this state of
mind is present when a crime is committed as a result of an uncontrollable burst of
passion provoked by prior unjust or improper acts or by a legitimate stimulus so
powerful as to overcome reason. To appreciate this circumstance, the following
requisites should concur: (1) there is an act, both unlawful and sufficient to produce
such a condition of mind; and (2) this act is not far removed from the commission of
the crime by a considerable length of time, during which the accused might recover
her normal equanimity.

Fallo:
WHEREFORE, the conviction of Marivic Genosa for parricide is hereby AFFIRMED.
However, there being two(2) mitigating circumstances and no aggravating circumstance
attending her commission of the offense, her penalty is REDUCED.

Inasmuch as Marivic has been detained for more than the minimum penalty hereby
imposed upon her, the director of the Bureau of Corrections may immediately RELEASE
her from custody upon due determination that she is eligible for parole, unless she is
being held for some other lawful cause.

VIVAR, 1
CRIMINAL LAW DIGESTS

No Payment of the amount malversed <<<

Manuel v. Sandiganbayan
G.R. No. 158413, February 8, 2012
MENDOZA, J:

Doctrine:
Payment of the amount malversed will only serve as a mitigating circumstance akin to
voluntary surrender, as provided for in paragraph 7 of Article 13 in relation to paragraph
10 of the same Article of the Revised Penal Code.

Facts:
Mallare and Gosudan, Mayor and Treasurer, respectively, of the Municipality of Infanta,
Pangasinan were charged with the crime of malversation of public funds.

When COA Auditor Emilie S. Ritua (Ritua) requested Gosudan to immediately produce
the missing funds and to explain why there was a shortage in the accounting of
municipal funds, she failed to immediately do so. The best that she could do was to
explain that the subject amount was lent to some municipal officials and employees.
Gosudan presented an informal list of the borrowers who were granted "vales" or
"pautang".

On January 9, 2002, Mallare and Gosudan filed their Motion To Reopen Proceedings.
On May 20, 2002, the Sandiganbayan issued its Resolution granting the Motion To
Reopen Proceedings and allowing the reception of Mallare’s testimony.

On June 10, 2003, Celso M. Manuel, Evangelista A. Meru and Florante A. Miano
(petitioners) filed a petition for certiorari with prayer for the issuance of a writ of
preliminary injunction and/or temporary restraining order dated May 30, 2003,
docketed as G.R. No. 158413, particularly assailing the Sandiganbayan’s Order
granting the reopening of the subject criminal case.

Thereafter, on July 21, 2003, the Sandiganbayan issued a resolution, affirming its
September 17, 2001 Decision which convicted Mallare and Gosudan of the crime of
Malversation of Public Funds beyond reasonable doubt after its reception of additional
evidence during the re-opened proceedings.

Issue:
Whether or not the Sandiganbayan was correct in finding Mallare and Gosudan guilty
beyond reasonable doubt of the crime of Malversation of Public Funds.

Ruling:
Yes, The failure of a public officer to have duly forthcoming any public fund or property
with which he is chargeable, upon demand by any duly authorized officer, shall be prima
facie evidence that he has put such missing funds or property to personal uses.

To sustain a criminal conviction for the crime of Malversation of Public Funds under
Article 217 of the Revised Penal Code, as amended, all the following elements must be
present:
1. That the offender is a public officer;
2. That he had custody or control of funds or property by reason of the duties of his
office;
3. That those funds or property were public funds or property for which he was
accountable; and
4. That he appropriated, took, misappropriated or consented or, through abandonment
or negligence, permitted another person to take them.

All the elements of the crime of Malversation of Public Funds were present in this case
considering that 1) Mallare and Gosudan were public officers being the Mayor and
Municipal Treasurer, respectively, of Infanta, Pangasinan; 2) Gosudan, as Municipal
Treasurer, had custody of public funds thereby making her accountable for these
funds;

VIVAR, 1
CRIMINAL LAW DIGESTS

3) Godusan granted loans to herself and her co-employees; and 4) Mallare signed the
confirmation letter stating that he had outstanding loans received from Gosudan.

Mallare and Gosudan were accountable for public funds or property as Municipal
Mayor and Treasurer, respectively, they had the sworn duty to safely keep said funds
and disburse the same in accordance with standard procedure because the subject
funds belong to the municipality and must only be used for the benefit of the
municipality.

Fallo:
WHEREFORE, the petition is DENIED. The September 17, 2001 decision of the
Sandiganbayan in Criminal Case No. 25673 for Malversation of Public Funds is
AFFIRMED.

VIVAR, 1
CRIMINAL LAW DIGESTS

Voluntary Surrender <<<

People v. Callet
G.R. No. 135701, May 9,
2002 PUNO, J:

Doctrine:
Voluntary surrender requires that the offender had not been actually arrested; that he
surrendered himself to a person in authority or to the latter’s agent; and that the
surrender was voluntary.

Facts:
The accused, ELBERT CALLET y SABANAL was charged with Murder before the
Regional Trial Court of Negros Oriental, Dumaguete City, Branch 30. The crime was
allegedly committed as follows:
 on September 15, 1996, at 5:00 p.m., the victim, Alfredo Senador, his 12-year
old son, Lecpoy Senador, and Eduardo Perater were at the flea market of
barangay Tambulan, Tayasan, Negros Oriental. There were many people in the
vicinity. Some were playing cara y cruz while others were playing volleyball.
 Alfredo, Lecpoy and Eduardo were beside each other as they watched a cara y
cruz game. Alfredo sat close to the ground, with his buttocks resting on his right
foot. Lecpoy and Eduardo sat on a piece of wood and on a stone, respectively.
 Out of nowhere, the accused, Elbert Callet, appeared behind Alfredo and
stabbed the latter on the left shoulder near the base of the neck with a 9-inch
hunting knife. Instinctively, Alfredo stood up and managed to walk a few
meters. When he fell on the ground, Lecpoy and Eduardo rushed to help him but
to no avail. Alfredo died shortly thereafter.
 The accused claims that his liability should be mitigated by the fact that he had
no intention to commit so grave a wrong. We are not persuaded.

Issue:
Whether or not voluntary surrender of the accused mitigates his liability.

Ruling:
Yes. Voluntary surrender of the accused mitigates his liability.
Voluntary surrender requires that the offender had not been actually arrested; that he
surrendered himself to a person in authority or to the latter’s agent; and that the
surrender was voluntary.

The records reveal that the accused ran toward the municipal building after the
stabbing incident. On his way to the municipal building, he admitted to Barangay
Tanods Nilo Callet and Jesus Dagodog that he stabbed the victim. Although he did not
immediately turn over his weapon to them for fear of retaliation from the victim’s
relatives, he did so as soon as they reached the municipal building. Undoubtedly, the
conduct he displayed was spontaneous as it shows his interest to give himself up
unconditionally to the authorities, thus saving the State the trouble and expenses
necessarily incurred in his search and capture.

Fallo:
IN VIEW WHEREOF, the decision appealed from, finding the accused, ELBERT CALLET,
guilty beyond reasonable doubt of Murder in Criminal Case No. 12995, and sentencing
him to suffer reclusion perpetua and to pay the legal heirs of the victim, ALFREDO
SENADOR, the amount of P50,000.00 as civil indemnity, and to pay the costs, is
AFFIRMED.

VIVAR, 1
CRIMINAL LAW DIGESTS

Passion or Obfuscation <<<

People vs. Adlawan


G.R. No. 131839, January 30,
2002 YNARES-SANTIAGO, J:

Doctrine:
The requisites of this mitigating circumstance are: (1) that there be an act, both unlawful
and sufficient to produce such a condition of mind; and (2) said act which produced the
obfuscation was not far removed from the commission of the crime by a considerable
length of time, during which the perpetrator might recover his normal equanimity.

Facts:
At dawn of November 15, 1992, the deceased, together with prosecution witnesses
Benjamin Basubas and Quirino Cinco, and a certain Oliver Bonayan, were inside a
fenced disco area in Sitio Oril, Mandaue City.

All of a sudden, accused-appellant drew a gun from his waist, pointed it at the
deceased, saying, "this is the one." He immediately fired the gun, hitting the deceased
on the chest. The necropsy report shows that the cause of death: SHOCK,
IRREVERSIBLE. Secondary to Massive Hemorrhage due to Multiple Gun Shot Wounds.

But the accused claimed that at around 2:00 in the morning of November 15, 1992,
while he was inside a fenced disco area in Sitio Oril, Mandaue City, he heard somebody
shouting and when he turned to the source of the disturbance, he saw his father, lying
on the ground unconscious and with a bloodied face. Accused dashed to his father
whom he thought was already dead.

As he tried to lift him, he saw the deceased holding a gun and told him, "Do you want
to follow your father?" Thereafter, accused lunged at the deceased, twisted his hand,
forcing the muzzle of the gun to be pointed at the deceased’s chest. Suddenly, the gun
went off, causing the deceased to fall in a canal. Accused was able to get hold of the gun
and again fired at the deceased. Thereafter, he fled and hid in Manila until January 23,
1997, when he finally decided to surrender to Mayor Alfredo M. Ouano and P/Supt.
Rolando Borres.

The lower court rendered decision and found the accused guilty of Murder and to serve
the indeterminate penalty by imprisonment of TEN (10) YEARS and ONE (1) DAY of
prision mayor as minimum to SEVENTEEN (17) YEARS and ONE (1) DAY of reclusion
temporal as maximum.

Issue:
Whether or not the accused is entitled for the mitigating circumstance of passion or
obfuscation.
Whether or not the accused is entitled for the privileged mitigating circumstance for
self defense/ defense of relative.

Ruling:
The Trial Court correctly appreciated the qualifying circumstance of treachery on the
part of the accused. The essence of treachery is the sudden and unexpected attack,
depriving the victim of any real chance to defend himself, thereby ensuring its
commission without risk to the aggressor. At the time accused-appellant fired the first
shot, the deceased was unarmed and had his hands raised.

The privileged mitigating circumstance of incomplete self-defense cannot be appreciated


in favor of accused-appellant. Unlawful aggression is a condition sine qua non for self-
defense, whether complete or incomplete. From the version of the prosecution, which
the Court finds credible, the deceased did not commit any unlawful aggression
towards accused-appellant.

The acts of the deceased immediately prior to the shooting did not constitute unlawful

VIVAR, 1
CRIMINAL LAW DIGESTS

aggression. Unlawful aggression requires an actual, sudden and unexpected attack, or


imminent danger thereof, and not merely a threatening or intimidating attitude. It
must be such as to put in real peril the life of the person defending himself and not a
mere imagined threat.

Nevertheless, the mitigating circumstance of passion or obfuscation should be


appreciated to mitigate accused-appellant's criminal liability. The requisites of this
mitigating circumstance are: (1) that there be an act, both unlawful and sufficient to
produce such a condition of mind; and (2) said act which produced the obfuscation
was not far removed from the commission of the crime by a considerable length of
time, during which the perpetrator might recover his normal equanimity.

In the case at bar, accused-appellant thought his father whose face was bloodied and
lying unconscious on the ground was dead. Surely, such a scenario is sufficient to
trigger an uncontrollable burst of legitimate passion. His act, therefore, of shooting the
deceased, right after learning that the latter was the one who harmed his father,
satisfies the requisite of the mitigating circumstance of passion or obfuscation.

The decision of the Regional Trial Court convicting accused-appellant of the crime of
Murder, is AFFIRMED with the MODIFICATION that accused-appellant is sentenced to
suffer the indeterminate penalty of ten (10) years and one (1) day of prision mayor, as
minimum, to seventeen (17) years, four (4) months and one (1) day of reclusion temporal,
as maximum.

Fallo:
WHEREFORE, in view of all the foregoing, the Decision of the Regional Trial Court of
Mandaue City, Branch 28, in Criminal Case No. DU-3463, convicting accused-appellant
of the crime of Murder, is AFFIRMED with the MODIFICATION that accused-appellant
is sentenced to suffer the indeterminate penalty of ten (10) years and one (1) day of
prision mayor, as minimum, to seventeen (17) years, four (4) months and one (1) day
of reclusion temporal, as maximum, and to pay in addition to the P50,000.00 death
indemnity and the costs, the amount of P50,000.00 as moral damages; P10,000.00 as
temperate damages and the amount of P443,700.00 for the loss of earning capacity of
the deceased.

VIVAR, 1
CRIMINAL LAW DIGESTS

Immediate vindication <<<

People v. Espina
G.R. No. 132325, July 16, 2002
YNARES-SANTIAGO, J:

Doctrine:
The act of the victim of urinating on the accused in front of guests undoubtedly
insulted and humiliated the accused, a circumstance coming within the purview of a
‘grave offense’ under Article 13, paragraph 5, of the Revised Penal Code, and for which
the mitigating circumstance of acting in immediate vindication of a grave offense
should be appreciated in favor of the accused.

Facts:
Romeo Bulicatin, Rogelio Espina, Samson Abuloc who were having a drinking spree
and playing cards during an association meeting, when accused-appellant Romeo
Espina arrived, Bulicatin asked three (3) bottles of “kulafu” wine from him and he
acceded by buying three (3) bottles of “kulafu” wine from the store of Eufronia Pagas.
Later on that afternoon, Bulicatin again demanded another bottle of “kulafu” wine
from accused- appellant but this time, the latter refused to give in to the demand.
Bulicatin then proceeded to where accused-appellant was playing cards and without
any warning, urinated on the latter and clipped him under his arms. Accused-appellant
got angry. He however did not engage Romeo in any altercation but instead went
home.

Later on in the evening, while the trio were still having a drinking spree, they heard
accused-appellant calling Bulicatin from outside, saying, “Borgs, get out because I have
something to say.” The trio came down from the house. Rogelio and Samson were
ordered to lie on the ground, Bulicatin was still at the stairway and when he turned his
back towards accused-appellant, the latter shot him, hitting him at the back. Bulicatin
ran away but he was chased by accused-appellant who fired two (2)more shots at him.

On appeal he alleged that the court erred by overlooking and misinterpreting some
significant facts in convicting him.

Issue:
Whether or not appellant’s contentions tenable? [NO]
Whether or not appellant entitled to any mitigating circumstance? [YES]

Ruling:
1. NO
The court held that, contrary to the claim of accused-appellant, the trial court did
not overlook his contention that he could not have committed the offenses charged
because at the time of the incident, he was unconscious due to a stab wound. In
fact, the trial court treated the same as a defense of denial and alibi. Indeed, these
defenses cannot prevail over the categorical and positive identification of accused-
appellant by prosecution witness Abuloc who was not shown to have any ill motive
to testify falsely against him.

Moreover, it is doctrinally settled that the assessment of the credibility of witnesses


and their testimonies is a matter best undertaken by the trial court because of its
unique opportunity to observe the witnesses first hand and to note their demeanor,
conduct and attitude under grilling examination. In the case at bar, the trial court
did not err in giving credence to the version of the prosecution. The facts and
circumstances alleged to have been overlooked by the trial court are not material
to the case and will not affect the disposition thereof.

2. YES
The trial court however correctly appreciated the mitigating circumstance of
having acted in immediate vindication of a grave offense. As the evidence on record
show, accused-appellant was urinated on by the victim in front of the guests. The
act of the victim, which undoubtedly insulted and humiliated accused-appellant,

VIVAR, 1
CRIMINAL LAW DIGESTS

came

VIVAR, 1
CRIMINAL LAW DIGESTS

within the purview of a “grave offense” under Article 13, paragraph 5, of the
Revised Penal Code. Thus, this mitigating circumstance should be appreciated in
favor of accused-appellant. Nevertheless the decision of the lower court was
affirmed with modifications, his sentenced was reduced to prision mayor.

Fallo:
WHEREFORE, the Decision of the Regional Trial Court of Tagbilaran, Branch 47, in
Criminal Case Nos. 8194 and 8195 finding the accused-appellant Romeo Espina guilty
beyond reasonable doubt of the crime of murder, is AFFIRMED with MODIFICATION
that the accused-appellant is hereby sentenced to suffer the indeterminate penalty of
eight (8) years and one (1) day of prision mayor, as minimum, to seventeen (17) years,
four (4) months and one (1) day of reclusion temporal, as maximum; and to pay the
heirs of Romeo Bulicatin the sum of P50,000.00 as death indemnity and to pay the
costs.

VIVAR, 1
CRIMINAL LAW DIGESTS

No evident premeditation <<<

Dorado v. People
G.R. No. 216671, October 3, 2016
MENDOZA, J:

Doctrine:
For evident premeditation to be appreciated, the following must be proven beyond
reasonable doubt: (1) the time when the accused determined to commit the crime; (2)
an act manifestly indicating that the accused clung to his determination; and (3)
sufficient lapse of time between such determination and execution to allow him to reflect
upon the circumstances of his act.

Facts:
This is a petition for review on certiorari seeking to reverse and set aside the decision
of CA which affirmed the RTC, finding Jerwin Dorado guilty of frustrated murder.

On April 15, 2004, at around 11:00 o'clock in the evening, Ronald was talking to his
friends Raniel, Delon Busar, Annan Luna, Jerome Amergo and a certain Erwin
(Ronald's group) along A. Reyes Street, Lower Bicutan, Taguig. At that very time,
Dorado, carrying a sumpak, and his friends, Confessor and Cabiaso (Dorado's group),
arrived and threw stones and bottles at Ronald's group. Ronald's group scampered for
shelter toward the talipapa and hid inside to avoid being hit by the stones and bottles.
When Ronald thought that Dorado's group was no longer-in the vicinity, they came out
of hiding. Dorado's group, however, was out there waiting for them. When they finally
surfaced, Dorado's group resumed throwing stones at Ronald's group. During the
commotion, Dorado fired his sumpak and hit Ronald between the eyes. Ronald fell
unconscious for about ten (10) minutes while Dorado's group ran away. Thereafter,
Ronald was brought to the Rizal Medical Center by Raniel and Delon Busan.

Ruling of the RTC – Dorado is guilty beyond reasonable doubt of the crime of
frustrated murder but also acquitted all the accused for the charge of violation of RA
7610 because the prosecution failed to establish Ronald’s minority but appreciated the
privileged mitigating circumstance of minority in Dorado’s favour as he was still a
minor at the time of the incident.

CA – affirmed the RTC

Issue:
Whether or not the CA gravely erred in affirming the conviction of the petitioner for the
crime charged.

Ruling:
Yes, the petition should be granted. The conviction is hereby reversed and set aside by
reason of the exempting circumstance of minority. He thereby to the total social welfare
and development officer of the loyalty for the appropriate intervention program.

The Court finds merit in the petition. Dorado was a minor at the time of the commission
of the crime. Upon perusal of the records will readily show that Dorado was a sixteen
(16) year old minor at the time of the commission of the crime on March 15, 2004. The
Informations filed against him consistently stated his minority. For said reason, he must
benefit from the provisions of R.A. No. 9344, or the Juvenile Justice and Welfare Act of
2006, as amended. Even though the said law was enacted on April 28, 2006, the same
must still be retroactively applied for the benefit of Dorado pursuant to the well-
entrenched principle in criminal law — favorabilia sunt amplianda adiosa restrigenda
(penal laws which are favorable to the accused are given retroactive effect).

To recapitulate, R.A. No. 9344 provides that only those minors above fifteen (15) years
but below eighteen (18) years of age who acted with discernment shall not be
exempted from criminal responsibility.

VIVAR, 1
CRIMINAL LAW DIGESTS

"The discernment that constitutes an exception to the exemption from criminal


liability of a minor x x x who commits an act prohibited by law, is his mental capacity
to understand the difference between right and wrong, and such capacity may be
known and should be determined by taking into consideration all the facts and
circumstances accorded by the records in each case, the very appearance, the very
attitude, the very comportment and behavior of said minor, not only before and during
the commission of the act, but also after and even during the trial."

After a judicious study of the records, the Court finds that the prosecution did not
make an effort to prove that Dorado, then a sixteen (16)-year old minor, acted with
discernment at the time of the commission of the crime. The RTC decision simply
stated that a privileged mitigating circumstance of minority in favor of Dorado must
be appreciated as it was proven that he was a minor at the time of the incident.
Glaringly, there was no discussion at all on whether Dorado acted with discernment
when he committed the crime imputed against him.

Discernment cannot be presumed even if Dorado intended to do away with Ronald.


Discernment is different from intent.

The distinction was elaborated in Guevarra v. Almodovar. Thus, it is this intent which
comprises the third element of dolo as a means of committing a felony, freedom and
intelligence being the other two. On the other hand, We have defined the term
"discernment," as used in Article 12(3) of the RPC, in the old case of People vs.
Doquena, 68 Phil. 580(1939), in this wise:"The discernment that constitutes an
exception to the exemption from criminal liability of a minor under fifteen years of age
but over nine, who commits an act prohibited by law, is his mental capacity to
understand the difference between right and wrong.

From the foregoing, it is clear that the terms "intent" and "discernment" convey two
distinct thoughts. While both are products of the mental processes within a person, the
former refers to the desire of one's act while the latter relate to the moral significance
that person ascribes to the said act. Hence, a person may not intend to shoot another
but may be aware of the consequences of his negligent act which may cause injury to
the same person in. negligently handling an air rifle. It is not correct, therefore, to
argue, as petitioner does, that since a minor above nine years of age but below fifteen
acted with discernment, then he intended such act to be done. He may negligently
shoot his friend, thus, did not intend to shoot him, and at the same time recognize the
undesirable result of his negligence.

Considering that there was no determination of discernment by the trial court, the
Court cannot rule with certainty that Dorado was criminally responsible. As earlier
stated, there can be no presumption of discernment on the part of the CICL. In the
absence of such determination, it should be presumed that the CICL acted without
discernment. Lastly, it was only frustrated homicide because evident premeditation
was not duly proven. The essence of evident premeditation is the execution of
the criminal act is preceded by cool thought and reflection upon the resolution
to carry out the criminal intent within a space of time sufficient to arrive at a
calm judgment.

Fallo:
WHEREFORE, the petition is GRANTED. The judgment of conviction of Jerwin Dorado
is hereby REVERSED and SET ASIDE by reason of the exempting circumstance of
minority. He is hereby referred to the local social welfare and development officer of
the locality for the appropriate intervention program.
He is also ordered to pay the private complainant, Ronald Bonion, civil indemnity in
the amount of P30,000.00 and moral damages in the amount of P30,000.00.

VIVAR, 1
CRIMINAL LAW DIGESTS

The amounts of damages awarded shall have an interest at the rate of 6% per annum
from the date of finality of judgment until fully paid.

Let copies of this decision be furnished the two houses of Congress for their information
and guidance in future legislation regarding children in conflict with the laws.

VIVAR, 1
CRIMINAL LAW DIGESTS

No evident premeditation <<<

People v. Macaspac
G.R. No. 198954, February 22, 2017
BERSAMIN, J:

Doctrine:
Was the lapse of time between the determination and execution — a matter of three (3)
minutes, based on the records — sufficient to allow him to reflect upon the
consequences of his act? By quickly returning to the group with the knife, he let no
appreciable time pass to allow him to reflect upon his resolve to carry out his criminal
intent. It was as if the execution immediately followed the resolve to commit the crime.
As such, the third requisite was absent.

Facts:
At around 8:00 in the evening of July 7, 1988, Macaspac along with four other persons
including one Robert Jebulan. In the course of their drinking spree, an argument
ensued between Macaspac and Jebulan. It became so heated that, Macaspac uttered to
the group: “Hintayin nyo ako d’yan, wawalisin ko kayo, “and then left. After around
three minutes, Macaspac returned wielding a knife. He confronted and taunted
Jubulan, saying, “Ano?”. Jebulan simply replied, “Tama na”. At that point, Macaspac
suddenly stabbed Jebulan on the lower right area of his chest and ran away. The others
witnessed the stabbing of Jebulan. He was rushed to the hospital but was rushed dead
on arrival.

Macaspac initially invoked self-defense, testifying that he and Jebulan had scuffed for
the possession of the knife, and that he had then stabbed Jebulan once he seized
control of the knife. However, he later on claimed that Jebulan had been stabbed by
accident when he fell on the knife. He denied being the person with whom Jebulan had
the argument, which he insisted had been between Barcomo and one Danny.
According to him, he tried to pacify their argument, but his efforts angered Jebulan,
who draw out the knife and tried to stab him. He fortunately evaded the stab thrust of
Jebulan, whom he struck with a wooden chair to defend himself. The blow caused
Jebulan to fall on the knife, puncturing his chest.

On February 19, 2008, the RTC found Macaspac guilty beyond reasonable doubt of
murder and sentenced him to an imprisonment of reclusion perpetua. On appeal, the
CA affirmed conviction of Macaspac with modification of the imposition of the civil
liability. The case elevated to the Supreme Court.

Issue:
Whether or not the CA erred in affirming Macaspac’s conviction for murder on the
ground that the Prosecution did not establish his guilt for murder beyond reasonable
doubt

Ruling:
NO.
The Court sees no misreading by the RTC and the CA of the credibility of the witnesses
and the evidence of the parties. On the contrary, the CA correctly observed that
inconsistencies had rendered Macaspac’s testimony doubtful as to shatter his
credibility. The Court cannot uphold the CA’s conclusion on the attendance of
treachery.

According to the facts, he did not mount the attack with surprise because the heated
argument between him and the victim and his angry threat of going back “to sweep
them” had sufficiently forewarned the latter of the impending lethal assault. The
requisites for the appreciation of evident premeditation are: (1) the time when the
accused determined to commit the crime; (2) an act manifestly indicating that the
accused had clung to his determination to commit the crime; and, the lapse of a
sufficient length of time between the determination and execution to allow him to reflect
upon the consequences of his act.

VIVAR, 1
CRIMINAL LAW DIGESTS

The first and second requisites were established. But it is the essence of this
circumstance that the execution of the criminal act be preceded by cool thought and
reflection upon the resolve to carry out the criminal intent during the space of time
sufficient to arrive at a calm judgment. By quickly returning to the group with a knife,
he let no appreciable time pass to allow him to reflect upon his resolve to carry out his
criminal intent. It was as if the execution immediately followed the resolve to commit a
crime. As such, the third requisite was absent.

Without the prosecution having sufficiently proved the attendance of either treachery
or evident premeditation, Macaspac was guilty only of homicide for the killing of
Jebulan. Macaspac shall suffer the indeterminate penalty of eight years of prision
mayor, as minimum, to 14 years, eight months and 1 day of reclusion temporal.

Anent the liabilities, the Court deem a modification to be necessary to align with
prevailing jurisprudence.

Fallo:
WHEREFORE, the Court DECLARES accused-appellant RODRIGO MACASPAC y ISIP
guilty beyond reasonable doubt of HOMICIDE, and SENTENCES him to suffer the
indeterminate penalty of EIGHT YEARS OF PRISIÓ N MAYOR, as minimum, to 14
YEARS, EIGHT MONTHS AND ONE DAY OF RECLUSION TEMPORAL, as maximum; to
pay to the heirs of the late Robert Jebulan: (a) P50,000.00 as civil indemnity; (b)
P50,000.00 as moral damages; and (c) P50,000.00 as temperate damages, plus
interest on all damages hereby awarded at the rate of 6% per annum from the finality
of the decision until fully paid. The accused shall further pay the costs of suit.

VIVAR, 1
CRIMINAL LAW DIGESTS

Quasi-recidivism <<<

People v. Macariola
G.R. No. L-40757, January 24,
1983 MELENCIO-HERRERA, J:

Doctrine:
Contrary to the stand of the Solicitor General’s Office, the special aggravating
circumstance of quasi-recidivism, under Article 160 of the Revised Penal Code, is
attendant. The accused committed this new felony while serving sentence for Robbery
imposed upon him by final judgment. Pursuant to the same provision, the maximum
period of the penalty prescribed by law for this new offense should thus be imposed.

Facts:
On September 21, 1971, between the hours of 8:00 and 9:30 o'clock in the morning,
Romeo de la Peñ a No. 29820-P, an inmate at the New Bilibid Prisons in Muntinlupa,
Rizal, was standing in his cell when suddenly he was approached and stabbed by
appellant Ricarte Macariola with an improvised weapon called 'matalas'. The
stabbing incident took place while they were inside their cell at Brigade No. 2-D and
was witnessed by fellow inmates Romeo Sato, Fernando Gomez, now deceased, a
certain Alvarez and Severino Pingkian. Romeo de la Peñ a shouted, 'Inay ,Inay 'and ran
to a 'kubol'. He was pursued by the appellant, and by inmates Nelson Binas and a
certain 'Bugok'. De la Peñ a dropped to the ground and lay prostrate inside the 'kubol'.
Again appellant stabbed him followed by thrusts from Nelson Binas and 'Bugok'. De la
Peñ a died inside the 'kubol'.

Meanwhile, Fernando Gomez, considered the 'Mayor' at Brigade No. 2-D since 1970,
upon hearing the victim shout 'papatayin ako, papatayin ako,' entered the cell and saw
the victim in a lying position with the appellant bending over him with a blood stained
knife in his hand. Immediately, he reported the incident to prison keeper Alfredo
Manzano who opened the door of the cell. Alberto Supetran, another prison keeper,
entered the cell. Appellant then approached the latter to whom he surrendered the
improvised weapon ('matalas') which he used in stabbing the victim. On the same day,
an investigation of the stabbing incident was conducted. When appellant was
investigated, he admitted having stabbed the victim Romeo de la Peñ a.

An autopsy was conducted on the body of the victim by Dr. Cristino S. Garcia, medico-
legal officer of the NBI. The Necropsy Report prepared by him shows that the victim
sustained a total of sixteen stab wounds, of which, wounds nos. 11 and 12 were
considered fatal. Stab wound No. 11 involved the left lobe of the liver and penetrated
the left ventricle of the heart. Stab wound No. 12, was five in number and penetrated
the upper and middle lobe of the right lung. These wounds produced severe
hemorrhage and caused the death of the victim.

On the other hand, the accused, presented his own versions (Self-defense):

Accused was confined in the same Brigade 2-D along with said victim, they were
gambling in which said accused won all the money. After giving ‘balato’ to the victim,
he snatched all the money and kicked accused while trying to get back such. While
sitting in the floor he pulled out his ‘matalas’. The victim was also trying to reach for
his own ‘matalas’ in his pillows but he a never got hold of it for the accused already
stabbed him and pulled him out to not reach the bed. The victim is taller, bigger and
huskier than the accused; that the victim was a boxer; that their cell was closed and
there was no exit: that there was no place to run away from the victim: that the
accused had to use the "matalas " in older to disable the victim, thinking that his life
was in imminent danger.

VIVAR, 1
CRIMINAL LAW DIGESTS

Issue:
Whether or not the court erred in finding defendant-appellant guilty beyond
reasonable doubt of the crime of Murder. (Yes, attended with Treachery)

Whether the court erred in finding defendant-appellant a recidivist. (No, Quasi-


Recidivist: Special Aggravating Circumstance)

Ruling:
The commission of the crime was attended by treachery. It may be that, at the
inception, treachery cannot be appreciated as the sudden assault made by the accused,
as testified to by prosecution witness Romeo Sato, was merely an immediate
retaliation for the act of kicking by the victim, thereby placing the latter on his guard.
Yet, although absent initially, if the attack is continued and the crime is consummated
with treachery, it may still be taken into consideration.

Even though an attack may be begun under conditions not exhibiting the feature of
alevosia yet if the assault is continued and the crime consummated with alevosia, such
circumstance may be taken into consideration as a qualifying factor in the offense of
murder.

Treachery exists when the offender employs means, methods or forms which tend
directly and specially to insure the execution of the offense without risk to the accused
arising from the defense which the victim might make. The concurrence of the two
conditions necessary for treachery to exist are present in this case, namely: (1) the
employment of means, method or manner of execution which would insure the
offender's safety from any defensive or retaliatory act on the party of the offended party,
which means that no opportunity is given the latter to defend himself or to retaliate; and
(2) such means, method or manner of execution was deliberately or consciously chosen.

According to the accused's own version, after he was kicked by the victim and as the
latter turned to retrieve something from under his pillow, the accused drew his
improvised deadly weapon from his waist, pulled the victim, and stabbed the latter on
the back. Then, according to prosecution witness, Romeo Sato, the victim ran to the
"kubol" where he was pursued by the accused and two other inmates and as the victim
lay prostrate on the floor the accused and two other inmates kept on stabbing him
without let up. The risk to the accused arising from any defense that the victim might
have made had ceased the moment the victim fled and hid under his "tarima " after
being initially wounded. Yet, the accused went in pursuit and continued attacking the
victim to s death. According to the accused himself, after he had stabbed the victim the
latter sought refuge under his "tarima"; but the accused pulled him from under and
continued assaulting him even as he was lying prostrate with half of his body under
the " tarima "; The, victim was in no position to retaliate. He was unarmed and
completely defenseless. The state of helplessness of the victim is evidenced by the 16
stab wounds he received.

Contrary to the stand of the Solicitor General's Office, the special aggravating
circumstance of quasi-recidivism, under Article 160 of the Revised Penal Code, is
attendant. The accused committed this new felony while serving sentence for Robbery
imposed upon him by maximum period of the Penalty prescribed by law for this new
offense should thus be imposed.

The presence of the mitigating circumstance of sufficient provocation is of no


consequence as quasirecidivism cannot be offset by any ordinary mitigating
circumstance.

Fallo:
WHEREFORE, the judgment appealed from is modified as to the penalty imposed and the
accused, Ricarte Macariola, is hereby sentenced to suffer reclusion perpetua. The indemnity
awarded by the Trial Court of P10,000.00 is increased to P12,000.00.22 The judgment is
affirmed in all other respects. Costs against accused-appellant Ricarte Macariola.

VIVAR, 1
CRIMINAL LAW DIGESTS

Abuse of superior strength <<<

People v. Nazareno
G.R. No. 196434, October 24, 2012
ABAD, J:

Doctrine:
There is abuse of superior strength when: 1) the aggressors purposely use excessive
force rendering the victim unable to defend himself; 2) the notorious inequality of
forces creates an unfair advantage for the aggressor.

Facts:
On November 10, 1993 David Valdez, Magallanes, and Francisco attended the wake of
a friend. While there, they drank liquor with accused Nazareno and Saliendra. A heated
argument ensued between Magallanes and Nazareno but their companions pacified
them. On the next day, David, Magallanes, and Francisco returned to the wake.

Nazareno and Saliendra also arrived and told the three not to mind the previous
night’s altercation. At around 9:30 in the evening, while David, Francisco, and their
friend, Aida Unos were walking on the street, Nazareno and Saliendra blocked their
path. Nazareno boxed Francisco who fled but Saliendra went after him with a balisong.
Francisco, who succeeded in hiding saw Nazareno hit David on the body with a stick
while Saliendra struck David’s head with a stone. David ran towards a gasoline station
but Nazareno and Saliendra, aided by some barangay tanods, caught up with him. As
David fell, the barangay tanods took over the assault. This took place as Magallanes
stood about five meters across the highway unable to help his friend. Afterwards, Unos
brought David to the UST hospital. Dr. Rebosa performed surgery on David’s head but
the latter died on November 14, 1993 due to massive intra-cranial hemorrhage.

Accused’s Version/Defense
Nazareno claimed that he left his house at around 9:30 in the evening on November 11,
1993 to buy milk. While on a street near his house, he noted a commotion taking place
nearby. He then bumped into Saliendra. Nazareno proceeded home and went to bed.
His wife Isabel supported his testimony, claiming that she asked her husband on that
night to buy milk for their children. When Nazareno returned home, he informed her of
the commotion outside and how someone bumped into him. Unos testified that she saw
Saliendra chasing David as the latter hang on the rear of a running jeepney. She claimed
that she did not see Nazareno around the place.

Crime Charged: Murder

RTC: found Nazareno GUILTY BEYOND REASONABLE DOUBT OF MURDER, qualified


by abuse of superior strength and aggravated by treachery.

CA: AFFIRMED WITH MODIFICATION the decision of the RTC; finding no treachery,
it convicted Nazareno of MURDER qualified by abuse of superior strength.

Issue:
Whether or not the qualifying circumstance of abuse of superior strength attended the
killing of David (YES)

Ruling:
The killing of David should be characterized as one of murder qualified by abuse of
superior strength.

There is abuse of superior strength when:


1. The aggressors purposely use excessive force rendering the victim unable to
defend himself.
2. The notorious inequality of forces creates an unfair advantage for the aggressor.

VIVAR, 1
CRIMINAL LAW DIGESTS

Here, Nazareno and Saliendra evidently armed themselves beforehand, Nazareno


with a stick and Saliendra with a heavy stone. David was unarmed. The two chased
him even as he fled from them. And when they caught up with him, aided by some
unnamed barangay tanods, Nazareno and Saliendra exploited their superior
advantage and knocked the defenseless David unconscious. He evidently died from
head fracture caused by one of the blows on his head.

Fallo:
WHEREFORE, the Court AFFIRMS the assailed Decision of the Court of Appeals in
CA-G.R. CR-H.C. 01308 dated December 17, 2010, that found Chito Nazareno guilty
beyond reasonable doubt of the crime of murder qualified by abuse of superior
strength in Criminal Case 94-133117.

The Court also AFFIRMS the penalty of reclusion perpetua imposed on accused
Nazareno but MODIFIES the award of damages to P141,670.25 as actual damages,
P75,000.00 as civil indemnity, P75,000.00 as moral damages, and P30,000.00 as
exemplary damages, and to pay the costs.

VIVAR, 1
CRIMINAL LAW DIGESTS

Incomplete Self-Defense- Absent <<<

People v. Dulin
G.R. No. 171284, June 29, 2015
BERSAMIN, J:

Doctrine:
Retaliation is not the same as self-defense. In retaliation, the aggression that the victim
started already ceased when the accused attacked him, but in self-defense, the
aggression was still continuing when the accused injured the aggressor.

Case Summary:
Batulan initially stabbed Dulin but after grappling, Dulin was able to gain possession
of the weapon. Batulan still pursued the attack and this eventually led to his death due
to various laceration wounds. Dulin claims that what he did was due to self-defense.
The court did not agree because the unlawful aggression, a key element in self-defense,
had already ended when the act of killing Batulan was done.

Facts:
Statement of the Case: That on or about August 22, 1990, in the Municipality of
Tuguegarao, Province of Cagayan, and within the jurisdiction of this Honorable Court,
the said accused, Alfredo Dulin y Narag alias Freddie, armed with a sharp blade(d)
instrument, with intent to kill, with evident premeditation and with treachery did then
and there willfully, unlawfully and feloniously attack, assault and stab one, Francisco
Batulan, inflicting upon him several stab wounds on the different parts of his body
which caused his death.

 Tamayao was on Tamayao Street in Atulayan Norte, Tuguegarao at about 10:00


o’clock in the evening of August 22, 1990 when a young man came running from
the house of Vicente Danao towards the house of Batulan, shouting that his Uncle
Totoy (Batulan) had been stabbed. Tamayao rushed towards Danao’s house, which
was about 30 meters from his own house, and there he saw Dulin stabbing Batulan
who was already prostrate face down. Dulin was on top of Batulan, as if kneeling
with his left foot touching the ground. Dulin was holding Batulan by the hair with
his left hand, and thrusting the knife at the latter with his right hand. Seeing this,
Tamayao ran towards Batulan’s house to inform Estelita Batulan, the victim’s wife
who was his aunt, about the incident. He went home afterwards.

 There has been a long standing grudge between Batulan and Dulin, and of seeing
them fighting in April 1990. He recalled Dulin uttering on two occasions: He will
soon have his day and I will kill him.

 Cabalza, a barangay tanod, was in his house around 10:00 o’clock in the evening of
August 22, 1990 when he heard the commotion in Danao’s house which was facing
his house. It was Carolina, Danao’s daughter, screaming for help. He thus sought
out a fellow barangay tanod. On his return to the scene, he found Batulan at the
door of Danao’s house, with Dulin wielding a sharp pointed instrument, about 6-7
inches long. Fearing for his safety, he rushed to the Barangay Hall to seek the
assistance of Edwin Cabalza and Nanding Buenaflor to bring Batulan to the
Provincial Hospital in Carig, Tuguegarao.

 At the hospital, she was told that her husband had sustained two wounds in the
back and several stab wounds in the front, and was being attended to at the
hospital’s intensive care unit (ICU) before he expired.

 Batulan was attended to at the Cagayan Valley Regional Hospital on August 22, 1990
by Dr. Macaraniag, who said that the victim was in a state of shock from his 12 stab
wounds (lacerations). He died eventually.

 Defenders Side: He was attacked first.

VIVAR, 1
CRIMINAL LAW DIGESTS

 RTC rendered its decision convicting Dulin of murder.


 CA affirmed decision.

Issue:
1. Whether or not Dulin was actual in the act of self-defense. NO
2. Whether or not Dulin’s actions may be considered as an incomplete form of self-
defense? NO

Ruling:
1. NO.
 The CA observed that although Batulan had initiated the attack against Dulin
the unlawful aggression from Batulan effectively ceased once Dulin had
wrested the weapon from the latter.
 Batulan, albeit the initial aggressor against Dulin, ceased to be the aggressor
as soon as Dulin had dispossessed him of the weapon. Even if Batulan still
went after Dulin despite the latter going inside the house of Danao, where
they again grappled for control of the weapon, the grappling for the weapon
did not amount to aggression from Batulan for it was still Dulin who held
control of the weapon at that point.
 Whatever Dulin did thereafter – like stabbing Batulan with the weapon –
constituted retaliation against Batulan. In this regard, retaliation was not the
same as self-defense. In retaliation, the aggression that the victim started
already ceased when the accused attacked him, but in self-defense, the
aggression was still continuing when the accused injured the aggressor. As
such, there was no unlawful aggression on the part of Batulan to justify his
fatal stabbing by Dulin.

2. NO
Like in complete self-defense, Dulin should prove the elements of incomplete
self- defense by first credibly establishing that the victim had committed
unlawful aggression against him. With Batulan’s aggression having already
ceased from the moment that Dulin divested Batulan of the weapon, there
would not be any incomplete self-defense. Moreover, as borne out by his
stabbing of Batulan several times, Dulin did not act in order to defend himself or
to repel any attack, but instead to inflict injury on Batulan.

Fallo:
WHEREFORE, the Court MODIFIES the judgment promulgated on August 26, 2005 by
finding ALFREDO DULIN YNARAG guilty beyond reasonable doubt of HOMICIDE, and
SENTENCES him to suffer the indeterminate sentence of EIGHT YEARS AND ONE DAY
OF PRISION MAYOR, AS THE MINIMUM, TO 14 YEARS, EIGHT MONTHS AND ONE
DAY OF RECLUSION TEMPORAL, with full credit of his preventive imprisonment;
ORDERS him to pay to the heirs of Francisco Batulan ₱50,000.00 as civil indemnity,
₱50,000.00 as moral damages, and ₱25,000.00 as temperate damages, plus interest of
6% per annum on each item reckoned from the finality of this decision until full
payment; and DIRECTS him to pay the costs of suit.

NOTES:
The test for the presence of unlawful aggression under the circumstances are:
(a) there must be a physical or material attack or assault;
(b) the attack or assault must be actual, or, at least, imminent; and
(c) the attack or assault must be unlawful.

Unlawful aggression is of two kinds:


(a) actual or material unlawful aggression; and
(b) imminent unlawful aggression

VIVAR, 1
CRIMINAL LAW DIGESTS

Abuse of superior strength <<<

Fantastico v. Malicse
G.R. No. 190912, January 12,
2015 PERALTA, J:

Doctrine:
There is no treachery where the attack was not preconceived and deliberately adopted
but was just triggered by the sudden infuriation on the part of the accused because of
the provocative act of the victim.

Facts:
Sometime in the afternoon of June 27, 2017, Elpidio Malicse, Sr. heard his sister
Isabelita’s throwing invectives to her son. Elpidio, who was then under the influence of
alcohol, confronted Isabelita but got cursed which prompted him to slap the latter. The
barangay chairman intervened and led Elpidio to go home. After drinking coffee at
their house, Elpidio went alone to the house of Isabelita to seek for reconciliation.
Upon reaching the house, the sons of Isabelita (Titus and Salvador) and in-law Gary
Fantastico, were there as he looked for her sister. Elpidio was cursed by Titus
prompting the latter to kick the door and a fight emerged. Elpidio was mauled by the
sons Isabelita (Salvador, Titus, Gary, Rolly, Nestor, Eugene and Tommy) having
weapons of rattan stick tomahawk axe and pipe. As a result, Elpidio obtained severe
wounds in the head and body and fractured his legs and knees. The neighbors begged
the assailants to stop until a certain Mang Gil tried to break them off. Elpidio
pretended to be dead but the betting has just stopped when a bystander suddenly
fainted because of the incident. The victim was them bought to the Philippine General
Hospital.

A case of attempted murder was filed at the RTC of Manila which ruled guilty beyond
reasonable doubt for Gary Fantastico and Rolando Villanueva; and acquitted Titus
Iguiron, Saligan Iguiron and Tommy Ballesteros. A motion for reconsideration was
denied and the CA, upon appeal, affirmed the decision of the RTC. Hence, this petition.

Issue:
Whether or not, there is an abuse of superior strength as aggravating circumstance in
the commission of the crime.

Ruling:
Yes. The SC ruled that an aggravating circumstance of abuse of superior strength was
present during the commission of the crime. In the present case, prosecution, however,
has not proved that there was treachery in the commission of the crime. Instead, a
qualifying circumstance of abuse of superior strength was appreciated. The SC stated
that, “the sequence of the injuries is proven by victim's testimony. But it was a
lopsided attack as the victim was unarmed, while his attackers were all armed (rattan
stick, tomahawk and lead pipe). And the victim was also drunk. This establishes the
element of abuse of superior strength. The suddenness of the blow inflicted by
Salvador on Elpidio when he entered the premises show that the former was ready to
hit the victim and was waiting for him to enter. It afforded Elpidio no means to defend
himself. And Salvador consciously adopted the said actuation. He hit Elpidio twice on
the head. Villanueva, Fantastico and Iguiron were all armed while Elpidio, inebriated,
had nothing to defend himself with. There is clearly present here the circumstance of
abuse of superior strength”.

Abuse of superior strength is present whenever there is a notorious inequality of


forces between the victim and the aggressor, assuming a situation of superiority of
strength notoriously advantageous for the aggressor selected or taken advantage of by
him in the commission of the crime. The fact that there were two persons who
attacked the victim does not per se establish that the crime was committed with abuse
of superior strength, there being no proof of the relative strength of the aggressors and
the victim. The evidence must establish that the assailants purposely sought the
advantage, or that they had the deliberate intent to use this advantage. To take
advantage of superior strength means to purposely use excessive force out of

VIVAR, 1
CRIMINAL LAW DIGESTS

proportion to the means of

VIVAR, 1
CRIMINAL LAW DIGESTS

defense available to the person attacked. The appreciation of this aggravating


circumstance depends on the age, size, and strength of the parties. Thus, in the case at
bar, the act of the assailant evidently showed an abuse of strengthen; thus, the petition
of the accused was denied and found them guilty of the crime of attempted murder.

Fallo:
WHEREFORE, the Petition for Review on Certiorari dated January 20, 2010 of
petitioners Gary Fantastico and Rolando Villanueva is hereby DENIED. Consequently,
the Decision dated August 31, 2007 and Resolution dated January 7, 2010 of the Court
of Appeals are hereby AFFIRMED with the MODIFICATION that the petitioners are
sentenced to an indeterminate penalty of imprisonment from six (6) years of prisió n
correccional, as minimum, to eight (8) years and one (1) day of prisió n mayor, as
maximum. Petitioners are also ORDERED to pay P17,300.00 as actual damages, as well
as P10,000.00 moral damages as originally ordered by the RTC. In addition, interest is
imposed on all damages awarded at the rate of six percent (6%) per annum from date
of finality of judgment until fully paid.

VIVAR, 1
CRIMINAL LAW DIGESTS

Nighttime; evident premeditation; treachery; abuse of confidence- Absent <<<

People v. Bokingco
G.R. No. 187536, August 10,
2011 PEREZ, J:

Doctrine:
To warrant a finding of evident premeditation, the prosecution must establish the
confluence of the following requisites: (a) the time when the offender was determined to
commit the crime; (b) an act manifestly indicating that the offender clung to his
determination; and (c) a sufficient interval of time between the determination and the
execution of the crime to allow him to reflect upon the consequences of his act.

Facts:
The victim, Noli Pasion (Pasion) and his wife, Elsa, owned a pawnshop, which formed
part of his house. He also maintained two (2) rows of apartment units at the back of his
house. A certain Vitalicio was spin-drying his clothes and he heard a commotion from
Apartment No. 3. He headed to said unit to check and saw Bokingco hitting something
on the floor. Upon seeing Vitalicio, Bokingco allegedly attacked him with a hammer in
his hand. Vitalicio was hit several times. Vitalicio bit Bokingco’s neck and managed to
push him away. Bokingco tried to chase Vitalicio but was eventually subdued by a co-
worker. Pasion was found dead in the kitchen of Apartment No. 3.

Elsa testified that she was in the master’s bedroom on the second floor of the house
when she heard banging sounds and her husband’s moans. She immediately went
down. Before reaching the kitchen, Col blocked her way. Col suddenly sprayed tear gas
on her eyes and poked a sharp object under her chin. Col then instructed her to open
the vault of the pawnshop but Elsa informed him that she does not know the
combination. Col dragged her towards the back door by holding her neck. Before they
reached the door, Elsa saw Bokingco open the screen door and heard him tell Col: "tara,
patay na siya."Col immediately let her go and ran away with Bokingco.

Bokingco testified that he was sleeping when he was awakened by Pasion who
appeared to be intoxicated. The latter wanted to know why he did not see Bokingco at
the construction site. When Bokingco replied that he just stayed at the apartment,
Pasion suddenly hit him in the head. This prompted Bokingco to take a hammer and
hit Pasion repeatedly. Col alleged that he went home to Cainta, Rizal, where he was
apprehended and brought to Camp Olivas. Bokingco pointed to him as the person who
killed Pasion. He insisted that he doesn’t know Bokingco very well. On preliminary
investigation, Bokingco admitted that he conspired with Col to kill Pasion and that
they planned the killing several days before because they got "fed up" with Pasion.

Lower Court(s):
RTC: the accused MICHAEL BOKINGO and REYNANTE COL guilty beyond reasonable
doubt of the crime of MURDER, and there being the two aggravating circumstances of
nighttime and abuse of confidence to be considered against both accused and the
mitigating circumstance of voluntary plea of guilty in favor of accused Bokingo only,
hereby sentences each of them to suffer the penalty of DEATH.

CA: affirmed the findings of the trial court but reduced the penalty to reclusion perpetua.

Issue:
Whether OR NOT the qualifying circumstances were properly appreciated to convict
appellant Bokingco of murder.

Ruling:
Bokingco made two (2) separate and dissimilar admissions: first, in his extrajudicial
confession taken during the preliminary investigation where he admitted that he and
Col planned the killing of Pasion; and second,... when he testified in open court that he
was only provoked in hitting Pasion back when the latter hit him in the head. On the

VIVAR, 2
CRIMINAL LAW DIGESTS

basis of his extrajudicial confession, Bokingco was charged for murder qualified by
evident premeditation and treachery.

We agree with appellants that treachery cannot be appreciated to qualify the crime to
murder in the absence of any proof of the manner in which the aggression was
commenced. For treachery to be appreciated, the prosecution must prove that at the
time of the attack, the... victim was not in a position to defend himself, and that the
offender consciously adopted the particular means, method or form of attack
employed by him.

Nobody witnessed the commencement and the manner of the attack. While the
witness Vitalicio... managed to see Bokingco hitting something on the floor, he failed to
see the victim at that time.

To warrant a finding of evident premeditation, the prosecution must establish the


confluence of the following... requisites: (a) the time when the offender was determined
to commit the crime; (b) an act manifestly indicating that the offender clung to his
determination; and (c) a sufficient interval of time between the determination and the
execution of the crime to allow him to reflect... upon the consequences of his act.

It is indispensable to show how and when the plan to kill was hatched or how much
time had elapsed before it was carried out.

In the instant case, no proof was shown as to how and... when the plan to kill was
devised. Bokingco admitted in court that he only retaliated when Pasion allegedly hit
him in the head.
Despite the fact that Bokingco admitted that he was treated poorly by Pasion, the
prosecution failed to... establish that Bokingco planned the attack.

It was during the preliminary investigation that Bokingco mentioned his and Col's plan
to kill Pasion.

Bokingco's confession was admittedly taken without the assistance of counsel.

The finding that nighttime attended the commission of the crime is anchored on the
presumption that there was evident premeditation. Having ruled however that evident
premeditation has not been proved, the aggravating circumstance of nighttime cannot
be properly... appreciated. There was no evidence to show that Bokingco purposely
sought nighttime to facilitate the commission of the offense.

Abuse of confidence could not also be appreciated as an aggravating circumstance in


this case. Taking into account that fact that Bokingco works for Pasion, it may be
conceded that he enjoyed the trust and confidence of Pasion. However, there was no
showing that he took... advantage of said trust to facilitate the commission of the
crime.

A downgrade of conviction from murder to homicide is proper for Bokingco for failure of
the prosecution to prove the presence of the qualifying circumstances.

Fallo:
WHEREFORE, the appeal is GRANTED. The Decision of the Court of Appeals in CA-G.R. CR-H.C.
No. 00658 is REVERSED and SET ASIDE. Appellant Reynante Col is ACQUITTED on ground of
reasonable doubt. The Bureau of Corrections is ordered to cause the immediate release of
accused-appellant, unless he is being lawfully held for another cause, and to inform this Court
of action taken within ten (10) days from notice.
Appellant Michael Bokingco is found GUILTY beyond reasonable doubt of the crime of
Homicide. He is hereby sentenced to suffer the penalty of six years (6) and one (1) day of
prision mayor as minimum to 14 years, eight (8) months and one (1) day of reclusion temporal,
as maximum Appellant is further ordered to indemnify the heirs of Noli Pasion in the amount of
Seventy five thousand pesos (P75,000.00); Fifty thousand pesos (P50,000.00) as moral
damages; Twenty five thousand pesos (P25,000.00) as temperate damages; Fifteen thousand
pesos (P15,000.00) as attorney's fees; and to pay the costs.

VIVAR, 2
CRIMINAL LAW DIGESTS

Treachery <<<

People v. Tabarnero
G.R. No. 168169, February 24, 2010
LEONARDO-DE CASTRO, J:

Doctrine:
Aggravating Circumstances; Treachery; Abuse of Superior Strength; Despite the yelling
which should have warned the victim of a possible attack, the mere fact that the
accused’s companions held the hands of the victim while the accused stabbed him was
considered by this Court to constitute alevosia; The act of the accused in stabbing the
victim while two persons were holding him clearly shows the deliberate use of
excessive force out of proportion to the defense available to the person attacked.

Facts:
Late at night, Gary Tabarnero went to the house of Ernesto Canatoy where Gary used
to reside as the live-in partner of Ernesto’s stepdaughter, Mary Jane Acibar. Gary and
Ernesto had a confrontation during which, Ernesto was stabbed 9 times. Gary and his
father, Albert Tabernero were charged with murder and on March 27, 2000, RTC
issued warrants of arrest. On April 22, 2001, Gary surrendered to Barangay Tanod
Edilberto Alarma while Albert remained at large.

Defense: During the arraignment Gary pleaded not guilty but in the pre-trial
conference he admitted killing Ernesto but in self-defense. When the Reverse trial
ensued, Gary testified that he lived with Mary Jane for 2 years at Ernesto’s house but
left the shortly before the incident because Ernesto stopped his planned marriage with
Mary Jane, who was pregnant at that time.

Shortly before the incident, Gary was at his house together with a friend, his brother,
his mother, and father, herein co-accused. Overcome with his emotion over being
separated from Mary Jane, he went to Ernesto’s house but was not able to enter. He
shouted his pleas outside, asking what he did wrong and how much he loves Mary
Jane. When he was about to leave, Ernesto struck him with a lead pipe, the former was
aiming at Gary’s head, but Gary was able to block the blow with his hands.

Gary embraced Ernesto, but the latter strangled him. At that point, Gary took the
bladed weapon tucked at Ernesto’s back and stabbed him. Gary was stunned and did
not notice that his father was coming. When asked by his father about what happened,
Gary responded: “Nasaksak ko po ata si Ka Erning”. Then, both fled out of fear.

On August 5, 2001, Alberto was apprehended and denied the accusations. He further
said that they ran away in different directions, he immediately went to Norzagaray, and
did not know where Gary proceeded.

Edilberto Alarma, the barangay tanod, testified that while he was on a meeting, Gary
arrived and told him of his intention to surrender and that he was responsible for the
incident. He was brought to the Police Station were the surrender was entered in a
blotter report.

Prosecution: Emerito, brother of Mary Jane, who was inside their house at the time the
incident happened, testified that he saw (7-8m away) his stepfather Ernesto being
held by two persons while Gary and Ernesto were stabbing him with fan knives. Gary
lost count of the number of thrusts made by Gary and Alberto but the last stab was
made by the latter. Emerito further confirmed that Gary and Mary Jane were live-in
partners and his father got mad when he knew about their relationship because they
treated Gary as a member of their family.

SP02 Ronnie Morales, asked Ernesto at the hospital who stabbed him and answered
that the assailants were Gary and Alberto. Ernesto however died before he was able to
sign the Sinumpaang Salaysay.

VIVAR, 2
CRIMINAL LAW DIGESTS

RTC convicted both Gary and Alberto of murder; appealed to SC but was referred to
CA. CA affirmed with modifications as regards the damages.

In an appeal to SC, the accused contended that the RTC erred in not considering the
justifying circumstance of self-defense and the mitigating circumstance of voluntary
surrender interposed by Gary.

Issue:
1.) Should the justifying circumstance of self-defense be considered on the part of Gary?
2.) Is Gary entitled to the mitigating circumstance of voluntary surrender?

Ruling:
1.) NO. Gary’s contention that the unlawful aggression on the part of Alberto was when
the latter struck him of a lead pipe, his pleas outside their house could not be
considered as sufficient provocation, and that his defense was reasonable, are
unmeritorious. His testimony is insufficient and self-serving. The alleged initial
attack on him when he was about to leave seemed to be all-convenient considering
that no one witnessed the start of the fight. The nine stab wounds inflicted on
Ernesto indicate an intent to kill and not merely to defend himself.

He further argues that even if he is not qualified to the justifying circumstance of


self-defense, he is still entitled to the mitigating circumstance of incomplete self-
defense under Article 13(1). The court however, ruled that Gary failed to prove the
presence of unlawful aggression which is an indispensable element of self-defense
whether complete or incomplete. Hence, he is not entitled to the mitigating
circumstance.

2.) NO. In order that the mitigating circumstance of voluntary surrender may be credited
to the accused, the following should be present: (a) the offender has not actually been
arrested; (b) the offender surrendered himself to a person in authority; and (c) the
surrender must be voluntary. A surrender, to be voluntary, must be spontaneous, i.e.,
there must be an intent to submit oneself to authorities, either because he acknowledges
his guilt or because he wishes to save them the trouble and expenses in capturing him.
In the case at bar, appellant surrendered to the authorities after more than one year
(almost one year and six months from the October 23, 1999 incident, and almost one
year and one month from the issuance of the warrant of arrest against him on March
27, 2000 ) had lapsed since the incident and in order to disclaim responsibility for the
killing of the victim. This neither shows repentance nor acknowledgment of the crime
nor intention to save the government the trouble and expense necessarily incurred in
his search and capture. Besides, at the time of his surrender, there was a pending
warrant of arrest against him. Hence, he should not be credited with the mitigating
circumstance of voluntary surrender.

Others:
*Alberto is a principal by direct participation having actually participated in stabbing
Ernesto. Ernesto’s dying declaration that it was the father and son, Gary and Alberto
who stabbed him, proved to be very persuasive and entitled to the highest credence.
*Killing of Ernesto is qualified by treachery because he was held by two other persons
while he was being stabbed, which rendered him defenseless and unable to effectively
repel or evade the assault.

Fallo:
WHEREFORE, the Decision of the Court of Appeals in CA-G.R. CR.-H.C. No. 00027
dated April 29, 2005 is hereby AFFIRMED, with the MODIFICATION that appellants
Alberto and Gary Tabarnero are further ordered to pay the heirs of Ernesto Canatoy
the amount of P75,000.00 as civil indemnity.

VIVAR, 2
CRIMINAL LAW DIGESTS

Treachery- Absent <<<

People v. Vilbar
G.R. No. 186541, February 1, 2012
LEONARDO-DE CASTRO, J:

Doctrine:
Where the meeting between the accused and the victim was casual and the attack was
done impulsively, there is no treachery even if the attack was sudden and unexpected.

Facts:
On May 5, 2000 at around 7:00pm, Guilbert Patricio (victim) arrived on the store of his
wife located in the Ormoc City public market from work. He was met by their child whom
he then carried in his arms. Moments later, Guilbert noticed a man urinating at one of
the tables in front of their store. The man urinating was among those engaged in a
drinking spree in a nearby store. Guilbert immediately admonished the man urinating
but the latter paid no attention and continued relieving himself. Guilbert then put
down his child when Vicente Vilbar (accused) rose from his seat, approached Guilbert,
drew out a knife and stabbed him below his breast. Thereafter, the accused, as well as
his companions, scampered away. At the time, the wife of the victim was getting her
child from Guilbert and easily recognized the accused because he would sometimes
drink at their store. Guilbert was immediately brought to the hospital and later
expired 11:35 of the same evening.

On the point of view of the accused: Denial was his main plea in exculpating himself of
the charged against him. He claimed that in the evening of May 5, 2000, he and his wife
went to the public market to collect receivables out of the sale of meat. Afterwards,
they chanced upon his wife’s acquaintances who were engaged in a drinking spree.
Among them were Dodong Danieles (Dodong for brevity) and his younger brother.
They invited him (the accused) and his wife to join them. While they were drinking,
Dodong had an altercation with Guilbert that stemmed from Dodong’s younger
brother who had earlier urinated at the Patricio’s store premises. Suddenly, Dodong
assaulted Guilbert and stabbed him. Fearing that he might be implicated in the
incident, the accused fled. When he learned that the victim was brought to the
Hospital, he went there to verify the victim’s condition. He was able to talk with the
mother and the wife of Guilbert as well as the police. He was thereafter invited to the
precinct so that the police can get his statement. Half a month later, he was arrested
and charged for the death of Guilbert Patricio.

On August 6, 2001, the RTC promulgated its Decision finding accused-appellant guilty
of murder. However, the Court of Appeals on its decision dated February 14, 2008,
subsequently held accused-appellant liable only for the lesser crime of homicide.
Hence, this petition.

Issue:
1. Whether or not the accused-appellant should be held liable for murder as held
by the RTC or homicide as adjudged by the Court of Appeals.
2. Whether or not credibility should be given on the prosecution’s witnesses since
there were alleged inconsistencies in their testimony.

Ruling:
1. No. The accused-appellant should be held liable for homicide as adjudged by
the Court of Appeals in the absence of the qualifying circumstance of
treachery. The jurisprudence provides in a number of cases that, treachery
cannot be appreciated simply because the attack was sudden and unexpected.

In People v. Escoto, it was held that: “We cannot presume that treachery was
present merely from the fact that the attack was sudden. The suddenness of an
attack, does not of itself, suffice to support a finding of alevosia, even if the
purpose was to kill, so long as the decision was made all of a sudden and the
victim’s helpless position was accidental ”

VIVAR, 2
CRIMINAL LAW DIGESTS

In People v. Bautista, it was held: “. . . The circumstance that an attack was


sudden and unexpected to the person assaulted did not constitute the
element of alevosia necessary to raise homicide to murder, where it did
not appear that the aggressor consciously adopted such mode of attack to
facilitate the perpetration of the killing without risk to himself. Treachery
cannot be appreciated if the accused did not make any preparation to kill the
deceased in such manner as to insure the commission of the killing or to make
it impossible or difficult for the person attacked to retaliate or defend himself ”

Applying these principles to the case at bar, the prosecution has not proven that the
killing was committed with treachery. It was not shown that he consciously adopted
the mode of attacking the victim from behind to facilitate the killing without risk to
himself. Moreover, the mere fact that the accused-appellant stabbed the victim in a
public market at 7:00pm wherein the place was well-lighted and teeming with people.
He showed no concern that the people in the immediate vicinity might retaliate in
behalf of the victim. There are no indications that accusedappellant deliberately
planned to stab the victim at said time and place.

Therefore, accused-appellant should be held liable for homicide in the absence of the
qualifying circumstance of treachery.

2. Yes. Credibility should be given on the prosecution’s witnesses.

Case laws mandate that “when the credibility of a witness is in issue, the
findings of fact of the trial court, its calibration of the testimonies of the
witnesses and its assessment of the probative weight thereof, as well as its
conclusions anchored on said findings are accorded high respect if not
conclusive effect. This is more true if such findings were affirmed by the
appellate court, since it is settled that when the trial court’s findings have been
affirmed by the appellate court, said findings are generally binding upon this
Court.”

The inconsistencies and inaccuracies in the testimonies of the prosecution witnesses


are relatively trivial, minor, and do not impeach their credibility. The positive
identification and categorical statements of the prosecution witnesses that it was
accused-appellant who stabbed Guilbert prevail over accusedappellant’s self-serving
denial.

Furthermore, these minor contradictions were expected from said witnesses as they
differ in their impressions of the incident and vantage point in relation to the victim
and the accusedappellant. Therefore, credibility should be given on the prosecution’s
witnesses.

Fallo:
WHEREFORE, the instant appeal of accused-appellant is hereby DENIED for lack of
merit. The Decision dated February 14, 2008 of the Court of Appeals in CA-G.R. CR.-
H.C. No. 00270 is hereby AFFIRMED with MODIFICATION. Accused-appellant Vicente
Vilbar is found GUILTY of the crime of HOMICIDE, for which he is SENTENCED to
imprisonment of twelve (12) years of prision mayor, as minimum, to seventeen (17) years
and four (4) months of reclusion temporal, as maximum, and ORDERED to pay the
heirs of Guilbert Patricio the amounts of P50,000.00 as moral damages, P50,000.00 as
civil indemnity, and P25,000.00 as temperate damages.

VIVAR, 2
CRIMINAL LAW DIGESTS

Treachery; unlicensed firearms <<<

People v. Matibag
G.R. No. 206381, March 25, 2015
PERLAS-BERNABE, J:

Doctrine:
As the RTC and CA held, the special aggravating circumstance of use of unlicensed
firearm, which was duly alleged in the Information, should be appreciated in the
imposition of penalty. Presidential Decree No. (PD) 1866, as amended by Republic Act
No. (RA) 8294, treats the unauthorized use of an unlicensed firearm in the commission
of the crimes of homicide or murder as a special aggravating circumstance: Section 1.
Presidential Decree No. 1866, as amended, is hereby further amended to read as
follows: Section 1. Unlawful Manufacture, Sale, Acquisition, Disposition or Possession
of Firearms or Ammunition or Instruments Used or Intended to be Used in the
Manufacture of Firearms or Ammunition.—x x x. x x x x If homicide or murder is
committed with the use of an unlicensed firearm, such use of an unlicensed firearm
shall be considered as an aggravating circumstance.

Facts:
Matibag was charged with the crime of Murder and entered a plea of not guilty during
his arraignment. The prosecution asserted that Enrico Clar de Jesus Duhan, who just
came from a meeting with the other officers of the homeowners' association, was walking
along Iron Street when Matibag confronted Duhan, and asked, "ano bang pinagsasasabi
mo?" Duhan replied "wala," and without warning, Matibag delivered a fist blow hitting
Duhan on the left cheek and causing him to teeter backwards. Matibag then pulled out
his gun and shot Duhan, who fell face-first on the pavement. While Duhan remained in
that position, Matibag shot him several more times. PO2 Tom Falejo, a member of the
PNP, positively identified Matibag and stated on record that he arrested the latter. The
victim suffered gunshot wounds in the head and chest which led to his death.

In his defense, Matibag alleged that he was at the despedida party of his neighbor
when Duhan arrived. Wanting to settle a previous misunderstanding, Matibag
approached Duhan and extended his hand as a gesture of reconciliation. However,
Duhan pushed it away and said, "putang ina mo, ang yabang mo," thereby provoking
Matibag to punch him in the face. Matibag saw Duhan pull something from his waist
and fearing that it was a gun and Duhan was about to retaliate, Matibag immediately
drew his own gun, shot Duhan, and hurriedly left the place.

The CA agreed with the RTC's findings that: (a) treachery attended the killing of Duhan
as the attack on him was sudden; and (b) an unlicensed firearm was used in
committing the crime, which is considered as a special aggravating circumstance.

Issue:
Whether or not the CA correctly upheld the conviction of Matibag for Murder.

Ruling:
The appeal is bereft of merit. There is no indication that the trial court, whose findings
the CA affirmed, overlooked, misunderstood or misapplied the surrounding facts and
circumstances of the case. Hence, the Court defers to the trial court on this score.
Matibag is charged with the crime of Murder.

Evidently, the treacherous manner by which Matibag assaulted Duhan negates


unlawful aggression. The prosecution was able to prove that the attack was so sudden
and unexpected, and the victim was completely defenseless. On the other hand,
Matibag's version that he saw Duhan pull something from his waist (which thereby
impelled his reaction), remained uncorroborated. In fact, no firearm was recovered
from the victim. Hence, by these accounts, Matibag's allegation of unlawful aggression
and, consequently, his plea of self-defense cannot be sustained. The foregoing
considered, the Court upholds Matibag's conviction for the crime of Murder, qualified
by treachery, as charged.

VIVAR, 2
CRIMINAL LAW DIGESTS

Aggravating Circumstances; Special Aggravating Circumstances; Use of Unlicensed


Firearm; If homicide or murder is committed with the use of an unlicensed firearm,
such use of an unlicensed firearm shall be considered as an aggravating circumstance.
—As the RTC and CA held, the special aggravating circumstance of use of unlicensed
firearm, which was duly alleged in the Information, should be appreciated in the
imposition of penalty. Presidential Decree No. (PD) 1866, as amended by Republic Act
No. (RA) 8294, treats the unauthorized use of an unlicensed firearm in the
commission of the crimes of homicide or murder as a special aggravating
circumstance: Section 1. Presidential Decree No. 1866, as amended, is hereby further
amended to read as follows: Section 1. Unlawful Manufacture, Sale, Acquisition,
Disposition or Possession of Firearms or Ammunition or Instruments Used or
Intended to be Used in the Manufacture of Firearms or Ammunition.—x x x. x x x x If
homicide or murder is committed with the use of an unlicensed firearm, such use of an
unlicensed firearm shall be considered as an aggravating circumstance.

Fallo:
WHEREFORE, the appeal is DENIED. The Decision of the Court of Appeals finding
accused-appellant Daniel Matibag y De Villa GUILTY beyond reasonable doubt of the
crime of Murder, defined and penalized under Article 248 of the Revised Penal Code,
as amended, is hereby AFFIRMED with MODIFICATION.

VIVAR, 2
CRIMINAL LAW DIGESTS

Disguise; Treachery <<<

People v. Feliciano
G.R. No. 196735, May 5,
2014 LEONEN, J:

Doctrine:
In criminal cases, disguise is an aggravating circumstance because, like nighttime, it
allows the accused to remain anonymous and unidentifiable as he carries out his
crimes.

Facts:
 On December 8, 1994, seven (7) members of the Sigma Rho fraternity were eating
lunch at the Beach House Canteen of the UP Diliman, when they were attacked by
several masked men carrying baseball bats and lead pipes. Some of them sustained
injuries that required hospitalization. One of the Sigma Rho members, Dennis
Venturina, died from his injuries. An information for murder was filed against
several members of the Scintilla Juris fraternity, who wore masks and/or other
forms of disguise, conspiring, confederating with other persons whose true names,
identities and whereabouts have not as yet been ascertained, and mutually helping
one another, with intent to kill, qualified with treachery, and with evident
premeditation, taking advantage of superior strength, armed with baseball bats,
lead pipes, and cutters, did then and there willfully, unlawfully and feloniously
attack, assault and employ personal violence upon the person of Dennis Venturina.
Separate informations were also filed against them for the attempted murder and
frustrated murder of Sigma Rho fraternity members.

 One of the members (Arnel Fortes) ran but when he looked back, he saw Danilo
Feliciano Jr hitting Venturina. He was also able to see Warren Zipangan and George
Morano at the scene.

 The Sigma Rho counsel, Atty. Frank Chavez, told the UP Police that the victims will
give their statements to the NBI. Meanwhile, Venturina was transferred from the
UP Infirmary to St.Luke’s Hospital. He died after two days.

Defense: The accused provided different alibis, each claiming that they were not at the
scene of the crime.

RTC:
 “Although each victim had a very strong motive to place his fraternity rivals
permanently behind bars, not one of them testified against all of them. If the
prosecution eyewitnesses, who were all Sigma Rhoans, were simply bent on
convicting Scintilla Juris members for that matter, they could have easily tagged
each and every single accused as a participant.. each eye witness named only one or
two and some were candid enough to say that they did not see who delivered the
blows against them.”
 All of the accused-appellants were found guilty of the murder of Dennis Venturina
and the attempted murder of 5 victims.
 Because one of the penalties meted out was reclusion perpetua, the case was
brought to this court on automatic appeal. However, due to the amendment of the
Rules on Appeal, the case was remanded to the Court of Appeals.

CA: Affirmed the RTC decision, but modified the liabilities and found that the accused-
appellants were guilty of attempted murder only against 2 members (Natalicio and
Flores), and NOT against the other 3 (Mangrobang, Lachica, and Gaston). It is the
appellate court's reasoning that because Lachica and Mangrobang "were no longer
chased by the attackers," it concluded that accused-appellants "voluntary desisted
from pursuing them and from inflicting harm to them, which shows that they did not
have the intent to do more than to make them suffer pain by slightly injuring them." It
also pointed out that the wound inflicted on Gaston "was too shallow to have been
done with an intent to kill." Thus, it concluded that the accused-appellants would have

VIVAR, 2
CRIMINAL LAW DIGESTS

been guilty only of slight physical injuries.

VIVAR, 2
CRIMINAL LAW DIGESTS

Issue:
Whether accused-appellants’ constitutional rights were violated when the information
against them contained the aggravating circumstance of the use of masks despite the
prosecution presenting witnesses to prove that the masks fell off.

Ruling:
NO. Every aggravating circumstance being alleged must be stated in the information.
Failure to state an aggravating circumstance, even if duly proven at trial, will not be
appreciated as such. It was, therefore, incumbent on the prosecution to state the
aggravating circumstance of “wearing masks and/or other forms of disguise” in the
information in order for all the evidence, introduced to that effect, to be admissible by
the trial court. In criminal cases, disguise is an aggravating circumstance because it
allows the accused to remain anonymous and unidentifiable as he carries out his
crimes. The introduction of the prosecution of testimonial evidence that tends to prove
that the accused were masked but the masks fell off does not prevent them from
including disguise as an aggravating circumstance. What is important in alleging
disguise as an aggravating circumstance is that there was a concealment of identity
by the accused. The inclusion of disguise in the information was, therefore, enough to
sufficiently apprise the accused that in the commission of the offense they were being
charged with, they tried to conceal their identity. The appellate court, however,
incorrectly ruled out the presence of treachery in the commission of the offense. The
victims were eating lunch on campus and were not at a place where they would be
reasonably expected to be on guard for any sudden attack by rival fraternity men. The
swiftness and the suddenness of the attack using lead pipes and baseball bats gave no
opportunity for the victims to retaliate or even to defend themselves. Treachery,
therefore, was present in this case. Further, the information charges conspiracy
among the accused. Conspiracy presupposes that “the act of one is the act of all.” This
would mean all the accused had been one in their plan to conceal their identity even
if there was evidence later on to prove that some of them might not have done so.

Fallo:
WHEREFORE, the decision of the Court of Appeals in CA-G.R. CR No. 01158 dated
November 26, 2010 is AFFIRMED insofar as the accused appellants Danilo Feliciano,
Jr., Julius Victor Medalla, Christopher Soliva, Warren L. Zingapan, and Robert Michael
Beltran Alvir are found GUILTY beyond reasonable doubt of Murder in Criminal Case
No. Q95-61133 with the MODIFICATION that they be found GUILTY beyond
reasonable doubt of Attempted Murder in Criminal Case Nos. Q95-61136, Q95-61135,
Q95- 61134, Q95-61138, and Q95-61137.

VIVAR, 2
CRIMINAL LAW DIGESTS

Entrapment <<<

People v. Naelga
G.R. No. 171018, September 11, 2009
CHICO-NAZARIO, J:

Doctrine:
The presumption of innocence of an accused in a criminal case is a basic constitutional
principle, fleshed out by procedural rules that place on the prosecution the burden of
proving that the accused is guilty of the offense charged by proof beyond reasonable
doubt.

Facts:
Naelga was apprehended for selling shabu through a buy-bust operation. PO2
Sembran, posing as a buyer, approached Naelga and asked if he was a security guard
and what he could suggest so that he could keep himself awake and not be caught
sleeping in his post. Naelga passed his finger under his nose as if sniffing something
andPO2 asked what he meant, Naelga said bato or shabu. PO2 Sembran said he was
willing to try it and asked to buyP500 worth. He initially gave P400 in marked bills
which Naelga used to get the shabu he was about to sell PO2Sembran. Naelga was
then apprehended during their exchange of the shabu and the P100 balance. Naelga
initially denied the accusation claiming that he was selling CDs when PO2 Sembran
approached him asking for shabu. Naelga then claimed that he was not certain that
what he bought for PO2 Sembran was shabu but that he gave it toPO2 and then the
latter handcuffed him. The RTC found him guilty and sentenced him to life
imprisonment. CA affirmed.

Naelga elevated the case to the SC questioning the RTC’s reliance on the credibility of
the policemen who conducted the buy-bust operation on the following grounds: (1)
Material inconsistencies and gross contradictions in the testimonies of the police officers
destroyed their credibility; (2) The police officers failed to observe the proper guidelines
in securing the chain of custody of the prohibited drugs; this alleged failure to follow
proper procedure raises doubts as to whether the specimen examined by the forensic
chemist and presented in court was indeed the one retrieved from accused-appellant.
Thus, there can be no presumption of regularity. He claimed that it was PO2 Sembran
who instigated him to sell shabu.

The Solicitor Gen on the other hand, argued that the conviction should be sustained as
the alleged inconsistencies are minor and inconsequential, and therefore, does not
negate the occurrence of a buy-bust operation.

Issue:
WON the RTC erred in finding the accused guilty beyond reasonable doubt based on the
disputable presumption that the police officers regularly performed their function.

Ruling:
NO.
The general rule is that it is no defense that a decoy solicitation was used to facilitate
the entrapment. Mere deception by the detective will not shield the defendant if the
offense was committed by him free from the influence or instigation of the detective.
In this case, the law enforcers already received reports that accused was engaged in
illegal drug trade. There is no instigation as PO2 Sembran merely pretended to buy. In
an entrapment, ways and means are resorted to for the purpose of trapping and
capturing the lawbreakers in the execution of their criminal plan. In instigation, the
instigator practically induces the would-be defendant into the commission of the
offense, and himself becomes a co-principal. Entrapment is no bar to prosecution and
conviction; in instigation, the defendant would have to be acquitted.

While accused-appellant claims that it was PO2 Sembran who approached and asked
him to buy shabu for him, the same cannot be considered as an act of instigation, but
an act of “feigned solicitation.” Instigation is resorted to for purposes of entrapment,
based on the tip received from the police informant that accused-appellant was

VIVAR, 2
CRIMINAL LAW DIGESTS

peddling

VIVAR, 2
CRIMINAL LAW DIGESTS

illegal drugs in the public market of Rosales. In fact, it was accused-appellant who
suggested to PO2 Sembran to use shabu; and, despite accused-appellant’s statement
that he did not know anybody selling shabu, he still took the money from PO2
Sembran and directly went to Urdaneta, where he claimed to have bought the illegal
drug. Then he returned to the Rosales public market and gave the drug to PO2
Sembran.

As to his contention on the custody of the seized shabu, the SC found that there is no
broken chain in its custody. Nevertheless, assuming that the presumption of regularity
in the performance of official duty will not apply due to the failure to comply with the
guidelines, the same will not automatically lead to the exoneration of the accused.
Accused-appellant’s conviction was based not solely on the presumption of regularity,
but on the documentary and real evidence; and, more importantly, on the oral
evidence of prosecution witnesses, whom we found to be credible. One witness is
sufficient to prove the corpus delicti - that there was a consummated sale between the
poseur-buyer and the accused - there being no quantum of proof as to the number of
witnesses to prove the same. To emphasize, accused-appellant himself verified in his
testimony that the said transaction took place.

Fallo:
WHEREFORE, premises considered, the Decision of the Court of Appeals in CA-G.R. CR
No. 00304 convicting accused-appellant ELLY NAELGA of violation of Section 5, Article
II of Republic Act No. 9165, and sentencing him to suffer the penalty of life imprisonment
and to pay a fine of P500,000.00 is hereby AFFIRMED.

VIVAR, 2
CRIMINAL LAW DIGESTS

Cruelty and uninhabited place <<<

People v. Dizon
G.R. No. 134802, October 26, 2001
PER CURIAM

Doctrine:
Aggravating Circumstance; Cruelty; The test of cruelty is whether the accused
deliberately and sadistically augmented the wrong by causing another wrong not
necessary for its commission, or inhumanly increased the victim’s suffering, or
outraged, or scoffed at his person or corpse.

Facts:
On July 7, 1997, around 9:30 p.m., private complainant Arlie Rosalin, 21 years old,
was walking along the vicinity of Roosevelt Avenue Q.C.. Unknowingly by her, Accused
Appellant was behind her and suddenly seized her, pointing a fan knife to the side of
her neck, and announced a holdup. After appellant stripped her of her valuables, he
instructed her to walk with him past Roosevelt and Muñ oz to a dark and empty
basketball court in Project 7 Q.C.

Appellant kissed private complainant on the lips, neck, and breasts, which he also
mashed. He likewise bit her nipple at least three times, as well as the right side of her
back and vagina. He then forced her to bend forward over the hood of a taxi and, in this
position, forcefully penetrated her vagina with his organ. After satisfying himself in
this fashion, appellant ordered private complainant to hold and massage his penis, He
then forced her to put his foul-smelling penis into her mouth, forcing her to admire his
Bolitas ,. After that he pushed private complainant to the ground, he went down on her
and proceeded to ravish her all over again. After all these, appellant still refused to let
go of private complainant. Instead, he made her sit astride over him, and to make sure
she would not be able to escape, held her tightly by the hair with both hands.
Thereafter a struggle ensued and the victim was able to escape.

Three days later, the victim, accompanied by the police returned to the vicinity of
Muñ oz market and there they found the accused-appellant who is actually working as
a tricycle dispatcher. And there she was able to identify the offender by his tattoos and
face. The policemen captured him.

After trial, Appellant is sentenced with the Supreme Penalty of Death, for robbery with
rape and attended by two aggravating circumstances of cruelty and uninhabited place.

Upon review, appellant contended that the lower court erred in appreciating the
aggravating circumstances of cruelty and uninhabited place against the accused.

Issue:
Whether or not the acts of the appellant constituted Cruelty.

Ruling:
The court held that the element of cruelty undoubtedly 'attended the commission of the
crime in this case. As recounted by private complainant, appellant not only raped her,
but subjected her to various dehumanizing indignities, such as making her fondle and
put his foul-smelling penis in her mouth, forcing her to admire his bolitas , and
demanding that she assume embarrassing and indelicate positions. Furthermore, he
viciously slammed her head against the hood of the taxi, banged her head against the
wall, and slapped her hard in the face whenever she failed to answer any of his
questions. All these wrongs were no longer necessary insofar as appellant's purpose
of raping private complainant was concerned. By subjecting her to these unwarranted
physical and moral abuses on top of raping her, appellant deliberately and inhumanly
augmented her pain and sufferings, thus, committing cruelty.

VIVAR, 2
CRIMINAL LAW DIGESTS

In the case at bar, two (2) aggravating circumstances attended the commission of the
robbery with rape, thus the trial court correctly imposed on the appellant the supreme
penalty of death.

Fallo:
WHEREFORE, the Court AFFIRMS the decision of the Regional Trial Court of Quezon
City, Branch 219 in Crim. Case No. Q-71910 finding appellant Renato Dizon y Zuela
guilty beyond reasonable doubt of the crime of robbery with rape under paragraph
one, Art. 294 of the Revised Penal Code, as amended by RA 7659, attended by two (2)
aggravating circumstances, and sentencing him to suffer the penalty of death, to pay
victim Arlie Rosalin P200,000.00 as moral damages; and P9,500.00 as actual damages,
with the MODIFICATION that he shall further pay the victim P50,000.00 as civil
indemnity; and P25,000.00 as exemplary damages.

In accordance with Section 25 of Republic Act No. 7659, amending Article 83 of the
Revised Penal Code, upon finality of this decision, let certified copies thereof, as well as
the records of this case, be forwarded without delay to the Office of the President for
possible exercise of executive clemency.

VIVAR, 2
CRIMINAL LAW DIGESTS

Treachery; Band <<<

People v. Ancheta
G.R. No. 138306-07, December 21, 2001
BELLOSILLO, J:

Doctrine:
Aggravating Circumstances; Treachery; Treachery cannot be considered where the
witnesses did not see the commencement of the assault and the importance of such
testimonies cannot be overemphasized considering that treachery cannot be
presumed nor established from mere suppositions.

Facts:
1. This is an appeal from the decision 1 dated October 16, 1998 of the Regional
Trial Court of Cabanatuan City, Branch 30, convicting the appellant Felipe "Boy"
Ulep of the crime of robbery with homicide and sentencing him to suffer the
penalty of reclusion perpetua.
2. Appellant, together with William Ancheta, Edgardo "Liling" Areola, Antos Dacanay,
Lito dela Cruz and Ely Calacala, was charged with the crime of robbery with
multiple homicide and frustrated murder in an Information dated November 2,
1987.
3. That on or about the 20th day of March, 1987, at 12:00 o'clock to 1:00 o'clock in
the afternoon, at Manggahan, Bicos, Rizal, Nueva Ecija, Philippines, and within
the jurisdiction of this Honorable Court, the above-named accused, conspiring,
confederating and helping one another, did then and there wilfully, unlawfully
and feloniously, through force and intimidation upon persons, take, rob and
carry away thirty (30) cavans of clean palay valued at P4,500.00,belonging to
Alfredo Roca, to his damage and prejudice, and in order to successfully carry out
the robbery.
4. The accused, pursuant to the same conspiracy, willfully, unlawfully and
feloniously, with evident premeditation and with treachery, and with intent to kill,
fired their guns at Marjun Roca, which caused his death, shot at Benita
Avendañ o Roca and Febe Roca and hurled a grenade against them and both of
them died as consequence of the wounds they sustained; and also fired upon
Alfredo Roca with their firearms, thus performing all the acts of execution
which would produce the crime of murder as a consequence but which,
nevertheless, did not produce it by reason of the timely running for cover by the
said Alfredo Roca.

Version of the Prosecution:


1. Alfredo Roca testified that between 12:00 noon and 1:00 p.m. of March 20,
1987, he was in his farm in Manggahan, Rizal, Nueva Ecija to thresh palay. With
him at that time were Marjun Roca, Benita Roca, Febe Roca and daughter
Virgilita Roca-Laureaga. He, Benita and Febe were about to take their lunch
inside his hut. Marjun and Virgilita were done eating and were standing
outside.
2. At this point, Alfredo noticed the arrival of an owner-type jeep with trailer
which stopped at a spot not far from his hut. He recognized the occupants as
accused Antos Dacanay, Edgardo "Liling" Areola, William Ancheta, Lito de la
Cruz, Ely Calacala and appellant Felipe "Boy" Ulep who all alighted from the
jeep.
3. Dacanay, Areola and Ancheta stood on one side of the irrigation canal facing
Marjun Roca who was standing on the other side. From a distance of 10 to 12
meters, Alfredo saw Dacanay suddenly pull out a gun and shoot Marjun on the
head, causing the latter to fall to the ground. As he lay on the ground, Marjun
was again shot, this time by Areola and Ancheta.
4. Thereafter, Ulep, de la Cruz and Calacala started firing at Alfredo's hut. Alfredo
was not hit, however, because he was able to get out of the hut and dive into the
irrigation canal in the nick of time. However, Benita and Febe were fatally hit by
the initial volley of gunfire. The assailants fired at Alfredo in the canal but they
did not hit him.
5. Ancheta then hurled a grenade which exploded near the hut. When the group

VIVAR, 2
CRIMINAL LAW DIGESTS

ran out of bullets, Alfredo emerged from the canal and hid inside his hut. He saw
the group load onto the trailer 35 sacks of palay, each containing an average of

VIVAR, 2
CRIMINAL LAW DIGESTS

50 kilos valued at P4.50 per kilo. Alfredo owned the stolen palay. Appellant Ulep
and his companions then boarded their jeep and left.

Issue:
Whether or not the in the Commission of the crime the generic aggravating
circumstances of the Treachery, was committed with a band? YES

Ruling:
Only treachery and band were established. There was treachery as the events narrated
by the eyewitnesses pointed to the fact that the victims could not have possibly been
aware that they would be attacked by appellant and his companions. There was no
opportunity for the victims to defend themselves as the assailants, suddenly and
without provocation, almost simultaneously fired their guns at them. The essence of
treachery is the sudden and unexpected attack without the slightest provocation on
the part of the person attacked.

We deem it necessary to reiterate the principle laid down by the Court en banc in the
case of People vs. Escote, Jr. 15 on the issue of whether treachery may be appreciated
in robbery with homicide which is classified as a crime against property. This Court
held: Treachery is a generic aggravating circumstance to robbery with homicide
although said crime is classified as a crime against property and a single and
indivisible crime.

In fine, in the application of treachery as a generic aggravating circumstance to


robbery with homicide, the law looks at the constituent crime of homicide which is a
crime against persons and not at the constituent crime of robbery which is a crime
against property. Treachery is applied to the constituent crime of "homicide" and not
to the constituent crime of "robbery" of the special complex crime of robbery with
homicide. The crime of robbery with homicide does not lose its classification as a
crime against property or as a special complex and single and indivisible crime simply
because treachery is appreciated as a generic aggravating circumstance. Treachery
merely increases the penalty for the crime conformably with Article 63 of the Revised
Penal Code absent any generic mitigating circumstance.

In sum then, treachery is a generic aggravating circumstance in robbery with homicide


when the victim of homicide is killed by treachery. The offense was also proven to
have been executed by a band. A crime is committed by a band when at least four
armed malefactors act together in the commission thereof. In this case, all six accused
were armed with guns which they used on their victims. Clearly, all the armed
assailants, including appellant, took direct part in the execution of the robbery with
homicide.

Under Article 294 (1) of the Revised Penal Code, the crime of robbery with homicide
carries the penalty of reclusion perpetua to death. Inasmuch as the crime was
committed on March 20, 1987 which was prior to the effectivity of RA 7659 on December
31, 1993, the penalty of death cannot be imposed even if the aggravating
circumstances of treachery and band attended its commission. Only the single
indivisible penalty of reclusion perpetua is imposable on appellant.

With respect to damages, we affirm the award of P50,000 as civil indemnity each for
the death of Marjun, Febe and Benita Roca. In addition, moral damages must be
granted in the amount of P50,000 for each of the deceased victims. The amount of
P7,875 is also due to Alfredo Roca as reparation for the 35 sacks of palay stolen from
him, each valued at P225. The heirs of the victims are likewise entitled to exemplary
damages in the sum of P20,000 for each of the three victims due to the aggravating
circumstances that attended the commission of the crime. However, the award of
burial expenses cannot be sustained because no receipts were presented to
substantiate the same. Nonetheless, the victims' heirs are entitled to the sum of
P25,000 as temperate damages in lieu of actual damages, pursuant to the case of
People vs. Abrazaldo.

VIVAR, 2
CRIMINAL LAW DIGESTS

Fallo:
WHEREFORE, the Decision of the trial court appealed from convicting accused-
appellant SPO1 Eduardo Ancheta y Rodigol of Murder in Crim. Case No. C-44939 and
Frustrated Murder in Crim. Case No. C-44940, is MODIFIED.

In G.R. No. 138306 (Crim. Case No C-44939), accused-appellant SPO1 Eduardo Ancheta
y Rodigol is found guilty of HOMICIDE and is sentenced to an indeterminate prison
term of six (6) years eight (8) months and ten (10) days of prision mayor minimum as
minimum to twelve (12) years six (6) months and twenty (20) days of reclusion temporal
minimum as maximum. He is also ordered to pay the heirs of Julian Ancheta
P50,000.00 as death indemnity, P54,200.00 as actual and compensatory damages, plus
the costs.

VIVAR, 2
CRIMINAL LAW DIGESTS

Badge <<<

Ambagan, Jr. v. People


G.R. No. 204481-82, October 14, 2015
VELASCO, JR., J:

Doctrine:
A testimony that an accused made a statement which allegedly initiated the shootout,
such testimony not being entirely credible, cannot sustain the conviction of said
accused as principal by inducement.

Facts:
Accused Mayor Ambagan Jr. was charged and convicted by the Sandiganbayan with
two counts of homicide as principal by inducement or the deaths of SPO2 Reynaldo
Santos (Santos) and Domingo Bawalan. The prosecution presented statements from
two persons who was said to be directly present during the shooting.

The first witness (Bawalan) said that the shooting started after he heard the mayor
said “GE, IYAN PALA ANG GUSTO MO, MGA KASAMA BANATAN N’YO NA YAN”.

However, the second witness (Patam) contradicts this when he said that he instead
pushed the mayor out of the road where the shooting incident occurred and that
he did not hear the mayor saying those words which could have provoked and
initiate the shooting of the victims.

Further, evidence provides that Rene Amparo (one of Mayor Ambagan’s men) has
negative paraffin test which would lead to the fact that it is not the Mayor’s men
who initiated the shooting but rather from the deceased Rey Santos.

The Sandiganbayan convicted Ambagan of the crime of double homicide. Aggrieved,


petitioner moved for reconsideration of the aforequoted ruling. The Sandiganbayan,
however, would deny petitioner's motion through its assailed October 31, 2012
Resolution. Hence, the instant petition.

Issue:
Whether or not the accused Albert Ambagan is criminally liable as a principal by
inducement?

Ruling:
NO. The requisites for the conviction of a person as principal by inducement are (1)
that the inducement be made with the intention of procuring the commission of the
crime and (2) that such inducement be the determining cause of the commission by
the material executor. The Sandiganbayan would have been correct if the statement
was indeed made by the petitioner immediately before the shooting incident. However,
there are substantial inconsistencies in the testimonies of star prosecution witnesses
Patam and Ronnel Bawalan, which refer not only to minor details but even to facts
constituting important aspects of the case, seriously eroding the weight of the
evidence of the prosecution, and casting reasonable doubt on the culpability of
petitioner.

1. Only Ronnel Bawalan allegedly heard petitioner make the utterance. Between the
two star witnesses, Patam was in closer proximity to Ambagan and was thus in
a better position to know whether the statement was indeed made by the latter.
However, Patam made no such testimony despite the several opportunities he
could have done so.
2. The testimony of Ronnel Bawalan is not entirely credible, as (1) Ronnel testified
that he saw petitioner engage Santos in a verbal altercation in the middle of the
street, but thereafter claimed he did not see petitioner’s whereabouts when the
shooting began, (2) if the facts were as Ronnel stated, petitioner would have been
caught in the middle of the crossfire he himself allegedly ordered, (3) both
petitioner and Patam testify that petitioner wanted to go back (as Patam was
taking him into the house of Javier) during the exchange of gunshots – if Ronnel

VIVAR, 2
CRIMINAL LAW DIGESTS

was correct in saying that petitioner initiated the shootout, it would not make
sense why petitioner would want to go back to the shootout he started, (4) if
petitioner wanted to start the shootout, he would have drawn his own pistol,
(5) if it is true that, per Ronnel, only Santos from his group was armed, that
meant Santos was able to shoot down 3 of petitioner’s men before he fell
himself, (6) Ronnel first claimed that he personally witnessed the deaths of the
other individuals in the shootout (Causaren and Jamon) but reversed himself
during trial and claimed that he learned of such deaths from other sources, and
(7) Ronnel is biased against petitioner, as deceased Santos is his cousin-in-law
and deceased Domingo is his brother.

Fallo:
WHEREFORE, premises considered, the petition is hereby GRANTED. Petitioner Albert
G. Ambagan, Jr. is ACQUITTED of the two counts of homicide subject of Criminal
Case Nos. 28259 and 28260 on reasonable doubt. The June 21, 2012 Decision of
the Sandiganbayan First Division in Crim. Case Nos. 28259-60 is hereby MODIFIED
accordingly as follows: Ambagan, Jr. vs. People, 772 SCRA 489, G.R. Nos. 204481-82
October 14, 2015.

VIVAR, 2
CRIMINAL LAW DIGESTS

Badge <<<

People v. Dulay
G.R. No. 193854, September 24, 2012
PERALTA, J:

Doctrine:
Under the RPC, an accused may be considered a principal by direct participation, by
inducement, or by indispensable cooperation. To be a principal by indispensable
cooperation, one must participate in the criminal resolution, a conspiracy or unity in
criminal purpose and cooperation in the commission of the offense by performing
another act without which it would not have been accomplished.

Facts:
 Dina Dulay was introduced to AAA, then 12 yrs. old, by the latter’s sister as
someone who is nice. Thereafter, Dulay convinced AAA to accompany her at a
wake at Parañ aque City. Before going to the said wake, they went to a casino,
and thereafter to Sto. Niñ o, to look for Dulay’s boyfriend, but he was not there.
When they went to Bulungan Fish Port to ask for some fish, they saw Dulay's
boyfriend. Afterwards, AAA, Dulay and the latter's boyfriend proceeded to the
Kubuhan located at the back of the fish port. There, Dulay suddenly pulled
AAA inside a room where a man known by the name "Speed" was waiting. AAA
saw "Speed" give money to Dulay and heard "Speed" tell Dulay to look for
a younger girl. Thereafter, "Speed" wielded a knife, tied AAA's hands to the
papag, and raped her. AAA asked for Dulay's help when she saw the latter
peeping into the room while she was being raped, but Dulay did not do so. After
the rape, "Speed" and Dulay told AAA not to tell anyone what had happened or
else they would get back at her.Upon going to Laguna, AAA told her sister what
happened and the latter informed their mother about it. AAA, her sister and
mother, filed a complaint at the barangay. Thereafter, the barangay officials
referred the complaint to the police station.
 Upon conducting physical examination upon AAA, Dr. Tan issued a Medico-
Legal Report stating that there was no evident injury in the body of AAA, but
medical evaluation cannot exclude sexual abuse. During her testimony, Dr. Tan
explained that such impression or conclusion pertains to the ano-genital
examination and also stated that she found multiple abrasions on the back
portion of the body of AAA.
 RTC: Dulay GUILTY of RAPE as a co-principal by indispensable coopeRation.
 CA: Affirmed. Dulay cooperated in the perpetration of the crime of rape committed
by "Speed" by acts without which the crime would not have been consummated,
since she prepared the way for the perpetration thereof, convinced the victim to
go with her under the guise of looking for her boyfriend and upon arrival at the
kubuhan, she pulled the victim inside a room where "Speed" was waiting,
delivered the victim to him, and then after receiving some amount of money from
"Speed" she settled in another room together with her boyfriend so that "Speed"
might freely consummate the rape with violence and intimidation, as he did.

Issue:
Whether or not the accused should be held liable as a co-principal for the crime of Rape.

Ruling:
 NO, in order to be considered as a principal by indispensable cooperation, one
must participate in the criminal resolution, a conspiracy or unity in criminal
purpose and cooperation in the commission of the offense by performing
another act without which it would not have been accomplished.
 The Supreme Court held that the accused did not participate in the criminal
resolution of the crime of Rape but merely delivered AAA to “Speed.” Nothing in
the evidence presented by the prosecution does it show that the acts committed
by appellant are indispensable in the commission of the crime of rape. The
events narrated by the CA, from the time appellant convinced AAA to go with
her until appellant received money from the man who allegedly raped AAA, are

VIVAR, 2
CRIMINAL LAW DIGESTS

not

VIVAR, 2
CRIMINAL LAW DIGESTS

indispensable in the crime of rape. Anyone could have accompanied AAA and
offered the latter's services in exchange for money and AAA could still have
been raped. Even AAA could have offered her own services in exchange for
monetary consideration and still end up being raped. Thus, this disproves the
indispensable aspect of the appellant in the crime of rape. It must be
remembered that in the Information, as well as in the testimony of AAA, she was
delivered and offered for a fee by appellant, thereafter, she was raped by
"Speed."
 However, the accused is still liable for violation of Section 5 (a) of R.A. 7610, or
the Special Protection of Children Against Abuse, Exploitation and
Discrimination Act or a) Those who engage in or promote, facilitate or induce
child prostitution which include, but are not limited to, the following: (1) Acting
as a procurer of a child prostitute.
 Paragraph (a) essentially punishes acts pertaining to or connected with child
prostitution. It contemplates sexual abuse of a child exploited in prostitution. In
other words, under paragraph (a), the child is abused primarily for profit.
 The elements of paragraph (a) are:
1. the accused engages in, promotes, facilitates or induces child prostitution;
2. the act is done through, but not limited to, the following means:
a. acting as a procurer of a child prostitute;
b. inducing a person to be a client of a child prostitute by means of written
or oral advertisements or other similar means;
c. taking advantage of influence or relationship to procure a child as a
prostitute;
d. threatening or using violence towards a child to engage him as a
prostitute; or
e. giving monetary consideration, goods or other pecuniary benefit to a
child with intent to engage such child in prostitution;

Fallo:
WHEREFORE, the appeal of appellant Dina Dulay y Pascual is hereby DISMISSED.
However, the Decision of the CA is hereby MODIFlED as appellant is not guilty beyond
reasonable doubt of the crime of rape, but of violating Section 5 (a), Article III of R.A.
7610, as amended, for which she is sentenced to fourteen (14) years and eight (8)
months of reclusion temporal, as minimum, to twenty (20) years of reclusion
temporal, as maximum. Appellant is also ORDERED to pay AAA the amount of
P50,000.00 as civil indemnity.

VIVAR, 2
CRIMINAL LAW DIGESTS

Badge <<<

People v. Gambao
G.R. No. 172707, October 1, 2013
PEREZ, J:

Doctrine:
Jurisprudence is instructive of the elements required, in accordance with Article 18 of
the Revised Penal Code, in order that a person may be considered an accomplice,
namely,
(1) that there be community of design; that is knowing the criminal design of the
principal by direct participation, he concurs with the latter in his purpose; (2) that he
cooperates in the execution by previous or simultaneous act, with the intention of
supplying material or moral aid in the execution of the crime in an efficacious way;
and
(3) that there be a relation between the acts done by the principal and those attributed
to the person charged as accomplice.

Case Summary:
On 3 July 2005, AAA was introduced to the accused during the wake of a relative of
AAA. Thereafter, the accused convinced AAA to accompany her at a wake at GI San
Dionisio, Paranaque City. However, before they went to the wake, they went to look for
the boyfriend of the accused. They went to Bulungan Fish Port were they found the
boyfriend of the accused. They proceeded to the kubuhan, located at the back of the
Bulungan Fish Port. Upon arrival, the accused suddenly pulled AAA inside a room
where a man known only as “Speed” was waiting. AAA saw “Speed” give the accused
some money, then the latter left. “Speed” wielded a knife and tied AAA’s hands to the
papag and raped her. AAA asked for appellant’s help when she saw the latter peeping
into the room while she was being raped, but appellant did not do so. After the rape,
“Speed” and appellant told AAA not to tell anyone what had happened or else they
would get back at her. AAA, accompanied by her sister and mother filed a complaint
for Rape. The RTC rendered a decision finding the accused guilty as a co-principal by
indispensable cooperation for the crime of Rape.

Facts:
Lucia Chan (Chan) was a fish dealer based in Manila. She usually expected fish
deliveries, which were shipped by her suppliers from the provinces. Sometime in the
afternoon of 11 August 1998, two persons, one of whom was identified as Theng
Dilangalen (Dilangalen), went to Chan’s residence at FB Harrison St., Pasay City to
inquire about a certain passport alleged to have been mistakenly placed inside a box of
fish to be delivered to her. Unable to locate said passport, the two left. The next morning,
Dilangalen, together with another companion identified as Tony Abao (Abao), returned
looking for Chan but were told that she was out. When the two returned in the afternoon,
Chan informed them that the fish delivery had yet to arrive. Chan offered instead to
accompany them to the airport to retrieve the box of fish allegedly containing the
passport. Dilangalen and Abao declined and told Chan that they would be back later
that evening.

Dilangalen, accompanied by an unidentified person who remains at large, returned to


Chan’s residence that evening. Chan’s houseboy ushered them in and Chan met them
by the stairs. Thereat, the unidentified companion of Dilangalen pointed his gun at
Chan’s son, Levy Chan (Levy), and the house companions. As the unidentified man
forcibly dragged Chan, her son Levy tried to stop the man by grabbing his mother’s
feet. Seeing this, Dilangalen pointed his gun at Levy’s head forcing the latter to release
his grip on Chan’s feet. Levy thereafter proceeded to the Pasay Police Headquarters to
report the incident.

Chan was forced to board a "Tamaraw FX" van. After travelling for about two hours,
the group stopped at a certain house. Accused-appellant Edwin Dukilman (Dukilman)
warned Chan not to shout as he had his gun pointed at her mouth. Chan was
ordered to go with two women, later identified in court by Chan as appellants Monette
Ronas (Ronas) and Nora Evad (Evad). Chan was brought inside a house and was made

VIVAR, 2
CRIMINAL LAW DIGESTS

to lie down on a bed, guarded by Ronas, Evad, Dukilman and Jaman Macalinbol

VIVAR, 2
CRIMINAL LAW DIGESTS

(Macalinbol). Ronas and Evad threatened Chan that she would be killed unless she
paid 20 Million Pesos.

On 13 August 1998, Chan was awakened by Evad and was asked to board the
"Tamaraw FX" van. After travelling for about ten minutes, the van stopped and the
group alighted. Chan was brought to a room on the second floor of the house.
Inside the room were three persons whom Chan identified in court as Macalinbol, Raul
Udal (Udal) and Halil Gambao (Gambao). Another woman, later identified as
Thian Perpenian (Perpenian), arrived. At about 9:00 o’clock in the evening, a man
who was later identified as Teng Mandao (Mandao), entered the room with a handgun
and asked Chan "Bakit kayo nagsumbong sa pulis?” Another man, whom

Chan identified in court as Eddie Karim (Karim), ordered Mandao out of the room.
Karim informed Chan that he was sent by their boss to ask her how much money she
has. Chan was instructed to talk to her son through a cell phone and she gave
instructions to her son to get the ₱75,000.00 she kept in her cabinet. The group then
talked to Chan’s son and negotiated the ransom amount in exchange for his
mother’s release. It was agreed upon that Levy was to deliver ₱400,000.00 at the
"Chowking" Restaurant at Buendia Avenue.

Inspectors Narciso Ouano, Jr. (Inspector Ouano) and Cesar Mancao (Inspector Mancao),
who were assigned at the Pasay City area to conduct the investigation regarding the
kidnapping, were informed that the abductors called and demanded for ransom in
exchange for Chan’s release. During their surveillance the following day, Inspectors
Ouano and Mancao observed a Red Transport taxicab entering the route which led to
the victim’s residence. The inspectors observed that the occupants of the taxicab kept
on looking at the second floor of the house. The inspectors and their team tailed the
taxicab until Pansol, Calamba, Laguna, where it entered the Elizabeth Resort and
stopped in front of Cottage 1. Convinced that the woman the team saw in the cottage
was the victim, they sought clearance from Philippine Anti Organized Crime Task Force
(PAOCTF) to conduct a rescue operation.

On 14 August 1998, P/Insp. Vicente Arnado (Inspector Arnado) received information


that the abductors acceded to a ₱400,000.00 ransom money to be delivered at
"Chowking" Restaurant at Buendia Avenue at around 2:00 am. Upon learning of
the information, the team immediately and strategically positioned themselves
around the vicinity of the restaurant. At about 2:00 am, a light blue "Tamaraw FX"
van with 4 people on board arrived. The four took the ransom money and headed
towards the South Luzon Expressway. The surveillance team successfully intercepted
the van and arrested the 4 men, later identified in court as Karim, Abao, Gambao and
Dukilman. The team was also able to recover the ₱400,000.00 ransom.

At about 5:00 o’clock in the morning of the same day, the police team assaulted
Cottage No. 1, resulting in the safe rescue of Chan and the apprehension of seven of
her abductors, later identified in court as Dilangalen, Udal, Macalinbol, Mandao,
Perpenian, Evad and Ronas.

Issue:
Whether or not Thian Perpenian be considered as an accomplice to the crime.

Ruling:
Yes, Thian should be considered as an accomplice to the crime. Settled is the rule that
being present and giving moral support when a crime is being committed will make a
person responsible as an accomplice in the crime committed. As a rational person,
Thian was expected to report the incident to the police authorities. Here, Thian failed
to report the said incident and to add to that, she even spent the night at the cottage.
Thus, being present and giving moral support when a crime is being committed is
attended.

Also, the testimony of the victim that she saw Thian entered the room talking to other
accused of things not related to the crime is not sufficient evidence to hold Thian as
principal to the crime. Settled is the rule that in case of doubt, the participation of the

VIVAR, 2
CRIMINAL LAW DIGESTS

offender will be considered as that of an accomplice rather than that of a principal.


Therefore, the Court opines that Thian should not be held liable as a co-principal, but
rather only as an accomplice to the crime

Fallo:
WHEREFORE, the 28 June 2005 Decision of the Court of Appeals in CA-G.R. CR-H.C.
No. 00863 is hereby AFFIRMED WITH MODIFICATIONS. Accused-appellants HALIL
GAMBAO y ESMAIL, EDDIE KARIM y USO, EDWIN DUKILMAN y SUBOH, TONY ABAO
y SULA, RAUL UDAL y KAGUI, THENG DILANGALEN y NANDING, JAMAN
MACALINBOL y KATOL, MONETTE RONAS y AMPIL and NORA EVAD y MULOK are
found guilty beyond reasonable doubt as principals in the crime of kidnapping for
ransom and sentenced to suffer the penalty of Reclusion Perpetua, without eligibility
of parole. Accused-appellant THIAN PERPENIAN y RAFON A.K.A. LARINA PERPENIAN
is found guilty beyond reasonable doubt as accomplice in the crime of kidnapping for
ransom and sentenced to suffer the indeterminate penalty of six (6) months and one
(1) day of Prision Correccional, as minimum, to six (6) years and one (1) day of Prision
Mayor, as maximum. Accused-appellants are ordered to indemnify the victim in the
amounts of P100,000.00 as civil indemnity, P100,000.00 as moral damages and
P100,000.00 as exemplary damages apportioned in the following manner: the
principals to the crime shall jointly and severally pay the victim the total amount of
P288,000.00 while the accomplice shall pay the victim P12,000.00, subject to Article
110 of the Revised Penal Code on several and subsidiary liability.

The Court orders the Correctional Institute for Women to immediately release THIAN
PERPENIAN A.K.A. LARINA PERPENIAN due to her having fully served the penalty
imposed on her, unless her further detention is warranted for any other lawful causes.

Let a copy of this decision be furnished for immediate implementation to the Director of
the Correctional Institute for Women by personal service. The Director of the
Correctional Institute for Women shall submit to this Court, within five (5) days from
receipt of a copy of the decision, the action he has taken thereon.

VIVAR, 2
CRIMINAL LAW DIGESTS

Badge <<<

People v. Cupino
G.R. No. 125688, April 03, 2000
PANGANIBAN, J:

Doctrine:
To hold an accused guilty as a co-principal by reason of conspiracy, he must be shown
to have performed an overt act in pursuance or furtherance of the complicity. Hence,
conspiracy, exists in a situation where at the time the malefactors were committing the
crime, their actions impliedly showed unity of purpose among them, a concerted effort
to bring about the death of the victim.

The cooperation that the law punishes is the assistance knowingly or intentionally
rendered which cannot exist without previous cognizance of the criminal act intended
to be executed. It is therefore required in order to be liable either as a principal by
indispensable cooperation or as an accomplice that the accused must unite with the
criminal design of the principal by direct participation.

Facts:
In the evening of August 16, 1989 at around 9:45pm, during the celebration of a certain
fiesta in Patag, Cagayan de Oro City, suspect Mr. Ramon Galos & victim Mr. Gromyko
Valiente were having a heated argument. Such incident was happening in front of a
particular store called Dod’s situated at the crossing of the town of Patag.

Suddenly a fist fight occurred between Mr. Galos and Mr. Valiente. During this
instance, the appellants arrived in the scene. Aftewards, all three individuals (Galos,
Cupino and Dejoras) ganged up against and beat up the victim Mr. Valiente . This
resulted for the victim to run away but likewise the three accused pursued him.

When Mr. Galos was able to reach the victim, he suddenly stabbed him twice in
his stomach using a small bolo. This caused the victim to fall down and thus crawl on
the floor. Then afterwards, Mr. Cupino pulled the bolo embedded within the
victim’s body and tried to stab him as well also. But Mr. Dejoras tried to prevent
this from occurring by grabbing his friend’s hand which held the small bolo.

During this process, Dejoras wasn’t able to successfully grab hold of his friend’s
hand but instead caught the knife with its blade. This instantly injured the hand of
Mr. Dejoras. But nonetheless, Mr. Cupino proceeded with his intent of stabbing the
victim. Then both Galos and Cupino immediately fled the scene. The victim, Mr.
Valiente, was left lying on the floor soaked with his own blood. He was
eventually brough to the hospital but later died. Mr. Dejoras, on the other hand,
went to a hospital in order to seek treatment upon the hand wound he incurred.

RTC: (1) appellants Dejoras, Galos, and Cupino were guilty of murder, as the killing
was qualified by treachery; (2) conspiracy was proven by the chain of circumstantial
evidence submitted; and (3) the aggravating circumstance of superior strength was
absorbed by treachery and may no longer be used to increase the penalty to its
maximum period.

Issue:
Whether or not the appellant-accused were guilty as charged.

Ruling:
RTC’s decision insofar as the conviction of Cupino and Galos, Dejoras should be
acquitted.

In People v. Elijorde, a case with similar facts, we said:


Indeed, with respect to accused Reynaldo Punzalan, the Court cannot assert with moral
certainty that he is guilty of murder. Conspiracy must be proved as indubitably as
the crime itself through clear and convincing evidence, not merely by conjecture.
To hold an accused guilty as a co-principal by reason of conspiracy, he must be

VIVAR, 2
CRIMINAL LAW DIGESTS

shown to have performed an overt act in pursuance or furtherance of the


complicity. Hence, conspiracy, exists in a situation where at the time the malefactors
were committing the crime, their actions impliedly showed unity of purpose among them,
a concerted effort to bring about the death of the victim. In a great majority of cases,
complicity was established by proof of acts done in concert, i.e., acts which yielded the
reasonable inference that the doers thereof were acting with a common intent or design.
Therefore, the task in every case is determining whether the particular acts established
by the requisite quantum of proof do reasonably yield that inference.

Thus, in Elijorde we found one of the accused, a certain Punzalan, innocent under the
circumstances. Similarly, in the present case, we find Appellant Vincent Dejoras not
guilty.

Unlike the trial court, we are quite mindful of the testimony of Prosecution Eyewitness
Bahian regarding Appellant Dejoras' participation in that bloody incident on the eve of
the fiesta. His answers to the propounded questions merely established that Dejoras
joined Galos and Cupino when they approached the victim. The prosecution filled,
however, to show, what Dejoras specifically did that proved his participation in the
conspiracy. Rather, what the said eyewitness said was that Dejoras tried to prevent
Cupino from stabbing the victim, clearly showing that he did not support the criminal
intent and conspiracy of the other two accused. These incontrovertible data lead to one
conclusion: there is reasonable doubt on whether Dejoras conspired with Galos and
Cupino in killing Valliente. We are therefore constrained to exonerate him. Indeed, guilt
must be proven beyond reasonable doubt. In this case, there is reasonable doubt on the
culpability of Appellant Dejoras as a principal.

Dejoras cannot be held liable as an accomplice, either. In Elijorde, we said:

The cooperation that the law punishes is the assistance knowingly or intentionally
rendered which cannot exist without previous cognizance of the criminal act intended
to be executed. It is therefore required in order to be liable either as a principal by
indispensable cooperation or as an accomplice that the accused must unite with
the criminal design of the principal by direct participation.

The acts of Appellant Dejoras showed that he was not aware of his companions'
intent to kill Valliente; at the very least, there is reasonable doubt as to his
knowledge thereof. In any event, community of design, the first of the requisite
elements that must be present before a person may be held liable as an accomplice, is
lacking. On the prosecution's theory that Dejoras may have inflicted injury on the
victim when he joined in the fray, we have combed the records and found no basis for
this speculation. We note that the eyewitness could not recount the details of the
brawl, but merely provided a general picture, saying that everything happened so fast.
Hence, we find no basis for Appellant Dejoras' liability even for physical injuries.

Fallo:
WHEREFORE, we AFFIRM the appealed Decision insofar as it found Appellant Ignacio
Cupino GUILTY of MURDER and sentenced him to reclusion perpetua. Appellant
Cupino is solely responsible for paying the heirs of the victim, Gromyko Valliente, the
amounts of P50,000 as indemnity ex delicto, P30,000 as actual damages and P50,000
as moral damages. Appellant Vincent Dejoras is ACQUITTED and ordered RELEASED
from custody IMMEDIATELY, unless he is being legally held for another cause. In this
regard, the Director of the Bureau of Corrections is directed to report his compliance,
within five (5) days from receipt hereof. Costs against Appellant Cupino.

VIVAR, 2
CRIMINAL LAW DIGESTS

Badge <<<

People v. Agulos
G.R. No. 121828, June 27, 2003
CALLEJO, SR., J:

Doctrine:
To hold an accused guilty as a co-principal by reason of conspiracy, he must be shown
to have performed an overt act in pursuance or furtherance of the conspiracy. The
mere presence of an accused at the situs of the crime will not suffice; mere knowledge,
acquiescence or approval of the act without cooperation or agreement to cooperate on
the part of the accused is not enough to make him a party to a conspiracy.

Facts:
On June 7, 1998, Edmar Aguilos, Odilon Lagliba y Abregon and appellant Rene Gayot
Pilola were charged with the murder of Joselito Capa.

Of the three accused, Odilon Lagliba was the first to be arrested and tried, and
subsequently convicted of murder. The decision of the trial court became final and
executory. Accused Edmar Aguilos remains at large while accused Ronnie Diamante
reportedly died a month after the incident. Meanwhile, herein appellant Rene Gayot
Pilola was arrested. He was arraigned on March 9, 1994, assisted by counsel, and
pleaded not guilty to the charge. Thereafter, trial of the case ensued.

Prosecution:
The witness Elisa Rolan was inside their store in Mandaluyong at around 11:30PM.
Joselito Capa (deceased) and Julian Azul Jr. were drinking beer when Aguilos and
Odilon arrived at the store. Joselito and Julian invited them to join their drinking which
they accepted. Later on, a heated argument between the two parties turned into an
altercation. Elisa was able to pacify them. However, as Joselito and Julian were
about to leave, Edmar and Odilar returned. Edmar and Julian got into a fist fight.
Joselito tried to stop the fight however, this caused Odilon to pull out his knife and
stab Joselito in the neck. Ronnie and the appellant (Rene), who were across the
street, saw their gangmate Odilon stabbing the victim and decided to join the fray.
They pulled out their knives, rushed to the scene and stabbed Joselito. Julian was
able to run away.

Appellant
The appellant denied stabbing the victim and interposed the defense of alibi. He claims
he was inside his cousin’s house and was suffering from ulcer.

The appellant also argues that the prosecution failed to prove that he conspired
with Ronnie and Odilon in stabbing the victim to death. He could not have conspired
with Odilon as the incident was only a chance encounter between victim, the appellant
and his co-accused. In the absence of a conspiracy, the appellant cannot be held liable
as a principal by direct participation.

Crime Charged: Murder


Lower Court Ruling: WHEREFORE, this Court finds RENE GAYOT PILOLA of 606 Nueve
de Febrero Street, Mandaluyong City, GUILTY beyond reasonable doubt of Murder.

Issue:
Whether or not the trial court erred in finding that there was conspiracy.

Ruling:
NO.
Supreme Court: We are not persuaded by the ruminations of the appellant.

There is conspiracy when two or more persons agree to commit a felony and decide to
commit it. Conspiracy as a mode of incurring criminal liability must be proved
separately from and with the same quantum of proof as the crime itself. Conspiracy

VIVAR, 2
CRIMINAL LAW DIGESTS

need not be proven by direct evidence. After all, secrecy and concealment are essential
features of a successful conspiracy.

Conspiracy may be implied if it is proved that two or more persons aimed by their acts
towards the accomplishment of the same unlawful object, each doing a part so that
their combined acts, though apparently independent of each other, were, in fact,
connected and cooperative. There may be conspiracy even if an offender does not
know the identities of the other offenders, and even though he is not aware of all
the details of the plan of operation or was not in on the scheme from the
beginning. One need only to knowingly contribute his efforts in furtherance of
it.

Fallo:
WHEREFORE, the Decision, dated May 3, 1995, of Branch 164 of the Regional Trial
Court of Pasig City in Criminal Case No. 73615, finding appellant Rene Gayot Pilola
GUILTY beyond reasonable doubt of the crime of murder is AFFIRMED WITH
MODIFICATION. The appellant is hereby directed to pay to the heirs of the victim
Joselito Capa the amount of ₱50,000 as civil indemnity; the amount of ₱50,000 as
moral damages; and the amount of ₱25,000 as exemplary damages.

VIVAR, 2
CRIMINAL LAW DIGESTS

Badge <<<

People v. Watimar
G.R. No. 121651-52, August 16, 2000
YNARES-SANTIAGO, J:

Doctrine:

Facts:
The accused Fernando Watimar (50) raped his daughter Myra Watimar (20) at the
point of a knife and with threat to kill on two separate occasions (March 26, 1992 and
Nov 28, 1992).

Myra slept in the same room with her brothers and sisters: Bernarndo, Marilou,
Leonardo, Ariel and Lea. Myra testified that, on March 26 at around 2AM, she felt
someone on top of her. She then realized it was her father and tried to resist. However,
Fernando threatened her with a knife and told her not to resist. The second incident
happened on Nov 28 at 10 PM while she was cooking alone, her father suddenly kissed
her and pulled her to the place where they used to sleep.

The lower court found him guilty for the two cases of rape.

The accused claimed that in Nov 1992 he was working as a truck helper and that he
was unable to go home for a period of one month due to this work. He also claimed
that the false accusations were instigated by father in law. This was because his
brother in law Celestino (youngest brother of his wife) was the one who caused the
pregnancy of his daughter.

Accused appellant also contends that it would have been impossible to consummate
the rape because they were in the room with the victim’s 5 siblings. Her resistance
would have caused a commotion which would have alerted her brothers and sisters.

Crime Charged: Rape


Lower Court Ruling:
In Crim. Case No. 5513-AF, the Court finding the accused Fernando Watimar guilty
beyond reasonable doubt of the crime of RAPE, hereby sentences him to suffer the
penalty of RECLUSION PERPETUA

In Crim. Case No. 5513-AF, the Court finding the accused Fernando Watimar guilty
beyond reasonable doubt of the crime of RAPE, hereby sentences him to suffer the
penalty of RECLUSION PERPETUA.

Issue:
Whether or not the lower court erred in finding the accused guilty of rape.

Ruling:
NO
SUPREME COURT: The possibility of rape is not negated by the presence of even the
whole family of the accused inside the same room with the likelihood of being discovered.
Rape may be committed even when the rapist and the victim are not alone, or while the
rapist’s spouse was asleep, Rape does not necessarily have to be committed in an
isolated place and can in fact be committed in places which to many would appear to
be unlikely and high-risk venues for sexual advances.

Accused-appellant further contends that the victim did not do everything in her power
to prevent the assault on her virtue. The law does not impose upon a rape victim the
burden of proving resistance, especially where there is intimidation. In fact, physical
resistance need not be established in rape when intimidation is exercised upon the
victim and she submits herself against her will to the rapist’s lust because of fear for
her life or personal safety.

VIVAR, 2
CRIMINAL LAW DIGESTS

Accused-appellant, lastly, faults the complainant for tarrying for three (3) long years
before telling her mother about his nefarious conduct despite ‘countless’ opportunities
to seek the aid of her mother and other relatives particularly her grandfather who
lived a mere ten (10) meters from her house.

This Court has consistently held that delay in reporting rape incidents in the face of
physical violence cannot be taken against the victim. A rape victim’s action is oftentimes
overwhelmed by fear rather than reason. It is fear, springing from the initial rape, that
the perpetrator hopes to build up a climate of extreme psychological terror, which would,
he hopes, numb his victim to silence and submissiveness.

Fallo:
WHEREFORE, in view of all the foregoing, the Decision appealed from, finding accused-
appellant guilty beyondreasonable doubt of two counts of rape and sentencing him to
reclusion perpetua for each crime, is AFFIRMED with the MODIFICATIONS that the
accused-appellant is ordered to pay the victim Myra Watimar for each count of rape
the amounts of P50,000.00 by way of civil indemnity ex delicto, P50,000.00 as moral
damages and P25,000.00 by way of exemplary damages.

VIVAR, 2
CRIMINAL LAW DIGESTS

Badge <<<

People v. Arrojado
G.R. No. 130492, January 31,
2001 MENDOZA, J:

Doctrine:
The Revised Rules of Criminal Procedure took effect on December 1, 2000, requiring
that every complaint or information state not only the qualifying but also the aggravating
circumstances. This provision may be given retroactive effect in the light of the well
settled rule that “statutes regulating the procedure of the court will be construed as
applicable to actions pending and undetermined at the time of their passage.
Procedural laws are retroactive in that sense and to that extent. Thus herein
aggravating circumstance cannot be appreciated against accused-appellant for failure
to allege the same in the information.

Facts:
On or about June 1, 1996, the accused, armed with a knife, with intent to kill, with
treachery and evident premeditation, attack, assault, and stab one Mary Ann Arrojado,
on the different parts of the body. Thereby inflicting upon her serious and mortal
wounds which were the direct and immediate cause of her death. That by reason of
her death, her heirs incurred actual and moral damages. The information was read and
explained to accused-appellant in his native dialect, after which he pleaded not guilty.

Prosecution’s Version
Accused-appellant Salvador Arrojado and the victim Mary Ann Arrojado are first
cousins, their fathers being brothers. The victim's father, Alberto Arrojado, who was
living in Canada, suffered a stroke for which reason he decided to come home to Roxas
City and financially supported by the victim's sister Asuncion.

Starting February 15, 1996, accused-appellant lived with the victim and her father. He
helped care for the victim's father, for which he was paid a P1,000.00 monthly salary.

In the early morning of June 1, 1996, accused-appellant went to the house of his
cousin, Erlinda Arrojado Magdaluyo, and reported that the victim had committed
suicide. The victim, who was bloodied, was lying on her left side facing the bedroom
door with her hands clasped together. On her bed was a rosary and a crucifix. Near her
was a knife. Erlinda recognized it to be the knife kept in the kitchen. Erlinda also
noticed that the electric fan was turned on full blast, while all the windows were closed
except the window on the east side which was slightly open.

As he went to the other room, where the victim's father stayed, accused-appellant told
Erlinda that he was afraid he might be suspected as the one responsible for the
victim's death. The police noticed that the victim's room "was very neat as if nothing
happened." The police saw no signs of forcible entry. They made a sketch of the
victim's position in relation to the whole house and took pictures of her.

Dr. Lourdes Roldan, of the Roxas City Health Office, conducted the post-mortem
examination of the victim at 1:30 p.m. of June 1, 1996. Her findings revealed that the
victim sustained the following stab wounds 10 stab wounds, 5 of which are fatal.

Dr. Roldan listed "hemorrhagic shock” as the victim's immediate cause of death and
"multiple stab wounds" as the antecedent cause.

Erlinda Arrojado Magdaluyo testified that the relationship between the victim and
accused-appellant had been strained as the victim constantly picked on accused-
appellant even for the slighest mistake. Erlinda remembered the scolding that the
victim gave accused-appellant on May 27, 1996 over the loss of keys. The victim also
entrusted her jewelries and bank accounts to Erlinda, but later returned the same
thereafter.

VIVAR, 2
CRIMINAL LAW DIGESTS

Another relative, Thelma Arrojado, corroborated Erlinda's testimony that she knew
the latter to be a snob ("suplada") and overly strict. Because they did not get along
with the victim, Thelma and her husband eventually left. She testified that accused was
angry at the victim and in fact passed by her store thrice complaining to her of the
victim's maltreatment of him.

Accused’s Version/Defense
Accused testified in his behalf. He told that Alberto Arrojado asked him for food, so
accused went to the kitchen to find out if the victim had already prepared breakfast.
When he found that the victim was not in the kitchen, he proceeded to the victim's room.
From the doorway, he saw the victim lying on her bed, bloodied. He thought that the
victim had committed suicide because the victim had told him that she felt tied down
taking care of her father. She in fact once remarked that "It would be better that my
father and I commit suicide."

Accused said that the victim scolded him only once and that was for buying rotten
cabbage. He said that the victim was the one who was constantly being scolded by her
father who often found fault with her.

Crime charged: murder


RTC: guilty beyond reasonable doubt of the heinous crime of murder

In the service of his sentence consisting of deprivation of liberty, the accused, who is a
detention prisoner and not otherwise disqualified, shall be credited with the full time
of his confinement under preventive imprisonment, provided he voluntarily agrees in
writing to abide by the same disciplinary rules imposed on convicted prisoners,
pursuant to Art. 29 of the RPC.

Issue:
1) Whether or not the Regional Trial Court erred in finding that Mary Ann Arrojado
was stabbed ten times at her home.
2) Whether or not the Regional Trial Court erred in ruling that Mary Ann Arrojado
could not have committed suicide.
3) Whether or not the Regional Trial Court erred in finding that the house of Mary
Ann Arrojado was totally closed and locked against intruders.
4) Whether or not the Regional Trial Court erred in finding that accused-appellant
was abused and oppressed by Mary Ann Arrojado that led accused-appellant to
kill Mary Ann Arrojado.

Ruling:
1. No, the victim indeed sustained ten stab wounds.

That accused-appellant only saw one wound while Erlinda Magdaluyo saw two
wounds on the victim does not necessarily mean that the other wounds were
inflicted upon the victim afterwards. The two might have simply missed seeing
the other wounds. In accused-appellant's case, it may be because he did not go
inside the room but only viewed the body from a distance. Dr. Roldan, who
conducted a post-mortem examination, testified that the victim actually
sustained ten wounds.

Accused-appellant also argues that the varying depths of wound despite the fact
that they had the same surface length of 3 cm. could only mean that after the
victim was found dead, she was again stabbed with a knife or knives other than
the one beside her. The contention is without merit. The variance in depth does
not necessarily mean that more than one weapon was used.

2. No, the victim did not commit suicide.

Somewhat inconsistently with his claim that the victim committed suicide,
accused-appellant disputes the trial court's conclusion that only one of those
residing in the house could have killed the victim because the police found no

VIVAR, 2
CRIMINAL LAW DIGESTS

sign of a break-in. Accused-appellant says that in the morning of June 1, 1996,


he found that the kitchen door leading outside was open. Accused-appellant's
contention must fail. Accused-appellant admitted that it did not occur to him
that an intruder was in the house in the evening of May 31, 1996 because "No
person could get inside because the windows were closed and besides the doors
were closed." Accused-appellant never told Erlinda that the kitchen door was
open that morning. Indeed, Erlinda testified that "it is not possible that
somebody would enter the house as the doors were securely locked… with
additional barrel bolts, and the windows have grills."

3. No, the house of Mary Ann Arrojado was totally closed and locked against
intruders.

To be sure, the evaluation of the trial court of the credibility of witnesses will
not be disturbed on appeal unless it is shown that it overlooked certain facts or
circumstances. In sum, the following circumstances point to accused-appellant
as the perpetrator of the crime:
a. Accused-appellant, the victim, and the latter’s father were the only ones
living in the house in which the crime was committed in the evening of
May 31, 1996.
b. No one from the outside can gain entry since all doors of the house were
locked and the windows had grills.
c. Accused-appellant had access to the victim’s bedroom because the
bedroom doors were left unlocked so that the victim could check on her
father’s condition
d. The murder weapon was a kitchen knife readily accessible to the
occupants of the house. As the Solicitor General observed, common sense
dictates that if an outsider entered the house with the intent to kill the
victim, he would have brought his own weapon to ensure the execution
of his purpose.
e. None of the victim’s belongings was missing or disturbed, indicating
that the motive for the crime was not gain but revenge.
f. Judging from the number and severity of the wounds (10 stab wounds,
half of which were fatal), the killer felt deep-seated resentment and
anger toward the victim. Accused-appellant had admitted those feelings
to Erlinda Arrojado Magdaluyo and Thelma Arrojado.
g. Aside from accused-appellant, no one was known to harbor a grudge
against the victim.
h. As the Solicitor General also pointed out, accused-appellant’s behavior in
the morning of June 1, 1996 was inconsistent with someone who had
just found his cousin and employer, a person he claims to get along with,
dead. By his testimony, he did not even go inside the room to check on
her condition on the lame excuse that he was afraid. He also did not
inform his neighbors about the incident for the equally flimsy reason that
he did not know them nor did he go to the police.

Under Rule 133, Section 4 of the Rules on Evidence, circumstantial


evidence is sufficient for conviction if:
i. there is more than one circumstance;
ii. the facts from which the inference are derived are proven; and
iii. the combination of all circumstances is such as to produce a
conviction beyond reasonable doubt.

As the foregoing discussion shows, these requisites have been established in this case.

4. No, the RTC correctly appreciated the qualifying circumstance of


treachery against accused-appellant. Anent the first requisite, Dr. Roldan
testified that based on her findings, the victim was not in a position to fight
the assailant and that she might have been stabbed while she was asleep. As
regards the second requisite, the number and nature of the wounds
sustained by the victim lead to no other conclusion that accused-appellant
employed means in

VIVAR, 2
CRIMINAL LAW DIGESTS

killing the victim which tended directly and specially to ensure its execution
without risk to himself arising from the defense; so many wounds, a total 10, half
of which were fatal, if he had not deliberately adopted such manner of attack.

Abuse of superior strength also attended the killing since accused-


appellant, a man and armed with a knife, attacked the victim, an unarmed and
defenseless woman. However, since abuse of superior strength is absorbed
in treachery, there is no need to appreciate it separately as an independent
aggravating circumstance.

The trial court correctly held that there was no proof of evident premeditation
since the requisites thereof have not been established in this case.

No generic aggravating circumstance of dwelling since the latter and the


victim lived in the same house.

The aggravating circumstance of abuse of confidence is present. For this


aggravating circumstance to exist, it is essential to show that the confidence
between the parties must be immediate and personal such as would give the
accused some advantage or make it easier for him to commit the criminal
act. The confidence must be a means of facilitating the commission of the
crime, the culprit taking advantage of the offended party's belief that the
former would not abuse said confidence. In this case, while the victim may have
intimated her fear for her safety for which reason she entrusted her jewelry and
bank book to Erlinda Arrojado Magdaluyo, her fears were subsequently allayed
as shown by the fact that she took back her personal effects from Erlinda.
Thinking that accused-appellant would not do her any harm, because he was
after all her first cousin, the victim allowed accused-appellant to sleep in the
same room with her father and left the bedroom doors unlocked.

Death Penalty(moot) and Retroactivity of the Law/Rules of Court


The murder in this case took place after the effectivity of R.A. 7659 on
December 31, 1993 which increased the penalty for murder from reclusion
temporal maximum to death to reclusion perpetua to death. In view of the
presence of the aggravating circumstance of abuse of confidence and in accordance
with Art. 63(1) of the Revised Penal Code, the trial court should have imposed
the penalty of death on accused-appellant.

However, on December 1, 2000, the Revised Rules of Criminal Procedure took


effect, requiring that every complaint or information state not only the
qualifying but also the aggravating circumstances. This provision may be
given retroactive effect in the light of the well settled rule that "statutes
regulating the procedure of the court will be construed as applicable to actions
pending and undetermined at the time of their passage. Procedural laws
are retroactive in that sense and to that extent." The aggravating
circumstance of abuse of confidence not having been alleged in the information,
the same therefore could not be appreciated to raise accused-appellant's
sentence to death.
In People vs. Lucas, the penalty of reclusion perpetua remains indivisible
notwithstanding the fixing of its duration from twenty (20) years and one (1) day
to forty (40) years, the trial court erred in imposing on accused-appellant the
penalty of 30 years of reclusion perpetua. In line with the ruling in Lucas, accused-
appellant should suffer the entire extent of forty (40) years of reclusion
perpetua.

Fallo:
WHEREFORE, the decision of the Regional Trial Court, Branch 19, Roxas City, is AFFIRMED
with the MODIFICATION that accused-appellant Salvador Arrojado is sentenced to suffer the
penalty of reclusion perpetua in its entire duration and to its full extent. Furthermore, he is
ordered to pay the heirs of the victim Mary Ann Arrojado the amount of P50,000.00 as civil
indemnity and the further sum of P50,000.00 as moral damages and the costs.

VIVAR, 2
CRIMINAL LAW DIGESTS

Badge <<<

People v. Nelmida
G.R. No. 184500, September 11, 2012
PEREZ, J:

Doctrine:
When various victims expire from separate shots, such acts constitute separate and
distinct crimes.

In a complex crime, two or more crimes are actually committed, however, in the eyes of
the law and in the conscience of the offender they constitute only one crime, thus, only
one penalty is imposed. There are two kinds of complex crime. The first is known as
compound crime, or when a single act constitutes two or more grave or less grave
felonies while the other is known as complex crime proper, or when an offense is a
necessary means for committing the other.

Facts:
Wenceslao Nelmida and Ricardo Ajok, with 10 other accused, allegedly ambushed the
vehicle of Mayor Tawan-tawan of Salvador, Lanao del Norte. Together with the Mayor
are his security escorts, 2 of which died due to the ambush while the others have been
injured, although not severely, but needed to be hospitalized. In the conduct of the said
alleged ambush, appellants and their co-accused brought Samuel (whose task was to
identify the vehicle) to a waiting shed where a pick-up service vehicle boarded by
Mayor Tawan-tawan and his group would pass. Appellants and their co-accused,
thereafter, assembled themselves on both sides of the road and surreptitiously waited
for the vehicle. The moment it passed by the waiting shed, appellants and their co-
accused opened fire and rained bullets thereon resulting in the killing and wounding of
the victims. Immediately, appellants and their co-accused fled to escape.

The prosecution presented their witnesses. Samuel, who became a witness, categorically
pointed to Nelmida and Ajok as two of the people who participated in the commission of
the ambush as well as named all the other co-accused. The same was given as testimony
by other witnesses, including the other surviving victims of the ambush as they were,
accordingly, of the vantage point where they saw the perpetrators of the offense.

In their defense, both herein accused interposed their respective alibis corroborated
by testimonies of their respective relatives, and being that they both left their
respective domiciles after the incident, posed their separate explanations. In the case
of Nelmida, that such was prompted by the fear and restlessness of his wife so that she
wanted to go to an environment further from the place of the ambush; and for Ajok,
that such was prompted by fear of being harassed as what he said was happening to
the others with the same political affiliation as him.

CRIME CHARGED: Amended Information: DOUBLE MURDER with MULTIPLE


FRUSTRATED MURDER and DOUBLE ATTEMPTED MURDER.
RTC: GUILTY beyond reasonable doubt of double murder with multiple frustrated
murder and double attempted murder and imposing upon them the penalty of
reclusion perpetua.
CA: AFFIRMED the RTC Decision.

Issue:
Whether or not there complex crime?

Ruling:
No, in a complex crime, two or more crimes are actually committed, however, in
the eyes of the law and in the conscience of the offender they constitute only one
crime, thus, only one penalty is imposed. There are two kinds of complex crime. The
first is known as compound crime, or when a single act constitutes two or more grave
or less grave felonies while the other is known as complex crime proper, or when an
offense is a necessary means for committing the other.

VIVAR, 2
CRIMINAL LAW DIGESTS

The classic example of the first kind is when a single bullet results in the death of two
or more persons. A different rule governs where separate and distinct acts result in a
number killed. Deeply rooted is the doctrine that when various victims expire from
separate shots, such acts constitute separate and distinct crimes.

Evidently, there is in this case no complex crime proper. And the circumstances
present in this case do not fit exactly the description of a compound crime.

From its factual backdrop, it can easily be gleaned that the killing and wounding of
the victims were not the result of a single discharge of firearms by the appellants
and their co-accused. Appellants and their co-accused performed not only a single
act but several individual and distinct acts in the commission of the crime. Thus,
Article 48 of the Revised Penal Code would not apply for it speaks only of a "single
act."

Appellants should be convicted of SEPARATE CRIME – two (2) counts of murder and
seven (7) counts of attempted murder – and NOT of a COMPLEX CRIME

Fallo:
WHEREFORE, premises considered, the Decision of the Court of Appeals in CA-G.R. HC
No. 00246 dated 18 June 2008 is hereby MODIFIED, as follows: (1) appellants are
found guilty beyond reasonable doubt of two (2) counts of murder thereby imposing upon
them the penalty of reclusion perpetua for each count; (2) appellants are also found
guilty beyond reasonable doubt of seven (7) counts of attempted murder thereby
imposing upon them the indeterminate penalty of 4 years and 2 months of prision
correccional, as minimum, to 10 years of prision mayor, as maximum, for each count;
(3) other than the civil indemnity and moral damages already awarded by the trial
court and the appellate court, appellants are further ordered to pay, jointly and
severally exemplary and temperate damages in the amount of P30,000.00 and
P25,000.00, respectively, to the heirs of each deceased victims; and (4) appellants are
also directed to pay, jointly and severally, Macasuba, Mosanip, PFC Tomanto, PFC
Angni and Juanito the amount of P40,000.00 each as moral damages, P25,000.00
each as temperate damages and P30,000.00 each as exemplary damages.
Costs against appellants.

VIVAR, 2
CRIMINAL LAW DIGESTS

Badge <<<

People v. Punzalan
G.R. No. 199892, December 10, 2012
LEONARDO-DE CASTRO, J:

Doctrine:
When a single act constitutes two or more grave or less grave felonies, or when an offense
is a necessary means for committing the other, the penalty for the most serious crime
shall be imposed, the same to be applied in its maximum period.

In view of the enactment of RA 9346, prohibiting the imposition of the death penalty,
the penalty for the killing of each of the two victims is reduced to reclusion perpetua
without eligibility for parole.

Facts:
In August 2002, Seaman 1st Class (SN1) Arnulfo Andal, SN1 Antonio Duclayna, SN1
Evelio Bacosa, SN1 Cesar Domingo, SN1 Danilo Cuya, and SN1 Erlinger Bundang were
among the members of the Philippine Navy sent for schooling at the Naval Education
and Training Command (NETC) at San Miguel, San Antonio, Zambales.

On August 10, 2002, at around 5:00 or 6:00 in the afternoon, they went to the
“All-in-One” Canteen to have some drink. Later, at around 10:00 in the evening, they
transferred to a nearby videoke bar, “Aquarius,” where they continued their drinking
session. Shortly thereafter, a heated argument between SN1 Bacosa and appellant
ensued regarding a flickering light bulb inside “Aquarius.” When SN1 Bacosa
suggested that the light be turned off (“Patayin ang ilaw”), appellant who must have
misunderstood and misinterpreted SN1 Bacosa’s statement belligerently reacted asking,
“Sinong papatayin?,” thinking that SN1 Bacosa’s statement was directed at him.
SN1 Cuya tried to pacify SN1 Bacosa and appellant, while SN1 Bundang apologized to
appellant in behalf of SN1 Bacosa. However, appellant was still visibly angry,
mumbling unintelligible words and pounding his fist on the table.

To avoid further trouble, the navy personnel decided to leave “Aquarius” and return to
the NETC camp. They walked in two’s, namely, SN1 Bundang and SN1 Domingo in the
first group, followed by the group of SN1 Bacosa and SN1 Cuya, and SN1 Andal and
SN1 Duclayna in the last group, with each group at one arm’s length distance from the
other. Along the way, they passed by the NETC sentry gate which was being manned
by SN1 Noel de Guzman and F1EN Alejandro Dimaala at that time. SN1 Andal and SN1
Duclayna even stopped by to give the sentries some barbecue before proceeding to follow
their companions.

Soon after the navy personnel passed by the sentry gate, SN1 De Guzman and F1EN
Dimaala flagged down a rushing and zigzagging maroon Nissan van with plate
number DRW 706. The sentries approached the van and recognized appellant, who was
reeking of liquor, as the driver. Appellant angrily uttered, “kasi chief, gago ang mga
‘yan!,” while pointing toward the direction of the navy personnel’s group. Even before
he was given the go signal to proceed, appellant shifted gears and sped away
while uttering, “papatayin ko ang mga ‘yan!”

While F1EN Dimaala was writing the van’s plate number and details in the logbook, he
suddenly heard a loud thud. Meanwhile, SN1 De Guzman saw how the van sped away
towards the camp and suddenly swerved to the right hitting the group of the walking
navy personnel prompting him to exclaim to F1EN Dimaala, “chief, binangga ang
tropa!” SN1 De Guzman then asked permission to go to the scene of the incident and
check on the navy personnel.

When they were hit by the vehicle from behind, SN1 Cuya and SN1 Bacosa were
thrown away towards a grassy spot on the roadside. They momentarily lost
consciousness. When they came to, they saw SN1 Duclayna lying motionless on the
ground. SN1 Cuya tried to resuscitate SN1 Duclayna, while SN1 Bacosa tried to chase

VIVAR, 2
CRIMINAL LAW DIGESTS

the van. SN1 Domingo was not hit by the van as he was in the first group and was
pushed away from the path of the speeding van. He was able to see the vehicle’s
plate number. He also tried to chase the van with SN1 Bacosa but they turned around
when the vehicle made a U-turn as they thought that it would come back for them. The
vehicle, however, sped away again when other people started to arrive at the scene of the
incident.

CRIME CHARGED: Complex Crime of Double Murder qualified by treachery with


Multiple Attempted Murder attended by aggravating circumstance of use of motor
vehicle
RTC: GUILTY of Complex crime of Double Murder qualified by treachery with multiple
Attempted Murder attended by the aggravating circumstance of use of motor vehicle and
is hereby sentenced to suffer the penalty of Reclusion Perpetua.
CA: Affirmed

Issue:
Whether or not appellant guilty of the complex crime of murder with multiple attempted
murder?

Ruling:
Yes, the felony committed by appellant as correctly found by the RTC and the
Court of Appeals, double murder with multiple attempted murder, is a complex
crime contemplated under Article 48 of the Revised Penal Code: Art. 48. Penalty
for complex crimes. – When a single act constitutes two or more grave or less grave
felonies, or when an offense is a necessary means for committing the other, the penalty
for the most serious crime shall be imposed, the same to be applied in its maximum
period. Appellant was animated by a single purpose, to kill the navy personnel,
and committed a single act of stepping on the accelerator, swerving to the right
side of the road ramming through the navy personnel, causing the death of SN1
Andal and SN1 Duclayna and, at the same time, constituting an attempt to kill SN1
Cuya, SN1 Bacosa, SN1 Bundang and SN1 Domingo.

The crimes of murder and attempted murder are both grave felonies as the law attaches
an afflictive penalty to capital punishment (reclusion perpetua to death) for murder
while attempted murder is punished by prision mayor, an afflictive penalty. Under
Article 248 of the Revised Penal Code, as amended, murder is punishable by reclusion
perpetua to death.

Article 63[56] of the same Code provides that if the penalty prescribed is composed
of two indivisible penalties, as in the instant case, and there is an aggravating
circumstance the higher penalty should be imposed. Since use of vehicle can be
considered as an ordinary aggravating circumstance, treachery, by itself, being
sufficient to qualify the killing, the proper imposable penalty the higher sanction is
death. In view of the enactment of Republic Act No. 9346, prohibiting the imposition of
the death penalty, the penalty for the killing of each of the two victims is reduced to
reclusion perpetua without eligibility for parole.

Fallo:
WHEREFORE, the appeal is hereby DENIED. The Decision dated April 29, 2011 of the Court of
Appeals in CA-G.R. CR.-H.C. No. 02816 affirming the conviction of appellant Arturo Punzalan, Jr.
for the complex crime of double murder with multiple attempted murder, imposing upon him the
penalty of reclusion perpetua and ordering him to pay the following:
(a) To the respective heirs of SN1 Arnulfo Andal and SN1 Antonio Duclayna:
(i) P75,000.00 civil indemnity;
(ii) P75,000.00 moral damages;
(iii) P30,000.00 exemplary damages; and
(iv) P25,000.00 temperate damages;
(b) To the heirs of SN1 Andal, P2,172,270.21 for loss of earning capacity;
(c) To each of the surviving victims, SN1 Danilo Cuya, SN1 Evelio Bacosa, SN1 Erlinger Bundang
and SN1 Cesar Domingo:
(i) P40,000.00 moral damages; and
(ii) P30,000.00 exemplary damages; and
(d) To SN1 Cuya, SN1 Bacosa and SN1 Bundang, P25,000.00 temperate damages each is
AFFIRMED.

VIVAR, 2
CRIMINAL LAW DIGESTS

Badge <<<

Santiago v. Garchitorena
G.R. No. 109266, December 2, 1993
QUIASON, J:

Doctrine:
For delito continuado to exist there should be a plurality of acts performed during a
period of time; unity of penal provision violated; and unity of criminal intent or purpose,
which means that two or more violations of the same penal provisions are united in one
and same instant or resolution leading to the perpetration of the same criminal purpose
or aim.

Facts:
Petitioner Miriam Defensor-Santiago, the then Commission of Immigration and
Deportation (CID) Commissioner, was charged in Criminal Case No. 16698 of the
Sandiganbayan with violation of Section 3(e) of R.A. No. 3019, as amended,
otherwise known as the Anti-Graft and Corrupt Practices Act, allegedly committed
by her favoring "unqualified" aliens with the benefits of the Alien Legalization Program
wherein Santiago, approved the application for legalization of the stay of about 32
aliens who arrived in the Philippines in violation of Executive Order No. 324 which
does not allow the legalization of the same, thereby causing undue injury to the
government and giving unwarranted benefits and advantages to said aliens in the
discharge of the official and
administrative functions of said accused.

She filed a petition for certiorari and prohibition to enjoin the Sandiganbayan from
proceeding with Criminal Case No. 16698 on the ground that said case was
intended solely to harass her as she was then a presidential candidate. She also
moved to inhibit Sandiganbayan Presiding Justice Garchitorena from the case and to
defer her arraignment pending action on her motion to inhibit. Her motion was denied
by the Sandiganbayan.

Santiago filed a motion for a bill of particulars stating that while the Information alleged
that she had approved the application for legalization of "aliens" and gave them indirect
benefits and advantages it lacked a list of the favored aliens. According to her, unless
she was furnished with the names and identities of the aliens, she could not
properly plead and prepare for trial.

She contended in this case that the public prosecutors filed 32 Amended Informations
against her, after manifesting to the Sandiganbayan that they would only file one
Amended Information. She also questioned in her opposition to the motion to admit
the 32 Amended Informations, the splitting of the original information.

She even claimed that the Amended Informations filed against her did not charge any
offense punishable under Section 3 (e) of R.A. No. 3019 because the official acts
complained of therein were authorized under E.O. 324 and that the Board of
Commissioners of the Bureau of Investigation adopted the policy of approving
applications for legalization of spouses and unmarried, minor children of "qualified
aliens" even though they had arrived in the Philippines after December 31, 1983. She
concludes that the Sandiganbayan erred in not granting her motion to quash the
Informations.

Issue:
Whether or not there was only one crime that was committed in Santiago’s case wherein
there should only be one Information to be filed against her.

VIVAR, 2
CRIMINAL LAW DIGESTS

Ruling:
Yes, technically, there was only one crime that was committed in petitioner
Santiago's
case, and hence, there should only be one Information to be filed against her.

The 32 Amended Informations charge what is known as delito continuado or


"continued crime" and sometimes referred to as "continuous crime”.

Where only one single criminal act of approving the application for legalization
of 32 aliens was committed on the same period of time, the 32 Informations
should be consolidated into only one. In the case at bench, the original Information
charged petitioner Santiago with performing a single criminal act — that of her
approving the application for legalization of aliens not qualified under the law to enjoy
such privilege.

The original Information also averred that the criminal act committed by petitioner: (i)
was in violation of a law—Executive Order No. 324 dated April 13, 1988, (ii) caused an
undue injury to one offended party, the Government, and (iii) was done on a single day,
i.e., on or about October 17, 1988. The 32 Amended Informations reproduced verbatim
the allegation of the original Information, except that instead of the word “aliens” in the
original information, each Amended Information states the name of the individual whose
stay was legalized.

The 32 Amended Informations aver that the offenses were committed on the same period
of time, i.e., on or about October 17, 1988. The strong probability even exists that the
approval of the application for the legalization of the stay of the 32 aliens was done by
a single stroke of the pen, as when the approval was embodied in the same document.

For delito continuado to exist there should be a plurality of acts performed during a
period of time; unity of penal provision violated; and unity of criminal intent or
purpose, which means that two or more violations of the same penal provisions are
united in one and the same intent or resolution leading to the perpetration of the
same criminal purpose or aim. A delito continuado consists of several crimes but
in reality there is only one crime in the mind of the perpetrator.

The concept of delito continuado, although an outcrop of the Spanish Penal Code, has
been applied to crimes penalized under special laws. Under Article 10 of the Revised
Penal Code, the Code shall be supplementary to special laws, unless the latter provide
the contrary. Hence, legal principles developed from the Penal Code may be applied in
a supplementary capacity to crimes punished under special laws. The question
surrounding the concept of delito continuado is that whether a series of criminal acts
over a period of time creates a single offense or separate offenses.

At the hearing of the motion for a bill of particulars, the public prosecutors manifested
that they would file only one Amended Information embodying the legalization of stay
of the 32 aliens. Hence, in this case, the Office of the Special Prosecutor of the Office
of the Ombudsman is directed to consolidate the 32 Amended Informations
(Criminal Cases Nos. 18371 to 18402) into one information charging only one
offense under the original case number, No. 16698

Fallo:
WHEREFORE, the Resolution dated March 3, 1993 in Criminal Case No. 16698 of the
Sandiganbayan (First Division) is AFFIRMED and its Resolution dated March 11, 1993
in Criminal Case No. 16698 is MODIFIED in the sense that the Office of the Special
Prosecutor of the Office of the Ombudsman is directed to consolidate the 32 Amended
Informations (Criminal Cases Nos. 18371 to 18402) into one information charging
only one offense under the original case number, i.e., No. 16698. The temporary
restraining order issued by this Court on March 25, 1993 is LIFTED insofar as to the
disqualification of Presiding Justice Francis Garchitorena is concerned.

VIVAR, 2
CRIMINAL LAW DIGESTS

Badge <<<

People v. Patriarca, et. al.


G.R. No. 135457, September 29, 2000
BUENA, J:

Doctrine:
Amnesty commonly denotes a general pardon to rebels for their treason or other high
political offenses, or the forgiveness which one sovereign grants to the subjects of
another, who have offended, by some breach, the law of nations. Amnesty looks
backward, and abolishes and puts into oblivion, the offense itself; it so overlooks and
obliterates the offense with which he is charged, that the person released by amnesty
stands before the law precisely as though he had committed no offense.

Case Summary:
Accused-appellant Jose Patriarca, Jr. appealed to the Court when he was convicted
of murder and was sentenced with reclusion perpetua by the RTC Sorsogon in
1990. Accused and his companions in NPA were charged with the murder of Alfredo
Arevalo, as well as two other individuals. He denied the abduction of the victims in
the three criminal cases filed against him but was found guilty by the RTC.

The issue here is whether the trial court erred in finding the accused-appellant
guilty of the crime of murder in pursuance or in furtherance of rebellion.

The Court ruled that lower courts erred in convicting the accused because accused-
appellant applied for amnesty in 1994, and his application was favorably granted by
the National Amnesty Board. As provided by Article 89, par. 3, criminal liability is
totally extinguished by amnesty. Hence, the Court took judicial notice of the grant of
amnesty upon accused-appellant Jose N. Patriarca, and deemed it as binding and
effective, once granted.

Facts:
 This case involved the appeal of the accused-appellant, as regards to the
decision of the Regional Trial Court of Sorsogon, Sorsogon, convicting him of
murder and sentencing him to reclusion perpetua.
 Accused-appellant, Patriarca along with others, were charged with the murder
of Alfredo Arevalo in 1990. He was also charged with murder for the killing of
de Borja and Cadag.
 He pleaded not guilty to the crimes charged, and joint trial of the three cases
was conducted.
 A prosecution witness testified that his house was requested by the Patriarca
and his companions for resting purposes. The witness took cognizance of a
hogtied person, and heard gunshots which lead to the execution of the said
hogtied person.
 Another witness, the mother of the victim, Arevalo, testified that the accused
told her to not let her son join the military. She was later informed that her son
was abducted by the accused and his companions and was notified that her son
was killed by Patriarca.
 During the trial, accused appellant's defense involved denying the abduction of
the victims in the three criminal cases filed against him.
 In 1998, the RTC rendered a decision convicting the accused.
 Hence, this appeal was filed by the accused appellant, contending that the trial
court allegedly committed an error in its verdict of conviction.

Issue:
Whether the trial court erred in finding the accused-appellant guilty of the crime of
murder in pursuance or in furtherance of rebellion.

VIVAR, 2
CRIMINAL LAW DIGESTS

Ruling:
Yes. The accused-appellant applied for amnesty under Proclamation 724 in 1994, and
his application was favorably granted by the National Amnesty Board.

Amnesty commonly denotes a general pardon to rebels for their treason or other high
political offenses, or the forgiveness which one sovereign grants to the subjects of
another, who have offended, by some breach, the law of nations. Amnesty looks
backward, and abolishes and puts into oblivion, the offense itself; it so overlooks and
obliterates the offense with which he is charged, that the person released by
amnesty stands before the law precisely as though he had committed no offense.

In his application for amnesty, the accused-appellant admitted joining the NPA in 1977,
and participated in its several armed activities, including the liquidation of Alfredo
Arevalo. The grant of amnesty to the accused-appellant became final when there has
been no motion for reconsideration filed by any party within 15 days from receipt of
notice.

As provided by Article 89, par. 3, criminal liability is totally extinguished by amnesty.


Hence, the Court took judicial notice of the grant of amnesty upon accused-appellant
Jose N. Patriarca, and deemed it as binding and effective, once granted.

Fallo:
WHEREFORE, IN VIEW OF THE FOREGOING, the decision of the Regional Trial
Court at Sorsogon, Sorsogon, Branch 52 in Griminal Case No. 2773 is REVERSED
and SET ASIDE. Accused-appellant Jose N. Patriarca, Jr. is hereby ACQUITTED of
the crime of murder.

Pursuant to Resolution No. D-99-8683, 11 Criminal Case Nos. 2663 and 2664, which
are both filed in the Regional Trial Court, Branch 53, Sorsogon, Sorsogon, 12 are
ordered DISMISSED. The release of Jose N. Patriarca who is presently detained at the
Provincial Jail of Sorsogon is likewise ORDERED unless he is being detained for some
other legal cause.

The Director of Prisons is ordered to report within ten (10) days his compliance with
this decision.

VIVAR, 2
CRIMINAL LAW DIGESTS

Badge <<<

Batulanon v. People
G.R. No. 139857, September 15, 2006
YNARES-SANTIAGO, J:

Doctrine:
There is no complex crime of estafa through falsification of private document; If the
falsification of a private document is committed as a means to commit estafa, the
proper crime to be charged is falsification; If the estafa can be committed without the
necessity of falsifying a document, the proper crime to be charged is estafa.

Facts:
Herein petitioner assails the decision of the Court of Appeals in affirming with
modification the decisions rendered by the Regional Trial Court for Criminal Case Nos.
3453, 3625, 3626 and 3627, convicting Leonila Batulanon of estafa through
falsification of commercial documents.

Complainant Polomolok Credit Cooperative Incorporated (PCCI) employed Batulanon


as its Cashier/Manager from May 1980 up to December 22, 1982. She was in charge of
receiving deposits from and releasing loans to the member of the cooperative. During
an audit conducted in December 1982, certain irregularities concerning the release of
loans were discovered. Thereafter, four informations for estafa thru falsification of
commercial documents were filed against Batulanon.

That on 4 separate dates, the accused in her capacity as Cashier / Manager falsified
Cash/Check Voucher No. 30-A of PCCI in the name of Erlinda Omadlao by then and
there making an entry therein that the said Erlinda Omadlao, Gonafreda Oracion,
Ferlyn Arroyo, Dennis Batulanon, were granted a loan, in Philippine Currency, and by
signing on the appropriate line thereon the their signatures showing that they
correspondingly received the loan, thus making it appear that the said names were
granted a loan and received the amount when in truth and in fact the said persons were
never granted a loan, never received the same, and never signed the cash/check voucher
issued in their name.

Respondent pleaded not guilt to the charges. The prosecution on the other hand
presented witnesses who testified that accused indeed filed and approved the said
loans and saw her sign the forms making it appear like the personal signatures were
made by the victims.

Issue:
Whether or not respondent is guilty of falsification of private document.

Ruling:
Although the offense charged in the information is estafa through falsification of
commercial document, appellant could be convicted of falsification of private
document under the well-settled rule that it is the allegations in the information that
determines the nature of the offense and not the technical name given in the preamble
of the information.

The elements of falsification of private document under Article 172, paragraph 236 of
the Revised Penal Code are: (1) that the offender committed any of the acts of falsification,
except those in paragraph 7, Article 171; (2) that the falsification was committed in any
private document; and (3) that the falsification caused damage to a third party or at
least the falsification was committed with intent to cause such damage.

In Criminal Case Nos. 3625, 3626, and 3453, Batulanon's act38 of falsification falls
under paragraph 2 of Article 171, i.e., causing it to appear that persons have
participated in any act or proceeding when they did not in fact so participate. This is
because by signing the name of Omadlao, Oracion, and Arroyo in Cash Voucher Nos.
30A, 237A, and 267A, respectively, as payee of the amounts appearing in the

VIVAR, 2
CRIMINAL LAW DIGESTS

corresponding cash vouchers, Batulanon made it appear that they obtained a loan and
received its proceeds when they did not in fact secure said loan nor receive the
amounts reflected in the cash vouchers.

The prosecution established that Batulanon caused the preparation of the Cash
Vouchers in the name of Omadlao and Oracion knowing that they are not PCCI
members and not qualified for a loan from the cooperative. In the case of Arroyo,
Batulanon was aware that while the former is a member, she did not apply for a loan
with the cooperative.

Medallo categorically declared that she saw Batulanon forge the signatures of Oracion
and Arroyo in the vouchers and made it appear that the amounts stated therein were
actually received by these persons. As to the signature of Arroyo, Medallo's credible
testimony and her familiarity with the handwriting of Batulanon proved that it was
indeed the latter who signed the name of Arroyo. Contrary to Batulanon's contention,
the prosecution is not duty-bound to present the persons whose signatures were forged
as Medallo's eyewitness account of the incident was sufficient. Moreover, under Section
22, Rule 132 of the Rules of Court, the handwriting of a person may be proved by any
witness who believes it to be the handwriting of such person because he has seen the
person write, or has seen writing purporting to be his upon which the witness has acted
or been charged, and has thus acquired knowledge of the handwriting of such person.

The Court of Appeals correctly ruled that the subject vouchers are private documents
and not commercial documents because they are not documents used by merchants or
businessmen to promote or facilitate trade or credit transaction1 nor are they defined
and regulated by the Code of Commerce or other commercial law. Rather, they are
private documents, which have been defined as deeds or instruments executed by a
private person without the intervention of a public notary or of other person legally
authorized, by which some disposition or agreement is proved, evidenced or set forth.

As there is no complex crime of estafa through falsification of private document, it is


important to ascertain whether the offender is to be charged with falsification of a private
document or with estafa. If the falsification of a private document is committed as a
means to commit estafa, the proper crime to be charged is falsification. If the estafa can
be committed without the necessity of falsifying a document, the proper crime to be
charged is estafa.

Fallo:
WHEREFORE, the Decision appealed from is AFFIRMED with the following
MODIFICATIONS:

(1) In Criminal Case Nos. 3625, 3626 and 3453, Leonila Batulanon is found GUILTY of
three counts of falsification of private documents and is sentenced to suffer the
penalty of six (6) months of arresto mayor, as minimum, to four (4) years and two (2)
months of prision correccional, as maximum, for each count, and to indemnify
complainant Polomolok Credit Cooperative Incorporated the amount of P11,660.00
with interest at the rate of 6% per annum from November 28, 1994 until finality of this
judgment. The interest rate of 12% per annum shall be imposed from finality of this
judgment until its satisfaction; and
(2) In Criminal Case No. 3627, Leonila Batulanon is found GUILTY of estafa and is
sentenced to suffer the penalty of three (3) months of arresto mayor, as minimum, to
one (1) year and eight (8) months of prision correccional, as maximum. She is likewise
ordered to indemnify Polomolok Credit Cooperative Incorporated the sum of P5,000.00
with interest at the rate of 6% per annum from November 28, 1994 until finality of this
judgment. The interest rate of 12% per annum shall be imposed from finality of this
judgment until its satisfaction.

VIVAR, 2
CRIMINAL LAW DIGESTS

Badge <<<

Fransdilla v. People
G.R. No. 197562, April 20, 2015
BERSAMIN, J:

Doctrine:
The complex crime of robbery in an inhabited house by armed persons and robbery with
violence against or intimidation of persons was committed when the accused, who held
firearms, entered the residential house of the victims and inflicted injury upon the
victims in the process of committing the robbery. Hence, the penalty is that imposed for
the robbery in an inhabited house, the more serious crime. All the accused are liable
because the act of one is the act of all.

Facts:
On February 20, 1991 between 3 o'clock and 4 o'clock in the afternoon, at private
complainants' residence at No. 24, Mabait St., Teachers Village, Quezon City, private
complainant Lalaine Yreverre saw appellant Aurora Engson in front of their gate. Upon
noticing Aurora, Lalaine went to the gate and asked Aurora what is their purpose, as
there were four (4) of them. Aurora then inquired about Cynthia Yreverre, n Lalaine's
sister. The latter replied that Cynthia was in the Japanese Embassy and asked Aurora
if there was any other person whom she wanted to talk to. It was then that Aurora told
Lalaine that she was from the Philippine Overseas Employment Agency (POEA). It was
upon said pretension that Lalaine offered herself to instead talk to her and allowed her
to enter their house. When they were already having a conversation, Aurora asked
Lalaine if she could use the telephone, which the latter acceded to and handed her a
cordless telephone. Lalaine noticed that Aurora seemed to keep on dialing the
telephone and even said that the person she was calling did not know how to use the
telephone. But still, Aurora kept on dialing the telephone.

Thereafter, appellant Aurora asked for a cigarette. After Lalaine gave Aurora the
cigarette, the four (4) other men outside the gate, who were with Aurora, suddenly
came inside the house. The four (4) men stood behind Aurora who was still dialing the
telephone. When Aurora told that she could not contact the person she was calling, she
asked Lalaine if she could use the comfort room, which the latter again permitted.
Aurora stood up, put down the telephone, got her bag and went to the comfort room.
When Aurora came back, she sat down again but in crossed-legs as she said she was
having a menstrual period. Upon saying that, Lalaine's attention was focused on her. At
this juncture, accused Edgardo Cacal poked a gun at Lalaine's neck and announced
that it was a hold-up. While appellant Edgardo Cacal was poking a gun at Lalaine's
neck, accused Danilo Cuanang and the two (2) other men proceeded to the kitchen. In
the kitchen, Danilo and his two (2) other companions herded their maids, private
complainant's niece and cousin inside the bodega.

Accused Cacal who was still poking the gun at Lalaine's neck, thereafter, pulled
Lalaine's hair and dragged her upstairs and brought her inside Cynthia's room. The
gun still being poked at Lalaine, Cacal looked around the room and when he spotted
upon the vault he dropped Lalaine, opened the door and called for his companions to
come along. Accused Cuanang came up and the two (Cacal and Cuanang) carried the
vault and brought it downstairs. But before they went downstairs, they threatened
Lalaine not to follow them and to just stay in the room, but Lalaine opened the door and
followed them.

When Lalaine was halfway downstairs, accused Cacal turned his back and saw her.
Accused Cacal then brought her inside her room. Inside the room, Cacal pushed her
towards her bed and she fell. Cacal told her to just stay, and then he searched the
room. Lalaine managed to stand up but Cacal slapped her. While sitting, accused
Cuanang came and tied her arms at her back. While she was being tied, appellant
Aurora Fransdilla peeped inside the room. It was also at the time that accused Cacal
and Cuanang searched the entire room and took all the jewelries and things they saw.

VIVAR, 2
CRIMINAL LAW DIGESTS

When Cuanang and Cacal left the room, Lalaine followed them. While in the middle
downstairs, she saw Cacal, Cuanang and their two other companions tucking their
guns around their waists. Appellants and their coaccused then left the house on
board two
(2) cars that were waiting for them just outside the house, and one of which, a black
Colt Mirage, was driven by accused Manuel Silao, together with appellant Edgardo
Silao who was seated at the front passenger seat.

At this point, Lalaine shouted for help, thereafter, a relative came by to help and untied
her. Lalaine then called her sister Cynthia and related the incident. Cynthia reported
the incident to the police authorities. Not too long thereafter, the police investigated the
incident.

Issue:
1. WON the correction by the CA of the Indeterminate Sentence necessary to conform
to the letter and spirit of the Indeterminate Sentence Law.
2. WON the CA was correct in classifying the crime committed as the complex crime of
robbery in an inhabited house by armed men under Article 299 of the Revised Penal
Code and robbery with violence against or intimidation of persons under Article 294
of the Revised Penal Code.

Ruling:
1. Yes. The correction by the CA of the Indeterminate Sentence was necessary to
conform to the letter and spirit of the Indeterminate Sentence Law. Section 1 of the
ISL provides that:

… In imposing a prison sentence for an offense punished by the Revised Penal Code,
or its amendments, the court shall sentence the accused to an indeterminate
sentence the maximum term of which shall be that which, in view of the
attending circumstances, could be properly imposed under the rules of the said
Code, and the minimum which shall be within the range of the penalty next
lower to that prescribed by the Code for the offense; and if the offense is punished
by any other law, the court shall sentence the accused to an indeterminate
sentence, the maximum term of which shall not exceed the maximum fixed by said
law and the minimum shall not be less than the minimum term prescribed by the
same. (As amended by Act No. 4225).

In the present case, the trial judge fixed the indeterminate sentence at
"imprisonment of TWELVE (12) YEARS AND ONE (1) DAY to FOURTEEN (14) YEARS
and EIGHT (8) MONTHS of reclusion temporal as minimum to SEVENTEEN (17)
YEARS, FOUR (4) MONTHS and ONE (1) DAY to TWENTY (20) YEARS of reclusion
temporal as maximum" which was a patent elementary error. Such fixing
contravened the letter and spirit of the Indeterminate Sentence Law.

The CA justifiably deemed it necessary to correct the indeterminate sentence. Under


Section 1, supra, the minimum of the indeterminate sentence is a penalty "within
the range of the penalty next lower to that prescribed by the [Revised Penal] Code for
the offense," and the maximum is "that which, in view of the attending circumstances,
could be properly imposed under the rules of the said Code." Considering that the
clear objective of the Indeterminate Sentence Law is to have the convict serve the
minimum penalty before becoming eligible for release on parole pursuant to the
Indeterminate Sentence Law, both the minimum and the maximum penalties must
be definite, not ranging. This objective cannot be achieved otherwise, for determining
when the convict would be eligible for release on parole would be nearly impossible
if the minimum and the maximum were as indefinite as the RTC fixed the
indeterminate sentence. Indeed, that the sentence is an indeterminate one relates
only to the fact that such imposition would leave the period between the minimum
and the maximum penalties indeterminate "in the sense that he may, under the
conditions set out in said Act, be released from serving said period in whole or in
part."

VIVAR, 2
CRIMINAL LAW DIGESTS

2. Yes. The CA was correct in classifying the crime committed as the complex crime of
robbery in an inhabited house by armed men under Article 299 of the Revised Penal
Code and robbery with violence against or intimidation of persons under Article 294
of the Revised Penal Code.

Citing Napolis v. Court of Appeals, the CA ruled that all the accused, including
Fransdilla, were guilty of committing the complex crime of robbery in an inhabited
house under Article 299, Revised Penal Code, and robbery with intimidation or
violence under Article 294, Revised Penal Code. Thus, it held that the penalty for
the complex crime under Article 48 of the Revised Penal Code was that for the
more serious offense, to be imposed in its maximum period. Taking into
consideration that no mitigating or aggravating circumstances were present, it set
the indeterminate sentence of 12 years of prison mayor, as minimum, to 17 years
and four months of reclusion temporal, as maximum.

Paragraph 5, of Article 294, is the relevant provision, under which the penalty is
prision correccional in its maximum period to prison mayor in its medium period.

On the other hand, paragraph (a) 4 (because Fransdilla pretended to be from the
POEA) and paragraph (b) 2 (because the accused brought the vault down from
Cynthia's upstairs bedroom and forced it open outside the place where the robbery
was committed), supra of Article 299 is applicable hence, the penalty for the crime
is reclusion temporal.

Under Article 48 of the Revised Penal Code, the penalty for the complex crime is
that for the more serious felony, which, in this case, was the robbery in an
inhabited house by armed men punishable by reclusion temporal, to be imposed in
the maximum period (i.e., 17 years, four months and one day to 20 years ). Hence,
the maximum of the indeterminate sentence of 12 years of prision mayor, as
minimum, to 17 years and four months of reclusion temporal, must be
corrected to 17 years, four months and one day of reclusion temporal.

Fallo:
WHEREFORE, the Court DENIES the petition for review on certiorari and AFFIRMS in
all respects the conviction of accused AURORA ENGSON FRANSDILLA for the complex
crime of robbery in an inhabited house by armed men under Article 299 of the Revised
Penal Code and robbery with violence against and intimidation of persons under
Article 294 of the Revised Penal Code, subject to the following MODIFICATIONS,
namely: (1) she shall suffer the indeterminate sentence of 12 years of prision mayor,
as minimum, to 17 years, four months and one day of reclusion temporal, as
maximum; (2) the award of P200,000.00 as exemplary damages is deleted for lack of
legal basis; and (3) and the actual damages of P2,250,000.00 shall earn interest of 6%
per annum reckoned from the filing of the information until full payment.

The petitioner shall pay the costs of suit.

VIVAR, 2
CRIMINAL LAW DIGESTS

Badge <<<

Zafra v. People
G.R. No. 176317, July 23, 2014
BERSAMIN, J:

Doctrine:
To determine the maximum periods of the penalties to be imposed on the petitioner,
therefore, we must be guided by the following rules, namely: (1) the penalties provided
under Article 217 of the Revised Penal Code (RPC) constitute degrees; and (2)
considering that the penalties provided under Article 217 of the RPC are not composed
of three periods, the time included in the penalty prescribed should be divided into three
equal portions, which each portion forming one period, pursuant to Article 65 of the
RPC.

Facts:
Zafra was the only Revenue Collection Agent of BIR RD 3 in San Fernando, La Union.
He received tax payments were revenue official receipt (ROR) was issued. Original
copy of the receipt was given to the taxpayer while a copy was retained by the revenue
officer. Every month, appellant submitted BIR Form 12.31 of the Monthly Report of
Collections (MRC) indicating the numbers of the issued RORs, date of collection, name
of taxpayer, the amount collected and the kind of tax paid. An audit team from Head
Office was tasked to audit the cash and non-cash accountabilities of Zafra. Upon
reviewing of Certificate Authorizing Registration (CAR) with PNB’s ROR and MRC with
Zafra’s ROR, the MRC has lesser amount of paid taxes than the CAR. Some RORs in CAR
were declared to be paid but when checked to MCR, it was cancelled. The audit team
found 18 RORs as accountability of Zafra, in the amount of Php614,151.93. Thru a
demand letter, Zafra was required to pay the said amount. But the latter ignored the
letter.

BIR then filed 18 cases for malversation of public funds through falsification of public
document against Zafra. Zafra denied the allegations, claiming that he never received
tax payments; and that his subordinates received the payments and issued RORs. Zafra
was convicted by RTC of the charges.

On appeal, CA affirmed the RTC’s decision. CA held that the audit team’s demand letter
to Zafra, which he failed to rebut, raised a prima facie presumption that he put to his
personal use the missing funds. Moreover, even if it were to subscribe to the
petitioner’s insistence that it had been his assistants, not him, who had collected the
taxes and issued the RORs, he was nonetheless liable, because his duty as an
accountable officer had been to strictly supervise his assistants; and that by failing to
strictly supervise them he was responsible for the shortage resulting from the non-
remittance of the actual amounts collected.

Issue:
Whether or not the imposed penalties of RTC is correct.

Ruling:
No. The Court held that the imposed penalties must be corrected.

To determine the maximum periods of the penalties to be imposed on the petitioner, the
rules are: (1) the penalties provided under Article 217 of the Revised Penal Code
constitute degrees; and (2) considering that the penalties provided under Article 217 of
the Revised Penal Code are not composed of three periods, the time included in the
penalty prescribed should be divided into three equal portions, which each portion
forming one period, pursuant to Article 65 of the Revised Penal Code.

RTC erred in pegging the maximum terms within the minimum periods of the penalties
prescribed under Article 217 of the RPC and by fixing indeterminate sentences on some
counts despite the maximum of the imposable penalties being reclusion perpetua.

VIVAR, 2
CRIMINAL LAW DIGESTS

The Court held that Indeterminate Sentence Law was applicable, except where the
imposable penalty was reclusion perpetua. Considering that each count was a complex
crime without any modifying circumstances, the maximum term of the penalty for each
count is the maximum period, except for the count dealt with in Criminal Case No. 4635
involving the misappropriated amount of ₱4,869.00, for which the corresponding
penalty for malversation was prision mayor in its minimum and medium periods.
However, because such penalty for malversation was lower than the penalty of prision
mayor imposable on falsification of a public document under Article 171 of the Revised
Penal Code, it is the penalty of prision mayor in its maximum period that was applicable.

Hence, the Court gave the proper penalty to be imposed, as follows:

Fallo:
WHEREFORE, the Court AFFIRMS the decision promulgated on August 16, 2006 by
the Court of Appeals subject to the modification of the penalties imposed as stated in
this decision.

ACCORDINGLY, the dispositive portion of the consolidated decision rendered on


February 17, 2004 by the Regional Trial Court is hereby AMENDED to read as follows:

VIVAR, 2
CRIMINAL LAW DIGESTS

Badge <<<

People v. Lopez
G.R. No. 181441, November 14, 2008
CARPIO, J:

Doctrine:
Finding of the lower courts that the prosecution sufficiently established appellant’s
guilt beyond reasonable doubt for violation of Sections 5 and 11, Article II of RA 9165
sustained.

Inconsistencies on the existence of a pre-arranged signal and the markings on the buy-
bust money pertain to peripheral matters and do not refer to the actual buy-bust
operation itself—that crucial moment when the appellant was caught selling shabu—
which might warrant a reversal of appellant’s conviction.

Facts:

VIVAR, 2
CRIMINAL LAW DIGESTS

Issue:
Whether the appellant is guilty beyond reasonable doubt of violation of (1) Section 5 and
11 of RA 9165.

Ruling:

Fallo:
WHEREFORE, the Court AFFIRMS the 25 September 2007 Decision of the Court of
Appeals in CA-G.R. CR-HC No. 02031 with the MODIFICATION that the penalty in
Criminal Case No. 3189 shall be imprisonment for twelve (12) years and one (1) day,
as minimum, to twenty (20) years, as maximum, and a fine of P300,000.

VIVAR, 2
CRIMINAL LAW DIGESTS

Badge <<<

People v. Leonardo
G.R. No. 181036, July 6, 2010
PEREZ, J:

Doctrine:
Applying the Indeterminate Sentence Law, the appellant shall be entitled to a minimum
term to be taken within the range of the penalty next lower to that prescribed by
Republic Act No. 7610. The penalty next lower in degree is prision mayor medium to
reclusion temporal minimum, the range of which is from 8 years and 1 day to 14 years
and 8 months.

Facts:
 The case involved the conviction of Leonardo, who was found guilty of the crime
of rape and was sentenced to suffer the penalty of reclusion perpetua or
imprisonment for 20 to 40 years.
 The case arose from a complaint filed by the victim, who alleged that Leonardo,
her stepfather, had sexually abused her when she was still a minor. The victim
testified that the abuse had occurred on several occasions, and that she had
initially been afraid to report it to anyone. However, she eventually disclosed
the abuse to her mother, who then reported it to the authorities.
 During the trial, Leonardo denied the allegations and argued that the victim had
fabricated the story. However, the trial court found the victim's testimony to be
credible and convincing, and concluded that the prosecution had proven the
elements of the crime of rape beyond reasonable doubt.
 On appeal, the Supreme Court affirmed the trial court's decision, stating that it
had no reason to doubt the victim's testimony. The Court noted that rape cases
often lack corroborating evidence, and that the victim's testimony, if credible, can
be sufficient to sustain a conviction.
 The Court also emphasized the importance of protecting children from sexual
abuse, and noted that the crime of rape is a heinous offense that deserves
severe punishment. In affirming Leonardo's conviction, the Court sent a strong
message that sexual abuse will not be tolerated, and that perpetrators will be
held accountable for their actions.

Issue:
Whether the prosecution was able to prove the elements of the crime of rape beyond
reasonable doubt.

Ruling:
The trial court and the Supreme Court addressed these issues and concluded that the
prosecution had indeed proven the elements of rape beyond reasonable doubt, and
that the victim's testimony was credible and convincing. The defense of denial and
alibi raised by the accused was not sufficient to overcome the prosecution's evidence,
and the penalty of reclusion perpetua was deemed appropriate under the
circumstances of the case.

Fallo:
WHEREFORE, premises considered, the Decision of the Court of Appeals in CA-G.R. CR-H.C. No.
01092 dated 28 May 2007 finding herein appellant guilty beyond reasonable doubt in Criminal
Case Nos. 545-V-02, 549-V-02, 552-V-02, 553-V-02, 544-V-02 and 348-V-02 of six counts of
rape and in Criminal Case Nos. 546-V-02, 547-V-02, 548-V-02, 554-V-02 and 555-V-02 of five
counts of sexual abuse under Section 5(b), Article III of Republic Act No. 7610 is hereby
AFFIRMED with the following MODIFICATIONS: (1) the maximum term of the indeterminate
sentence to be imposed upon the appellant for each count of sexual abuse under Section 5(b),
Article III of Republic Act No. 7610 should be 17 years, 4 months and 1 day of reclusion
temporal;
(2) the awards of civil indemnity and moral damages for each count of sexual abuse under Section
5(b), Article III of Republic Act No. 7610 are reduced from P25,000.00 to P20,000.00 and from
P25,000.00 to P15,000.00, respectively; and (3) a fine of P15,000.00 is imposed on the appellant
also for each count of sexual abuse under Section 5(b), Article III of Republic Act No. 7610. Costs
against appellant.

VIVAR, 2
CRIMINAL LAW DIGESTS

Badge <<<

People v. Matias
G.R. No. 186469, June 13, 2012
PERLAS-BERNABE, J:

Doctrine:
he penalty for sexual abuse under Sec. 5 (b), Article III of Republic Act (R.A.) 7610 is
reclusion temporal medium to reclusion perpetua, while rape under Article 266-A of the
Revised Penal Code (RPC) is penalized with reclusion perpetua.

Facts:
In the evening of 6 June 2004, AAA, a minor, having been born on 23 April 1991, was
on her way to the vegetable stall of a certain “Manuela.” Suddenly, accused pulled her
towards a house that was under construction. There, he forced her to lie on a bamboo
bed, removed her shorts and underwear, and inserted his finger and then his penis
into her vagina. Accused threatened to kill her if she reported the incident to anyone.

When AAA arrived home, she narrated to her mother and aunt what happened.
Thereafter, they reported the incident to the barangay and then the Baler District
Police Station. After trial, the RTC, convicted accused of “rape” under Article III, Sec.
5(b) of
R.A. No. 7610. The CA affirmed the RTC Decision in toto.

Issue:
Whether or not accused was correctly convicted of “rape” under Article III, Sec. 5(b)
of R.A. No. 7610

Ruling:
Yes. Under Article III, Sec. 5(b) of R.A. No. 7610 in relation to R.A. No. 8353, if the
victim of sexual abuse is below 12 years of age, the offender should not be prosecuted
for sexual abuse, but for statutory rape under Article 266-A(1)(d) of the Revised Penal
Code (RPC) and penalized with reclusion perpetua. On the other hand, if the victim is
12 years or older, the offender should be charged with either sexual abuse under
Article III, Sec. 5(b) of R.A. No. 7610 or rape under Article 266-A (except paragraph
1[d]) of the RPC. However, the offender cannot be accused of both crimes for the same
act because his right against double jeopardy will be prejudiced. A person cannot be
subjected twice to criminal liability for a single criminal act. Likewise, rape cannot be
complexed with a violation of Article III, Sec. 5(b) of R.A. No. 7610. Under Section 48 of
the RPC, a felony under the Revised Penal Code (such as rape) cannot be complexed
with an offense penalized by a special law.

In this case, the lower courts erred in convicting accused of “rape” under Article III,
Sec. 5(b) of R.A. No. 7610 and sentencing him to reclusion perpetua based on their
finding that AAA was a minor below 12 years old at the time of the commission of the
offense on 6 June 2004. However, a scrutiny of the records shows that AAA was born
on 23 April 1991, which would make her 13 years old at the time of the commission of
the offense on 3 June 2004. Thus, accused can be prosecuted and convicted either
under Sec. 5 (b), Article III of R.A. No. 7610 for sexual abuse, or under Article 266-A of
the RPC, except for rape under paragraph 1(d). It bears pointing out that the penalties
under these two laws differ: the penalty for sexual abuse under Sec. 5(b), Article III of
R.A. No. 7610 is reclusion temporal medium to reclusion perpetua, while rape under
Article 266- A of the RPC is penalized with reclusion perpetua.

Fallo:
WHEREFORE, the appeal is DISMISSED. The August 19, 2008 Decision of the Court of Appeals
in CA-G.R. CR-H.C. No. 02781 finding appellant Jover Matias y Dela Fuente guilty beyond
reasonable doubt of sexual abuse under Section 5 (b), Article III of Republic Act No. 7610 is
AFFIRMED with MODIFICATIONS as to penalty and the amount of damages awarded. Appellant
is sentenced to suffer the penalty of 12 years of prision mayor as minimum to 17 years, 4
months and 1 day of reclusion temporal as maximum, and ordered to pay the private
complainant the amount of P50,000.00 as moral damages. The rest of the assailed Decision

VIVAR, 2
CRIMINAL LAW DIGESTS

stands.

VIVAR, 2
CRIMINAL LAW DIGESTS

Badge <<<

Legrama v. Sandiganbayan
G.R. No. 178626, June 13, 2012
PERALTA, J:

Doctrine:
Although restitution is akin to voluntary surrender, as provided for in paragraph 7 of
Article 13, in relation to paragraph 10 of the same Article of the Revised Penal Code,
restitution should be treated as a separate mitigating circumstance in favor of the
accused when the two circumstances are present in a case, which is similar to
instances where voluntary surrender and plea of guilty are both present even
though the two mitigating circumstances are treated in the same paragraph 7, Article
13 of the Revised Penal Code.

Facts:
Office of the Provincial Auditor of the Commission on Audit (COA), Zambales directed
an Audit Team to conduct an examination of the cash and account of petitioner Cecilia
Legrama, the Municipal Treasurer of the Municipality of San Antonio, Zambales.

The COA prepared a Special Cash Examination Report on the Cash and Accounts of
petitioner which contained the findings that petitioners cash accountability has a
shortage in the amount of P1,152,900.75. From the total amount of the shortage,
petitioner was able to restitute the initial amount of P60,000.00.

Consequently, petitioner and Romeo D. Lonzanida (Lonzanida), the Municipal Mayor


were charged in an Informationwith the crime of Malversation of Public Funds. Both
petitioner and Lonzanida voluntarily surrendered and posted their respective cash
bonds.

Sandiganbayan rendered a Decision acquitting Lonzanida (lack of proof that is


conspired with petitioner). However, the tribunal concluded that petitioner malversed
the total amount of P1,131,595.05 and found her guilty of the crime of Malversation of
Public Funds.

The amount involved in the instant case is more than Php22,000.00. Hence,
pursuant to the provisions of Article 217 of the Revised Penal Code, the penalty
to be imposed is reclusion temporal in its maximum period to reclusion
perpetua.

Considering the absence of any aggravating circumstance and the presence of


two mitigating circumstances, viz., accused Legramas voluntary surrender and
partial restitution of the amount involved in the instant case, and being entitled
to the provisions of the Indeterminate Sentence Law, she is hereby sentenced to
suffer an indeterminate penalty of 4 years, 2 months and 1 day of prision
correccional, as minimum, to 10 years and 1 day of prision mayor, as maximum.

Further, she is ordered to pay the amount of Php299,204.65, representing the


balance of her incurred shortage after deducting therein the restituted
amountof Php832,390.40 and the Php200.00 covered by an Official Receipt
dated August 18, 1996 issued in the name of the Municipality of San Antonio
(Exhibit 22). She is also ordered to pay a fine equal to the amount malversed
which is Php1,131,595.05 and likewise suffer the penalty of perpetual special
disqualification and to pay costs.
Issue:
1. Whether Petitioner is guilty of Malversation of Public funds?
2. Whether Petitioner is entitled of 2 mitigating circumstances? YES (voluntary
surrender and restitution)

VIVAR, 2
CRIMINAL LAW DIGESTS

Ruling:

Petitioner is guilty of Malversation


Undoubtedly, all the elements of the crime are present in the case at bar. First, it is
undisputed that petitioner was the municipal treasurer at the time material to this
case. Second, it is the inherent function of petitioner, being the municipal treasurer, to
take custody of and exercise proper management of the local governments funds.
Third, the parties have stipulated during the pre-trial of the case that petitioner
received the subject amount as public funds and that petitioner is accountable for the
same. Fourth, petitioner failed to rebut the prima facie presumption that she has put
such missing funds to her personal use.

Verily, in the crime of malversation of public funds, all that is necessary for conviction
is proof that the accountable officer had received the public funds and that he failed to
account for the said funds upon demand without offering sufficient explanation why
there was a shortage. In fine, petitioners failure to present competent and credible
evidence that would exculpate her and rebut the prima facie presumption of
malversation clearly warranted a verdict of conviction.

Petitioner is entitled to 2 mitigating circumstances: Voluntary surrender and Restitution

As for the appropriate penalty, since the amount involved is more than P22,000.00,
pursuant to the provisions of Article 217 of the Revised Penal Code, the penalty to be
imposed is reclusion temporal in its maximum period to reclusion perpetua.

However, as aptly concluded by the Sandiganbayan, petitioner enjoys the mitigating


circumstances of voluntary surrender and restitution. Although restitution is akin to
voluntary surrender, as provided for in paragraph 7 of Article 13, in relation to
paragraph 10 of the same Article of the Revised Penal Code, restitution should be treated
as a separate mitigating circumstance in favor of the accused when the two
circumstances are present in a case, which is similar to instances where voluntary
surrender and plea of guilty are both present even though the two mitigating
circumstances are treated in the same paragraph 7, Article 13 of the Revised Penal
Code. Considering that restitution is also tantamount to an admission of guilt on the
part of the accused, it was proper for the Sandiganbayan to have considered it as a
separate mitigating circumstance in favor of petitioner.

Taking into consideration the absence of any aggravating circumstance and the
presence of two mitigating circumstance, i.e., petitioners voluntary surrender and
partial restitution of the amount malversed, the prescribed penalty is reduced to prision
mayor in its maximum period to reclusion temporal in its medium period, which has a
range of ten (10) years and one (1) day to seventeen (17) years and four (4) months. In
accordance with paragraph 1, Article 64 of the Revised Penal Code.] and considering
that there are no other mitigating circumstance present, the maximum term should now
be the medium period of prision mayor maximum to reclusion temporal medium, which
is reclusion temporal minimum and applying the Indeterminate Sentence Law, the
minimum term should be anywhere within the period of prision correccional maximum
to prision mayor medium. Hence, the penalty imposed needs modification. Accordingly,
petitioner is sentenced to suffer the indeterminate penalty of four (4) years, two (2)
months and one (1) day of prision correccional, as minimum, to twelve (12) years, five
(5) months and eleven (11) days of reclusion temporal, as maximum.

Fallo:
WHEREFORE, premises considered, the petition is DENIED. The Decision dated
January 30, 2007 and the Resolution dated May 30, 2007 of the Sandiganbayan are
AFFIRMED with MODIFICATION. Petitioner is hereby sentenced to suffer the
indeterminate penalty of four (4) years, two (2) months and one (1) day of prision
correccional, as minimum term, to twelve (12) years, five (5) months and eleven (11)
days of reclusion temporal, as maximum term.

VIVAR, 2
CRIMINAL LAW DIGESTS

Badge <<<

Flordeliz v. People
G.R. No. 186441, March 3,
2010 NACHURA, J:

Doctrine:
Rape through Sexual Assault; The insertion of petitioner’s fingers into the victim’s
vagina constituted the crime of rape through sexual assault under Republic Act (R.A.)
No. 8353 or “The Anti-Rape Law of 1997.”

Acts of Lasciviousness; Sexual intercourse or lascivious conduct covers not only a


situation where a child is abused for profit but also where one—through coercion,
intimidation or influence—engages in sexual intercourse or lascivious conduct with a
child.

Facts:
Sometime in March 1995, ABC, the wife of petitioner and the mother of private
complainants AAA and BBB, left for Malaysia as an overseas worker. AAA and BBB
were left under the care and custody of petitioner. They resided in a small house in
Quezon Hill, Baguio City. 5

In April 1995, while sleeping with BBB and AAA, who was then eleven (11) years old,
petitioner woke up AAA, touched her vagina, then played with it. AAA cried and told
petitioner that it was painful. The latter stopped, but warned AAA not to tell anyone
about it; otherwise, she would be harmed. Petitioner allegedly committed the same
acts against AAA repeatedly.

Petitioner and his daughters later transferred residence and lived with the former's
siblings. Not long after, petitioner was convicted of homicide and imprisoned in
Muntinlupa City. Consequently, AAA and BBB lived with their grandparents in La
Trinidad, Benguet. 7 While petitioner was incarcerated, AAA and BBB visited him and
sent him two greeting cards containing the following texts, among others: "happy
valentine"; "ur the best dad in the world"; "I love you papa, love BBB, Love BJ"; "till we
meet again"; portrait of Jesus Christ with a heart, "this is for you dad"; "flordeliz, AAA
P., love AAA and Iyos." 8

In 2001, petitioner was released on parole. He would frequently fetch AAA and BBB from
their grandparents' house during weekends and holidays and they would stay with
him in Gabriela Silang, Baguio City. 9 ISHaTA

Unsatisfied with the abuses committed against AAA, petitioner allegedly started
molesting BBB in May 2002. 10 In 2003, BBB spent New Year's Day with her father. On
January 3, 2003, while they were sleeping, petitioner inserted his two (2) fingers into
BBB's vagina. 11 BBB did not attempt to stop petitioner because of fear. Thereafter,
they slept beside each other. 12 BBB suffered the same ordeal the following night. 13

On February 8, 2003, BBB visited petitioner. Again, petitioner held her vagina, played
with it and inserted his fingers, which caused her pain. 14

The same incident allegedly took place on August 3, 2003. 15 On October 26, 2003, a
day before AAA's birthday, while BBB was with petitioner, the latter committed the same
dastardly act. This time, it was for a longer period. 16

During All Saints' Day of 2003, BBB spent two nights with her father and, during those
nights (November 1 and 2), she experienced the same sexual abuse. 17 The same thing
happened on December 28, 2003. 18

Notwithstanding the repeated incidents of sexual abuse committed against her, BBB
did not reveal her ordeal to anybody because of fear for her life and that of her
mother.

VIVAR, 2
CRIMINAL LAW DIGESTS

AAA and BBB had the chance to reveal their horrifying experiences when their mother
ABC arrived for a vacation. AAA immediately told ABC what petitioner did to her.
When confronted by ABC, BBB likewise admitted the repeated abuses committed by
petitioner. ABC forthwith reported the incidents to the National Bureau of
Investigation.

After conducting medical examinations on AAA and BBB, the attending physician
remarked that there was a "disclosure of sexual abuse and she noted the presence of
hymenal notch in posterior portion of hymenal rim that may be due to previous blunt
force or penetrating trauma suggestive of abuse.

RTC: charged with the crimes of Acts of Lasciviousness, 22 committed against AAA,
and nine (9) counts of Qualified Rape through Sexual Assault, 23 committed against
BBB.

CA: affirmed petitioner's conviction with a modification of the amount of his civil
liabilities

Issue:
Whether or not the court erred in in affirming the judgment of conviction.

Ruling:
NO.

Fallo:
WHEREFORE, premises considered, the Court of Appeals’ July 29, 2008 Decision and
February 16, 2009 Resolution in CA-G.R. CR No. 30949 are AFFIRMED with
MODIFICATIONS. The Court finds petitioner Salvador Flordeliz y Abenojar:
1. GUILTY of seven (7) counts of RAPE Through Sexual Assault in Criminal Case Nos.
23072-R, 23073-R, 23074-R, 23076-R, 23077-R, 23079-R, and 23080-R. He is
sentenced to suffer the indeterminate penalty of ten (10) years of prision mayor, as
minimum, to seventeen (17) years and four (4) months of reclusion temporal, as
maximum, for each count. Petitioner is ordered to indemnify BBB P30,000.00 as civil
indemnity; P30,000.00 as moral damages; and P30,000.00 as exemplary damages, for
each count;

2. GUILTY of ACTS OF LASCIVIOUSNESS in Criminal Case No. 23145-R. He is


sentenced to suffer the indeterminate penalty of thirteen (13) years, nine (9) months and
eleven (11) days of reclusion temporal, as minimum, to sixteen (16) years, five (5) months
and ten (10) days of reclusion temporal, as maximum. He is likewise ordered to pay a
fine of P15,000.00 and to indemnify AAA P20,000.00 as civil indemnity, P15,000.00 as
moral damages, and P15,000.00 as exemplary damages;

3. NOT GUILTY in Criminal Case Nos. 23075-R and 23078-R.

VIVAR, 2
CRIMINAL LAW DIGESTS

Badge <<<

Imbo v. People
G.R. No. 197712, April 20, 2015
PEREZ, J:

Doctrine:
With the aggravating circumstance of relationship and applying the Indeterminate
Sentence Law, the penalty imposed by the lower courts of seventeen (17) years, four (4)
months of reclusion temporal correctly does not exceed the maximum of the penalty
range of reclusion temporal in its medium period (14 years, 4 months and 1 day to 17
years and 4 months).

Facts:
Imbo was charged with crime of acts of lasciviousness upon his own daughter, 11
years old, a minor, by then and there forcing her to remove her shorts, mashing her
breasts and private parts and kissing her, thereby subjecting him to sexual abuse, with
lewd design and against her will, which act debases, degrades or demeans the intrinsic
worth of dignity of his daughter as a human being, to the damage and prejudice of the
said offended party.

Imbo claimed that his wife CCC, AAA's mother merely fabricated such a story that he
had raped his daughter. Ultimately, Imbo claimed that on the night in question, within
the period from14 October 2003 to 25 January 2004, no crime occurred, his days
ending as did his workday which were from 8:30 a.m. to 5:00 p.m.

The RTC convicted Imbo of the crime of Acts of Lasciviousness and was sentenced to
suffer an indeterminate sentence of FOURTEEN (14) YEARS, EIGHT (8) MONTHS OF
RECLUSION TEMPORAL ASMINIMUM TO SEVENTEEN (17) YEARS, FOUR (4) MONTHS
OF RECLUSION TEMPORAL AS MAXIMUM in accordance with Section 5 of Republic
Act No. 7610, otherwise known as the Special Protection of Children Against Child
Abuse, Exploitation and Discrimination.

Issue:
Whether Imbo is liable for the Crime of Acts of Lasciviousness and is subject to the
Penalty Imposed under Section 5 of RA 7610 despite the fact that the information
failed to indicate its applicability.

Ruling:
Yes. Imbo has undoubtedly committed the Acts of Lasciviousness as all the elements of
the crime was sufficiently proven through the lone testimony of AAA which the court
has held in more than one occasion as more than sufficient to establish the guilt of the
accused. He is liable since under Sec 5 of RA 7610 such as in this When AAA a child
was coerced by her Father to indulge in acts of lasciviousness as sufficiently proven in
the information.

With regard to the Penalty, the court found that the RTC's decision should be modified
by applying the Indeterminate Sentence Law. According to the Courts The Indeterminate
Sentence Law is applicable to prison sentence both for an offense punished by the RPC
and an offense punished "by any other law." The correct application of the Indeterminate
Sentence Law has long been clarified in People v. Simon which ruled that the
underscored portion of Section 1of the Indeterminate Sentence Law, i.e. the "offense is
punished by any other law," indubitably refers to an offense under a special law where
the penalty imposed was not taken from and is without reference to the RPC.

The minimum term should be within the range of the penalty next lower to that
prescribed by the RPC, i.e. reclusion temporal in its minimum period of twelve (12) years
and one (1) dayto fourteen (14) years and eight (8) months.

As for the maximum term of the imposable penalty on petitioner, the lower courts
while correct, should have mentioned Section 31(c), Article XIl of R.A. No. 7610. The
provision

VIVAR, 2
CRIMINAL LAW DIGESTS

takes into consideration the relationship between the parties, petitioner being AAA's
father. With the aggravating circumstance of relationship and applying the
Indeterminate Sentence Law, the penalty imposed by the lower courts of seventeen (17)
years, four (4) months of reclusion temporal correctly does not exceed the maximum of
the penalty range of reclusion temporal in its medium period (14 years, 4 months and
1 day to 17 years and 4 months).

Fallo:
WHEREFORE, the appeal is DENIED. The Decision dated February 17, 2011 of the
Court of Appeals in CA-G.R. CR No. 32804 and the Judgment of the Regional Trial
Court, Branch 94, Quezon City in Criminal Case No. 04-124565 are AFFIRMED WITH
MODIFICATION. Petitioner Nonito Imbo Y Gamores is sentenced to an indeterminate
penalty of twelve (12) years and one (1) day of reclusion temporal as minimum and
seventeen (17) years and four (4) months of reclusion temporal as maximum.
Petitioner Nonito Imbo Y Gamores is ordered to pay:

(1) Fine in the amount of P15,000.00;

(2) Civil Indemnity in the amount of P20,000.00;

(3) Moral damages in the amount of P15,000.00;

(4) Exemplary damages in the amount of P15,000.00; and

(5) Interest on all monetary awards for damages at the rate of six percent (6%)
per annum from the date of finality of this Decision until full satisfaction thereof.

VIVAR, 2
CRIMINAL LAW DIGESTS

Badge <<<

Villareal v. People
G.R. No. 151258, February 1, 2012
SERENO, J:

Doctrine:
Reckless imprudence consists in voluntary, but without malice, doing or falling to do
an act from which material damage results by reason of inexcusable lack of precaution
on the part of the person performing or failing to perform such act, taking into
consideration his employment or occupation, degree of intelligence, physical condition
and other circumstances regarding persons, time and place.

If death resulted from an act executed without malice or criminal intent — but with
lack of foresight, carelessness, or negligence — the act must be qualified as reckless or
simple negligence or imprudence resulting in homicide.

Facts:

In February 1991, seven freshmen law students of the Ateneo de Manila University
School of Law were subjected to traditional forms of Aquilan “initiation rites” which
lasted for three days. They were tormented physically and psychologically.

Accused non-resident or alumni fraternity members Fidelito Dizon and Artemio Villareal
demanded that the rites be reopened. The head of initiation rites, Nelson Victorino,
initially refused but he reopened the initiation rites. The fraternity members, including
Dizon and Villareal, then subjected the neophytes to “paddling” and to additional
rounds of physical pain. Lenny received several paddle blows, one of which was so
strong it sent him sprawling to the ground. The neophytes heard him complaining of
intense pain and difficulty in breathing.

After their last session of physical beatings, Lenny could no longer walk. After an hour
of sleep, the neophytes were suddenly roused by Lenny’s shivering and incoherent
mumblings. When his condition worsened, the Aquilans rushed him to the hospital.
Lenny was pronounced dead on arrival.

The trial court rendered judgment holding the 26 accused guilty beyond reasonable
doubt of the crime of homicide, penalized with reclusion temporal under Article 249 of
the Revised Penal Code.

The CA set aside the finding of conspiracy by the trial court and modified the criminal
liability of each of the accused according to individual participation. Nineteen of the
accused-appellants were acquitted, as their individual guilt was not established by
proof beyond reasonable doubt. Four of the accused-appellants– Vincent Tecson,
Junel AnthonyAma, Antonio Mariano Almeda, and Renato Bantug, Jr. (Tecson et al.)–
were found guilty of the crime of slight physical injuries and sentenced to 20 days of
arresto menor. Two of the accused-appellants– Fidelito Dizon and Artemio Villareal
were found guilty beyond reasonable doubt of the crime of homicide under Article 249 of
the Revised Penal Code.

Issue:

1) Whether or not the CA committed grave abuse of discretion, amounting to lack or


excess of jurisdiction, when it set aside the finding of conspiracy by the trial court and
adjudicated the liability of each accused according to individual participation (NO)

2) Whether or not the CA committed grave abuse of discretion when it pronounced


Tecson, Ama, Almeda, and Bantug guilty only of slight physical injuries (YES)

3) Whether or not accused Dizon is guilty of homicide (NO)

VIVAR, 2
CRIMINAL LAW DIGESTS

Ruling:

1) NO. Grave abuse of discretion cannot be attributed to a court simply because it


allegedly misappreciated the facts and the evidence. Mere errors of judgment are
correctible by an appeal or a petition for review under Rule 45 of the Rules of Court,
and not by an application for a writ of certiorari. Pursuant to the rule on double
jeopardy, the Court is constrained to deny the Petition contra Victorino et al. – the 19
acquitted fraternity members.

A verdict of acquittal is immediately final and a re-examination of the merits of such


acquittal, even in the appellate courts, will put the accused in jeopardy for the same
offense.

2) YES. The CA committed grave abuse of discretion amounting to lack or excess of


jurisdiction in finding Tecson, Ama, Almeda, and Bantug criminally liable for slight
physical injuries.

Article 4(1) of the Revised Penal Code dictates that the perpetrator shall be liable for
the consequences of an act, even if its result is different from that intended. Thus, once
a person is found to have committed an initial felonious act, such as the unlawful
infliction of physical injuries that results in the death of the victim, courts are required
to automatically apply the legal framework governing the destruction of life. This rule
is mandatory, and not subject to discretion. The accused cannot be held criminally
liable for physical injuries when actual death occurs.

Attributing criminal liability solely to Villareal and Dizon – as if only their acts, in and
of themselves, caused the death of Lenny Villa – is contrary to the CA’s own findings.
From proof that the death of the victim was the cumulative effect of the multiple injuries
he suffered, the only logical conclusion is that criminal responsibility should redound
to all those who have been proven to have directly participated in the infliction of
physical injuries on Lenny.

3) NO. The Court cannot sustain the CA in finding the accused Dizon guilty of homicide
under Article 249 of the Revised Penal Code on the basis of the existence of intent to
kill. Animus interficendi cannot and should not be inferred unless there is proof beyond
reasonable doubt of such intent. Instead, the Court adopts and reinstates the finding of
the trial court in part, insofar as it ruled that none of the fraternity members had the
specific intent to kill Lenny Villa.

Fallo:
WHEREFORE, premises considered, the Motion for Partial Reconsideration of petitioner
Gerarda H. Villa in connection with G.R. Nos. 178057 & 178080 is hereby DENIED. The
Motion for Reconsideration filed by the Office of the Solicitor General concerning G.R.
Nos. 155101 and 154954 is also DENIED.
The respective Motions for Clarification or Reconsideration of Antonio Mariano
Almeda, Junel Anthony D. Ama, Renato Bantug, Jr., and Vincent Tecson are likewise
DENIED. In light of the finding that Caloocan City Regional Trial Court Branch 130
acted without or in excess of its jurisdiction in taking cognizance of the
aforementioned Applications for Probation, we hereby ANNUL the entire probation
proceedings and SET ASIDE all orders, resolutions, or judgments issued in connection
thereto. We, however, CLARIFY that Antonio Mariano Almeda, Junel Anthony D. Ama,
Renato Bantug, Jr., Vincent Tecson, and Fidelito Dizon are eligible to apply or reapply
for probation in view of our recent ruling in Colinares v. People of the Philippines, 88
without prejudice to their remaining civil liability, if any.
Furthermore, we issue a CORRECTION of the dispositive portion of our Decision dated
1 February 2012 and hereby delete the phrase "and one (1) day" located in the fourth
sentence of the first paragraph thereof. The sentence shall now read as follows: "They
are hereby sentenced to suffer an indeterminate prison term of four (4) months of arresto
mayor, as minimum, to four (4) years and two (2) months of prisió n correccional, as
maximum."

VIVAR, 2
CRIMINAL LAW DIGESTS

Badge <<<

Colinares v. People
G.R. No. 182748, December 13, 2011
ABAD, J:

Doctrine:
Probation is a disposition under which a defendant, after conviction and sentence, is
released subject to conditions imposed by the court and to the supervision of a probation
officer.

Sec. 4, Presidential Decree 968 also known as the Probation Law of 1976, provides:
Grant of Probation. Subject to the provisions of this Decree, the trial court may, after it
shall have convicted and sentenced a defendant, and upon application by said defendant
within the period for perfecting an appeal, suspend the execution of the sentence and
place the defendant on probation for such period and upon such terms and conditions
as it may deem best; Provided, That no application for probation shall be entertained or
granted if the defendant has perfected the appeal from the judgment of conviction.

Probation may be granted whether the sentence imposes a term of imprisonment or a


fine only. An application for probation shall be filed with the trial court. The filing of the
application shall be deemed a waiver of the right to appeal.

An order granting or denying probation shall not be appealable.

Case Summary:
This case is about a) the need, when invoking self-defense, to prove all that it takes; b)
what distinguishes frustrated homicide from attempted homicide; and c) when an
accused who appeals may still apply for probation on remand of the case to the trial
court.

Facts:
The public prosecutor of Camarines Sur charged the accused Arnel Colinares (Arnel)
with frustrated homicide before the Regional Trial Court (RTC) of San Jose, Camarines
Sur, in Criminal Case T-2213.

Complainant Rufino P. Buena (Rufino) testified that at around 7:00 in the evening on
June 25, 2000, he and Jesus Paulite (Jesus) went out to buy cigarettes and
from nowhere, Arnel sneaked behind and struck Rufino twice on the head with a huge
stone. Rufino fell unconscious as Jesus fled.

Ananias Jallores (Ananias) testified that he was walking home when he saw Rufino
lying by the roadside and tried to help but someone struck him with something hard
on the right temple, knocking him out and later learned that Arnel had hit him. Paciano
Alano (Paciano) testified that he saw the whole incident since he happened to be
smoking outside his house. He sought the help of a barangay tanod and they brought
Rufino to the hospital. Arnel claimed self-defense that while he was on his way home
that evening when he met Rufino, Jesus, and Ananias who were all quite drunk, he
asked Rufino where he supposed the Mayor of Tigaon was but, rather than reply,
Rufino pushed him, causing his fall. Jesus and Ananias then boxed Arnel several times
on the back. Rufino tried to stab Arnel but missed. The latter picked up a stone and,
defending himself, struck Rufino on the head with it. When Ananias saw this, he
charged towards Arnel and tried to stab him with a gaff. Arnel was able to avoid the
attack and hit Ananias with the same stone. Arnel then fled and hid in his sister’s
house. On September 4, 2000, he voluntarily surrendered at the Tigaon Municipal
Police Station.

Diomedes testified that he, Rufino, Jesus, and Ananias attended a pre-wedding party
on the night of the incident. His three companions were all drunk. On his way home,
Diomedes saw the three engaged in heated argument with Arnel.

VIVAR, 2
CRIMINAL LAW DIGESTS

RTC RULING
On July 1, 2005 the RTC rendered judgment, finding Arnel guilty beyond reasonable
doubt of frustrated homicide and sentenced him to suffer imprisonment from two years
and four months of prision correccional, as minimum, to six years and one day of prision
mayor, as maximum. Since the maximum probationable imprisonment under the law
was only up to six years, Arnel did not qualify for probation.

CA RULING
The CA entirely affirmed the RTC decision but deleted the award for lost income in the
absence of evidence to support it.

Issue:
Whether or not he may still apply for probation on remand of the case to the trial court
Given a finding that Arnel is entitled to conviction for a lower offense and a reduced
probationable penalty - YES

Ruling:
Given a finding that Arnel is entitled to conviction for a lower offense and a reduced
probationable penalty, whether or not he may still apply for probation on remand of the
case to the trial court.

SC: Taken in its entirety, there is a dearth of medical evidence on record to support the
prosecution’s claim that Rufino would have died without timely medical intervention.
Thus, the Court finds Arnel liable only for attempted homicide and entitled to the
mitigating circumstance of voluntary surrender. Ordinarily, Arnel would no longer be
entitled to apply for probation, he having appealed from the judgment of the RTC
convicting him for frustrated homicide.

But, the Court finds Arnel guilty only of the lesser crime of attempted homicide and
holds that the maximum of the penalty imposed on him should be lowered to
imprisonment of four months of arresto mayor, as minimum, to two years and four
months of prision correccional, as maximum. With this new penalty, it would be but fair
to allow him the right to apply for probation upon remand of the case to the RTC. In a
real sense, the Court’s finding that Arnel was guilty, not of frustrated homicide, but only
of attempted homicide, is an original conviction that for the first time imposes on him a
“probationable” penalty. Had the RTC done him right from the start, it would have found
him guilty of the correct offense and imposed on him the right penalty of two years and
four months maximum. This would have afforded Arnel the right to apply for probation.

Fallo:
WHEREFORE, the Court PARTIALLY GRANTS the petition, MODIFIES the Decision
dated July 31, 2007 of the Court of Appeals in CA-G.R. CR 29639,FINDS petitioner
Arnel Colinares GUILTY beyond reasonable doubt of attempted homicide, and
SENTENCES him to suffer an indeterminate penalty from four months of arresto
mayor, as minimum, to two years and four months of prision correccional, as
maximum, and to pay Rufino P. Buena the amount of P20,000.00 as moral damages,
without prejudice to petitioner applying for probation within 15 days from notice that
the record of the case has been remanded for execution to the Regional Trial Court of
San Jose, Camarines Sur, in Criminal Case T-2213.

VIVAR, 2
CRIMINAL LAW DIGESTS

Badge <<<

Bala v. Hon. Martinez


G.R. No. L-67301, January 29,
1990 SARMIENTO, J:

Doctrine:
 No application for probation shall be granted if defendant has perfected the appeal
from the judgment of conviction; Presidential Decree 1990, not to be given retroactive
effect.
 Expiration of probation period alone does not automatically terminate probation, a
final order of discharge from the court is required.
 Probation period may be shortened or made longer but not to exceed the period set
in the law.
 An order revoking probation or modifying the terms thereof is unappealable.
 Probation is a mere privilege and rests on the court’s sound discretion.

Facts:
Herein petitioner, Manuel V. Bala, had been indicted for removing and substituting the
picture of Maria Eloisa Criss Diazen which had been attached to her United States of
America passport, with that of Florencia Notarte, in effect falsifying a genuine public or
official document. On January 3, 1978, he was held guilty as charged.

The Court of Appeals affirmed in toto the lower court's decision.

After the case had been remanded to the court of origin for execution of judgment, the
petitioner applied for and was granted probation by the respondent judge in his order
dated August 11, 1982. The petitioner was then placed under probation for a period of
one (1) year, subject to the terms and conditions enumerated therein.

On September 23, 1982, the probationer (petitioner) asked his supervising probation
officer for permission to transfer his residence from BF Homes to Phil-Am Life
Subdivision in Las Pin9 as specifically 33 Jingco Street. The probation officer verbally
granted the probationer's request as he found nothing objectionable to it.

By the terms of the petitioner's probation, it should have expired on August 10, 1983,
one year after the order granting the same was issued. But, the order of final discharge
could not be issued because the respondent probation officer had not yet submitted his
final report on the conduct of his charge.

Respondent People of the Philippines, through Assistant City Fiscal Jose D. Cajucom of
Manila, filed a motion to revoke the probation of the petitioner alleging that the
petitioner had violated the terms and conditions of his probation before Branch XX of
the Regional Trial Court of Manila, presided over by the respondent judge.

The petitioner filed his opposition to the motion on the ground that he was no longer
under probation, his probation period having terminated on August 10, 1983, as
previously adverted to. As such, no valid reason existed to revoke the same, he
contended.

Issue:
Whether or not petitioner’s probation was terminated by the expiration of the probation
period.

Ruling:
No, petitioner’s probation was not terminated by the expiration of the probation period.

The present law on probation, Presidential Decree (P.D.) 1990, which amends section
4 of P.D. 968, clearly states that "no application for probation shall be entertained or
granted if the defendant has perfected the appeal from the judgment of conviction."
However, in the case at bar, P.D. 1990 is inapplicable. P.D. 1990, which went in force

VIVAR, 2
CRIMINAL LAW DIGESTS

on January 15, 1985 cannot be given retroactive effect because it would be prejudicial
to the accused.

Probation is revocable before the final discharge of the probationer by the court, contrary
to the petitioner's submission Section 16 of PD 968 8 is clear on this score:

Sec. 16. Termination of Probation. — After the period of probation and upon
consideration of the report and recommendation of the probation officer, the
court may order the final discharge of the probationer upon finding that he has
fulfilled the terms and conditions of his probation and thereupon the case is
deemed terminated.

Thus, the expiration of the probation period alone does not automatically terminate
probation. Nowhere is the ipso facto termination of probation found in the provisions of
the probation law. Probation is not coterminous with its period. There must first be
issued by the court of an order of final discharge based on the report and
recommendation of the probation officer. Only from such issuance can the case of the
probationer be deemed terminated.

The period of probation may either be shortened or made longer, but not to exceed the
period set in the law. This is so because the period of probation, like the period of
incarceration, is deemed the appropriate period for the rehabilitation of the probationer.
In the instant case, a review of the records compels a revocation of the probation without
the need of further proceedings in the trial court which, after all, would only be an
exercise in futility. If we render justice now, why should we allow the petitioner to further
delay it. Probationer Manuel Bala failed to reunite with responsible society. Precisely he
was granted probation in order to give him a chance to return to the main stream, to
give him hope — hope for self-respect and a better life. Unfortunately, he has continued
to shun the straight and narrow path. He thus wrecked his chance. He has not reformed.

By his actuations, probationer-petitioner Manuel V. Bala has ridiculed the probation


program. Instead of utilizing his temporary liberty to rehabilitate and reintegrate himself
as a productive, law abiding, and socially responsible member of society, he continued
in his wayward ways — falsifying public or official documents.

At any time during the probation, the court may issue a warrant for the arrest of a
probationer for violation of any of the conditions of probation. The probationer, once
arrested and detained, shall immediately be brought before the court for a hearing which
may be informal and summary, of the violation charged. If the violation is established,
the court may revoke or continue his probation and modify the conditions thereof. If
revoked, the court shall order the probationer to serve the sentence originally
imposed. An order revoking the grant of probation or modifying the terms and
conditions thereof shall not be appealable.

The probation having been revoked, it is imperative that the probationer be arrested
so that he can serve the sentence originally imposed. The expiration of the probation
period of one year is of no moment, there being no order of final discharge as yet, as we
stressed earlier. Neither can there be a deduction of the one year probation period
from the penalty of one year and one day to three years, six months, and twenty-one
days of imprisonment because an order placing the defendant on "probation" is not a
"sentence," but is in effect a suspension of the imposition of the sentence. It is not a
final judgment but an "interlocutory judgment" in the nature of a conditional order
placing the convicted defendant under the supervision of the court for his reformation,
to be followed by a final judgment of discharge, if the conditions of the probation are
complied with, or by a final judgment if the conditions are violated."

Lastly, probation is a mere privilege. Accordingly, the grant of probation rests solely
upon the discretion of the court. This discretion is to be exercised primarily for the
benefit of organized society, and only incidentally for the benefit of the accused. If the
probationer has proven to be unrepentant, as in the case of the petitioner, the State is

VIVAR, 2
CRIMINAL LAW DIGESTS

not barred from revoking such a privilege. Otherwise, the seriousness of the offense is
lessened if probation is not revoked.

The Petition is dismissed and the probation of the petitioner is hereby revoked

Fallo:
WHEREFORE, the Petition is DISMISSED and the probation of the petitioner is hereby
REVOKED. Further, the trial court is ORDERED to issue a warrant for the arrest of the
petitioner and for him to serve the sentence originally imposed without any deduction.
Costs against the petitioner.

VIVAR, 2
CRIMINAL LAW DIGESTS

Badge <<<

Urbano v. Comelec
G.R. No. 168550, August 10,
2006 TINGA, J:

Doctrine:
Perfection of an appeal is a relinquishment of the alternative remedy of availing of the
Probation Law, the purpose of which is to prevent speculation or opportunism on the
part of the accused who, although already eligible, did not at once apply for probation,
but did so only after failing in his appeal.

During the period of probation, the probationer is not disqualified from running for a
public office because the accessory penalty of suspension from public office is put on
hold for the duration of the probation.

Facts:
Norma L. Mejes (Mejes) filed a petition to disqualify Moreno from running for Punong
Barangay on the ground that the latter was convicted by final judgment of the crime of
Arbitrary Detention. The Comelec en banc granted her petition and disqualified
Moreno. Moreno filed an answer averring that the petition states no cause of action
because he was already granted probation. Allegedly, following the case of Baclayon v.
Mutia, the imposition of the sentence of imprisonment, as well as the accessory
penalties, was thereby suspended. Moreno also argued that under Sec. 16 of the
Probation Law of 1976 (Probation Law), the final discharge of the probation shall
operate to restore to him all civil rights lost or suspended as a result of his conviction
and to fully discharge his liability for any fine imposed. However, the Comelec en banc
assails Sec. 40(a) of the Local Government Code which provides that those sentenced
by final judgment for an offense involving moral turpitude or for an offense punishable
by one (1) year or more of imprisonment, within two (2) years after serving sentence,
are disqualified from running for any elective local position. Since Moreno was
released from probation on December 20, 2000, disqualification shall commence on
this date and end two (2) years thence. The grant of probation to Moreno merely
suspended the execution of his sentence but did not affect his disqualification from
running for an elective local office. On his petition, Moreno argues that the
disqualification under the Local Government Code applies only to those who have
served their sentence and not to probationers because the latter do not serve the
adjudged sentence. The Probation Law should allegedly be read as an exception to the
Local Government Code because it is a special law which applies only to probationers.
Further, even assuming that he is disqualified, his subsequent election as Punong
Barangay allegedly constitutes an implied pardon of his previous misconduct.

Issue:
Whether or not Moreno’s probation grant him the right to run in public office?

Ruling:
Yes. Sec. 16 of the Probation Law provides that "[t]he final discharge of the probationer
shall operate to restore to him all civil rights lost or suspended as a result of his
conviction and to fully discharge his liability for any fine imposed as to the offense for
which probation was granted." Thus, when Moreno was finally discharged upon the
court's finding that he has fulfilled the terms and conditions of his probation, his case
was deemed terminated and all civil rights lost or suspended as a result of his
conviction were restored to him, including the right to run for public office. It is
important to note that the disqualification under Sec. 40(a) of the Local Government
Code covers offenses punishable by one (1) year or more of imprisonment, a
penalty which also covers probationable offenses. In spite of this, the provision does
not specifically disqualify probationers from running for a local elective office.
Probation Law should be construed as an exception to the Local Government Code.
While the Local Government Code is a later law which sets forth the qualifications and
disqualifications of local elective officials, the Probation Law is a special legislation
which applies only to probationers. It is a canon of statutory construction that a later
statute, general in its terms and not

VIVAR, 2
CRIMINAL LAW DIGESTS

expressly repealing a prior special statute, will ordinarily not affect the special provisions
of such earlier statute.

Fallo:
WHEREFORE, the petition is GRANTED. The Resolution of the Commission on
Elections en banc dated June 1, 2005 and the Resolution of its First Division dated
November 15, 2002, as well as all other actions and orders issued pursuant thereto,
are ANNULLED and SET ASIDE. The Commission on Elections is directed to proceed in
accordance with this Decision. No pronouncement as to costs.

VIVAR, 2
CRIMINAL LAW DIGESTS

Badge <<<

Jalosjos v. Comelec
G.R. No. 193237, October 09, 2012
CARPIO, J:

Doctrine:
The COMELEC properly cancelled Jalosjos’ certificate of candidacy. A void certificate of
candidacy on the ground of ineligibility that existed at the time of the filing of the
certificate of candidacy can never give rise to a valid candidacy, and much less to valid
votes. Jalosjos’ certificate of candidacy was cancelled because he was ineligible from
the start to run for Mayor. Whether his certificate of candidacy is cancelled before or
after the elections is immaterial because the cancellation on such ground means he
was never a valid candidate from the very beginning, his certificate of candidacy being
void ab initio. Jalosjos’ ineligibility existed on the day he filed his certificate of
candidacy, and the cancellation of his certificate of candidacy retroacted to the day he
filed it. Thus, Cardino ran unopposed. There was only one qualified candidate for
Mayor in the May 2010 elections – Cardino – who received the highest number of
votes.

The last sentence of Article 32 states that "the offender shall not be permitted to hold
any public office during the period of his perpetual special disqualification." Once the
judgment of conviction becomes final, it is immediately executory. Any public office
that the convict may be holding at the time of his conviction becomes vacant upon
finality of the judgment, and the convict becomes ineligible to run for any elective
public office perpetually. In the case of Jalosjos, he became ineligible perpetually to
hold, or to run for, any elective public office from the time his judgment of conviction
became final.

Case Summary:
Both Jalosjos and Cardino were candidates for Mayor of Dapitan City, Zamboanga del
Norte in the May 2010 elections. Jalosjos was running for his third term. Cardino filed
on 6 December 2009 a petition under Section 78 of the Omnibus Election Code to deny
due course and to cancel the certificate of candidacy of Jalosjos. Cardino asserted that
Jalosjos made a false material representation in his certificate of candidacy when he
declared under oath that he was eligible for the Office of Mayor. Cardino claimed that
long before Jalosjos filed his certificate of candidacy, Jalosjos had already been
convicted by final judgment for robbery and sentenced to prisió n mayor by the Regional
Trial Court, Branch 18 (RTC) of Cebu City.

Facts:
1. Both Jalosjos and Cardino were candidates for Mayor of Dapitan City, Zamboanga
del Norte in the May 2010 elections.
a. Jalosjos was running for his third term.
2. Cardino filed a petition under Section 78 of the Omnibus Election Code to deny due
course and to cancel the certificate of candidacy of Jalosjos alleging that:
a. Jalosjos made a false material representation in his COC when he declared
under oath that he was eligible for the Office of Mayor.
b. long before Jalosjos filed his certificate of candidacy, Jalosjos had already
been convicted by final judgment for robbery and sentenced to prisió n
mayor by the RTC.
c. Jalosjos has not yet served his sentence.
3. Jalosjos admitted his conviction but stated that he had already been granted
probation.
4. Cardino countered that the RTC revoked Jalosjos’ probation in an Order.
5. Jalosjos refuted Cardino and stated that the RTC issued an Order declaring that
Jalosjos had duly complied with the order of probation and that during the 2004
elections, the COMELEC denied a petition for disqualification filed against him on
the same grounds.
6. COMELEC First Division granted Cardino’s petition and cancelled Jalosjos’
certificate of candidacy Jalosjos "is not eligible by reason of his disqualification as
provided for in Section 40(a) of Republic Act No. 7160."

VIVAR, 2
CRIMINAL LAW DIGESTS

a. "Jalosjos has indeed committed material misrepresentation in his


certificate of candidacy when he declared, under oath, that he is eligible
for the office he seeks to be elected to when in fact he is not by reason of
a final judgment in a criminal case, the sentence of which he has not yet
served."
b. COMELEC First Division found that Jalosjos’ certificate of compliance of
probation was fraudulently issued; thus, Jalosjos has not yet served his
sentence.
c. The penalty imposed on Jalosjos was the indeterminate sentence of one
year, eight months and twenty days of prisió n correccional as minimum,
to four years, two months and one day of prisió n mayor as maximum.
7. COMELEC En Banc denied Jalosjos’ motion for reconsideration.
a. With the proper revocation of Jalosjos’ earlier probation and a clear
showing that he has not yet served the terms of his sentence, there is
simply no basis for Jalosjos to claim that his civil as well as political rights
have been violated.
b. Having been convicted by final judgment, Jalosjos is disqualified to run for
an elective position or to hold public office.
c. His proclamation as the elected mayor in the May 10, 2010 election does
not deprive the Commission of its authority to resolve the present
petition to its finality, and to oust him from the office he now wrongfully
holds.

Issue:
Whether or not COMELEC rightfully cancel the Jalosjos’ Certificate of Candidacy?

Ruling:
NO. SC denied petition.
1. Clearly, Lacuna instructs that the accessory penalty of perpetual special
disqualification "deprives the convict of the right to vote or to be elected to or hold
public office perpetually."
2. The accessory penalty of perpetual special disqualification takes effect
immediately once the judgment of conviction becomes final. The effectivity of
this accessory penalty does not depend on the duration of the principal penalty,
or on whether the convict serves his jail sentence or not. The last sentence of
Article 32 states that "the offender shall not be permitted to hold any public
office during the period of his perpetual special disqualification." Once the
judgment of conviction becomes final, it is immediately executory. Any public
office that the convict may be holding at the time of his conviction becomes
vacant upon finality of the judgment, and the convict becomes ineligible to run
for any elective public office perpetually. In the case of Jalosjos, he became
ineligible perpetually to hold, or to run for, any elective public office from the
time his judgment of conviction became final.
3. Perpetual special disqualification is a ground for a petition under Section 78 of
the Omnibus Election Code because this accessory penalty is an ineligibility,
which means that the convict is not eligible to run for public office, contrary to
the statement that Section 74 requires him to state under oath. As used in
Section 74, the word "eligible" means having the right to run for elective public
office, that is, having all the qualifications and none of the ineligibilities to run
for public office. As this Court held in Fermin v. Commission on Elections,17 the
false material representation may refer to "qualifications or eligibility." One
who suffers from perpetual special disqualification is ineligible to run for public
office. If a person suffering from perpetual special disqualification files a
certificate of candidacy stating under oath that "he is eligible to run for (public)
office," as expressly required under Section 74, then he clearly makes a false
material representation that is a ground for a petition under Section 78.
4. What is indisputably clear is that the false material representation of Jalosjos is
a ground for a petition under Section 78. However, since the false material
representation arises from a crime penalized by prisió n mayor, a petition under
Section 12 of the Omnibus Election Code or Section 40 of the Local Government
Code can also be properly filed. The petitioner has a choice whether to anchor his
petition on Section 12 or Section 78 of the Omnibus Election Code, or on Section

VIVAR, 2
CRIMINAL LAW DIGESTS

40 of the Local Government Code. The law expressly provides multiple remedies
and the choice of which remedy to adopt belongs to the petitioner.
5. The COMELEC properly cancelled Jalosjos’ certificate of candidacy. A void
certificate of candidacy on the ground of ineligibility that existed at the time of
the filing of the certificate of candidacy can never give rise to a valid candidacy,
and much less to valid votes. Jalosjos’ certificate of candidacy was cancelled
because he was ineligible from the start to run for Mayor. Whether his
certificate of candidacy is cancelled before or after the elections is immaterial
because the cancellation on such ground means he was never a valid candidate
from the very beginning, his certificate of candidacy being void ab initio.
Jalosjos’ ineligibility existed on the day he filed his certificate of candidacy, and
the cancellation of his certificate of candidacy retroacted to the day he filed it.
Thus, Cardino ran unopposed. There was only one qualified candidate for
Mayor in the May 2010 elections – Cardino – who received the highest number
of votes.

Fallo:
WHEREFORE, the Motion for Reconsideration in G.R. No. 193237 is DENIED, and the
Petition in G.R. No. 193536 is GRANTED. The Resolutions dated 10 May 2010 and 11
August 2010 of the COMELEC First Division and the COMELEC En Bane, respectively,
in SPA No. 09-076 (DC), are AFFIRMED with the MODIFICATION that Agapito J. Cardino
ran unopposed in the May 2010 elections and thus received the highest number of
votes for Mayor. The COMELEC En Bane is DIRECTED to constitute a Special City
Board of Canvassers to proclaim Agapito J. Cardino as the duly elected Mayor of
Dapitan City, Zamboanga del Norte.

Let copies of this Decision be furnished the Secretaries of the Department of Justice
and the Department of Interior and Local Government so they can cause the arrest of,
and enforce the jail sentence on, Dominador G. Jalosjos, Jr. due to his conviction for
the crime of robbery in a final judgment issued by the Regional Trial Court (Branch
18) of Cebu City in Criminal Case No. CCC-XIV140-CEBU. SO ORDERED.

VIVAR, 2
CRIMINAL LAW DIGESTS

Badge <<<

People v. Bayot
G.R. No. 200030, April 18, 2012
PEREZ, J:

Doctrine:
Art. 89. How criminal liability is totally extinguished.—Criminal liability is totally
extinguished: 1. By death of the convict, as to the personal penalties; and as to
pecuniary penalties, liability therefor is extinguished only when the death of the offender
occurs before final judgment.

Facts:
Appellant Nelson Bayot was charged with rape. The Information filed against him
states that on September 17, 1997, by means of force, violence and intimidation, did
then and there, willfully, unlawfully and feloniously have carnal knowledge of and/or
sexual intercourse with [AAA], 44 years old, against her will.

RTC: In its July 31, 2000 Decision, convicted appellant of the crime of rape and
sentenced him to suffer the penalty of reclusion perpetua and to pay AAA the amount
of P40,000.00 as indemnity with costs.

CA: In its Decision dated May 9, 2006, affirmed appellant’s conviction with the
modification increasing the award of indemnity from P40,000.00 to P50,000.00. It
likewise awarded moral damages in favor of AAA in the amount of P50,000.00.

However, in a letter dated May 29, 2006, the Penal Superintendent of the New Bilibid
Prison, informed the Court of Appeals that appellant died at the New Bilibid Prison
Hospital on December 4, 2004.

Nonetheless, the Public Attorney’s Office still appealed, on behalf of appellant, the
aforesaid Court of Appeals Decision to this Court (Supreme Court) dated May 31, 2006,
which was given due course by the Court of Appeals per Resolution dated January 19,
2007. The Court of Appeals also directed the Chief of the Judicial Records Division to
forward the entire records of the case to this Court.

Issue:
Whether or not appellant’s death on December 4, 2004, during the pendency of his
appeal before the Court of Appeals, extinguished his criminal liability for the crime of
rape and his civil liability solely arising from or based on said crime. (YES)

Ruling:
YES, the death of the accused pending appeal of his conviction extinguishes his
criminal liability, as well as the civil liability ex delicto.

Article 89(1) of the Revised Penal Code provides for the effect of death of the accused
on his criminal, as well as civil, liability. It reads:

ART. 89. How criminal liability is totally extinguished. – Criminal liability is


totally extinguished:
(1) By the death of the convict, as to the personal penalties; and as to pecuniary
penalties, liability therefor is extinguished only when the death of the
offender occurs before final judgment; x x x

Applying the foregoing provision, People vs. Bayotas had laid down the following
guidelines:
1. Death of the accused pending appeal of his conviction extinguishes his
criminal liability as well as the civil liability based solely thereon. As
opined by Justice Regalado, in this regard, “the death of the accused prior to
final judgment terminates his criminal liability and only the civil liability
directly

VIVAR, 2
CRIMINAL LAW DIGESTS

arising from and based solely on the offense committed, i.e., civil liability ex
delicto in senso strictiore."

2. Corollarily, the claim for civil liability survives notwithstanding the death
of the accused, if the same may also be predicated on a source of
obligation other than delict. Article 1157 of the Civil Code enumerates these
other sources of obligation from which the civil liability may arise as a result of
the same act or omission: law, contracts, quasi contracts, and quasi delict.

3. Where the civil liability survives, as explained in Number 2 above, an action


for recovery therefor may be pursued but only by way of filing a separate
civil action subject to the Rules on Criminal Procedure. This separate civil
action may be enforced either against the executor/administrator or the estate
of the accused, depending on the source of obligation upon which the same is
based as explained above.

4. Finally, the private offended party need not fear a forfeiture of his right to file this
separate civil action by prescription, in cases where during the prosecution of the
criminal action and prior to its extinction, the private-offended party instituted
together therewith the civil action. In such case, the statute of limitations on
the civil liability is deemed interrupted during the pendency of the criminal
case, conformably with the provisions of Article 1155 of the Civil Code, that
should thereby avoid any apprehension on a possible privation of right by
prescription.

From the foregoing, it is clear that the death of the accused pending appeal of
his conviction extinguishes his criminal liability, as well as the civil liability ex
delicto. The rationale, therefore, is that the criminal action is extinguished inasmuch
as there is no longer a defendant to stand as the accused, the civil action instituted
therein for recovery of civil liability ex delicto is ipso facto extinguished, grounded as it
is on the criminal case.

Evidently, it is already unnecessary to rule on appellant’s appeal. Appellant’s


appeal was still pending and no final judgment had been rendered against him at the
time of his death. Thus, whether or not appellant was guilty of the crime charged
had become irrelevant because even assuming that appellant did incur criminal
liability and civil liability ex delicto, these were totally extinguished by his death.

Fallo:
WHEREFORE, in view of the death of appellant Nelson Bayot y Satina, the Decision
dated 9 May 2006 of the Court of Appeals in CA-G.R. CEB-CR-H.C. No. 00269 is SET
ASIDE and Criminal Case No. 98-2025 before the RTC of Kabankalan City, Negros
Occidental, is DISMISSED. Costs de oficio.

VIVAR, 2
CRIMINAL LAW DIGESTS

Badge <<<

People v. Bautista
G.R. No. 168641, April 27, 2007
AUSTRIA-MARTINEZ, J:

Doctrine:
Prescription; It is a well-settled rule that the filing of the complaint with the fiscal’s office
suspends the running of the prescriptive period; The prescriptive period remains tolled
from the time the complaint was filed with the Office of the Prosecutor until such time
that respondent is either convicted or acquitted by the proper court.

Facts:
On June 12, 1999, a dispute arose between respondent and his co-accused Leonida
Bautista, on one hand, and private complainant Felipe Goyena, Jr., on the other.

Private complainant filed a Complaint with the Office of the Barangay of Malate,
Manila, butvno settlement was reached. The barangay chairman then issued a
Certification to file action dated August 11, 1999.

On August 16, 1999, private complainant filed with the Office of the City Prosecutor
(OCP) a Complaint for slight physical injuries against herein respondent and his co-
accused. After conducting the preliminary investigation, Prosecutor Jessica Junsay-Ong
issued a Joint Resolution dated November 8, 1999 recommending the filing of an
Information against herein respondent. Such recommendation was approved by the
City Prosecutor, represented by First Assistant City Prosecutor Eufrocino A. Sulla, but
the date of such approval cannot be found in the records. The Information was,
however, filed with the Metropolitan Trial Court (MeTC) of Manila, Branch 28 only
on June 20, 2000.

Respondent sought the dismissal of the case against him on the ground that by the
time the Information was filed, the 60-day period of prescription from the date of the
commission of the crime, that is, on June 12, 1999 had already elapsed.

MeTC: ruled that the offense had not yet prescribed.


Respondent elevated the issue to the RTC via a Petition for Certiorari.
RTC: denied said petition and concurred with the opinion of the MeTC.
Respondent then filed a Petition for Certiorari with the CA.
CA: On June 22, 2005, the CA rendered its Decision wherein it held that, the 60-day
prescriptive period was interrupted when the offended party filed a Complaint with
the OCP of Manila on August 16, 1999. Nevertheless, the CA concluded that the
offense had prescribed by the time the Information was filed with the MeTC,
reasoning that even if the 10-day period for the CP or ACP Sulla, to act on the
resolution is extended up to the utmost limit, it ought not have been taken as late as
the last day of the year 1999. Yet, the information was filed with the MeTC only on
June 20, 2000, or already nearly six (6) months into the next year. To use once again
the language of Article 91 of the RPC, the proceedings at the CPO was unjustifiably
stopped for any reason not imputable to him (the accused) for a time very much more
than the prescriptive period of only two (2) months. The offense charged had,
therefore, already prescribed when filed with the court on June 20, 2000.

Petitioner now comes before the Supreme Court seeking the reversal of the foregoing CA
Decision.

Issue:
Whether or not the prescriptive period began to run anew after the investigating
prosecutor’s recommendation to file the proper criminal information against respondent
was approved by the City Prosecutor. (NO)

VIVAR, 2
CRIMINAL LAW DIGESTS

Ruling:
The answer is in the negative.
It is not disputed that the filing of the Complaint with the OCP effectively interrupted
the running of the 60-day prescriptive period for instituting the criminal action for
slight physical injuries.

Article 91 of the Revised Penal Code provides thus:


ART. 91. Computation of prescription of offenses. - The period of
prescription shall commence to run from the day on which the crime is
discovered by the offended party, the authorities, or their agents, and
shall be interrupted by the filing of the complaint or information, and
shall commence to run again when such proceedings terminate
without the accused being convicted or acquitted, or are unjustifiably
stopped for any reason not imputable to him.

The term of prescription shall not run when the offender is absent from
the Philipppine Archipelago.
The CA and respondent are of the view that upon approval of the investigating
prosecutor's recommendation for the filing of an Information against respondent, the
period of prescription began to run again. The Court does not agree. It is a well-settled
rule that the filing of the complaint with the fiscal’s office suspends the running
of the prescriptive period.

The proceedings against respondent were not terminated upon the City
Prosecutor's approval of the investigating prosecutor's recommendation that an
Information be filed with the court. The prescriptive period remains tolled from
the time the complaint was filed with the Office of the Prosecutor until such
time that respondent is either convicted or acquitted by the proper court.
The Office of the Prosecutor miserably incurred some delay in filing the
Information but such mistake or negligence should not unduly prejudice the
interests of the State and the offended party.
The constitutional right of the accused to a speedy trial cannot be invoked by
the petitioner in the present petition considering that the delay occurred not in the
conduct of preliminary investigation or trial in court but in the filing of the
Information after the City Prosecutor had approved the recommendation of the
investigating prosecutor to file the Information.

The Office of the Solicitor General does not offer any explanation as to the delay
in the filing of the Information. The Court will not be made as an unwitting tool in
the deprivation of the right of the offended party to vindicate a wrong
purportedly inflicted on him by the mere expediency of a prosecutor not filing the
proper information in due time.
The Court will not tolerate the prosecutor’s apparent lack of a sense of
urgency in fulfilling their mandate. Under the circumstances, the more appropriate
course of action should be the filing of an administrative disciplinary action
against the erring public officials.

The Petition is hereby granted and the Decision of the RTC is hereby reinstated.
Fallo:
WHEREFORE, the Petition is hereby GRANTED. The Decision of the Court of Appeals in
CA-G.R. SP No. 72784 is hereby REVERSED and SET ASIDE and the Decision of the
Regional Trial Court of Manila in Civil Case No. 02-103990 is hereby REINSTATED.
Let the Secretary of the Department of Justice be furnished a copy of herein Decision
for appropriate action against the erring officials.

VIVAR, 2
CRIMINAL LAW DIGESTS

Badge <<<

People v. Pangilinan
G.R. No. 152662, June 13, 2012
PEREZ, J:

Doctrine:
Prescription of Offenses; Since BP Blg. 22 is a special law that imposes a penalty of
imprisonment of not less than thirty (30) days but not more than one year or by a fine
for its violation, it therefore prescribes in four (4) years in accordance with the aforecited
law.

Facts:
On September 16, 1997, private complainant Virginia Malolos filed with the Office of
the City Prosecutor an affidavit-complaint for estafa and violation of B.P. Blg. 22
against respondent Ma. Theresa Pangilinan alleging that respondent issued 9 checks in
favor of private complainant which were dishonored upon presentment for payment.

Respondent filed a civil case for accounting, recovery of commercial documents,


enforceability and effectivity of contract and specific performance against private
complainant withthe RTC, followed by "Petition to Suspend Proceedings on the Ground of
Prejudicial Question" before the Office of the City Prosecutor. The City Prosecutor
approved the petition for suspension.

Private complainant raised the matter before the Secretary of Justice who reversed
the resolution of the City Prosecutor and ordered the filing of Informations for
violation of B.P. Blg. 22 against respondent in connection with her issuance of 2
checks. The 7 other checks included in the affidavit-complaint filed on September 16,
1997 were, however, dismissed.

Consequently, two counts for violation of B.P. Blg. 22, were filed against respondent
Pangilinan on February 3, 2000 before the Office of the Clerk of Court, MeTC.

Respondent filed an "Omnibus Motion to Quash the Information and to Defer the
Issuance of Warrant of Arrest" alleging that her criminal liability has been extinguished
by reason of prescription.

MeTC: granted the motion in an order

Private complainant filed a notice of

appeal.

RTC: reversed the order of MeTC ruling that inasmuch as the Informations were filed
on February 3, 2000 although received by the Court itself only on June 7, 2000, they
are covered by the Rule as it was worded before the latest amendment. The criminal
action on two counts for violation of B.P. Blg. 22, had, therefore, not yet prescribed
when the same was filed with the court a quo considering the appropriate complaint
that started the proceedings having been filed with the Office of the Prosecutor on
September 16, 1997 yet. RTC in reversing the said decision, directed to proceed with
the hearing of the criminal cases.

CA: reversed the decision of RTC, thereby dismissing the criminal cases for the reason
that the cases for violation of B.P. Blg. 22 had already prescribed. It held that it
reckons the commencement of the period of prescription for violations of B.P. Blg. 22
imputed to respondent sometime in the latter part of 1995, as it was within this period
that the she was notified by the private complainant of the fact of dishonor of the
subject checks and, the 5-day grace period granted by law had elapsed. The private
respondent then had, pursuant to Section 1 of Act3326 , four years therefrom or
until the latter part of 1999 to file her complaint or information against the petitioner
before the proper court. The Informations against the petitioner having been filed on
February 3, 2000, the said cases had therefore, clearly prescribed.

VIVAR, 2
CRIMINAL LAW DIGESTS

Pursuant to Section 2 of Act 3326, as amended, prescription shall be interrupted when


proceedings are instituted against the guilty person.

It was held in the case of Zaldivia vs. Reyes that the proceedings referred to in Section
2 of Act No. 3326, as amended, are ‘judicial proceedings’, which means the filing of
the complaint or information with the proper court. Otherwise stated, the running of
the prescriptive period shallbe stayed on the date the case is actually filed in
court and not on any date before that, which is in consonance with Section 2 of
Act 3326, as amended. While the aforesaid case involved a violation of a municipal
ordinance, the SC, considering that Section 2 of Act 3326, as amended, governs the
computation of the prescriptive period of both ordinances and special laws, finds that
the ruling likewise applies to special laws, such as Batas Pambansa Blg. 22.

The OSG sought relief to the Supreme Court in the instant petition for review
contending that, while it admits that Act No. 3326, as amended by Act No. 3585 and
further amended by Act No. 3763, governs the period of prescription for violations of
special laws, it is the institution of criminal actions, whether filed with the court or with
the Office of the City Prosecutor, that interrupts the period of prescription of the offense
charged. It submits that the filing of the complaint-affidavit by private complainant on
September 16, 1997 interrupted the running of the prescriptive period of the subject
B.P. Blg. 22 cases.

Petitioner submits that the CA erred in its decision when it relied on the doctrine laid
down by this Court in the case of Zaldivia vs. Reyes, Jr. that the filing of the complaint
with the Office of the City Prosecutor is not the "judicial proceeding" that could have
interrupted the period of prescription, contending that in a catena of cases, the SC ruled
that the filing of a complaint with the Fiscal’s Office for preliminary investigation
suspends the running of the prescriptive period. It therefore concluded that the filing
of the Informations with the MeTC of Quezon City on February 3, 2000 was still within the
allowable period of four years within which to file the criminal cases for violation of
B.P. Blg. 22 in accordance with Act No. 3326, as amended.

Respondent reiterates the ruling of the CA that the filing of the complaint before the
City Prosecutor’s Office did not interrupt the running of the prescriptive period
considering that the offense charged is a violation of a special law, thus, claiming that
the cases relied upon by petitioner involved felonies punishable under the RPC and are
therefore covered by Article 91 of the RPC andSection 1, Rule 110 of the Revised Rules
on Criminal Procedure. Respondent pointed out that the crime imputed against her is
for violation of B.P. Blg. 22, which is indisputably a special law and as such, is governed
by Act No. 3326, as amended. She submits that a distinction should thus be made
between offenses covered by municipal ordinances or special laws , as in this case, and
offenses covered by the RPC.

Issue:
Whether or not the filing of the affidavit-complaint for estafa and violation of B.P. Blg.
22 against respondent with the Office of the City Prosecutor on September 16, 1997
interrupted the period of prescription of such offense (YES)

Ruling:
The Court finds that the CA reversively erred in ruling that the offense committed
by respondent had already prescribed. Indeed, Act No. 3326 entitled "An Act to
Establish Prescription for Violations of Special Acts and Municipal Ordinances and to
Provide When Prescription Shall Begin," as amended, is the law applicable to B.P. Blg.
22 cases. The law reads:

SECTION 1. Violations penalized by special acts shall, unless otherwise provided in


such acts, prescribe in accordance with the following rules: (a) xxx; (b) after four years
for those punished by imprisonment for more than one month, but less than two years;
(c) xxx.

VIVAR, 2
CRIMINAL LAW DIGESTS

SECTION 2. Prescription shall begin to run from the day of the commission of the
violation of the law, and if the same be not known at the time, from the discovery thereof
and the institution of judicial proceedings for its investigation and punishment.

The prescription shall be interrupted when proceedings are instituted against the
guilty person, and shall begin to run again if the proceedings are dismissed for reasons
not constituting jeopardy.

Since B.P. Blg. 22 is a special law that imposes a penalty of imprisonment of not
less than 30 days but not more than one year or by a fine for its violation, it
therefore prescribes in four (4) years in accordance with Act No. 3326. The running of
the prescriptive period, however, should be tolled upon the institution of
proceedings against the guilty person.

It has been held in a previous case that the filing of the complaint in the Municipal
Court even if it be merely for purposes of preliminary examination or investigation,
should, and thus, interrupt the period of prescription of the criminal responsibility,
even if the court where the complaint or information is filed cannot try the case on the
merits. Moreover, another case broadened such rule that the filing of the complaint
with the Fiscal’s Office also suspends the running of the prescriptive period of a
criminal offense.

There is no more distinction between cases under the RPC and those covered by
special laws with respect to the interruption of the period of prescription.

The ruling in Zaldivia vs. Reyes, Jr. is not controlling in special laws. It has been held in
a number of cases involving special laws that the institution of proceedings for
preliminary investigation against the accused interrupts the period of prescription.
The Court has even ruled that investigations conducted by the Securities and
Exchange Commission for violations of the Revised Securities Act and the Securities
Regulations Code effectively interrupts the prescription period because it is
equivalent to the preliminary investigation conducted by the DOJ in criminal cases.
In a case which is in all fours with the present case, the Court ruled that
commencement of the proceedings for the prosecution of the accused before
the Office of the City Prosecutor effectively interrupted the prescriptive period
for the offenses they had been charged under
B.P. Blg. 22. In the same case it was held that aggrieved parties, especially those who
do not sleep on their rights and actively pursue their causes, should not be allowed to
suffer unnecessarily further simply because of circumstances beyond their control,
like the accused’s delaying tactics or the delay and inefficiency of the investigating
agencies.

The Court follows the factual finding of the CA that "sometime in the latter part of
1995" is the reckoning date of the commencement of presumption for violations of B.P.
Blg. 22, such being the period within which herein respondent was notified by private
complainant of the fact of dishonor of the checks and the five-day grace period granted
by law elapsed.

The affidavit-complaints for the violations were filed against respondent on


September 16, 1997. The cases reached the MeTC of Quezon City only on February
13, 2000 because in the meanwhile, respondent filed a civil case for accounting
followed by a petition before the City Prosecutor for suspension of proceedings on the
ground of "prejudicial question". The matter was raised before the Secretary of Justice
after the City Prosecutor approved the petition to suspend proceedings. It was only
after the Secretary of Justice so ordered that the Informations for the violation of B.P. Blg.
22 were filed with the MeTC of Quezon City.

Clearly, it was respondent’s own motion for the suspension of the criminal
proceedings that caused the filing in court of the 1997 initiated proceedings only in
2000.

As laid down in jurisprudence, it is unjust to deprive the injured party of the right

VIVAR, 2
CRIMINAL LAW DIGESTS

to obtain vindication on account of delays that are not under his control. The only
thing the offended party must do to initiate the prosecution of the offender is to file
the

VIVAR, 2
CRIMINAL LAW DIGESTS

requisite complaint. Therefore, DOJ is ordered to re-file the Informations for violation of
B.P. Blg. 22 against the respondent.

Fallo:
IN LIGHT OF ALL THE FOREGOING, the instant petition is GRANTED. The 12 March
2002 Decision of the Court of Appeals is hereby REVERSED and SET ASIDE. The
Department of Justice is ORDERED to re-file the informations for violation of BP Blg.
22 against the respondent.

VIVAR, 2
CRIMINAL LAW DIGESTS

Badge <<<

SEC v. Interport Resource Corporation


G.R. No. 135808, October 6, 2008
CHICO-NAZARIO, J:

Doctrine:
The law on the prescription period was never intended to put the prosecuting bodies in
an impossible bind in which the prosecution of a case would be placed way beyond their
control, for even if they avail themselves of the proper remedy, they would still be barred
from investigating and prosecuting the case.
It is an established doctrine that a preliminary investigation interrupts the
prescription period.

Facts:
1. 6 Aug 1994 – Board of Directors of IRC approved a Memorandum of Agreement
(MoA) with Ganda Holdings Berhad (GHB)
a. Under the MoA, IRC acquired 100% or the entire capital stock of Ganda
Energy Holdings, Inc. (GEHI), which would own and operate a 102
megawatt gas turbine power-generating barge.
b. Also stipulated is that GEHI would assume a five-year power purchase
contract with National Power Corp. At that time, GEHI’s power-generating
barge was 97% complete and would go on-line by mid-Sept 1994.
c. In exchange, IRC will issue to GHB 55% of the expanded capital stock of
IRC (amounting to 40.88 billion shares – total par value of P488.44 million).
d. On the side, IRC would acquire 67% of the entire capital stock of
Philippine Racing Club, Inc. (PRCI). PRCI owns 25.724hectares of real
estate property in Makati.
e. Under the Agreement, GHB, a member of the Westmont Group of
Companies in Malaysia, shall extend or arrange a loan required to pay for
the proposed acquisition by IRC of PRCI.

2. 8 Aug 1994 – IRC alleged that a press release announcing the approval of the
agreement was sent through fax to Philippine Stock Exchange (PSE) and the
SEC, but that the fax machine of SEC could not receive it. Upon the advice of SEC,
IRC sent the press release on the morning of 9 Aug 1994.

3. SEC averred that it received reports that IRC failed to make timely public
disclosures of its negotiations with GHB and that some of its directors heavily
traded IRC shares utilizing this material insider information.

4. 6 Aug 1994 – SEC Chairman issued a directive requiring IRC to submit to SEC a
copy of its aforesaid MoA with GHB and further directed all principal officers of
IRC to appear at a hearing before the Brokers and Exchanges Dept (BED) of SEC
to explain IRC’s failure to immediately disclose the information as required by
the Rules on Disclosure of Material Facts by Corporations Whose Securities are
Listed in Any Stock Exchange or Registered/Licensed Under the Securities Act.

5. IRC sent a letter to SEC, attaching copies of MoA and its directors appeared to
explain IRC’s alleged failure to immediately disclose material information as
required under the Rules on Disclosure of Material Facts.

6. 19 Sept 1994 – SEC Chairman issued an Order finding that IRC violated the
Rules on Disclosure when it failed to make timely disclosure, and that some of
the officers and directors of IRC entered into transactions involving IRC shares
in violation of Sec 30,in relation to Sec 36 of the Revised Securities Act.

7. IRC filed an Omnibus Motion (later an Amended Omnibus Motion) alleging that
SEC had no authority to investigate the subject matter, since under Sec 8 of PD
902-A, as amended by PD 1758, jurisdiction was conferred upon the
Prosecution and Enforcement Dept (PED) of SEC.

VIVAR, 2
CRIMINAL LAW DIGESTS

8. IRC also claimed that SEC violated their right to due process when it ordered
that the respondents appear before SEC and show cause why no administrative,
civil or criminal sanctions should be imposed on them, and thus, shifted the
burden of proof to the respondents. They filed a Motion for Continuance of
Proceedings.

9. No formal hearings were conducted in connection with the Motions.

10.25 Jan 1995 – SEC issued an Omnibus Order: creating a special investigating
panel to hear and decide the case in accordance with Rules of Practice and
Procedure before the PED, SEC; to recall the show cause orders; and to deny the
Motion for Continuance for lack of merit.

11.Respondents filed a petition before the CA questioning the Omnibus Orders and
filed a Supplemental Motion wherein they prayed for the issuance of a writ of
preliminary injunction.

12.5 May 1995 – CA granted their motion and issued a writ of preliminary injunction,
which effectively enjoined SEC from filing any criminal, civil or administrative
case against the respondents.13)20 Aug 1998 – CA promulgated a Decision.
a. Determined that there were no implementing rules and regulations
regarding disclosure, insider trading, or any of the provisions of the
Revised Securities Acts which respondents allegedly violated.
b. It found no statutory authority for SEC to initiate and file any suit for civil
liability under Sec 8, 30 and 36 of the Revised Securities Act, thus, it ruled
that no civil, criminal or administrative proceedings may possibly be held
against the respondents without violating their rights to due process and
equal protection.
c. It further resolved that absent any implementing rules, the SEC cannot be
allowed to quash the assailed Omnibus Orders.
d. Further decided that the Rules of Practice and Procedure before the
PED did not comply with the statutory requirements contained in
the Administrative Code of 1997. Section 9, Rule V of the Rules of
Practice and Procedure before the PED affords a party the right to be
present but without the right to cross-examine witnesses presented
against him, in violation of Sec 12(3), Chap 3, Book VII of the
Administrative Code.

Issue:
Whether or not the instant case prescribed already? No

Ruling:
The instant case has not yet prescribed.

Respondents point out that the prescription period applicable to offenses punished
under special laws is 12 years. Since the offense was committed in 1994, they
reasoned that prescription set in as early as 2006 and rendered this case moot.

It is an established doctrine that a preliminary investigation interrupts the prescription


period. A preliminary investigation is essentially a determination whether an offense has
been committed, and whether there is probable cause for the accused to have committed
as offense.

Fallo:
IN VIEW OF THE FOREGOING, the instant Petition is GRANTED. This Court hereby
REVERSES the assailed Decision of the Court of Appeals promulgated on 20 August
1998 in CA-G.R. SP No. 37036 and LIFTS the permanent injunction issued pursuant
thereto. This Court further DECLARES that the investigation of the respondents for
violations of Sections 8, 30 and 36 of the Revised Securities Act may be undertaken by
the proper authorities in accordance with the Securities Regulations Code. No costs.

VIVAR, 2
CRIMINAL LAW DIGESTS

Badge <<<

Romualdez v. CA
G.R. No. 165510-33, July 28,
2006 YNARES-SANTIAGO, J:

Doctrine:
The accused may, at any time before he enters his plea, move to quash the complaint
and information on the ground that the criminal action or liability has been
extinguished, which ground includes the defense of prescription.

Case Summary:
This case lay down few situations wherein the prescription of a case may be
interrupted. It also explained the application of Article 90 and 91 of RPC and Act No.
3326.

Facts:
Petitioner is being charged with violations of Section 7 of RA No. 3019 for failure to file
his Statements of Assets and Liabilities for the period 1967-1985 during his tenure as
Ambassador Extraordinary and Plenipotentiary and for the period 1963-1966 during
his tenure as Technical Assistant in the Department of Foreign Affairs. The said offenses
were said to be discovered by SolGen Francisco Chavez on May 8, 1987, the date when
he filed a complaint against the petitioner with the PCGG. The preliminary investigations
conducted by the PCGG over the 24 offenses ascribed to Romualdez (of failure to file
annual statements of assets and liabilities) were held invalid by the court for lack of
jurisdiction of said offenses.

The nullity of the proceedings initiated by then Solicitor General Chavez in 1987 with
the PCGG and by the PCGG with the Sandiganbayan in 1989 was judicially settled. The
only proceeding that could interrupt the running of prescription of the 1987 case is
that which is filed or initiated by the offended party before the appropriate body or office.
The complaint was filed with the wrong body, the PCGG. Thus, the same could not have
interrupted the running of the prescriptive periods. On March 3, 2004, Office of the
Special Prosecutor initiated the preliminary investigation of Crim Case Nos. 13406-
13429 by requiring the petitioner to submit his counter-affidavit.

Issue:
Whether or not the filing of the complaint with the PCGG in 1987 as well as the filing of
the information with the Sandiganbayan to initiate Criminal Case Nos. 13406-13429 in
1989 interrupted the running of the prescriptive period such that when the Ombudsman
directed petitioner to file his counter-affidavit on March 3, 2004, the offenses have
already prescribed.

Ruling:
Yes. Section 11 of RA No. 3019 provides that all offenses punishable therein shall
prescribe in 15 years. However, prior to the amendment of Section 11 of R.A. No. 3019
by B.P. Blg. 195 which was approved on March 16, 1982, the prescriptive period for
offenses punishable under the said statute was only ten (10) years.

For offenses allegedly committed by the petitioner from 1962 up to March 15, 1982, the
same shall prescribe in 10 years. On the other hand, for offenses allegedly committed
by the petitioner during the period from March 16, 1982 until 1985, the same shall
prescribe in 15 years.

BUT Since the petitioners were absent in the Philippines from 1986 to April 27, 2000,
the respondents were saying that the charge could not have prescribed citing Article
91 of the Revised Penal Code which provides that "[t]he term of prescription should
not run when the offender is absent from the Philippine Archipelago." This was supported
by the dissent of Justice Carpio. He stated that Article 10 of the same Code makes
Article 91 "x x x supplementary to [special laws], unless the latter should x x x provide
the contrary." Nothing in RA 3019 prohibits the supplementary application of Article
91 to that law. Hence, applying Article 91, the prescriptive period in Section 11 of RA

VIVAR, 2
CRIMINAL LAW DIGESTS

3019,

VIVAR, 2
CRIMINAL LAW DIGESTS

before and after its amendment, should run only after petitioner returned to this
jurisdiction on 27 April 2000. There is no gap in the law. Where the special law is
silent, Article 10 of the RPC applies suppletorily.

HOWEVER, Court ruled that the law on prescription of offenses is found in Articles 90
and 91 of the Revised Penal Code for offenses punishable thereunder. For those
penalized under special laws, Act No. 3326 applies.

Fallo:
In view of the foregoing, the applicable 10-and-15-year prescriptive periods in the
instant case, were not interrupted by any event from the time they began to run on
May 8, 1987. The offenses committed by the petitioner for the years 1963-1982
prescribed 10 years from May 8, 1987 or on May 8, 1997. On the other hand, the
alleged offenses committed by the petitioner for the years 1983-1985 prescribed 15
years from May 8, 1987 or on May 8, 2002. Also, when the Office of the Special
Prosecutor initiated the preliminary investigation of Criminal Case Nos. 13406-13429
on March 3, 2004 by requiring the petitioner to submit his counter-affidavit, the
alleged offenses subject therein have already prescribed.

VIVAR, 2
CRIMINAL LAW DIGESTS

Badge <<<

People v. Patriarca, et. al.


G.R. No. 135457, September 29, 2000
BUENA, J:

Doctrine:
Amnesty commonly denotes a general pardon to rebels for their treason or other high
political offenses, or the forgiveness which one sovereign grants to the subjects of
another, who have offended, by some breach, the law of nations.

Criminal liability is totally extinguished by amnesty, which completely extinguishes


the penalty and all its effects.

Case Summary:
The accused-appellant in this case applied for amnesty under Section 1 of
Proclamation No. 724, which will completely extinguished the criminal liability and
penalty imposed to her on the crime she committed. It elaborates the crimes covered
by amnesty in the said Proclamation and relates its effect based on Paragraph 3 of
Article 89 of the Revised Penal Code.

Facts:
On August 16, 1990, an information for murder was filed against Jose Patriarca, Jr.,
alias "Ka Django," "Carlos Narra", "Ka Jessie," et al., charging them of murder. Accused-
appellant Jose Patriarca, Jr. was also charged with Murder for the killing of one Rudy
de Borja and a certain Elmer Cadag .

The defense presented accused Jose Patriarca, Jr. and Francisco Derla who
admitted that accused is a member of the NPA operating in Donsol, Sorsogon, but
denied ever abducting the victims in the three criminal cases filed against him.
Accused-appellant applied for amnesty under Proclamation No. 724 amending
Proclamation No. 347, dated March 25, 1994, entitled "Granting Amnesty to Rebels,
Insurgents, and All Other Persons Who Have or May Have Committed Crimes Against
Public Order, Other Crimes Committed in Furtherance of Political Ends, and Violations
of the Article of War, and Creating a National Amnesty Commission." His application
was favorably granted by the National Amnesty Board. Attached to appellant's brief is
the Notice of Resolution of the National Amnesty Commission (NAC).

Issue:
Whether or not Patriarca is may be granted amnesty under Proclamation No. 724
amending Proclamation No. 347.

Ruling:
Yes. Under Section 1 of Proclamation No. 724 reads thus:

"Section 1. Grant of Amnesty. - Amnesty is hereby granted to all persons who shall
apply therefor and who have or may have committed crimes, on or before June 1,
1995, in pursuit of their political beliefs, whether punishable under the Revised Penal
Code or special laws, including but not limited to the following: rebellion or
insurrection; coup d'etat; conspiracy and proposal to commit rebellion, insurrection,
or coup d'etat;…. disloyalty of public officers or employees; inciting to rebellion or
insurrection; sedition; conspiracy to commit sedition; inciting to sedition; illegal
assembly; illegal association; direct assault; indirect assault; resistance and
disobedience to a person in authority or agents of such person; tumults and other
disturbances of public order; unlawful use of means of publication and unlawful
utterances; alarms and scandals; illegal possession of firearms, ammunitions, and
explosives, committed in furtherance of, incident to, or in connection with the crimes
of rebellion and insurrection; and violations of Articles 59 (desertion), 62 (absence
without leave), 67 (mutiny or sedition), 68 (failure to suppress mutiny or sedition), 94
(various crimes), 96 (conduct unbecoming an officer and gentleman), and 97 (general
article) of the Articles of War; Provided, That the amnesty shall not cover crimes
against chastity and other crimes for personal ends."

VIVAR, 2
CRIMINAL LAW DIGESTS

Amnesty commonly denotes a general pardon to rebels for their treason or other high
political offenses, or the forgiveness which one sovereign grants to the subjects of
another, who have offended, by some breach, the law of nations. Amnesty looks
backward, and abolishes and puts into oblivion, the offense itself; it so overlooks and
obliterates the offense with which he is charged, that the person released by amnesty
stands before the law precisely as though he had committed no offense.

Paragraph 3 of Article 89 of the Revised Penal Code provides that criminal liability is
totally extinguished by amnesty, which completely extinguishes the penalty and all its
effects.

Fallo:
WHEREFORE, IN VIEW OF THE FOREGOING, the decision of the Regional Trial
Court at Sorsogon, Sorsogon, Branch 52 in Griminal Case No. 2773 is REVERSED
and SET ASIDE. Accused-appellant Jose N. Patriarca, Jr. is hereby ACQUITTED of
the crime of murder.

Pursuant to Resolution No. D-99-8683,11 Criminal Case Nos. 2663 and 2664, which
are both filed in the Regional Trial Court, Branch 53, Sorsogon, Sorsogon,12 are
ordered DISMISSED. The release of Jose N. Patriarca who is presently detained at the
Provincial Jail of Sorsogon is likewise ORDERED unless he is being detained for some
other legal cause.

The Director of Prisons is ordered to report within ten (10) days his compliance with
this decision.

VIVAR, 2

You might also like